Prep 2016
Prep 2016
Prep 2016
Question 1
A 9-month-old infant is referred to you by a community outreach program because of low
weight. He was born at term after an uncomplicated pregnancy and delivery. He was exclusively
breastfed until 6 months of age, when a few puréed baby foods were added to his diet. He
continues to breastfeed 6 times daily. His development and physical examination are
unremarkable. His current weight is 7.8 kg, length is 68 cm (Item Q1A), and head circumference
is 44 cm (Item Q1B).
Of the following, the MOST appropriate statement about monitoring this infant’s growth is
A. body mass index is the best parameter for assessing this infant’s growth
B. children should follow a consistent growth percentile line throughout the first 2 years of
life
C. this infant’s growth parameters indicate undernutrition
D. US Centers for Disease Control and Prevention growth charts are the best option for
monitoring growth of breastfed infants
E. World Health Organization growth charts should be used for monitoring children
younger than 2 years of age
The WHO infant charts are based on data collected longitudinally from 6 sites covering Europe,
Africa, south Asia, North America, and the Middle East. The children in this study had no
underlying disease and were born at term to nonsmoking mothers who had access to sufficient
resources. These children were exclusively breastfed until 4 months of age and continued to be
breastfed until at least 12 months of age, therefore the WHO growth charts are felt to represent
"ideal growth." By contrast, CDC growth charts were derived from cross-sectional studies in the
United States from 1963 to 1994, and the children from whom the data were derived were often
formula fed. Breastfed children tend to gain weight faster than formula-fed infants in the first 3
months of life, then slow their weight gain velocity. These changes are reflected in the
differences between the WHO and CDC growth charts. Therefore, the breastfed infant in the
vignette is more likely to have a growth pattern consistent with the WHO chart than the CDC
chart. Of note, both CDC and WHO growth charts for children older than 24 months of age are
derived from cross-sectional data, and the CDC charts are acceptable for use in older children.
For both the WHO and CDC growth charts, changes over time are the best reflection of actual
growth. Many children follow the same growth percentile lines throughout infancy, but it is also
normal for some infants to cross 1 or 2 major percentile lines between 6 and 18 months of age.
Growth parameters at birth reflect the intrauterine environment, but by later infancy, growth
parameters are more consistent with genetic potential. In fact, growth at 2 years of age correlates
well with mean parental height. In children younger than 3 years of age, the best assessment of
acute undernutrition is a weight-for-height below the fifth percentile. Body mass index has not
been validated for children younger than 2 years of age and would not be an appropriate
parameter to assess this infant.
PREP Pearls
• World Health Organization (WHO) growth charts better reflect “ideal growth” of breastfed
infants and are superior to US Centers of Disease Control and Prevention growth charts. The
WHO charts should be used to assess growth of children younger than 2 years of age.
• Many normally growing infants cross 1 or 2 major percentile lines between 6 and 18 months
of age as their growth changes from reflecting an intrauterine environment to reflecting
genetic potential.
• For children younger than 3 years of age, the best assessment of acute undernutrition is a
weight-for-height below the fifth percentile.
Suggested Readings
• Danner E, Joeckel R, Michalak S, Philllips S, Goday PS. Weight velocity in infants and
children. Nutr Clin Pract. 2009;24(1):76-79. doi:
http://dx.doi.org/10.1177/0884533608329663 .
• Grummer-Strawn LM, Renold C, Krebs NF. Use of World Health Organization and CDC
growth charts for children aged 0-59 months in the United States. Morbid Mortal Wkly Rep
MMWR. 2010;59(RR09):1-
15.http://www.cdc.gov/mmwr/preview/mmwrhtml/rr5909a1.htm.
• Keane V. Assessment of growth. In: Kleigman RM, Stanton BF, Schor NF, St Geme JW III,
Behrman RE, eds. Nelson Textbook of Pediatrics. 20th ed. Philadelphia, PA: Saunders
Elsevier; 2015: 84-89.
Question 2
A 17-year-old adolescent presents to the office with complaints of vaginal discharge for 3 days.
She has no significant past medical history, but she is the mother of an 18-month-old child. She
currently uses the etonogestrel implant for contraception. She is not on any other medications.
She reports 2 sexual partners in the past 3 months, and was last sexually active about 2 weeks
ago. Physical examination reveals a temperature of 37.2°C, heart rate of 72 beats/min,
respiratory rate of 16 breaths/min, blood pressure of 110/68 mm Hg, and weight of 54.5 kg. She
has no abdominal tenderness. On pelvic examination, she has a friable cervix with purulent
cervical discharge, but no cervical motion or adnexal tenderness. You are concerned that she has
gonococcal cervicitis.
Of the following, the BEST next steps in the diagnosis and management are
A. culture and treatment with ceftriaxone 250 mg intramuscularly as a single dose and
doxycycline 100 mg by mouth twice daily for 14 days
B. Gram stain smear and treatment with ceftriaxone 250 mg intramuscularly as a single dose
and azithromycin 1 g orally as a single dose
C. nucleic acid amplification test and treatment with azithromycin 1 g orally as a single dose
D. nucleic acid amplification test and treatment with ceftriaxone 250 mg intramuscularly as
a single dose
E. nucleic acid amplification test and treatment with ceftriaxone 250 mg intramuscularly as
a single dose and azithromycin 1g orally as a single dose
The azithromycin or doxycycline is included in the regimen because of the risk of co-infection
with Chlamydia trachomatis as well as the growing concern for cephalosporin-resistant Neisseria
gonorrhoeae. Azithromycin is preferred over doxycycline when possible because it is
administered as a single dose. Oral cephalosporins are no longer recommended as first-line
medications for treatment of N gonorrhoeae because of the concern for resistance.
Nucleic acid amplification tests have better overall sensitivity and specificity compared to other
diagnostic tests for gonococcal infections. Nucleic acid amplification tests are therefore
recommended for the detection of gonorrhea, except in cases of childhood sexual assault.
Currently, data are insufficient to recommend nucleic acid amplification tests for testing rectal
and oropharyngeal secretions in childhood sexual assault; instead, cultures should be used.
Cultures should also be considered when there is a concern for treatment failure to allow for
susceptibility testing.
PREP Pearls
• Gonorrhea is a common sexually transmitted disease. While patients can be asymptomatic,
symptoms of gonococcal cervicitis include cervicovaginal discharge, intermenstrual
bleeding, and lower abdominal pain.
• Nucleic acid amplification tests should be used for diagnosis in most situations.
• The current recommended treatment regimen is ceftriaxone 250 mg intramuscularly, and
either azithromycin 1 g orally as a single dose (preferred) or doxycycline 100 mg orally twice
a day for 7 days.
Suggested Readings
• Papp JR, Schachter J, Gaydos CA, Van Der Pol B. 4 Recommendations for the Laboratory-
Based Detection of Chlamydia trachomatis and Neisseria gonorrhoeae. MMWR Morbid
Mortal Wkly Rep.
2014;63(RR2).http://www.cdc.gov/mmwr/preview/mmwrhtml/rr6302a1.htm.
• Swygard H, Sena AC, Leone P, Cohen MS. Gonorrhea. In: Klausner JD, Hook EW III, eds.
Current Diagnosis & Treatment of Sexually Transmitted Diseases. New York, NY: McGraw-
Hill; 2007:99-107.
American academy of pediatrics 7
American Academy of Pediatrics PREP 2016
• US Centers for Disease Control and Prevention. Self-study STD modules for clinicians-
gonorrhea. US Centers for Disease Control and Prevention website.
http://www2a.cdc.gov/stdtraining/self-study/gonorrhea. Updated April 1, 2013.
• US Centers for Disease Control and Prevention. Sexually transmitted diseases treatment
Question 3
A 6-month-old infant presents for evaluation of a rash that developed 2 weeks ago. He has been
well and is taking no medications. The physical examination reveals numerous erythematous
papules located on the trunk and extremities, including the hands (Item Q3A), and feet (Item
Q3B).
A. cephalexin orally
B. an emollient topically
C. hydrocortisone topically
D. hydroxyzine orally
E. permethrin topically
Scabies is caused by infestation with the mite, Sarcoptes scabiei. Spread is primarily by direct
contact with an infested individual, although fomites may be responsible. Two to 3 weeks
following infestation, pruritus and rash develop. Lesions are erythematous papules, nodules
(Item C3A), and burrows (Item C3B) located in the interdigital spaces, wrist flexures, axillae,
and waist. In girls, the areolae may be involved; in boys, papules and nodules may affect the
penis (Item C3C) and scrotum. In infants, such as the one in the vignette, the eruption may be
generalized and vesiculopustules may be observed on the palms and soles.
Scabies is treated with permethrin cream 5% applied topically for 8 to 14 hours (ie, overnight).
The cream is applied to the entire skin surface from the neck to the toes. In infants and possibly
in young children and the elderly, the head (including the face) should be treated. Permethrin is
American academy of pediatrics 10
American Academy of Pediatrics PREP 2016
not completely ovicidal and, for this reason, a second treatment is recommended 7 to 14 days
later. The symptoms and signs of scabies represent a hypersensitivity reaction to the mite and its
products, therefore 2 to 4 weeks may be required for the pruritus and rash to subside. Household
or other close contacts may be infested, but not yet symptomatic, and as a result should receive a
single application of permethrin at the time the index case is first treated. Bed linens and clothing
should be laundered in hot water and dried at high temperature, although some advise a simpler
strategy of placing items in a dryer at 60°C for 10 min. Items that cannot be treated in this
manner may be stored in a sealed plastic bag for a minimum of 3 days.
PREP Pearls
• In children and adolescents, the lesions of scabies typically are located in flexural areas
(eg, between the digits, wrist flexors, etc). However, in infants, the eruption is
generalized and the palms and soles often are affected.
• In boys, papules and nodules often are present on the penis and scrotum.
• Topical permethrin is the treatment of choice for scabies in children.
• Asymptomatic household contacts should be treated at the time the index case is first
treated.
Suggested Readings
• Currie BJ, McCarthy JS. Permethrin and ivermectin for scabies. N Engl J Med.
2010;362(8):717-725. doi:http://dx.doi.org/10.1056/NEJMct0910329.
• Golant AK, Levitt JO. Scabies: a review of diagnosis and management based on mite
biology. Pediatr Rev. 2012;33(1):e1-e12. doi: http://dx.doi.org/10.1542/pir.33-1-e1.
Question 4
A 6-year-old boy is brought to the emergency department after being hit by a car while walking
across the street. He was thrown approximately 20 ft. His vital signs show a temperature of
37°C, pulse of 60 beats/min, blood pressure of 140/90 mm Hg, respiratory rate of 15
breaths/min, and SpO2 is 100% on 2 L of oxygen by nasal cannula. On neurologic examination,
the boy’s eyes are closed, but they open when his name is called. His right pupil is 2 mm and
briskly reactive. His left pupil is 4 mm and sluggishly reactive. He purposefully bats at the
examiner when a painful stimulus is applied. He moves all extremities equally. He does not say
words, but rather incomprehensibly moans. There are abrasions on his face and a boggy area
over his left temporal region. The boy is breathing comfortably on supplemental oxygen with
equal breath sounds. His extremities are warm and well-perfused. He has bruising over the left
upper quadrant of his abdomen, with no sign of chest or extremity trauma.
Of the following, the MOST appropriate next step in the boy’s management is to
The Monroe-Kellie doctrine states that because the volume of the calvarium is constant, ICP will
increase dramatically with an increase in the volume of its contents which include brain, blood,
and cerebrospinal fluid. Therefore, ICP will rise in the event of an intracranial hemorrhage or
cerebral edema without a compensatory decrease in another component. Hyperosmolar therapy,
such as intravenous hypertonic saline or mannitol, is a mainstay of treatment as it causes water to
shift from the intracellular and interstitial spaces of the brain into the serum, thus decreasing the
volume of the brain component. Mannitol, a sugar alcohol commonly used in the management of
increased ICP in both pediatric and adult traumatic brain injury, is a preferred therapy when
acute herniation is suspected or anticipated. In addition to its osmotic effect, which occurs within
15 to 30 minutes of administration, mannitol immediately decreases blood viscosity, causing
reflex vasoconstriction and thereby decreasing cerebral blood volume. The intravascular volume
status of a trauma patient receiving mannitol must be closely monitored. Because mannitol is
filtered in the glomerulus and not reabsorbed, the increased osmolality in the nephron leads to
osmotic diuresis. In contrast, hypertonic saline can be used both for intravascular fluid expansion
and to decrease ICP without causing osmotic diuresis, which is an added benefit in trauma cases.
It should be noted that the 2012 pediatric traumatic brain injury guidelines state that hypertonic
saline should be considered for the treatment of pediatric traumatic brain injury associated with
intracranial hypertension, but mannitol is not mentioned. However, both therapies continue to be
used by many major trauma centers.
The many causes of altered mental status in children, including coma, can be generally divided
into direct structural derangements of the central nervous system such as hydrocephalus, trauma,
and stroke, and medical causes such as hypoxia, infection, imbalances of metabolic supply and
demand, seizure, and toxins (Item C4A). Although presentation is dependent on the age of the
child, there is a continuum from normal mental status to coma, which includes confusion,
delirium, lethargy, and stupor. An early sign of altered mental status is confusion and
disorientation, in which the child cannot follow a conversation or lacks orientation to person,
place, or time. Delirium is a state of mental or motor excitement that can include fear, irritability,
and agitation. Lethargy is a sleepy state in which the child can be aroused with moderate
stimulation with immediate relapse into sleep. Stupor is a more unresponsive state in which the
child can only be aroused with vigorous or painful stimuli. It is important for the clinician to
recognize a child experiencing progression of signs and symptoms toward impending coma,
because this can be a harbinger of worsening illness or impending death (from herniation or loss
of airway and breathing).
As ICP rises and compensatory mechanisms are exhausted, herniation syndromes can occur,
leading to pupillary, extraocular, and motor derangements (Item C4B). Central herniation ensues
if brain structures are forced caudally into the foramen magnum. In the case of an expanding
temporal fossa lesion, such as an epidural hematoma, the medial temporal lobe (uncus) can
herniate through the tentorium.
Neither magnetic resonance imaging (MRI) nor diagnostic peritoneal lavage is the best next step
in this case. Although MRI of the brain will detect bleeding, hydrocephalus, cerebral vasculature
abnormalities, and tumors, it generally requires more than 1 hour spent in an inadequately
monitored environment. Although it has been replaced by focused abdominal ultrasonography of
trauma (FAST) in many trauma protocols, diagnostic peritoneal lavage can be used to detect
abdominal trauma, but as noted, is not the first priority in this hemodynamically stable child with
a mental status change. Similarly, a blood transfusion would not be indicated in this case without
evidence of acute blood loss or hemodynamic instability.
PREP Pearls
• Hyperosmolar therapy, such as 3% saline or mannitol administered intravenously, can be
used to treat an acute elevation in intracranial pressure.
• Altered mental status can be due to structural and medical causes.
• The clinician should be alert to the progression of signs and symptoms along the
continuum from normal mental status to coma, especially if it is rapid.
Suggested Readings
• Avner J. Altered states of consciousness. Pediatr Rev. 2006;27:331-338. doi:
http://dx.doi.org/10.1542/pir.27-9-331.
• Kochanek PM, Carney N, Adelson PD, et al. Guidelines for the acute medical
management of severe traumatic brain injury in infants, children, and adolescents: 2nd
edition. Pediatr Crit Care Med. 2012;13(suppl 1):S36-S40. doi:
http://dx.doi.org/10.1097/PCC.0b013e31823f435c.
• Rubenstein JS. Consultation with the specialist: Initial management of coma and altered
consciousness in the pediatric patient. Pediatr Rev. 1994;15:204-207. doi:
http://dx.doi.org/10.1542/pir.15-5-204.
Question 5
An 8-month-old infant with asplenia presents to the emergency department for evaluation of
irritability, fever, and rash. A physical examination reveals multiple purpuric lesions. Lumbar
puncture reveals a cerebrospinal fluid white blood cell count of 1,800/µL. He is treated with
vancomycin and cefotaxime, and admitted to the pediatric intensive care unit. His mother asks if
anything could have been done to prevent this illness.
Of the following, the BEST response to the mother’s question is that meningococcal conjugate
vaccine is
In the United States, there are 2 meningococcal quadrivalent polysaccharide protein conjugate
vaccines licensed for young children that offer protection against serogroups A, C, W, and Y. An
additional bivalent conjugate vaccine combined with Haemophilus influenzae vaccine protects
against serogroups C and Y. Since 2005, vaccination has been routinely recommended in
adolescents. In 2010, the recommendation for a booster dose at 16 years of age was made.
Although 60% of meningococcal disease occurs in children younger than 5 years of age, the two
licensed novel meningococcal serogroup B-specific vaccines available in the US are approved
for use in persons 10 to 25 years of age. Serogroup B-specific vaccines have been used
successfully in outbreak settings.
In the United States, one of the commercially available quadrivalent vaccines is licensed for
persons as young as 2 months of age and the bivalent vaccine that is combined with
Haemophilus influenzae is licensed for infants starting at 6 weeks of age.
PREP Pearls
• High risk groups should be vaccinated with meningococcal vaccine starting at 2 months
of age.
• Commercially available serogroup B-specific vaccines are licensed for persons 10 to 25
years of age
• Serogroup B is responsible for most infections in young children.
• For travel or residence in a hyperendemic region, meningococcal vaccination can begin at
2 months of age.
Suggested Readings
• American Academy of Pediatrics. Updated recommendations on the use of
meningococcal vaccines. Pediatrics. 2014;134(2):400-403. doi:
http://dx.doi.org/10.1542/peds.2014-1383.
American academy of pediatrics 17
American Academy of Pediatrics PREP 2016
• Centers for Disease Control and Prevention. Vaccines and Immunization: Meningococcal
Vaccination. Available at:http://www.cdc.gov/vaccines/vpd-vac/mening/default.htm.
Accessed April 12, 2015.
• Cohn AC, MacNeil JR, Clark TA, et al. Prevention and control of meningococcal
disease: recommendations of the advisory committee on immunization practices. MMWR
Morbid Mortal Wkly Rep. 2013;62(RR02):1-22
Question 6
A 16-year-old adolescent presents to the emergency room with chief complaints of flank pain
radiating to the groin and blood in the urine for the last 2 hours. There is no history of fever,
burning on urination, or trauma. The mother mentions that the patient had a similar episode 1
year ago associated with passage of tiny particles in the urine. Vital signs show a temperature of
38.3°C, heart rate of 100 beats/min, respiratory rate of 28 breaths/min, and blood pressure of
144/80 mm Hg. On physical examination, the patient is bent over in pain and has mild
dehydration. The patient refuses to let you examine her abdomen.
Of the following, the MOST sensitive imaging modality for this patient is
A. computed tomography
B. intravenous urography
C. magnetic resonance imaging
D. radiograph (kidney, ureter, bladder)
E. ultrasonography
Noncontrast (unenhanced) helical computed tomography (CT) is the most sensitive imaging
modality for evaluating patients with suspected urinary tract stones (adults and children).
Computed tomography can detect stones in the ureters that may not be detected by
ultrasonography, radiolucent stones (such as pure uric acid stones) that are not detected by plain
radiography, and very small stones (around 1 mm in diameter) that are often missed on plain
radiography or ultrasonography. Studies have also reported on the possible evaluation of the
chemical composition of the renal calculi from the CT images. Computed tomography also
provides better estimates of the size, number, and direction of the branches of a staghorn
calculus, which may further help in treatment decisions.
Noncontrast CT avoids the risk of intravenous contrast material and can be completed in less
than 5 minutes (with the newer multidetector CT machines). Therefore, the imaging can be done
without anesthesia in most patients. A urine pregnancy test should be done in all female patients
(of child-bearing age) prior to all radiation exposure.
A plain abdominal radiograph will detect radiopaque stones (such as calcium, struvite, and
cystine kidney stones) and miss radiolucent uric acid stones. Plain radiographs will also miss
small stones, stones overlying bony structures, and will provide no information on urinary
system obstruction. The current use of plain radiographs is limited to settings where renal
ultrasonography and CT are not available for imaging children with suspected renal stones.
Intravenous urography has been used previously in patients with suspected renal stones
following an initial plain film. Intravenous urography is a radiologic test for identifying the
details of the urinary system, including kidneys and ureters, and identifying upper urinary tract
obstruction. Delayed and prolonged excretion of the contrast medium and dilatation of the
collecting system are indicative of obstructive calculi of the kidneys. The intravenous urography
leads to less radiation exposure in comparison to a CT; however, it is also less sensitive in
detecting kidney stones. The current availability of ultrasonography, CT, and magnetic resonance
urography has replaced IVU in almost all diagnostic settings.
Magnetic resonance imaging, similar to plain radiography and ultrasonography, is not sensitive
for detecting small calculi. Calculi can be detected by magnetic resonance imaging if their foci of
signal void are large or they lie adjacent to tissues of high signal intensity. Signal defects on MRI
are nonspecific and could be due to calculi, clots, debris, or tumors.
PREP Pearls
• Noncontrast (unenhanced) helical computed tomography (CT) is the most sensitive
imaging test for evaluating patients with suspected urinary tract stones.
• Ultrasonography, though not as sensitive as helical CT, is a reasonable alternative to CT
to detect nephrolithiasis, especially when there are concerns about radiation exposure.
• Radiation exposure during CT can be reduced significantly while adjusting scanning
parameters based on the size and weight of the patient with little compromise of the
imaging quality.
Suggested Readings
• Mostafavi MR, Ernst RD, Saltzman B. Accurate determination of chemical composition
of urinary calculi by spiral computerized tomography. J Urol. 1998;159(3):673-675. doi:
http://dx.doi.org/10.1016/S0022-5347(01)63698-X.
• National Cancer Institute. Radiation risks and pediatric computed tomography (CT): a
guide for health care providers. National Institutes of Health website.
http://www.cancer.gov/cancertopics/causes/radiation/radiation-risks-pediatric-CT.
Updated June 7, 2012..
Question 7
You are caring for a 12-month-old boy with Down syndrome and short bowel syndrome
secondary to duodenal atresia. Following resection, he has 25 cm of small bowel remaining,
including 15 cm of duodenum and 10 cm of ileum, which includes the ileocecal valve. He
receives enteral feeds by continuous infusion, but requires parenteral nutrition for most of his
calories to maintain growth. His height and weight are currently at the 25th percentile for age.
Recent weekly laboratory studies to monitor his parenteral nutrition show stable electrolytes and
normal liver function tests. The results of his complete blood cell counts over time are shown in
Item Q7.
A. iron
B. vitamin B12
C. vitamin D
D. vitamin E
E. vitamin K
Children on long-term TPN require close observation and frequent laboratory monitoring. Item
C7 provides a summary of parameters to be measured during TPN use. Fluid balance is critical
when initiating and continuing TPN. Children on TPN are at risk for hypoglycemia and
hyperglycemia, particularly when TPN is initiated. Electrolyte supplementation must be tailored
to the needs of each child, with special attention to the age and underlying disorders of the child.
Calcium can be low in premature infants and in patients with renal losses. Phosphate can be
elevated in premature infants and is vital to the prevention of metabolic bone disease. Liver
function should be monitored to assess nutrition and to evaluate for evidence of parenteral
nutrition-associated liver disease.
Lipids provide a good energy source in a small volume and are critical to avoiding essential fatty
acid deficiency. Lipid utilization varies and underutilization can result in elevated triglyceride
levels. The goal is to maintain triglycerides at less than 150 mg/dL (1.7 mmol/L).
Fat soluble vitamins are absorbed throughout the duodenum, jejunum, and proximal ileum. Fat
soluble vitamins (vitamins A, D, E, and K) are less likely to be deficient in this child, as they are
supplemented in the TPN and deficiency is rare. Iron is absorbed in the proximal small
intestines. This child is at risk for chronic iron deficiency, but this is less likely than vitamin B12
deficiency.
PREP Pearls
• Close monitoring of glucose, fluid balance, and electrolytes is needed in the first days of
total parenteral nutrition (TPN) initiation.
• Children on chronic TPN are at risk for parenteral nutrition-associated liver disease.
Suggested Readings
• Kleinman RE. Pediatric Nutrition Handbook. 5th ed. Elk Grove Villlage, IL: American
Academy of Pediatrics; 2004:369-389.
Question 8
You are called to the neonatal intensive care unit to evaluate a 24-hour-old newborn with
progressive obtundation, seizures, and tachypnea. Pregnancy, labor, and delivery were
uneventful. The newborn initially did well during the first several hours after birth, but then
became lethargic, hypothermic, and developed poor feeding. Results of a comprehensive
metabolic panel including glucose, complete blood cell count with differential, and C-reactive
protein are all normal. The anion gap is normal. Blood gas results reveal a respiratory alkalosis.
Intravenous antibiotics are started and cultures are pending.
Of the following, the BEST next laboratory test for diagnosis and management is
A. serum ammonia
B. serum galactose-1-phosphate
C. serum toxicology screen
D. serum very long chain fatty acids
E. urine organic acids
Urea cycle disorders may present as a metabolic emergency, necessitating immediate recognition
and treatment to avoid irreversible brain damage. Severe hyperammonemia is treated with
dialysis and hemofiltration to rapidly reduce the plasma ammonia concentration, along with the
intravenous administration of arginine hydrochloride and nitrogen scavenger drugs to promote
the excretion of excess nitrogen through alternative pathways. Restriction of protein for 12 to 24
hours is essential, with calories provided through carbohydrates and fat. Care must be taken to
stabilize the patient with intravenous fluids and inotropic drugs if necessary.
Long term management mandates the use of specialized formulas, oral nitrogen-scavenging
drugs, dietary restriction of protein, and avoidance of hyperammonemic episodes. Patients are at
high risk of decompensation, necessitating hospitalization for close observation of clinical status
and ammonia levels if they have gastrointestinal or respiratory illnesses. Most patients are
routinely treated on a long term basis by biochemical or metabolic geneticists, in addition to their
primary care provider. They should have emergency protocols in place at home, at the primary
care provider’s office, and at the local hospital.
Classic patterns of unusual excretion of non-amino organic acids in urine are a first line test for
this group of disorders. Newborns appear well at delivery and for the first few days, followed by
metabolic decompensation. Symptomatology includes vomiting, poor feeding, neurologic
symptoms, and lethargy progressing to coma. Laboratory findings include metabolic acidosis
(not respiratory alkalosis as in the child in this vignette), ketosis, hyperammonemia, elevated
liver function tests, low blood sugar, and neutropenia. These disorders require immediate
recognition to optimize long term outcome via clinical interventions.
Tyrosinemia type 1 presents in infancy with significant liver involvement, and eventually renal
tubular dysfunction, growth failure, and rickets. Untreated children may present with repeated
neurologic crises involving a change in mental status, peripheral neuropathy, abdominal pain,
and occasionally respiratory failure. If untreated, many die before 10 years of age. Laboratory
abnormalities include increased succinylacetone concentration in the blood and urine; elevated
tyrosine, methionine, and phenylalanine on serum amino acids; and elevated tyrosine metabolites
on urine organic acids.
Galactosemia presents in the neonatal period with jaundice, hypotonia, scleral icterus, bruising,
bleeding, and cataracts in the face of rapidly progressive liver failure. Classic laboratory
abnormalities of galactosemia include positive urine-reducing substances, abnormal liver
function studies, coagulation abnormalities suggestive of a progressive bleeding diathesis,
elevated erythrocyte galactose-1-phosphate, and sepsis, especially due to Escherichia coli.
Peroxisomal biogenesis disorders are typically screened with serum very long chain fatty acids.
They can present in the neonatal period with hypotonia, poor feeding, dysmorphic facies,
seizures, and liver cysts with hepatic dysfunction. Bony stippling may occur in the patella or
long bones detectable by skeletal survey. Infants do not present with metabolic crises, but with
slowly progressive neurologic deterioration, dying in the first year of life.
PREP Pearls
• Urea cycle disorders clinically manifest with immediate decompensation in the first 24 to
72 hours of life with progressive respiratory alkalosis, obtundation, and
hyperammonemia in the presence of a normal anion gap.
• Hyperammonemia is very toxic to the brain and can cause irreversible damage without
immediate intervention. It is very important to check the serum ammonia level and
quickly implementation of measures to decrease the ammonia level as soon as possible
(dialysis, hemofiltration, nitrogen scavenger medications, and protein restriction).
Suggested Readings
• Mew NA, Lanpher BC, Gropman A, et al. Urea cycle disorders overview.
GeneReviews.http://www.ncbi.nlm.nih.gov/books/NBK1217/.
Question 9
The parents of a healthy 3-year-old boy call the office to report that they found him playing in
the “junk” drawer and saw him put his fingers in his mouth. They are concerned that he may
have swallowed something. He is in no respiratory distress and is not complaining of any pain.
He drank some milk, but has not had anything to eat since then. Various objects are present in
the drawer, including buttons, coins, batteries, safety pins, and small toys that contain magnets.
A radiograph is obtained and it is determined that the patient needs immediate intervention.
A. button
B. button battery
C. closed safety pin
D. magnet
E. quarter
common feature seen with larger button batteries and helps to distinguish them from other round
objects (ITEM C9CD). Among the response choices, the object that requires the most immediate
attention is the button battery.
Foreign body ingestion is primarily a pediatric problem. Management of foreign body ingestions
depends on the item ingested, anatomic location of the foreign body, and presence or absence of
symptoms. Coins are the most commonly reported foreign bodies ingested and most will traverse
the gastrointestinal (GI) tract without incident. Button batteries, sharp objects or toys, and
magnets require action because of the increased risk for serious complications.
Button batteries contain toxic heavy metals and alkaline compounds that are caustic to the
mucosa. Lithium cells are more likely to cause significant outcomes than the other chemistry
types (manganese dioxide, zinc-air, or silver oxide). Complications of button battery ingestions
have become increasingly frequent and devastating in parallel with increased household use of
20-mm lithium coin cells. Button batteries lodged in the esophagus, or at any point along the GI
tract, may lead to significant mucosal injury, even perforation. Button batteries that have passed
into the stomach may be monitored with serial abdominal radiographs every 12 hours. If there is
no progress through the GI tract over 24 hours, then surgical removal is necessary.
Unfortunately, the degree of injury may worsen long after battery removal.
Battery diameter is the most important predictor of a clinically significant outcome. All serious
outcomes or fatal cases reported have occurred with batteries of 20 mm or larger. Severe burns
can occur after only 2 to 2.5 hours. No clinically significant outcomes were observed with 15- to
18-mm battery cells. Young age (<4 years) and ingestion of more than 1 battery were also
associated with worse outcome. New (fully charged) 20- to 25-mm batteries are 3 times more
likely than spent cells to be associated with clinically significant outcomes. Unwitnessed
ingestions are common and are at risk for misdiagnosis and delay in removal, leading to worse
outcomes.
American academy of pediatrics 31
American Academy of Pediatrics PREP 2016
A plain anteroposterior (AP) radiograph that includes the neck, chest, and abdomen should be
obtained immediately on all patients who present with a history of possible button battery
ingestion, even if they are asymptomatic. The window of opportunity to remove an esophageal
lithium cell battery before injury occurs is less than 2 hours, so timing is critical. Endoscopic
removal is preferred because it allows direct visualization of tissue injury. Later complications
must be anticipated, depending on the battery position and orientation, as well as time before
removal. Patients should be monitored closely for the development of fistulas or perforations 1 to
3 weeks later and strictures weeks to months later.
Plain radiography of the neck, chest, and abdomen area should be performed in patients who
present with a recent history of possible foreign body ingestion to confirm the item ingested and
determine location. Symptoms that suggest esophageal impaction include feeding refusal,
drooling, difficulty or painful swallowing, or emesis. In these cases, urgent endoscopic
evaluation and removal of any impacted esophageal foreign body is mandatory. If the foreign
body has passed the gastroesophageal junction, and the object does not pose a danger (magnets,
sharps, button battery), then the patient's condition can be managed conservatively. Although
most objects will pass through the intestinal tract without incident, entrapment can occur at the
pylorus, the ligament of Treitz, or the ileocecal valve. Parents should be instructed to examine
the child’s stools closely for the object and if it is not recovered from the stools within 1 to 2
weeks after presentation, follow-up radiography should be performed. If the object is retained,
then surgical intervention is indicated, and is the appropriate management for all of the other
response choices given. If 2 or more magnets are ingested, there is a much greater risk for
entrapment of mucosa between magnets leading to perforation.
It is not recommended to induce vomiting or give cathartics for ingested foreign bodies, because
their effectiveness has not been proven. Alternative techniques for removal of esophageal foreign
bodies, such as use of a Foley catheter or advancement with bougienage, have been successful in
experienced hands, but should not be used in the case of button battery ingestion.
PREP Pearls
• Larger button batteries may reveal a double rim on plain radiography, which
distinguishes them from other round objects.
• Ingestions of lithium cell batteries with a diameter of 20 mm or greater have the worst
outcomes, with severe burns occurring after just 2 to 2.5 hours.
• Indications for immediate removal of an ingested foreign body include
• airway compromise
American academy of pediatrics 32
American Academy of Pediatrics PREP 2016
Suggested Readings
• Litovitz T, Whitaker N, Clark L, White NC, Marsolek M. Emerging battery-ingestion hazard:
clinical implications. Pediatrics. 2010125(6):1168-1177. doi:
http://dx.doi.org/10.1542/peds.2009-3037.
Question 10
You are seeing an 18-month-old boy who has had 2 skin abscesses in the perianal region and 1
abscess in the scalp, recurrent oral ulcers, 5 episodes of otitis media, and 2 episodes of
pneumonia in the previous year. Laboratory evaluation results are shown:
Test Results
% neutrophils 72
% lymphocytes 23
% monocytes 3
% eosinophils 1
% basophils 1
Of the following, the MOST appropriate next steps in the management of this child include
A. testing for the phagocyte oxidative burst by flow cytometry and starting prophylaxis
with allopurinol
B. testing for the phagocyte oxidative burst by flow cytometry and starting prophylaxis
with trimethoprim and sulfamethoxazole
C. testing for the presence of lymphoblasts by flow cytometry and starting prophylaxis
with allopurinol
D. testing for the presence of lymphoblasts by flow cytometry and starting prophylaxis
with trimethoprim and sulfamethoxazole
E. testing for the presence of lymphoblasts by flow cytometry and starting treatment
with prednisolone
Chronic granulomatous disease is the most common heritable defect of neutrophil function,
although other rarer disorders of neutrophil function exist. One example is the leukocyte
adhesion deficiency. Leukocyte adhesion deficiency is a group of genetic disorders that result in
the failure to express proteins necessary for the normal trafficking of leukocytes to areas of
infection. Without the ability to leave the blood stream and enter an area of infection, the
leukocytes are unable to contain infections. The resulting phenotype can be very similar to CGD.
Frequent bacterial infections in a young child, especially of skin, lung, and sinus, should raise
concern for an underlying immunodeficiency. The immune system can be divided into the innate
immune system and the adaptive immune system. The innate immune system is comprised of
barriers such as skin and mucosal membranes, and phagocytes such as the neutrophil,
macrophage, and natural killer cells. The adaptive immune system is comprised of B and T
lymphocytes and their subsets. Generally, abnormal viral infections suggest a deficiency of the
adaptive immune system, while frequent bacterial or fungal infections suggest a deficiency of the
innate immune system.
While children with leukemia are at risk for frequent bacterial infections, the complete blood cell
count reported no lymphoblasts, and the normal hemoglobin and platelet numbers do not suggest
bone marrow dysfunction. Thus, testing for lymphoblasts would not be warranted. Allopurinol is
used to reduce the risk of end organ damage from the accumulation of uric acid in the setting of
tumor lysis. As the presentation of the child in this vignette does not fit with a diagnosis of
leukemia, starting allopurinol would not be appropriate.
PREP Pearls
• A defect of the innate immune system should be suspected in a child presenting with frequent
bacterial infections.
• Chronic granulomatous disease is the most common disorder of neutrophil function.
• Chronic granulomatous disease is diagnosed through testing of the neutrophil oxidative burst
by flow cytometry and the initial management should include prophylaxis with trimethoprim
and sulfamethoxazole.
Suggested Readings
• Esfandbod M, Kabootari M. Images in clinical medicine: chronic granulomatous disease. N
Engl J Med. 2012;367(8):753. doi: http://dx.doi.org/10.1056/NEJMicm1106350.
Question 11
You see a 3-year-old boy for a health supervision visit. He has been healthy other than an ear
infection at 2 years of age. The mother reports no concerns about his health. His weight and
height are consistently at the 25th percentile and his physical examination is unremarkable. His
tympanic membranes appear normal and move appropriately with insufflation. You wish to
assess his language development, but he is cautious and shy in your office and speaks only a few
words quietly to his mother. Believing his mother to be an accurate and observant historian, you
question her about the boy's speech.
Of the following, the finding that BEST supports the need for a speech evaluation in this child is
Of the milestones listed, not being able to use a 3-word phrase would be of most concern for a 3-
year-old child and would be a good reason to refer him for further assessment. There is a
tremendous range of normal variation in speech, particularly between 1 to 2 years of age.
Therefore, keeping in mind signs of when a referral would be needed can help this seem less
confusing. The following red flag signs are far enough outside the norm that they would
demonstrate a clear need for further assessment:
o 6 months of age: lack of turning to sound or voice
o 9 months of age: lack of babbling consonant sounds
o 18 months of age: does not say "mama," "dada," or other names
o 24 months of age: failure to use single words
o 30 months of age: failure to use 2-word phrases
o 36 months of age: failure to use 3-word phrases
The ability to follow a 3-step direction and the ability to point to 5 to 6 specific colors are
expected normal range milestones for a 4-year-old child. The ability to use words to talk about
time would also be a normal range milestone at 4 years of age. Intelligibility of speech to
strangers typically follows this pattern: about 50% understandable at 2 years of age, about 75%
understandable at 3 years of age, and 100% understandable at 4 years of age. It is not uncommon
for a parent to feel they understand nearly everything their child says because of a high level of
familiarity, while a stranger understands much less.
PREP Pearls
• A speech evaluation would be needed for a 2-year-old child who cannot use single words to
communicate, or for a 3-year-old child who cannot use 3-word phrases.
• A 6-month-old infant who does not turn to sound or voice, and a 9-month-old infant who
does not make babbling consonant sounds should have a careful hearing evaluation.
• The parents' ability to interpret meaning in their children’s speech (that strangers cannot
understand) can be a reason for delayed recognition of a speech problem.
Suggested Readings
• McLaughlin MR. Speech and language delay in children. Am Fam Physician.
2011;83(10):1183-1188.
Question 12
A 2-month-old infant is seen in the emergency department for fever and respiratory distress. The
baby had rhinorrhea 1 week ago, has been tachypneic for 3 days, and developed lethargy today.
There is no history of a heart murmur. The baby is admitted for probable pneumonia, but after 2
days of antibiotics, is not improving as expected. She continues to fatigue easily with feeding,
which her mother states has been the case for the past week. Her birth weight was 2.7 kg and her
weight today is 5 kg (38th percentile). Her vital signs show a heart rate of 180 beats/min,
respiratory rate of 80 breaths/min, and blood pressure of 65/45 mm Hg taken in the right arm.
Her oxygen saturation is 92%. On physical examination, she has intercostal and subcostal
retractions, but is not coughing and has no stridor. Her lungs are clear with breath sounds heard
well to the bases. Her cardiac examination is significant for a soft S1 and S2 with no audible
murmur. Her abdominal examination shows a liver edge that is 4 cm below the right costal
margin. Her femoral pulses are palpable, but not strong. She has a capillary refill time of 3
seconds.
Of the following, the MOST likely etiology for the infant’s clinical symptoms is
A. aspiration
B. group B streptococcal sepsis
C. intussusception
D. viral bronchiolitis
E. viral myocarditis
Congestive heart failure can present at any age, with some presentations that are age-specific. In
the immediate newborn period, one may see a patient with critical (ductal dependent) cyanotic
congenital heart disease, such as hypoplastic left heart syndrome. These newborns may develop
signs of either pulmonary overcirculation with early CHF or poor systemic perfusion and shock.
This occurs because the cardiac output shift s, sometimes very quickly, when the relative
pulmonary and systemic vascular resistance changes. If the lungs become congested, the baby
will become tachypneic. In this setting, the systemic output will decrease in proportion to the
increase in pulmonary flow.
In the later newborn period, patients may become symptomatic with a large patent ductus
arteriosus. This lesion causes excess pulmonary blood flow directly from the aorta to the
pulmonary artery. This increases blood return to the pulmonary veins, the left atrium, and the left
ventricle, and may result in volume overload of the left ventricle. Tachycardia and tachypnea
may be seen.
At approximately 4 weeks of age, the pulmonary vascular resistance (PVR) will have dropped to
that seen in later childhood. As the PVR drops, so does the right ventricular pressure. As this
occurs, there will be increasing flow across any ventricular septal defect (VSD). The degree of
left to right shunt in this case will depend on the size of the defect and the drop in PVR.
Intracardiac shunts are described as the ratio of the pulmonary to systemic blood flow. In
someone with a normal heart with no shunt, the ratio is 1:1. An infant with a large left to right
shunt (such as 3:1 pulmonary flow to systemic flow) will have pulmonary vascular congestion
and tachypnea. There will be a volume load to the right and subsequently to the left ventricle.
Infants with this physiology are hyperdynamic and consequently have high caloric needs. They
will often have failure to thrive.
A child with an atrioventricular canal or endocardial cushion defect will usually present with
signs of CHF during the first few months of life. Those with a predominantly atrial level shunt
may present later than an infant with both a large atrial septal defect (ASD) and VSD. The timing
of presentation depends on when the PVR drops. The mechanism of CHF is similar to that
described for a VSD.
Many patients with an isolated secundum ASD do not exhibit symptoms of CHF until the second
or third decade of life (often during pregnancy when there is increased cardiac output).
Later in childhood, CHF may occur for many reasons, but it is a time when rheumatic heart
disease may present. The mitral valve is the most commonly affected, with the finding of valve
regurgitation. The mitral valve may become thickened and then will not close properly. This
creates an increase in the volume load of the left atrium and the left ventricle (LV). As the
regurgitation worsens, this large volume overload can dilate the LV. There will be both systolic
(forward squeeze) and diastolic (ability for the LV to relax and fill) dysfunction.
An older child or adolescent with new onset CHF should be suspected of having
cardiomyopathy. Often, the presentation is subtle and the decrease in function is tolerated until it
becomes precipitously worse. This form of CHF is a LV failure rather than a volume overload
issue. The symptoms of the infant in this critique are not of intermittent shock and there is no
abdominal distension, making intussusception less likely. If the baby had worsening pneumonia
as a result of aspiration, one would expect more severe hypoxemia and respiratory distress. For
all the other choices, including group B streptococcal sepsis, one would not expect symptoms of
heart failure such as the hepatomegaly and quiet tachypnea.
PREP Pearls
• Pediatric heart failure is often due to volume overload in infancy and early childhood.
• Heart failure can be easily mistaken for respiratory illness in infants.
Suggested Readings
• Andréoletti L, Lévêque R, Boulangnon C, Brasselet C, Fornes P. Viral causes of human
myocarditis. Arch Cardiovasc Dis. 2009;102(6-7):559-568. DOI:
http://dx.doi.org/10.1016/j.acvd.2009.04.010.
Question 13
A 15-year-old adolescent girl presents to the emergency department with chest pain and
shortness of breath. She describes the pain as a pressure in the center of her chest that is
worsening. Initially, she had no difficulty breathing, but states that she is now unable to take a
deep breath because of the pain. Her review of systems is positive for fevers up to 40°C for 1
week and generalized joint pain, with some swelling in the knees and wrists bilaterally.
The patient’s pulse rate is 88 beats/min, respiratory rate is 17 breaths/min, and her blood pressure
is 140/91 mm Hg. On physical examination, she appears pale. There is a confluent erythematous
rash with raised borders and central clearing over her trunk. She has cervical lymphadenopathy,
arthritis in the wrists and knees bilaterally, and a cardiac friction rub. Laboratory studies are
significant for an erythrocyte sedimentation rate of 40 mm/hour and a urinalysis with 2+ protein
and 3+ blood. The remainder of her laboratory studies are unremarkable. You obtain a chest
radiograph (Item Q13).
The range of signs and symptoms that can be seen in SLE is broad (Item C13A). The initial
presentation of SLE can mimic several infectious diseases and cancers. Specific antibody studies
are important to definitively diagnose the disease. To meet American College of Rheumatology
criteria for the diagnosis of SLE, 4 of 11 criteria must be present (Item C13B), though pediatric
patients with SLE often will not fully fit the adult defined criteria. It can be especially
challenging to differentiate SLE from acute rheumatic fever (ARF). Although SLE and ARF
share features such as fever, elevated acute phase reactants, rash, and arthritis or joint pain, they
can be distinguished using the Jones criteria (Item C13C), antibody testing, organ function
testing, and review of the patient’s clinical presentation.
The adolescent girl in the vignette has evidence of a pericardial effusion that can be life-
threatening. It is important for practitioners to be aware that serositis can affect the heart and
lungs of patients with SLE.
Many morbidities are associated with pediatric lupus, because of either the disease state or
immunosuppressive therapy. These can include:
• Lupus nephritis, which can cause hypertension, and with poor disease control can lead to
renal failure, need for dialysis, and even the need for renal transplant
• Premature atherosclerosis leading to myocardial infarction
• Increased risk of recurrent infection due to dysregulation of the immune system, from disease
as well as immunosuppressive therapy
• Increased risk of malignancy
• Osteoporosis, compression fractures, and avascular necrosis resulting from steroid treatment
• Infertility, offspring with neonatal lupus, and increased fetal wastage
• Increased risk for diabetes, obesity, and growth failure
American academy of pediatrics 47
American Academy of Pediatrics PREP 2016
PREP Pearls
• An elevated anti–double-stranded DNA antibody titer is specific for the diagnosis of
systemic lupus erythematosus (SLE).
• Antinuclear antibody (ANA) is positive in the majority of patients with SLE, but is not
specific for the disease.
• The presence of Smith antibody is specific for SLE.
Suggested Readings
• Levy D, Kamphuis S. Systemic lupus erythematosus in children and adolescents. Pediatr Clin
North Am. 2012;59:345-364.
Question 14
At a health supervision visit, the parents of a 13-month-old boy express concern that their son’s
legs are very bowed. After reviewing the child’s history and performing a physical examination,
you diagnose physiologic genu varum.
Of the following, the feature MOST likely to be associated with this diagnosis is
Vitamin D deficiency (rickets) can lead to a softening of the bones with resulting genu varum
and, in most cases, short stature. Many of these children have radiographic findings in addition to
genu varum, including widening of the growth plates and metaphyseal flaring.
Infantile tibia vara, sometimes referred to as infantile Blount disease, results from depression of
the medial tibial physis. Risk factors include walking at an early age and obesity. Children with
infantile tibia vara with progressive bowing may require surgical correction. Bracing for this
condition is controversial, because no controlled trials have demonstrated efficacy.
Other causes of nonphysiologic genu varum include skeletal dysplasias, such as achondroplasia,
and trauma-induced growth plate injuries, which lead to asymmetric bone growth resulting in
unilateral genu valgum or genu varum, depending on the location of injury. Breech presentation
is a risk factor for hip dysplasia but not for genu varum.
PREP Pearls
• Physiologic genu varum (bowing of the legs) generally resolves by age 2 years.
• Asymmetric bowing, and bowing that persists at age 3 years, should prompt evaluation for
idiopathic tibia vara.
Suggested Readings
• Sarwark JF, LaBella C. Pediatric Orthopaedics and Sports Injuries: A Quick Reference
Guide. 2nd ed. Elk Grove Village, IL: American Academy of Pediatrics; 2014:745 pp.
Question 15
A previously healthy 7-year-old boy presents to your office with a 4-day history of a “blister” on
his heel that his mother believes started after he got new shoes. The lesion began as a “small
bump,” but now has grown in size, became red, and is draining yellow fluid. The boy reports that
the lesion is increasingly painful. On physical examination, his temperature is 38°C, heart rate is
70 beats/min, respiratory rate is 20 breaths/min, and blood pressure is 100/60 mm Hg. You note
a 1-cm circular lesion with surrounding erythema and swelling on the posterior aspect of the
right heel that is draining purulent fluid. The remainder of the physical examination is
unremarkable.
Of the following, in addition to local wound care, the BEST treatment to prescribe for this
patient is oral
A. cefixime
B. clindamycin
C. doxycycline
D. penicillin
E. trimethoprim/sulfamethoxazole
Culture of the fluid from the boy’s lesion grew methicillin-resistant, clindamycin-susceptible S
aureus. S aureus colonizes the skin and mucous membranes of approximately 30% to 50% of
children and adults. More than 95% of staphylococcal disease manifests as localized infections
such as cellulitis, impetigo, furuncles, abscesses, and lymphadenitis. Invasive infections such as
bacteremia, pneumonia, pyomyositis, and osteomyelitis are less common. However, in the health
care setting, S aureus is the second most common cause of health care–associated bacteremia and
often causes pneumonia and surgical site infections. In addition, S aureus also can cause device-
related (eg, intravascular catheter, pacemaker, ventriculoperitoneal shunt) infections as well as
toxin-mediated (eg, toxic shock syndrome, scalded skin syndrome, food poisoning) illness.
The initial management of skin and soft tissue infections caused by community-associated S
aureus is dictated by the disease severity and the prevalence and susceptibility pattern of MRSA
isolates in the community (Item C15). For the boy in the vignette who was previously healthy
with low-grade fever and soft-tissue infection, clindamycin is an appropriate initial choice of
therapy because it has activity against streptococci as well as susceptible strains of MRSA and
MSSA.
Cephalexin (first-generation cephalosporin) can be used for the treatment of MSSA, but cefixime
(third-generation cephalosporin) lacks appropriate antistaphylococcal activity. Doxycycline has
activity against susceptible strains of MSSA and MRSA but is not useful for treating GAS and
generally is not recommended for children younger than 8 years. Although penicillin would be
an appropriate choice of therapy for patients with cellulitis caused by GAS, it would be unlikely
to be effective against S aureus infection which usually is penicillin-resistant. Trimethoprim-
sulfamethoxazole also is effective against susceptible strains of S aureus but, like doxycycline, is
not useful for treating GAS.
American academy of pediatrics 53
American Academy of Pediatrics PREP 2016
PREP Pearls
• More than 95% of staphylococcal disease manifests as localized infection (eg, cellulitis,
impetigo, furuncles, abscesses, and lymphadenitis).
• S aureus colonizes the skin and mucous membranes of 30% to 50% of children and adults.
• The initial management of skin and soft tissue infections caused by community-associated S
aureus is dictated by the disease severity and the prevalence and susceptibility pattern of
MRSA isolates in the community.
• Cephalexin is appropriate for treating MSSA, but is ineffective in treating MRSA.
• Clindamycin can treat susceptible strains of both MSSA and MRSA and also has activity
against group A streptococci.
Suggested Readings
• American Academy of Pediatrics. Group A streptococcal infections. In: Kimberlin DW,
Brady MT, Jackson, MA, Long SS, eds. Red Book: 2015 Report of the Committee on
Infectious Diseases. 30th ed. Elk Grove Village, IL: American Academy of Pediatrics;
2015:732-734.
Question 16
A 14-year-old male adolescent is brought to your office for a follow-up visit after being
evaluated and discharged from a local emergency department the prior evening. The patient’s
parents took him to the emergency department after they found him stumbling around in the
basement when they arrived home from work. At that time, he seemed confused, was unsteady
on his feet, and his speech was slurred. His symptoms gradually resolved over approximately 1
hour. The emergency department physician ordered numerous “tests,” including urine and blood
drug screens, which the parents were told were “normal.”
The adolescent’s parents have noticed changes in his behavior over the past 3 months since he
began spending time with a different group of friends. Recently, his grades have dropped and he
is no longer interested in playing soccer. His mother found him and one of his friends “hanging
out” in the family garage after they had told her they were going to the library. When she
confronted them, the boys began to giggle uncontrollably and appeared “high.” Both parents feel
that their son is more irritable and withdrawn recently. Otherwise, his complete review of
symptoms is unremarkable.
When you talk with the patient privately, he repeatedly denies drug or alcohol use. His vital signs
are normal, and a complete physical examination reveals no abnormalities.
Of the following, the BEST next step in the management of this patient is to
In evaluating patients with suspected substance abuse, pediatric providers must fully understand
the limitations of drug screening tests. Results of these tests are generally reported as simply
“positive” or “negative,” but these results can be misleading. To accurately interpret the results
of drug screening tests, clinicians must understand the type of testing performed and carefully
consider the clinical scenario in which the testing was ordered.
Urine drug screens are the most commonly used tests to identify substance use or exposure,
though some institutions routinely perform serum drug screening as well. Certain drug assays
can also be performed using hair, sweat, or saliva, but these assays are used much less
commonly. Specific tests included in basic drug screening panels vary by region and institution;
therefore, clinicians should know what drugs are included in the panel used by their laboratory.
In the United States, basic urine screening panels for drugs of abuse typically test for
amphetamines, cocaine, opioids, marijuana, and phencyclidine. Numerous drugs that are widely
abused by adolescents, including designer amphetamines, such as MDMA (ecstasy), synthetic
opioids (such as tramadol), synthetic marijuana, and inhalants, are not detected by many
laboratories’ routine drug screening panels. Thus, a negative result on drug screening cannot
definitively exclude substance abuse.
Most routine drug screening tests are immunoassays that yield a positive result if a drug of abuse
or its metabolite is present at or above an established threshold level at the time the sample is
obtained from a patient. The time frame during which drugs are detected on screening tests
varies, but for most drugs begins within minutes of exposure and lasts for a few days. A patient
may have a negative drug screening result, despite recent substance use, if the level of the drug
in the specimen tested falls below the threshold set for detection. As a result, patients may be
able to intentionally achieve false-negative results by ingesting large amounts of water before
giving a urine sample (diluting the concentration of a drug in their urine), ingesting masking
agents, adding water or other adulterants to their urine samples, or even submitting a urine
sample from another person or synthetic urine.
Positive drug screening results can also mislead clinicians. Many screening tests, especially
immunoassays, can yield false-positive results if cross-reacting substances are present in a test
specimen. For example, over-the-counter cold medications such as dextromethorphan can yield a
false-positive result for phencyclidine, whereas ingestion of poppy seeds can lead to a positive
result for opioids.
American academy of pediatrics 56
American Academy of Pediatrics PREP 2016
Although a positive result to a drug screen may indicate that a patient has been exposed to a drug
of abuse, routine screening tests generally do not indicate whether a patient is intoxicated or even
symptomatic from the drug detected. Clinicians should not attribute signs and symptoms
displayed by a patient to a positive result without considering and excluding other important
differential diagnoses. For instance, in the scenario of a teenager presenting to an emergency
department with fever and confusion and a urine drug screen that is positive for amphetamines, it
would be essential to consider and exclude life-threatening conditions such as encephalitis and
intracranial injury rather than immediately attributing these symptoms to acute amphetamine
intoxication.
There is no clear consensus among clinicians regarding appropriate indications for drug testing
to identify substance abuse among children, or follow-up for those undergoing treatment for
substance abuse. A recent clinical report from the American Academy of Pediatrics regarding
testing for drugs of abuse in children and adolescents provides clinicians with expert, evidence-
based guidelines on this topic.
For the boy in the vignette, who has an ongoing history of behavioral changes, declining school
performance, and symptoms suggestive of intermittent intoxication over the past few months,
referral to a mental health professional with expertise in substance abuse is the best next step in
management.
Considering their limitations, repeating the drug screening tests for this patient, either now or at a
random time in the next week, will not likely aid in confirming his diagnosis and would only
delay the most appropriate management, which is referral for substance abuse treatment.
Pediatric providers should never disregard ongoing symptoms that are suggestive of substance
abuse just because a drug screening test is negative.
Computed tomography of the brain would not contribute to the diagnosis or management of this
boy’s underlying problem. Given that his neurologic symptoms have been transient and self-
limited, he has displayed no other “red flags” for intracranial pathology (such as persistent or
worsening headaches, vomiting, or persistent neurologic deficits), his symptoms began shortly
after he began spending time with a new peer group, and he had a completely normal neurologic
examination at the time of emergency department follow-up, substance abuse is much more
likely to be the etiology of his recent symptoms than intracranial pathology.
Finally, the progressive behavioral changes, irritability, and declining school performance
displayed by this boy over the past 3 months are not consistent with normal behavior and
development. Referral to a mental health professional should be made promptly to prevent the
significant long-term morbidity and even mortality that can result from ongoing substance abuse.
PREP Pearls
• When interpreting results of drug screening tests, clinicians must understand the type of
testing performed and carefully consider the clinical scenario that prompted the testing.
American academy of pediatrics 57
American Academy of Pediatrics PREP 2016
• Pediatric providers should never disregard ongoing symptoms that are suggestive of
substance abuse just because a drug screening test is negative.
• Clinicians should not attribute signs and symptoms displayed by a patient to a positive drug
test without considering and excluding other important differential diagnoses.
Suggested Readings
• Crocetti M. Inhalants. Pediatr Rev. 2008;29:33.
http://pedsinreview.aappublications.org/content/29/1/33.
• Kulig JW. Tobacco, alcohol, and other drugs: the role of the pediatrician in prevention,
identification, and management of substance abuse. Pediatrics. 2005;115:816.
http://pediatrics.aappublications.org/content/115/3/816.
Question 17
While making rounds in the newborn nursery, you are called emergently to the delivery room for
the precipitous delivery of a neonate at 23 2/7 weeks gestation. You arrive to find a limp neonate
with a heart rate of 40 beats/min and no respiratory effort. You are told the neonate weighs 470
g. As you prepare to begin resuscitation, you note microcephaly, cutis aplasia on the scalp, and a
cleft lip and palate.
Of the following, the MOST appropriate indication for non-initiation of resuscitation for this
neonate is
C. fetal aneuploidy
D. parental request
The ethical principles of autonomy, beneficence, nonmaleficence, and justice are tightly woven
into care decisions at the edge of viability. No uniform agreement exists on the exact time of
gestation when this occurs. Viability has been defined as the point at which there is a reasonable
chance of survival with advanced medical support, with some ethicists arguing that resuscitation
should be provided to all neonates who have at least a 50% chance of survival. Neonatal
outcome data published in 2010 can be seen in Item C17, but these data may not reflect
subsequent improvements in neonatal care or center-to-center variability.
The sixth edition of the Neonatal Resuscitation Program of the American Academy of Pediatrics
and the American Heart Association has outlined clinical situations in which noninitiation of
resuscitation may be appropriate. These include confirmed gestation less than 23 weeks,
birthweight less than 400 g, anencephaly, and a confirmed lethal genetic disorder or
malformation. Ideally, ongoing discussions about care at the time of birth should be made jointly
with the family before delivery.
The birthweight of 470 g, gestational age of 23 2/7 weeks, and stigmata of trisomy 13 aneuploidy
in the neonate in the vignette may be indications for noninitiation of resuscitation, but only after
discussion with the family. If the wishes of the family are unknown when a neonate with these
American academy of pediatrics 60
American Academy of Pediatrics PREP 2016
findings is born, it is preferable to begin resuscitation pending discussion with the parents. Many
extremely premature infants are born with bradycardia and apnea, so it would not be appropriate
to assume that this indicates birth asphyxia and withhold resuscitation. If the neonate responds to
resuscitation, further discussions and decisions about ongoing care may occur in the nursery with
the family as active participants.
PREP Pearls
• The ethical principles of autonomy, beneficence, nonmaleficence, and justice are tightly
woven into care decisions for neonates at the edge of viability.
• Parental wishes regarding resuscitation should be respected when a neonate is born between
23 and 24 weeks of gestation.
Suggested Readings
• AAP Committee of the Fetus and Newborn. Noninitiation or withdrawal of care for high-risk
infants. Pediatrics. 2007;119:401-403. DOI: http://dx.doi.org/10.1542/peds.2006-3180.
Question 18
A 14-year-old African American adolescent presents to your office for evaluation of severe
headaches for the past 7 months. The headaches began shortly after the start of the school year.
She had never experienced a headache before that. The headaches involve the left side of her
head, are described as throbbing, and last 2 to 3 hours. During the headache, she feels nauseated
and sometimes vomits. Her symptoms are somewhat relieved if she lies down in a dark room.
The patient does not have any vision changes or weakness associated with the headaches, and
they do not worsen with coughing, sneezing, lying down, or sitting up. They do not wake her up
from sleep. She has not gained or lost weight recently. She does not take any medications other
than occasional ibuprofen for the headaches. Her father has migraine headaches and her younger
sister has sickle cell disease. Her blood pressure is 102/68 mm Hg, heart rate is 92 beats/min,
respiratory rate is 22 breaths/min, and her body mass index is 21. Her physical examination,
including neurological examination and fundoscopy, is unremarkable.
Although classic teaching is that morning headaches are suggestive of a brain tumor, in pediatric
patient’s migraines often occur in the morning. Time of day is not one of the diagnostic criteria
for pediatric migraine, possibly because this criterion does not differentiate migraine from
secondary headache very well. Red flags for pediatric headache include positional headache
(worsening with lying down implies increased intracranial pressure) and headache that wakes the
child from sleep.
In pediatric patients who have had migraine headaches for more than 6 months, with normal
neurological examination findings, and without features suggestive of neurologic dysfunction,
brain imaging is not needed. Family history of migraine is another reassuring factor that supports
a diagnosis of migraine headaches. If there are red flags associated with headache (positional,
sleep disrupting), the preferred brain imaging method is magnetic resonance imaging.
Magnetic resonance imaging of the brain shows the brain’s structure. This is an ideal first
imaging test if a structural malformation such as Chiari I malformation is suspected.
Symptomatic Chiari I malformation presents with diffuse headache that is worse with a Valsalva
maneuver, such as coughing, sneezing, laughing, or defecating. Symptomatic Chiari I
malformation is treated with neurosurgical decompression.
If there is an urgent clinical indication, such as new encephalopathy, new focal deficit, or recent
head trauma, then computed tomography of the head is the best imaging method because it is the
quickest. Other imaging modalities may be indicated when specific diagnoses are being
considered.
A computed tomography venogram is obtained to evaluate for cerebral sinus venous thrombosis.
These cases present with headaches that have migrainous features, such as severe unilateral head
pain, nausea, vomiting, and phonophobia or photophobia. The fundus examination will show
papillitis and there may be a partial or full cranial nerve VI palsy (inability to abduct the eye).
Treatment of cerebral sinus venous thrombosis is anticoagulation until the thrombus is resolved.
Transcranial Doppler ultrasonography is used to monitor flow velocity in the middle cerebral
artery in people with sickle cell disease. Decreased flow velocity is associated with moyamoya
disease. The headaches in moyamoya disease can be diffuse or unilateral, and are often provoked
by exercise or activity or associated with hemiparesis. Treatment of moyamoya disease is
surgical revascularization. The girl in the vignette does not have any signs of sickle cell disease
or moyamoya disease, so a transcranial Doppler ultrasound is not indicated.
PREP Pearls
• Intermittent migraine headaches for at least 6 months, normal neurologic examination
findings, and a family history of migraine are strongly suggestive of a diagnosis of migraine;
brain imaging is not indicated if all these criteria are met.
• Headaches that wake a child from sleep or worsen with lying down are suggestive of a
secondary cause, such as increased intracranial pressure, and would be an indication for
neuroimaging.
Suggested Readings
• Bigal ME, Arruda MA. Migraine in the pediatric population: evolving concepts. Headache.
2010;50:1130-1143. DOI:http://dx.doi.org/10.1111/j.1526-4610.2010.01717.x.
Question 19
A 17-year-old adolescent with type 2 diabetes mellitus presents to the emergency department
with altered mental status. He has had worsening polyuria and polydipsia for 10 days since
starting perphenazine 8 mg orally once every evening for the diagnosis of schizophrenia. His
only other medication is metformin 1,000 mg orally twice per day. On physical examination, his
pulse is 130 beats/min and blood pressure is 108/75 mm Hg. Capillary refill is 3 seconds and
extremities are cool.
Of the following, the MOST important first treatment to initiate for this patient is an intravenous
The mainstay of treatment for HHS is fluid resuscitation, as the degree of dehydration is more
profound than that seen in DKA. The goal of initial fluid therapy is expansion of the
intravascular and extravascular volume and restoration of normal renal perfusion. Vigorous fluid
replacement is recommended, with rates of fluid replacement more rapid than those
recommended for DKA. Fluid resuscitation that is not aggressive is associated with an increased
risk of complications. Increased mortality is observed in patients with unreversed shock over the
first 24 hours of admission who had received less than 40 mL/kg of intravenous fluids over the
first 6 hours of treatment. Therefore, aggressive isotonic fluid replacement is recommended
initially to restore perfusion, even if hypernatremia is present, followed by more hypotonic
(0.45%-0.75% saline solution) fluids.
Ketosis in HHS is usually minimal. Although mild acidosis can occur, it is typically the result of
hypoperfusion (lactic acidosis). Thus, in contrast to the recommended management of DKA,
insulin administration should be withheld until initial fluid resuscitation has been accomplished
and the serum glucose concentration is no longer decreasing from rehydration alone. When
insulin treatment is begun, continuous administration at 0.025 to 0.05 units/kg per hour is
recommended initially. The dose should then be titrated to achieve a decrease in glucose
concentration of 50 to 75 mg/dL per hour (2.7-4.1 mmol/L per hour). Insulin therapy is not
usually necessary for resolution of ketosis in HHS and should be suspended if the glucose
concentration drops more than 100 mg/dL per hour (5.5 mmol/L per hour). Insulin
administration as a bolus is never recommended. Treatment with an insulin drip is not the
appropriate initial treatment for HHS, and unless preceded by adequate fluid resuscitation, may
increase the risk of death.
PREP Pearls
• Fluid resuscitation with isotonic fluids is the mainstay of treatment for hyperglycemic
hyperosmolar syndrome (HHS).
• In contrast to the management of diabetic ketoacidosis, insulin administration in HHS should
be withheld until initial fluid resuscitation has been accomplished and the serum glucose
concentration is no longer decreasing from rehydration alone.
Suggested Readings
• Rosenbloom AL. Hyperglycemic hyperosmolar state: an emerging pediatric problem. J
Pediatr. 2010;156(2):180-184. DOI: http://dx.doi.org/10.1016/j.jpeds.2009.11.057.
Question 20
A 5-year-old boy is brought to the office for follow-up after an admission for loss of
consciousness. His electrocardiogram suggested long QT syndrome and he was prescribed
nadolol. When you see him for follow-up, it becomes clear that the family is not giving him the
medication as prescribed. You express your concern and review the need for medication again
with an explanation of the pathophysiology. The family is very concerned about adverse effects
and does not believe that the medication is necessary. They agree to follow-up in a month and
avoid medications that may worsen the QT prolongation. Two weeks later, a check for $1,000
arrives in your office from the family with a thank you card.
A. accepting cash gifts may cause the family to expect preferential treatment
E. cash gifts to the physician are permissible if they are used for research projects
PREP Pearls
• Accepting monetary gifts for personal use from patients is not recommended in any
circumstance.
• Expensive personal gifts are likely to disrupt the patient-physician relationship and create
inappropriate expectations.
Suggested Readings
• American Academy of Pediatrics. Appropriate boundaries in the pediatrician-family-patient
relationship. Pediatrics. 1999;104(2):334-336.
http://pediatrics.aappublications.org/content/104/2/334.full.pdf+html.
Question 21
During her health supervision visit, you note that a 7-year-old child’s stature has fallen from the
25th percentile last year to less than the third percentile now. The family reports that the child
has been fatigued and has had a poor appetite associated with nausea that began 6 months ago
when she was hospitalized for a kidney problem in another state. She was treated with a 1-month
course of oral prednisone at that time, but has not followed up since. Physical examination shows
a small child for her age, with a blood pressure of 132/88 mm Hg; the remainder of the
examination is unremarkable. Today, the urinalysis shows 4+ protein and 2+ blood. You have
arranged an appointment with a pediatric nephrologist for tomorrow. The parents ask you about
the child’s growth (Item Q21).
Inadequate energy and protein intake is an extremely important cause of growth failure in
children with CKD, and improvements in nutrition with supplementation to achieve adequate
caloric intake result in improved linear growth rates. However, nutrition alone does not fully
explain short stature related to CKD. Growth also improves with optimized fluid and electrolyte
control, treatment of metabolic acidosis, and use of erythropoietin to prevent severe anemia.
Metabolic bone disease related to CKD affects bone formation and growth plate structure, which
also can lead to linear growth impairment. Prolonged steroid use affects GH secretion, decreases
liver production of IGF-1, and contributes to abnormal bone formation, ultimately contributing to
short stature. The child in the vignette, however, received a relatively short course of oral
corticosteroids, so more complex mechanisms of growth impairment are likely to be involved in
this patient. Acquired hypothyroidism is common in nephropathic cystinosis and contributes to
inadequate growth, but coincident endocrine disorders are otherwise unlikely causes of growth
failure in children with CKD.
Unfortunately for children with severe CKD, neither dialysis nor renal transplantation eliminates
growth failure. The mean height 6 to 12 months after initiation of dialysis still shows standard
deviation scores of -1.71 compared to the general population. After transplantation, there is an
increase in mean height, but not enough to make up for the deficit. For children between 6 and
17 years of age at transplantation, there was no improvement in height standard deviation scores
after transplantation.
PREP Pearls
• Linear growth impairment is common in chronic kidney disease and is the result of multiple,
complex causes including inadequate nutrition, metabolic acidosis, fluid and electrolyte
imbalance, metabolic bone disease, severe anemia, and growth hormone insensitivity.
• Dialysis and renal transplantation for children with chronic kidney disease improves growth,
but will not make up for the deficits that already occurred.
Suggested Readings
• Cooke DW, Divall SA, Radovick S. Normal and aberrant growth. In: Melmed S, Polonsky
KS, Larsen PR, Kronenberg HM, eds. Williams Textbook of Endocrinology. 12th ed.
Philadelphia, PA: Saunders Elsevier; 2011:935-1053 .
Question 22
An adolescent who is 13 years of age presents to the office with severe lower abdominal pain.
Her mother reports that she has been complaining about this pain intermittently for the last 4
months. The patient and her mother deny any history of trauma. The patient has had no fever,
nausea, vomiting, diarrhea, or dysuria. She has had constipation and took over-the-counter stool
softeners. Her appetite is decreased when she has pain. She has had no weight loss. She is
premenarchal. She describes the pain as sharp. She says that the pain usually lasts for a few days
and is somewhat relieved by nonsteroidal anti-inflammatory medications. On physical
examination, she is afebrile. Her blood pressure and heart rate are normal for her age and body
mass index is at the 80th percentile. Her cardiovascular and pulmonary examinations are
unremarkable. She has a midline suprapubic mass with associated tenderness. She is sexual
maturity rating 4 for breasts and pubic hair. She has a bluish, bulging discoloration in her
perineal region. Her abdominal ultrasonography is shown in Item Q22.
A. agenesis of vagina
B. imperforate hymen
C. labial adhesion
E. uterine duplication
A low transverse vaginal septum can present with similar clinical symptoms. However, a low
transverse vaginal septum can be distinguished from an imperforate hymen with the Valsalva
maneuver. During the Valsalva maneuver, bulging is seen with an imperforate hymen, and not
with a low transverse vaginal septum.
Vaginal agenesis is typically characterized by absence of the proximal vagina and absence or
hypoplasia of the uterus. In addition to those findings, ultrasonography may reveal additional
abnormalities such as urinary tract anomalies.
Uterine duplication anomalies represent malformations related to failed fusion of the Müllerian
duct structures. Uterine duplication anomalies are often asymptomatic unless an obstruction is
present. Ultrasonography would reveal separate divergent uterine horns.
Labial adhesions occur typically in the prepubescent population before the production of
endogenous estrogen, which starts at puberty.
PREP Pearls
• Imperforate hymen often presents during adolescence as lack of menses and cyclical
abdominal pain with bluish bulging present at the introitus.
• Treatment of an imperforate hymen requires a hymenotomy, a surgical resection of the
membrane.
ABP Content Specifications(s)
• Recognize the clinical findings associated with imperforate hymen
Suggested Readings
• Bacon JL. Prepubertal labial adhesions: evaluation of a referral population. Am J Obstet
Gynecol. 2002;187(2):327-332. DOI: http://dx.doi.org/10.1067/mob.2002.126201.
Question 23
A 4-year-old girl presents for evaluation of bumps on the arm for several weeks. There have
been no associated symptoms and the girl is otherwise well. Her physical examination is
remarkable only for several papules located in the left antecubital fossa (Item Q23).
Of the following, the MOST likely diagnosis is
A. closed comedones
B. common warts
C. flat warts
D. milia
E. molluscum contagiosum
Several disorders can mimic MC, but in each, the lesions lack umbilication. These entities and
their differentiating features are summarized in Item C23C.
Individual lesions of MC may resolve in 1 to 2 months, but often are replaced by new papules.
As a result, the condition may persist for months to years (average 13 months). Molluscum
contagiosum is self-limited, therefore no intervention would be a reasonable choice for children
who have only a few asymptomatic lesions. If treatment is desired, several options exist.
Especially in young children who do not tolerate discomfort well, cantharidin (a blister beetle
extract) may be applied in the office to individual lesions. Within several hours of application, a
blister may form, after which the MC lesion resolves. Alternative painless topical agents that
may be applied at home include salicylic acid and a topical retinoid (eg, tretinoin or adapalene).
None of these topical therapies are US Food and Drug Administration-approved for the treatment
of molluscum contagiosum. For children who can tolerate discomfort, cryotherapy and curettage
are effective.
PREP Pearls
• Lesions that become enlarged and inflamed often represent the body’s inflammatory
response to the infection.
• Molluscum contagiosum is self-limited and does not require treatment.
• The lesions of molluscum contagiosum are skin-colored or “translucent” papules that have a
central umbilication.
Suggested Readings
• Butala N, Siegfried E, Weissler A. Molluscum BOTE sign: a predictor of imminent
resolution. Pediatrics. 2013;131(5):e1650-e1653. DOI: http://dx.doi.org/10.1542/peds.2012-
2933.
Question 24
A 15-year-old adolescent collapses while playing in a high school basketball game without any
preceding trauma. He is unconscious and not breathing.
Of the following, the BEST sequence of actions is to
B. ask someone to retrieve an automatic external defibrillator and start chest compressions
C. immobilize the cervical spine and attempt to palpate the carotid pulse
Appropriate life support responses for children include the algorithms of basic life support, in
which it is assumed that there is only 1 responder, and pediatric advanced life support, which
takes place in an environment in which many rescuers are involved and actions can be
undertaken simultaneously. Regardless of the environment, etiology, or age of the patient in
cardiac arrest, the effectiveness of life support efforts depends on the quality of the
cardiopulmonary resuscitation (CPR). Chest compressions and rescue breathing should be
immediately started. For CPR in children it is recommended to push hard (greater than 1/3 of the
anterior-posterior diameter of the chest), push fast (at least 100 compressions per minute),
minimize interruptions between compressions, and rotate the person giving compressions every 2
minutes. If no advanced airway, ie, an endotracheal tube or laryngeal mask airway, is present, a
15:2 compression-ventilation ratio should be followed. If an advanced airway is in place, 8 to 10
breaths per minute should be given with continuous chest compressions. CPR recommendations
for infants also include a 15:2 compression-ventilation ratio, however, newborns with cardiac
arrest of cardiac origin in the delivery room or neonatal intensive care unit should receive a 3:1
compression-ventilation ratio.
Because the boy in the vignette presented with collapse and apnea, he is unlikely to have a
perfusing rhythm. Palpation for a pulse in this setting may not be accurate, and could lead to a
delay in definitive care. Even rapid assessment of circulation/airway/breathing is not necessary
given the obvious gravity of the child’s status and could lead to further delays in treatment. The
cause of his collapse is likely cardiac, therefore jaw-thrust or chin-lift maneuvers would not be
helpful. Although rescue breathing should be performed, it is not the best next step, because the
therapy most likely to restore spontaneous circulation is cardioversion or defibrillation using an
AED. The precordial thump is no longer recommended in the latest American Heart Association
guidelines. In the absence of trauma, immobilization of the cervical spine is not recommended.
Although 911 should be called, it is likely that an AED is available in the high school
gymnasium and may be effective before the arrival of emergency medical services.
PREP Pearls
• Effective cardiopulmonary resuscitation includes pushing hard (to 1/3 of the anterior-
posterior diameter of the chest), pushing fast (100 compressions per minute) with minimal
interruptions, and rotating personnel performing compressions every 2 minutes.
• The first steps for a child with a sudden cardiac arrest are to call for an automatic external
defibrillator (AED) and to start chest compressions immediately, especially in a public place
where an AED is likely to be available.
Suggested Readings
• Kattwinkel J, Perlman JM, Aziz K, et al. Part 15: neonatal resuscitation—2010 American
Heart Association guidelines for cardiopulmonary resuscitation and emergency
cardiovascular care. Circulation. 2010;122:S909-S919.
DOI:http://dx.doi.org/10.1161/CIRCULATIONAHA.110.971119.
Question 25
A 7-week-old term infant presents to the emergency department in July for evaluation of fever.
He has had 3 days of diarrhea and yesterday was noted to have a rash. Vital signs show a
temperature of 38.5°C, respiratory rate of 50 breaths/min, heart rate of 196 beats/min, blood
pressure of 92/56 mm Hg, and his weight is 4.84 kg. On physical examination, he is irritable and
has a faint erythematous maculopapular rash principally over the lower extremities. Laboratory
data shows:
• White blood cells, 8,330/µL (8.33 x 109/L)
• Hemoglobin, 9.3 g/dL (93 g/L)
• Platelets, 479 x 103/µL (479 x 109/L)
• Differential count, 29% segmented neutrophils, 28% bands, 30% lymphocytes, 11%
monocytes, 2% eosinophils
• Cerebrospinal fluid (CSF) white blood cells, 128/µL
• CSF red blood cells, 5/µL
• CSF glucose, 50 mg/dL (2.8 mmol/L)
• CSF protein, 63 mg/dL
Of the following, the CSF fluid study that is MOST likely to establish the diagnosis in this child
is
A. arbovirus panel
D. viral culture
While arboviral infections, including West Nile virus, can present with a nonspecific febrile
illness or aseptic meningitis, they are a less common etiology in this age group, as they would
require contact with the respective vector. Additionally, diarrhea is not usually associated with
arboviral infections. The most common means of identifying arboviral infections is through the
measurement of antibodies in either CSF or serum. Immunoglobulin M is usually detectable by 3
to 8 days in patients with arboviral infections. However, if an arboviral infection is strongly
suspected and testing is negative within 10 days of the onset of illness, convalescent testing is
recommended. Nucleic acid amplification tests for some arboviral infections are available only
through select reference laboratories or the US Centers for Disease Control and Prevention.
While herpes simplex virus infection can present as a febrile illness with associated
meningoencephalitis, an exanthem, if present, would be expected to be vesicular in nature. In
patients with central nervous system (CNS) infection caused by herpes simplex virus, a PCR
assay is considered to have greater sensitivity than cell culture.
While it is possible to grow some enteroviruses in culture, PCR assays have superior sensitivity.
In the setting of CNS infection, the yield of CSF viral culture for enterovirus ranges between 0%
to 80%, depending on the serotype and cell line used. Patients with neurologic disease caused by
enterovirus 71 can have negative results with both CSF culture and PCR, and it is recommended
that throat and rectal swabs be obtained to identify enterovirus at these sites. Echoviruses 22 and
23 have been reclassified as human parechoviruses. They can cause clinical illness
indistinguishable from enteroviruses. Commercially available enteroviral PCR assays do not
detect parechoviruses.
PREP Pearls
• Depending on geographic location and season, enteroviral infection should be considered a
causative agent for acute febrile illness in a young infant.
• In enterovirus and herpes simplex virus central nervous system infections, cerebrospinal fluid
(CSF) polymerase chain reaction assays have greater sensitivity than viral culture.
• Antibody testing in either CSF or serum is the preferred method for diagnosing arboviral
infections.
Suggested Readings
• American Academy of Pediatrics. Enterovirus (nonpoliovirus). In: Kimberlin DW, Brady
MT, Jackson, MA, Long SS, eds. Red Book: 2015 Report of the Committee on Infectious
Diseases. 30th ed. Elk Grove Village, IL: American Academy of Pediatrics; 2015:333-335.
Question 26
A 14-year-old adolescent presents to the emergency room (ER) with severe abdominal pain for
the last 10 hours. There is no history of fever, burning on urination, or trauma. You note in the
electronic medical record that the patient had 2 other ER visits in the last year, with complaints
of abdominal pain and vomiting with spontaneous resolution of the symptoms.
On physical examination, the patient is anxious. The vital signs show a temperature of 37°C,
heart rate of 110 beats/min, respiratory rate of 24 breaths/min, and blood pressure of 129/82 mm
Hg. An abdominal examination reveals tenderness of the left upper abdomen and a mass on the
left side.
Complete blood cell count, blood urea nitrogen, serum creatinine, electrolytes, and liver function
tests all yield normal results. His urinalysis demonstrates a specific gravity of 1.035, pH of 6.0,
3+ blood, and no leukocyte esterase, protein, or nitrites. His urine microscopy shows 10 to 20 red
blood cells/high power field, less than 5 white blood cells/ high power field, and no crystals and
no bacteria. The significant finding on the patient’s abdominal ultrasound is shown in Item Q26.
Of the following, the MOST likely cause of the patient’s symptoms is
D. vesicoureteral reflux
E. Wilms tumor
In older patients, findings of hydronephrosis on ultrasonography are the clue to the possibility of
UPJ obstruction as the underlying cause of the patients’ symptoms (Item C26). It is important to
perform an ultrasonographic examination during episodes of acute pain, as this may be normal
once the pain subsides.
Diuretic renal scan (renal scan along with administration of a furosemide) is used to confirm the
diagnosis of urinary tract obstruction. It measures the drainage time of the radioisotope
(technetium-99m-mercaptoacetyltriglycine [Tc99mMAG3]) from the renal pelvis (evaluated as a
washout curve). The timed excretion of the radioisotope correlates with the degree of
obstruction. Administration of furosemide results in a prompt washout in nonobstructed kidneys,
while a half-life greater than 20 minutes to clear the isotope from the kidney is indicative of
obstruction.
Vesicoureteral reflux (VUR) is the retrograde passage of urine from the bladder to the kidneys.
Vesicoureteral reflux may present prenatally as hydronephrosis on maternal ultrasonographic
screening or be diagnosed in children after an episode of urinary tract infection (UTI) (usually
before 6-7 years of age). Vesicoureteral reflux is less likely the cause of hydronephrosis in this
patient, as the patient has no symptoms suggestive of current or prior UTIs.
Voiding cystourethrogram showing reflux of urine from the bladder to the upper urinary tract by
either contrast or radioisotope is diagnostic for VUR. Normally, reflux of urine is prevented by
compression of the intravesical ureter by the contracting bladder muscles. The shorter
American academy of pediatrics 86
American Academy of Pediatrics PREP 2016
intravesical ureter (which may be genetically linked) has been implicated in the failure of the
antireflux mechanism, thus leading to primary VUR. Secondary VUR occurs because of
abnormally high pressures in the bladder, leading to incompetence of the ureterovesical junction
and associated reflux. In more than 50% of patients, the reflux resolves spontaneously. Lower
grade of reflux, unilateral reflux, prenatal hydronephrosis, and diagnosis before 1 year of age
have been favorably associated with spontaneous resolution of VUR. Spontaneous resolution has
been reported in up to 60% to 80% of the patients with unilateral grade I to grade IV VUR.
Although patients with grade I and grade II VUR continue to have high rates of bilateral reflux
resolution, only 10% to 20% of patients with grade III and grade IV bilateral reflux experience
spontaneous resolution. Grade V reflux rarely resolves spontaneously, therefore these patients
usually require surgical intervention.
Patients with multicystic dysplastic kidney are usually asymptomatic. Multicystic dysplastic
kidney is usually suspected based on renal abnormalities detected on antenatal ultrasonography
or in neonates with abdominal mass on examination. Classic findings on renal ultrasonography
include multiple noncommunicating cysts with intervening dysplastic renal tissue. The
contralateral normal kidney has increased risk for congenital renal anomalies such as VUR.
Autosomal dominant polycystic kidney disease (ADPKD) is considered an adult onset disease,
with clinical manifestations developing later in life. Children with ADPKD are usually
asymptomatic. Patients are diagnosed with ADPKD during ultrasonographic evaluation for other
abdominal symptoms or by screening of children with strong family history. Infrequently,
children with ADPKD may present with hematuria (gross or microscopic), flank or abdominal
pain, infection (of the cyst), or asymptomatic hypertension. Patients with ADPKD have renal
macrocysts that are visible on ultrasonographic imaging of the kidneys.
The classic presentation of Wilms tumor is abdominal swelling with or without associated
symptoms, including abdominal pain, hematuria, and hypertension. Physical examination reveals
a firm, nontender, smooth mass that usually does not cross the midline.
PREP Pearls
• In patients with antenatally detected hydronephrosis and postnatal genitourinary
abnormalities, ureteropelvic junction (UPJ) obstruction is the most common and
vesicoureteral reflux is the second most common diagnosis.
• Older children with UPJ obstruction present with episodes of flank or abdominal pain (Dietl
crisis).
• Hydronephrosis on renal ultrasonography is the clue to the possibility of UPJ obstruction as
the underlying cause of the abdominal or flank pain in patients.
• Ultrasonography should be performed during episodes of acute pain, as the examination may
be normal once the pain subsides.
Suggested Readings
• Carmody JB, Carmody RB. Question from the clinician: management of prenatal
hydronephrosis. Pediatr Rev. 2011;32(12):e110-e112. DOI: http://dx.doi.org/10.1542/pir.32-
12-e110.
Question 27
You are caring for a 16-year-old adolescent diagnosed 1 year ago with Crohn disease, with
fistulas affecting the ileum and colon. Despite aggressive medical management, she required
resection of her ileum, including her ileocecal valve 4 months ago. Since her surgery, she has
had persistent diarrhea. She is currently on a regular, low fiber diet without supplementation.
Stool studies demonstrate normal bacterial flora, negative reducing substances, and are heme
negative.
Of the following, the patient’s diarrhea is MOST likely caused by malabsorption of
A. bile
B. fat
C. fructose
D. lactose
E. sucrose
Crohn disease may affect the small bowel and the colon. Malnutrition can occur in Crohn disease
caused by chronic inflammation, as an adverse effect of medication or due to surgical resection.
The most common cause of nutritional issues in Crohn disease is inadequate intake of calories or
protein. Patients with active disease have a significant increase in their nutritional needs that
exceed their ability to ingest sufficient calories. Additional nutritional deficiency concerns
resulting from gastrointestinal disorders are listed in Item C27.
PREP Pearls
• Patients with Crohn disease are at increased risk for malnutrition and nutritional deficiency.
• Understanding the anatomy following gastrointestinal resection will help to identify nutrients
that may require additional supplementation.
• Malabsorption of bile, disaccharides, and fats can result in worsening diarrhea.
Suggested Readings
• Critch J, Day AS, Otley A, King-Moore C, Teitelbaum JE, Shashidhar H. Use of enteral
nutrition for the control of intestinal inflammation in pediatric Crohn disease. J Pediatr
Gastroenterol Nutr. 2012;54(2):298–305.
DOI:http://dx.doi.org/10.1097/MPG.0b013e318235b397.
Question 28
You are called to the nursery to evaluate a male newborn who has intermittent cyanosis,
increased work of breathing, and feeding problems. The cyanosis improves with crying. You are
unable to pass an 8-French catheter through both nares. On physical examination, you note a
heart murmur, left-sided facial droop, abnormal ears with no lobes, a left iris coloboma, and
cryptorchidism.
Of the following, the MOST likely diagnosis is
B. branchiootorenal syndrome
C. CHARGE syndrome
E. VACTERL association
The diagnosis of CHARGE syndrome is based on clinical findings and imaging. The only known
gene to be associated with CHARGE syndrome is the CHD7 gene. This gene mutation is
detectable in 65% to 70% of people presenting with CHARGE syndrome. Less than 2% of
patients with CHARGE syndrome are caused by a gene deletion. This disorder is autosomal
dominant in inheritance. Clinical testing is available.
The patient in the vignette had all 4 major criterions and several minor criterions. Item C28C
shows a patient with CHARGE syndrome.
American academy of pediatrics 95
American Academy of Pediatrics PREP 2016
22q11.2 deletion syndrome presents with congenital heart disease, palatal abnormalities,
characteristic facial features, learning problems, hypocalcemia, immune deficiency, kidney
abnormalities, and hearing loss. Patients do not have colobomas or choanal atresia.
Renal coloboma syndrome presents with kidney abnormalities, retinal and optic nerve
colobomas, and sometimes hearing loss. Other multiple congenital anomalies are not seen. It is
caused by PAX2 gene mutations.
VACTERL association presents with a constellation of vertebral anomalies, anal atresia, cardiac
anomalies, tracheoesophageal fistula or esophageal atresia, and renal and limb anomalies. The
absence of colobomas, choanal atresia, ear deformities, and cranial nerve abnormalities
distinguishes it from CHARGE syndrome.
PREP Pearls
• CHARGE syndrome is a mnemonic for a constellation of clinical findings including
coloboma, heart defects, atresia choanae, retarded growth and development, genital
abnormalities, and ear anomalies.
• Neonates with this condition can present with multiple life-threatening conditions,
necessitating immediate evaluation of the heart, airway, gastrointestinal tract, genitourinary
tract, and hearing to improve morbidity and mortality and optimize outcome.
Suggested Readings
• Lalani SR, Hefner MA, Belmont JW, Davenport SLH. CHARGE syndrome.
GeneReviews.http://www.ncbi.nlm.nih.gov/books/NBK1117/ .
Question 29
A 12-year-old girl presents to your office with a 3-week history of nasal congestion and 2 days
of ear pain. Her appetite has been diminished slightly, but she has not been febrile. She has not
been sleeping well, but she has no other systemic complaints. On physical examination, you note
pale purplish nasal mucosa, boggy nasal turbinates, clear nasal secretions, and cobblestoning of
the posterior pharyngeal wall. The tympanic membranes are dull gray with air fluid bubbles
present. Her lungs are clear to auscultation.
Of the following, the MOST likely cause for this girl’s persistent nasal congestion is
A. acute sinusitis
B. allergic rhinitis
D. nonallergic rhinitis
Allergic rhinitis typically begins in childhood and persists into adulthood, often improving in
older adults. AR is an IgE-mediated hypersensitivity reaction to specific allergens in sensitized
patients that results in inflammation of the nasal mucosa. It may be classified as seasonal or
perennial, intermittent or persistent, and mild or moderate to severe. Patients with AR may
present with various signs and symptoms, such as snoring, mouth breathing, sneezing, sniffing,
congestion, rhinorrhea, nasal itching, postnasal drainage, and cough. The nasal symptoms may
be associated with eye complaints, such as red, watery, itchy eyes. In addition, headaches,
fatigue, poor sleep, plus decreased attention and daytime performance are common complaints.
There are several classic physical examination findings of AR. “Allergic shiners” is the
descriptive term for venous congestion and suborbital edema that appears as dark discoloration
under the eyes. Dennie-Morgan lines are accentuated lines or folds below the lower lids. The
“allergic salute,” pushing the tip of the nose up repeatedly, may lead to a transverse nasal crease.
“Allergic facies” consists of an elongated face, high-arched palate, and open mouth breathing.
Seasonal or intermittent AR, often caused by pollen from trees, grasses, or weeds, has symptoms
that are predictable and reproducible from year to year. Perennial or persistent AR may result
from continual exposure to indoor allergens like dust mites, cockroaches, mold spores, and
animal dander; outdoor allergens in subtropical regions with long pollinating seasons; or ongoing
occupational allergen exposure.
Children with 1 component of the atopic triad (allergic rhinitis, asthma, eczema) are 3 times
more likely to develop a second component. Comorbidities may exist with AR, such as poorly
controlled asthma, sinusitis, or otitis media. Approximately 50% of children with chronic otitis
media with effusion have AR and may present with symptoms of ear pain and air-fluid bubbles
on examination like the girl in this vignette. Although she has complained of nasal congestion for
3 weeks, the girl does not have purulent rhinorrhea, headache, facial pain, or chronic cough, as
would be expected in acute sinusitis. Patients with a nasal foreign body typically present with
unilateral nasal obstruction with purulent malodorous rhinorrhea. The symptoms of nonallergic
rhinitis are similar to AR, but triggers such as cold air, strong odors, or spicy foods can be
identified in the patient’s history. In this vignette, persistence of symptoms for 3 weeks and the
girl’s pale rather than erythematous turbinates are clues that this is not a case of recurrent upper
respiratory infection.
PREP Pearls
• In cases of suspected allergic rhinitis, the clinician should attempt to elicit any history of
exposure to allergens.
• Physical examination findings of pale bluish-purple nasal mucosa, boggy nasal turbinates,
clear nasal secretions, and cobblestoning of the posterior pharyngeal wall are suggestive of
allergic rhinitis.
• Allergic rhinitis, sinusitis, otitis media, and poorly controlled asthma are often comorbid
conditions.
Suggested Readings
• Gargiulo KA, Spector ND. Stuffy nose. Pediatr Rev. 2010;31(8):320-324; DOI:
http://dx.doi.org/10.1542/pir.31-8-320 .
• Mahr TA, Sheth K. Update on allergic rhinitis. Pediatr Rev. 2005;26(8):284-289.
Question 30
A 2-year-old boy is admitted to the hospital with newly diagnosed high risk neuroblastoma. He
has had a documented 2 kg weight loss over the last 2 months and has been refusing to eat for
several weeks. On physical examination, his weight is now 10 kg. He has evident muscle wasting
and weakness. His laboratory evaluation is remarkable for hypoproteinemia and
hypoalbuminemia. Nasogastric tube feedings are initiated and he has tolerated a gradual increase
in calories.
Of the following, the BEST caloric goal for this patient is
Children with cancer are at risk for malnutrition and weight loss. Multiple interacting factors are
responsible for the malnutrition seen in children with malignancy (Item C30).
Despite the nutritional challenges faced by children with cancer, there are several methods to
supplement nutrition during treatment. Oral caloric supplements can be added to the food or
drink patients are already consuming; enteral nutrition can be provided via nasogastric, gastric,
or gastro-jejunal tube; or nutrition can be provided parenterally through total parenteral nutrition
(TPN). Despite TPN’s ability to provide caloric supplementation, it has significant risks,
including increased risk of infection and liver damage. Whenever possible, it is preferable to
supplement caloric intake through enteral feeds. This allows for the administration of a balanced
feeding formula providing adequate lipid, protein, and carbohydrate support, as well as
permitting the child to eat as he wishes. For the child described in the vignette, nasogastric tube
feeds providing more than the basic calories needed for growth and development would be most
appropriate.
PREP Pearls
• Children with cancer experience malnutrition as a result of hypophagia and anorexia,
increased caloric needs, and malabsorption.
• Children with cancer have higher caloric needs than children without cancer.
• There are multiple routes to provide nutritional support during cancer therapy, including oral
caloric supplementation, enteral feeds, and parenteral nutrition.
• Enteral caloric supplementation is preferred over parenteral nutrition.
Suggested Readings
• Bauer J, Jürgens H, Frühwald MC. Important aspects of nutrition in children with cancer.
Adv Nutr. 2011;2(2):67-77.http://advances.nutrition.org/content/2/2/67.full .
Question 31
Item deleted
Question 32
A full term female newborn weighing 3,900 g is in the well-baby nursery and breastfed well on
day 1 of life. You are performing the discharge physical on day 2 after birth, but the routine
pulse oximetry screening has a saturation reading of 91%. The baby has a heart rate of 120
beats/min, respiratory rate of 40 breaths/min, and blood pressure is 65/45 mm Hg taken in the
right leg. The baby is awake, alert, and fussy. The chest examination is unremarkable. The
cardiac examination shows a single S2 and a 2/6 systolic murmur at the right upper sternal
border. There is no hepatosplenomegaly. The femoral pulses are easily palpable but diminished.
You are awaiting a call back from the neonatologist.
Of the following, the MOST appropriate next step in management is to
The decreased peripheral pulses in the neonate in the vignette point towards hypoplastic left
heart syndrome with reduction in peripheral circulation. In this lesion, the PDA is functioning as
if it is the aortic arch, much like it did in utero. Early recognition and initiation of PGE treatment
will help to prevent the decompensation that occurs if the PDA is allowed to close. If the PDA
closes, the baby will quickly become acidotic and hypotensive with resultant renal dysfunction.
If you think about the entire cardiac output as the bubble in a carpenter’s level, with the PDA
being the level, then anything that causes the flow to shift toward one end will rob the flow to the
other. Therefore, if the pulmonary vascular resistance (PVR) is acutely lowered by rapidly
ventilating a newborn, the cardiac output will flow toward the lungs and leave the systemic
circulation depleted. This will then cause decreased renal blood flow, decreased urine output, and
poor pulses with acidosis. The lungs will become overcirculated and pulmonary edema will
occur. This also happens naturally and gradually as the PVR drops during the first few days after
birth and helps dictate when surgery is optimally done. If the PVR is acutely raised with a
pneumothorax, for example, the baby will become quite hypoxemic.
Two other lesions that usually require the PDA for stabilization are pulmonary atresia and
tricuspid atresia. In both of these instances where there is no or limited blood flow into the
pulmonary veins, the PDA provides the pulmonary blood flow. The oxygen saturations in these
lesions may be lower than the 91%, as in the patient in this vignette with hypoplastic left heart
syndrome. In pulmonary atresia, a single S2 is audible. Blood flow in this lesion is from the right
atrium through either an atrial septal defect to the left atrium and the left ventricle (LV), or
through the tricuspid valve to the right ventricle and then through a ventricular septal defect to
the LV. In either case, there is mixing in the LV and the blood leaving the LV will then get to the
lungs via the aorta and PDA. Some patients may have collateral vessels that provide additional
flow from the aorta to the pulmonary arteries. Prostaglandin E is needed to keep the PDA open
and ensure pulmonary blood flow. In tricuspid atresia, A2 and a P2 may be audible if there is
flow from the right ventricle to the pulmonary artery. In tricuspid atresia, there will need to be an
adequate atrial level shunt, as well as a ventricular level shunt, to return blood to the right side of
the heart and allow for blood flow into the pulmonary artery. If there is inadequate ventricular
level shunting, then the PDA will be needed for pulmonary blood flow. Stabilization with PGE
prior to transport for both types of patients will allow pulmonary flow to be maintained.
If a newborn has ductal dependent systemic circulation, they may decompensate quickly as the
PDA closes. Following the 4 extremity blood pressures and allowing normal feedings may be
dangerous to the baby if the mesenteric circulation has been diminished. A fluid bolus will not
overcome the lack of flow in the descending aorta. One hundred percent O2 will cause the PDA
to close and the pulmonary vasculature to dilate. Dopamine will increase the systemic vascular
resistance, but without ductal patency in the setting of hypoplastic left heart syndrome, it will not
improve distal perfusion or prevent acidosis.
PREP Pearls
• Cardiac decompensation secondary to outflow tract atresia can be emergently managed by
maintaining the patency of the ductus arteriosus with prostaglandin.
• Screening of all newborns on day 2 of life has been implemented in the United States to
identify newborns with critical congenital heart disease who may be asymptomatic.
Suggested Readings
• Artman M, Mahoney L, Teitel D. Perinatal cardiovascular physiology. Neonatal Cardiology.
2nd ed. New York, NY: McGraw-Hill Professional; 2010:51-57.
Question 33
You are seeing a 4-year-old boy, diagnosed with juvenile idiopathic arthritis, for a health
supervision visit. His arthritis, which affects his wrists and knees, has been well controlled with
medication for the last 6 months. He has had 1 previous disease flare. He is meeting his
developmental milestones. The boy’s parents are concerned and ask for guidance about how his
chronic condition will affect his health, development, and school performance.
Of the following, the MOST appropriate recommendation for this boy’s family is to
A. apply for a 504 modification plan when the boy starts school
B. apply for an individualized educational plan when the boy starts school
The provisions for education related services are under the Individuals with Disabilities
Education Act (IDEA). Related services, as defined by IDEA, include speech-language
pathology, audiology, psychology, physical and occupational therapy, recreation, social work,
counseling, orientation, and mobility services. Educational services are based on the
classification of the student. To qualify for an Individualized Education Plan (IEP), a student
must qualify for special education. Conditions that qualify a student for special education include
sensory deficits such as hearing or visual impairment, physical impairment, chronic illness,
traumatic brain injuries, or cognitive disabilities. Students should have a medical referral for
evaluation and services. Psychoeducational assessments can identify children with a specific
learning disability, autism, emotional disturbances, or speech and language impairments for
special education services.
If a child does not qualify for special education, accommodations can still be granted through a
504 modification plan. Section 504 allows for children with chronic diseases or disabling
conditions to be provided with related services even when they do not qualify for special
education.
An important role of the practitioner is the early identification of children with developmental
delay and their referral to an early intervention program. To qualify for early intervention
services, a child must have a documented developmental delay. Infants and toddlers up to age 3
years can be referred to this program, and should be referred as early as possible to gain
maximum benefit. Children 3 years of age and older with developmental delay should be
referred to the local school district for evaluation and treatment.
Publicly funded special education is available until age 22 years, as well as Supplemental
Security Income (SSI) for those who qualify. For older individuals with chronic or disabling
conditions, the Rehabilitation Act, the Americans with Disabilities Act, and SSI can provide
financial assistance, employment assistance, and assistance with workplace accommodations.
PREP Pearls
• Early intervention programs provide education-related services to infants and toddlers with
developmental delay up to 3 years of age.
• Individualized Education Plans address the needs of children who qualify for special
education by authorizing appropriate education-related services.
• 504 modification plans provide education-related services to children with chronic or
disabling conditions who do not qualify for special education.
Suggested Readings
• American Academy of Pediatrics Council on Children With Disabilities. Provision of
educationally related services for children and adolescents with chronic diseases and
disabling conditions. Pediatrics. 2007;119:1218-1223.
DOI:http://dx.doi.org/10.1542/peds.2007-0885.
Question 34
A 12-year-old girl presents to your office for a health supervision visit prior to attending a
summer gymnastics camp. The girl is premenarcheal and there is no family history of scoliosis.
On physical examination, you note a right thoracic spine prominence with the Adam’s Forward
Bend test. Measurement of her spine using a scoliometer reveals an 8-degree angle of thoracic
rotation. History and physical examination are otherwise unremarkable.
Of the following, the next MOST appropriate step in management is to
Scoliosis is categorized by underlying etiology. Idiopathic scoliosis is the most common type,
affecting 1 in 50 individuals. Idiopathic scoliosis is generally diagnosed during preadolescence
or early adolescence. With the exception of very mild cases, scoliosis shows a female
predominance; girls are 10 times more likely than boys to have severe scoliosis that merits spinal
fusion surgery. Idiopathic scoliosis tends to progress during years of rapid growth. If the
curvature remains in the mild or moderate range by the time a teen reaches skeletal maturity,
further progression of the scoliosis is unlikely.
The most common cause of nonidiopathic scoliosis is neuromuscular (eg, related to cerebral
palsy, spinal muscular atrophy, or other neuromuscular conditions). Scoliosis is also associated
with certain genetic syndromes such as neurofibromatosis and Marfan syndrome.
Occult spinal cord pathology, such as tethered spinal cord or Chiari I malformation with syrinx,
can also cause scoliosis. “Red flags” for occult spinal cord pathology include young age at
scoliosis presentation (<10 years), atypical curve pattern (eg, a thoracic curve toward the left side
of the body), foot deformities (eg, high arch, toe contractures), and other neurologic signs and
symptoms on history and physical examination.
Radiographs are the “gold standard” for the diagnosis of scoliosis. The curvature must measure
10 degrees or more on radiographs to meet the criteria for scoliosis. Providers taking the history
of a patient with suspected scoliosis should ask about any history of worsening shoulder or back
asymmetry, neurologic symptoms including bowel or bladder dysfunction, weakness or pain
radiating into the extremities, and family history of scoliosis. Physical examination findings of
scoliosis can include asymmetry of the shoulders or muscles of the upper back, or a shift of the
pelvis. For the Adams forward bending test, the patient puts his/her palms together and attempts
to touch the floor with his/her fingertips without bending at the knees. Scoliometer
measurements can be performed with the patient in this position. Prominence of 1 side of the
thoracic and/or lumbar spine should prompt radiographic evaluation, particularly if the ATR is 5
degrees or greater.
The treatment for idiopathic scoliosis depends on the severity of curvature and the years of
growth remaining. Observation is the treatment of choice for mild scoliosis (10 to <25 degrees).
Bracing is recommended for individuals with moderate scoliosis (25 to <45 degrees) with
remaining growth potential. For children and adolescents with scoliosis curves greater than 45 to
50 degrees, spinal fusion surgery is usually required.
The girl in the vignette has physical examination findings consistent with scoliosis. A magnetic
resonance imaging scan is not warranted because she does not exhibit atypical features
suggesting occult spinal cord pathology. Bracing would be considered only if radiography
showed scoliosis in the moderate range. Observation would not be appropriate because
radiography could demonstrate that this girl may be a candidate for bracing without which her
scoliosis could worsen over the subsequent 12 months. Individuals with idiopathic scoliosis do
not need activity restriction.
PREP Pearls
• Evaluation for occult spinal pathology is recommended for children with scoliosis presenting
before age 10 years or those who present with a left-sided thoracic curvature.
• Scoliometer measures scoliosis indirectly, therefore radiographs should be obtained for
individuals with a measurement of 5 degrees or greater.
• Occult neurologic disease can cause scoliosis.
Suggested Readings
• Sarwark JF, LaBella C. Pediatric Orthopaedics and Sports Injuries: A Quick Reference
Guide. Elk Grove Village, IL: American Academy of Pediatrics; 2014.
Question 35
A previously healthy, 6-month-old full-term female infant presents to your office for follow-up 3
days after a visit to the emergency department for a fever of 39°C, diarrhea, and poor oral intake.
The infant’s mother reports that "they did some laboratory tests, gave her an antibiotic shot, and
said they thought she had a virus." On physical examination, her temperature is 37°C, heart rate
is 110 beats/min, respiratory rate is 26 breaths/min, and blood pressure is 85/60 mm Hg. The
infant is quiet and nontoxic appearing. Her mucous membranes are moist, capillary refill is 1 to 2
seconds, and you note clear rhinorrhea. The remainder of her physical examination is
unremarkable.
Individuals most likely to acquire infection from CoNS include those with indwelling
intravascular catheters or hardware such as cerebrospinal fluid shunts, pacemakers, vascular
grafts, prosthetic heart valves, prosthetic joints, and other implantable devices. In addition,
immunocompromised hosts are at increased risk of infection. Coagulase-negative staphylococci,
most frequently S epidermidis, are the leading cause of hospital-associated bacteremia in
neonates, especially those who are of less than 35 weeks’ gestation, have low birthweight (<
2,000 g), have intravenous catheters, and are receiving total parenteral nutrition.
If the child described in the vignette had an indwelling vascular catheter or other risk factor(s)
for infection because of CoNS, obtaining a repeat blood culture from the catheter and
administering intravenous vancomycin would be recommended. In such cases, admission to the
hospital is warranted for administration of intravenous antibiotics, repeat blood cultures, and
determination of whether the central catheter can be retained. Ceftriaxone is not useful in treating
CoNS, and oral antimicrobial therapy is not appropriate for bloodstream infections caused by
CoNS.
PREP Pearls
• The genus Staphylococcus has more than 30 species, most of which are coagulase-negative
staphylococci (CoNS)
• Isolation of CoNS in blood culture represents contamination in the majority of cases.
• Factors suggesting that CoNS may be a pathogen include:
• repeatedly positive blood cultures with the same CoNS species
• growth within 24 hours
• symptomatic infection (eg, fever, hypotension, lethargy)
• an intravenous catheter (or implantable device) in place for more than 3 days
• susceptibility testing indicating multiple antibiotic resistance
• immunocompromised host (including neonates)
Suggested Readings
• American Academy of Pediatrics. Staphylococcal infections. In: Kimberlin DW, Brady MT,
Jackson, MA, Long SS, eds. Red Book: 2015 Report of the Committee on Infectious
Diseases. 30th ed. Elk Grove Village, IL: American Academy of Pediatrics; 2015:715-732.
Question 36
You are seeing a 2-year-old girl brought to the emergency department after she was found
unresponsive at her uncle’s house. She had been playing normally and later was found lying
unresponsive in the hallway. The uncle called 911 after his attempts to awaken her were
unsuccessful. Upon arrival to the emergency department, the child is unresponsive. Her vital
signs include a heart rate of 70 beats/min, respiratory rate of 8 breaths/min, blood pressure of
64/38 mm Hg, temperature of 35.6°C, and a pulse oximeter reading of 80% on room air. There
are no obvious signs of traumatic injury. Her pupils are equal, 2 mm in diameter, with minimal
responsiveness. No gag reflex is appreciated and you are unable to elicit deep tendon reflexes.
Her glucose level is 70 mg/dL (3.9 mmol/L). The girl’s mother expresses concern that the uncle
keeps a lot of medicines in his house and she is worried the girl might have gotten into them.
Of the following, the MOST likely cause of this girl’s clinical findings is ingestion of
A. digoxin
B. diphenhydramine
C. hydrocodone
D. lithium
E. verapamil
Opioids are drugs with activity similar to that of opium or morphine that can be either naturally
occurring or synthetic. Opioid receptors exist throughout the central and peripheral nervous
systems and gastrointestinal tract. Opioids are prescribed for many indications including pain
management, cough suppression, sedation or anesthesia, and diarrhea alleviation; they are widely
available in multiple formulations for both prescribed and illicit use.
Over the past 25 years, opioid products available in the United States have increased threefold.
Use of these products has risen drastically, along with resultant addiction and overdose-related
fatalities. Because of their frequent and widespread use, opioids are accessible to many children
and adolescents. All pediatric providers should recognize the clinical findings associated with
acute opioid intoxication and understand how to manage this life-threatening condition
appropriately.
In young children, opioid intoxication typically arises from exploratory ingestion of prescription
or illicit opioids that are accessible in their environments. Some of these agents, such as codeine
and methadone pills, powdered heroin, or fentanyl patches, may cause profound respiratory
depression or even death with ingestion of only small quantities of pills or powder, or exposure
to a single opioid patch. In addition, widely available over-the-counter medications containing
diphenoxylate (eg, lomotil) or dextromethorphan may cause toxicity in young children and are
sometimes even administered to children by well-meaning caregivers for the relief of diarrhea
and cough, respectively.
The classic clinical triad of acute opioid toxicity consists of central nervous system depression,
respiratory depression, and miosis. Other typical clinical findings include hypothermia,
hyporeflexia, flushing, pruritus, bradycardia, hypotension, vomiting, and decreased bowel
sounds. In the case of massive overdose, respiratory toxicity can lead to significant hypoxia,
hypercarbia, and acute lung injury. Generalized seizure activity occurs less commonly, but has
been reported after intoxication with propoxyphene, meperidine, tramadol, fentanyl, and
pentazocine.
The initial management of opioid intoxication in both children and adults should focus on
stabilization of the airway, support of breathing, and hemodynamic support. In cases of known or
suspected opioid overdose, naloxone should be administered promptly. Naloxone, a pure opioid
antagonist, can be given intravenously, subcutaneously, intramuscularly, via endotracheal tube,
or by nebulizer. The onset of action of naloxone is rapid, usually within 1 minute of
administration. In the setting of acute opioid overdose, for children up to 5 years of age and those
weighing less than 20 kg, a dose of 0.1 mg/kg of naloxone should be administered. In older
children, a rapid dose of 2 mg should be given. Repeat doses may be given every 2 to 3 minutes,
American academy of pediatrics 118
American Academy of Pediatrics PREP 2016
Any young child presenting with central nervous system and respiratory depression secondary to
opioid toxicity should be hospitalized, even if symptoms improve with naloxone administration,
because repeat dosing or continuous infusion of naloxone may be needed. Young children should
be observed in a monitored setting for at least 24 hours after exposure because of the potential
for delayed toxicity.
Children with acute toxicity from digoxin typically present with vomiting, bradycardia with
atrioventricular heart block, hyperkalemia, and mental status changes. Although the girl in the
vignette exhibits mental status changes and bradycardia, her findings of pinpoint pupils and
respiratory depression are not typically seen with digoxin toxicity.
Children with acute toxicity from lithium may present with a range of clinical findings, including
gastrointestinal upset (nausea, vomiting, and diarrhea), dehydration, tremor, weakness,
hyperreflexia, slurred speech, visual disturbances, mental status changes, and even seizures with
severe toxicity. Lithium toxicity would not explain the significant respiratory depression, miosis,
or decreased reflexes observed in the girl in the vignette. Profound somnolence would also not be
a characteristic early finding in a child with acute lithium toxicity.
Verapamil is a calcium channel blocker used (most commonly in adult patients) in the treatment
of hypertension, coronary artery disease, and atrial fibrillation, and to prevent cerebral
vasospasm. The predominant clinical findings in patients with calcium channel blocker toxicity
are bradycardia and hypotension. Neurologic and respiratory system derangements can occur
from calcium channel blocker overdose, but these generally arise secondary to cardiovascular
toxicity and shock. Though the girl in the vignette presents with bradycardia and hypotension,
American academy of pediatrics 119
American Academy of Pediatrics PREP 2016
her constricted pupils, acute onset of somnolence, and profound respiratory depression are better
explained by overdose of the opioid hydrocodone.
PREP Pearls
• The classic clinical triad of central nervous system depression, respiratory depression, and
miosis should prompt clinicians to suspect opioid poisoning, even in the absence of a history
of opioid exposure.
• Initial management of suspected or confirmed opioid toxicity should include stabilizing the
patient’s airway, ensuring adequate ventilation, and prompt administration of the opioid
antagonist naloxone.
• Any young child presenting with central nervous system and respiratory depression
secondary to opioid toxicity should be hospitalized, even if symptoms improve with
naloxone administration, because repeat dosing or continuous infusion of naloxone may be
needed.
Suggested Readings
• Erickson TB. Opioids. In: Erickson TB, Ahrens WR, Aks SE, Baum CR, Ling LJ, eds.
Pediatric Toxicology: Diagnosis and Management of the Poisoned Child. New York, NY:
McGraw Hill; 2005;chap 60:409-414.
Question 37
A mother presents to labor and delivery at 36 weeks’ gestation with a chief complaint of
decreased fetal movement for the past 6 hours. The pregnancy history is significant for a known
fetal tachyarrhythmia that has been well controlled by maternal treatment with digoxin.
Continuous monitoring reveals a fetal heart rate of 160 beats/min with rare variable decelerations
seen with infrequent contractions. Further evaluation of fetal well-being includes a nonreactive
nonstress test and a biophysical profile of 4 out of 10. Due to the late preterm status of the
pregnancy and history of fetal arrhythmia, the obstetrician seeks your input in the subsequent
management of the mother and fetus.
Of the following, the MOST appropriate next step in management of the mother and fetus is to
Maternal perception of fetal movement has long been recognized as a measure of fetal well-
being; with any concerns of decreased fetal movement prompting further evaluation. If the
pregnancy is at 32 weeks’ gestation or greater, a nonstress test (NST) is often performed to
document the presence of fetal heart rate (FHR) accelerations with fetal movement. A reactive
NST will demonstrate 2 or more FHR accelerations within a 20-minute period and is considered
reassuring. Conversely, a nonreactive NST is nonspecific and should lead to further fetal
assessment with a biophysical profile (BPP). A BPP uses fetal ultrasound evaluation along with
an NST to assess 5 measures of fetal well-being: movement, tone, breathing, amniotic fluid
volume, and heart rate (NST). Each is assigned either 0 or 2 points, with a composite score of 8
to 10 being normal, 6 being equivocal, and 4 or less being abnormal. For an abnormal BPP,
induction of labor is recommended unless an obstetric contraindication necessitates caesarian
delivery.
The presence of a fetal arrhythmia makes in utero assessment more challenging. The early
identification of a fetal arrhythmia allows close fetal monitoring for complications such as
hydrops fetalis. Some fetal arrhythmias are amenable to therapies delivered transplacentally,
such as digoxin for supraventricular tachycardia. Undiagnosed arrhythmias presenting in the
third trimester may be interpreted as fetal distress, leading to premature delivery.
The mother in the vignette presented with a complaint of decreased fetal movement. The
subsequent nonreactive NST and biophysical profile of 4 out of 10 raise concern for fetal well-
being and suggests the need to induce labor. Continued observation with intermittent monitoring
or discharge from the hospital would not be recommended because of the increased risk of fetal
death with these findings. The maternal digoxin dosing does not need to be increased in this case,
because the fetal heart rate appears to be well controlled. At present, there is no evidence to
support urgent caesarian delivery.
PREP Pearls
• Maternal perceptions of fetal movement have long been recognized as a measure of fetal
wellbeing; any concerns of decreased fetal movement should prompt further evaluation.
• A reactive nonstress test (NST) will demonstrate 2 or more fetal heart rate accelerations
within a 20-minute period and is considered reassuring.
• A biophysical profile, which uses both fetal ultrasound and NST, contains 5 measures of fetal
wellbeing: movement, tone, breathing, amniotic fluid volume, and heart rate (NST). Each
measure is assigned either 0 or 2 points, with a composite score of 8 to 10 being normal, 6
being equivocal, and 4 or less abnormal.
Suggested Readings
• Farley D, Dudley DJ. Fetal assessment during pregnancy. Pediatr Clin North Am. 2009; 56:
489-504. DOI:http://dx.doi.org/10.1016/j.pcl.2009.03.001.
Question 38
A 17-year-old high school junior presents for evaluation of migraine headaches. He has had
infrequent migraine headaches since he was 10 years of age. Since school started 3 months ago,
his headaches have increased in frequency. He experiences 2 to 3 migraine headaches per week
and has to go home from school at least once a week. He has been taking ibuprofen 3 or 4 days
every week for the past month and it no longer relieves the migraine symptoms. He takes
isotretinoin for acne. His blood pressure is 118/68 mm Hg, heart rate is 86 beats/min, respiratory
rate is 16 breaths/min, and body mass index is 18. His physical examination, including
neurological examination and fundoscopy, is unremarkable.
C. prescribe butterbur
D. prescribe topiramate
For the boy in the vignette, who is nearly an adult, migraine prophylactic medications used for
adults would be effective. Butterbur and topiramate are both approved by the US Food and Drug
Administration for migraine prophylaxis in adults. Medication choices should be tailored to
individuals. For instance, topiramate can cause weight loss and should be used cautiously in a
person who is already thin.
Isotretinoin has been associated with pseudotumor cerebri and should be stopped if a person
develops signs of this condition. The boy in the vignette has typical migraines and his
fundoscopic examination findings are normal, so he does not have signs of pseudotumor cerebri.
Stopping isotretinoin is unlikely to help his headaches.
An abortive therapy plan for migraines starts with over-the-counter medications such as
acetaminophen or ibuprofen. If these are ineffective, prescription medications such as one of the
triptan class of medications can be tried. Caffeine is often helpful for migraines as well.
Antiemetics should be considered if nausea or emesis are present.
Medications for abortive therapy should not be used more than 2 to 3 times a week; otherwise
medication overuse headache can develop. The only treatment for medication overuse headache
is to discontinue the inciting medication. Patients should know that the headaches will transiently
worsen, but discontinuing the inciting medication is a necessary step as the migraines are
unlikely to improve in the setting of ongoing medication overuse.
PREP Pearls
• Limit migraine abortive medications (eg, ibuprofen and acetaminophen) to 2 to 3 days per
week, to prevent medication overuse headache.
• Isotretinoin has been associated with pseudotumor cerebri and should be discontinued if a
person develops signs of this condition.
Suggested Readings
• Blume HK. Pediatric headache: a review. Pediatr Rev. 2012;33(12):562-576. DOI:
http://dx.doi.org/10.1542/pir.33-12-562.
Question 39
You are contacted by your state’s newborn screening office about an 8-day-old male newborn
with a positive newborn screen for congenital adrenal hyperplasia. In your office, the newborn’s
vital signs include a pulse of 175 beats/min and a blood pressure of 75/40 mm Hg. His body
surface area is 0.2 m2. He has descended testes bilaterally and a normal phallus. He has no
hyperpigmentation.
Initial laboratory results include a serum sodium of 129 mEq/L (129 mmol/L), potassium of 6.6
mEq/L (6.6 mmol/L), glucose of 87 mg/dL (4.8 mmol/L), and bicarbonate of 16 mEq/L (16
mmol/L).
Based on the newborn’s diagnosis, clinical status, and laboratory results, you administer a
normal saline bolus of 20 mL/kg intravenously.
Of the following, the MOST important next step in the treatment of this patient is to administer
A. betamethasone, 10 mg intramuscularly
C. dexamethasone, 10 mg intravenously
E. methylprednisolone, 10 mg intravenously
The infant in the vignette has abnormal vital signs, and his electrolytes show that he is already
experiencing salt wasting and is dehydrated. Treatment with high-dose steroids (stress dose
steroids) is needed to mimic the high doses of steroids normally produced under stress in patients
with sufficient adrenal function.
The treatment of choice for adrenal crisis, of any cause, is fluid replacement, hydrocortisone
hemisuccinate intravenously, and if hypoglycemia is present, intravenous dextrose.
Hydrocortisone is quick acting and at an emergency stress dose (100 mg/m2) saturates all steroid
receptors, causing a mineralocorticoid and glucocorticoid effect. Intramuscular hydrocortisone is
commonly given to patients to take at home before coming to the hospital if they are severely ill,
but once in the hospital or emergency department, intravenous hydrocortisone should be used
because of its quick onset of action.
Pediatricians should recognize that some steroids, such as methylprednisolone, commonly used
in asthma, have no mineralocorticoid activity at any dose and would not be appropriate for this
patient. Dexamethasone and betamethasone can act quickly but have limited mineralocorticoid
effect, and would be used in much smaller doses than those listed in the responses. It is helpful to
be familiar with recommended hydrocortisone stress doses (Item C39).
PREP Pearls
• A newborn screen positive for congenital adrenal hyperplasia should prompt immediate
testing for serum electrolytes, glucose and a confirmatory 17-alpha-hydroxyprogesterone
level.
• High-dose intravenous hydrocortisone hemisuccinate (100 mg/m2 up to 100 mg maximum
per dose) is the steroid of choice for the treatment of adrenal crisis.
Suggested Readings
• Shulman DI, Palmert MR, Kemp SF; Lawson Wilkins Drug and Therapeutics Committee.
Adrenal insufficiency: still a cause of morbidity and death in childhood. Pediatrics.
2007;119:e484-e494. DOI: http://dx.doi.org/10.1542/peds.2006-1612.
Question 40
A 7-year-old boy is brought to the emergency department with wheezing, tachypnea, and
respiratory distress. He started to exhibit symptoms of an acute upper respiratory infection 2 days
ago. He has had dry, spasmodic coughing, and now complains of chest tightness. There has been
no loss of consciousness.
The child was born full term and has no prior history of hospitalization or respiratory failure. He
plays soccer 5 days per week and receives β-adrenergic agonist therapy 4 to 6 times per week for
exertional cough, chest tightness, and dyspnea; care providers report short term improvement
with this intervention. On these occasions, he has used albuterol prescribed to an older sibling
who has been treated for moderate persistent asthma and atopic dermatitis.
On physical examination, his heart rate is 130 beats/min and the respiratory rate is 24
breaths/min. Oxygen saturation is 98% in room air. His weight and height are at the 50th
percentile. Mild suprasternal retractions are noted. There is no appreciable murmur on cardiac
examination. Auscultation of lung fields reveals fair-to-moderate aeration with prolonged
expiratory phase and diffuse expiratory wheezing. The abdomen is soft and nontender.
Extremities are warm and well perfused, without cyanosis, clubbing, or edema.
You administer albuterol and ipratropium bromide via nebulizer, with only a modest
improvement in the boy’s respiratory signs and symptoms.
Of the following, the MOST appropriate next therapeutic intervention for this patient is
administration of
C. subcutaneous epinephrine
D. systemic corticosteroids
E. theophylline
Corticosteroids are an important component in the treatment of acute asthma as they reduce
airway hyperresponsiveness, inhibit migration and activation of inflammatory cells, and prevent
the late phase reaction to allergens. Assuming equal doses, systemic steroids are of similar
efficacy when administered orally, intramuscularly, or intravenously. Systemic steroids may be
given as prednisone, prednisolone, dexamethasone, or methylprednisolone. Although there is
some controversy as to optimal dosing interval, it has been demonstrated that the treatment of
severe acute asthma with systemic corticosteroids within 1 hour of presentation to the emergency
department lowers hospitalization rate and improves pulmonary function.
According to the most recent National Asthma Education and Prevention Program Expert Panel
Guidelines for the Diagnosis and Management of Asthma, the patient in the vignette
demonstrates symptoms most consistent with mild persistent asthma (Item C40). He has required
use of his sibling’s short acting albuterol more than twice weekly with exertional symptoms, and
demonstrates minor limitation in exertional activities.
Long acting β-agonists (LABAs) are contraindicated as a reliever for acute asthma
exacerbations. They are also contraindicated as monotherapy in adult and pediatric asthma. The
US Food and Federal Drug Administration has mandated that a “black box warning” label be
added to products that contain salmeterol, formoterol, or indacaterol, as these compounds carry
"an increased risk of severe exacerbation of asthma symptoms, leading to hospitalizations, in
pediatric and adult patients, as well as death in some patients using LABAs for the treatment of
asthma." The use of LABAs is indicated for treatment of moderate to severe persistent asthma
(step 3 or higher), and only in combination with an inhaled corticosteroid in those patients whose
asthma is not well controlled with use of low dose inhaled corticosteroid alone.
Theophylline has not been demonstrated to have significant efficacy in the treatment of acute
asthma in children. Other therapeutic steps, including intravenous magnesium sulfate and
terbutaline, may be utilized in the treatment of status asthmaticus, but these therapies are
classically employed after administration of a systemic corticosteroid.
PREP Pearls
• Systemic corticosteroids are indicated in the treatment of moderate-to-severe asthma
exacerbations.
• Long acting β-agonists are contraindicated in the therapy of acute asthma and should not be
used as chronic monotherapy in the treatment of asthma.
• Based on clinical history, a patient’s asthma severity should be evaluated as recommended by
the National Asthma Education and Prevention Program Expert Panel Guidelines for the
Diagnosis and Management of Asthma; the most appropriate chronic preventive therapy may
then be guided by treatment recommendations.
Suggested Readings
• Becker AB, Nelson NA, Simons ER. Inhaled salbutemol (albuterol) vs injected epinephrine
in the treatment of acute asthma in children. J Pediatr. 1983;102(3):465-469 .
Question 41
A 7-month-old infant is referred to you for evaluation of failure to thrive. The baby has been
breastfed since birth; puréed fruits and vegetables were started 2 months ago. The parents report
that the baby stopped rolling over 2 months ago and he has not been able to sit, even with
assistance. Last month, he received intravenous fluids in the emergency department when he
became dehydrated after a day of vomiting. Physical examination is notable for a thin child who
is alert but demonstrates poor tone. His liver is palpable 3 cm below the right costal margin. The
remainder of the examination is unremarkable. His growth charts are shown in Item Q41A and
Item Q41B.
Of the following, the MOST appropriate initial approach to this infant’s failure to thrive is to
For patients with failure to thrive but without abnormalities suggesting an underlying inborn
error of metabolism, a multidisciplinary approach is appropriate. Involvement of an occupational
or speech therapist to work on feeding techniques and texture tolerance is often a first step. Child
abuse specialists and agencies may be necessary when there is concern for neglect or factitious
disorder. Some infants, particularly those with known underlying conditions such as congenital
heart disease, low birth weight, or chronic lung disease, may require a high caloric intake to
support growth, and that intake can best be achieved by providing higher caloric density foods
such as fortified breast milk. However, because the infant in the vignette has findings suggestive
of underlying metabolic disease, evaluation for that takes precedence over changes in the
approach to feeding the infant.
PREP Pearls
• Failure to thrive (FTT) is a physical sign of a nutritional state that is inadequate to support
normal growth and development.
• An underlying metabolic cause of FTT is suggested by a history or physical examination
finding of severe, life-threatening disease; recurrent vomiting and dehydration;
developmental delay or regression; hypotonia, stroke, or seizure; organomegaly, particularly
hepatomegaly; cardiomyopathy; visual or hearing deficit; dysmorphic features; or
pancytopenia.
• For patients with FTT without a known or suspected underlying cause, a multidisciplinary
approach to feeding is preferred.
Suggested Readings
• Ficiciouglu C, An Haack K. Failure to thrive: when to suspect inborn errors of metabolism.
Pediatrics. 2009;124(3):972-979. DOI: http://dx.doi.org/10.1542/peds.2008-3724.
Question 42
A 21-year-old woman presents to the clinic for her first routine gynecologic examination. She
states that her last menstrual period was about 3 weeks ago. She reports mild cramping during
her cycle. She denies sexual activity. She has no other concerns or complaints. Upon physical
examination, she is afebrile and has normal vital signs. Her pelvic examination is significant for
a left-sided adnexal mass, but the rest of her examination is unremarkable. She is sent for a
pelvic ultrasonography, which reveals a 4 cm anechoic fluid-filled mass of the left ovary.
Of the following, the MOST appropriate next step in her management is
A. α-fetoprotein testing
B. CA-125 testing
C. culdocentesis
D. cystectomy
E. oral contraceptives
Functional cysts that are less than 5 cm usually self-resolve within 2 to 3 menstrual cycles. Oral
contraceptives can be used to prevent future cysts, but are not thought to aid in the resolution of a
cyst.
α-fetoprotein is a tumor marker that is frequently elevated in patients with malignant ovarian
germ cell tumors (MOGCT). On ultrasonography, an MOGCT would have a more complex and
heterogeneous appearance than a simple cyst.
The tumor marker CA-125 is associated with epithelial ovarian cancer. However, it can also be
elevated in young women with endometriosis. It is not indicated in the woman in this vignette
because the ultrasonographic features of the mass suggest a benign process.
A simple cyst of less than 5 cm in a premenopausal woman can be followed for resolution and
often does not require surgical intervention. Oral contraceptives can be used for prevention of
additional cysts. Larger cysts, symptomatic cysts, or cysts that are increasing in size may require
aspiration or cystectomy. Large cysts increase the risk of ovarian torsion because of their weight.
PREP Pearls
• Functional ovarian cysts are the most common type of ovarian mass in the postpubertal
female and are the result of ovulation.
• Hormonal contraceptives may help prevent future ovarian cysts, but they are not thought to
help with the resolution of existing cysts.
Suggested Readings
• Hoffman BL, Schorge JO, Schaffer JI, et al. Ovarian germ cell and sex cord-stromal tumors.
In: Hoffman BL, Schorge JO, Schaffer JI, et al, eds. Williams Gynecology. 2nd ed. New
York, NY: McGraw-Hill; 2012:879-897.
Question 43
A 6-year-old boy presents for evaluation of a wart on his hand for several months. Parental
attempts to “freeze” the wart with an over-the-counter preparation were painful and
unsuccessful. The physical examination reveals a 4-mm papule with a rough surface located on
the dorsum of the right hand.
Of the following, the MOST appropriate treatment for this boy is
A. cimetidine orally
B. cryotherapy
C. imiquimod topically
E. surgical excision
For those children able to tolerate some discomfort, cryotherapy is useful, causing necrosis of
wart tissue. Liquid nitrogen is the most effective cryogen, achieving a temperature of
approximately -195°C. It is applied using a spray device or cotton-tipped applicator. The
application is continued until a white ring extends 1 to 3 mm beyond the margin of the wart
(typically 10 to 15 seconds). Some advise a second treatment after the wart thaws. Families
should understand that a blister may form in 1 to 2 days. When the blister ruptures, the area may
be cleansed twice daily, followed by the application of a topical antibiotic and bandage. If a
residual wart remains (or if a blister did not form), salicylic acid treatment should be initiated as
American academy of pediatrics 140
American Academy of Pediatrics PREP 2016
Cimetidine has immunomodulatory effects, enhancing T-cell function and cytokine production.
Although not US Food and Drug Administration (FDA)-approved for the treatment of warts, it
may be of benefit (used in conjunction with topical salicylic acid) in those who have multiple or
resistant warts. Topical imiquimod has been used off-label, although its efficacy is limited by
poor absorption through the highly keratinized skin characteristic of common warts. Surgical
excision occasionally is considered for resistant warts, but carries a risk of scarring. Additional
treatments include intralesional injection of skin test antigens (eg, Candida, Trichophyton);
measles, mumps, and rubella vaccine or bleomycin; and immunotherapy with topical squaric
acid. None of these therapies are FDA-approved for the treatment of warts.
PREP Pearls
• When treating young children who have common warts, salicylic acid is the preferred initial
therapy. An important element of treatment is debridement of the wart using an emery board.
• Families should be informed that a blister may form 1 to 2 days after wart cryotherapy. When
the blister ruptures, the area may be cleansed twice daily, followed by the application of a
topical antibiotic and bandage.
Suggested Readings
• Craw L, Wingert A, Lara-Corrales I. Are salicylic acid formulations, liquid nitrogen or duct
tape more effective than placebo for the treatment of warts in paediatric patients who present
to ambulatory clinics? Paediatr Child Health. 2014;19(3):126-127.
Question 44
A 10-year-old girl collided with a telephone pole while riding her bike. Upon presentation to the
emergency department, she is awake, alert, and complaining of significant abdominal pain. Her
vital signs are a temperature of 37°C, heart rate of 140 beats/min, respiratory rate of 30
breaths/min, and blood pressure of 120/65 mm Hg, with an oxygen saturation of 95% on room
air. On physical examination, the girl’s pupils are equal, round, and reactive. She does not have
any cervical spine tenderness or signs of extremity trauma, and she is moving all extremities with
no deficits. There is bruising over the anterior aspect of her abdomen tracking to her left flank.
Computed tomography scans reveal no intracranial bleeding and no bleeding or contusion in the
chest. There is a significant hematoma in the duodenal wall, hemorrhage involving the pancreas,
and a grade IV splenic laceration. Over the next 24 hours, the girl’s breathing becomes
increasingly rapid and shallow, with grunting and hypoxia. She requires endotracheal intubation,
mechanical ventilation, and 60% oxygen to maintain her arterial oxygen saturation above 90%.
On chest radiography, she has bilateral infiltrates.
Of the following, the MOST likely cause of this child’s respiratory failure is
A. aspiration pneumonia
C. pancreatitis
E. septic shock
The diagnosis of ARDS is made clinically based on criteria that include respiratory failure, a
PaO2/FiO2 ratio of less than 200, bilateral pulmonary infiltrates on chest radiography, and a
noncardiogenic cause of pulmonary edema. Acute lung injury results from the same pathogenic
causes and uses the same clinical diagnostic criteria, except for the PaO2/FiO2ratio, which is
between 200 and 300. ARDS has both pulmonary and extrapulmonary causes. Direct lung injury
can be caused by pneumonia, ventilator-induced lung injury, chest trauma, aspiration
pneumonitis, acute chest syndrome, drowning, and smoke inhalation. Extrapulmonary causes
include sepsis, transfusion-related lung injury, burns, fat embolism, pancreatitis, trauma, or
systemic inflammation from numerous other causes.
The pathogenesis of ARDS involves the breakdown of the barrier between alveoli and
pulmonary capillaries, which leads to debris in the airspaces, including blood, pus, and
proteinaceous fluid. Debris in the airspaces and increased surface tension from low surfactant
production and function exacerbate alveolar collapse. This leads to hypoxia in areas that are
perfused but not ventilated, also known as V/Q mismatch, or shunt. In addition, fluid in the
interstitium leads to decreased lung compliance and low tidal volumes. Repetitive and forceful
opening and closing of lung units to maintain tidal volume can exacerbate the inflammatory
cascade, leading to the secretion of proinflammatory cytokines, continuing the cycle of increased
capillary permeability. Death can occur in severe cases of ARDS from worsening hypoxia,
ventilator-associated pneumonia, pulmonary hypertension, end-organ damage, prolonged
respiratory failure, or chronic lung disease.
For the girl in the vignette, the history does not support a diagnosis of aspiration pneumonia.
Neurogenic pulmonary edema generally does not occur without a brain injury. Postobstructive
pulmonary edema can occur after an airway obstruction is relieved, but this child did not have
airway obstruction. There is no fever, infection, or end-organ perfusion compromise to suggest
septic shock.
PREP Pearls
• Acute respiratory distress syndrome can be caused by direct lung injury or indirectly from
nonpulmonary conditions.
• Acute respiratory distress syndrome can cause death from hypoxia, end-organ damage, or
complications from prolonged ventilation.
• Acute respiratory distress syndrome is a clinical diagnosis based on respiratory failure,
bilateral lung infiltrates on radiographs, a ratio of arterial oxygen tension to fraction of
inspired oxygen of <200, and a noncardiogenic cause of respiratory failure.
American academy of pediatrics 143
American Academy of Pediatrics PREP 2016
Suggested Readings
• Carlo WA, Ambalavanan N. Conventional ventilation: traditional and new strategies. Pediatr
Rev. 1999;20(12):e117-e126.
http://pedsinreview.aappublications.org/content/20/12/e117?related-
urls=yes&legid=pedsinreview;20/12/e117 .
Question 45
You are seeing a 9-year-old boy for a health supervision visit. During a recent camping trip in
Arkansas, one of his bunkmates was bitten by a tick and developed fever and a rash 1 week later.
The bunkmate is currently hospitalized and has been diagnosed with ehrlichiosis. Your patient’s
mother is now very concerned about tick-borne infections and inquires as to the best means of
protecting her children in the future.
Of the following, the BEST advice you can provide is
Generally, ticks require humid environments for survival. The use of permethrin-embedded bed
nets when sleeping would protect a child from mosquito-borne infections but not tick-borne
infections.
PREP Pearls
• Prevention of tick-borne and mosquito-borne infections involves both personal protection
and environmental measures.
• To prevent tick-borne infections, avoid humid environments, use light-colored long-sleeved
shirts and long pants, and perform full body inspections with removal of attached ticks as
soon as detected.
• Diethyltoluamide (DEET)-containing products are considered safe for use in children 2
months of age and older.
Suggested Readings
• American Academy of Pediatrics. Prevention of mosquitoborne infections. In: Kimberlin
DW, Brady MT, Jackson, MA, Long SS, eds. Red Book: 2015 Report of the Committee on
Infectious Diseases. 30th ed. Elk Grove Village, IL: American Academy of Pediatrics;
2015:213-215 .
Question 46
A 6-month-old infant presents with fever, vomiting, and gross hematuria. He was born to a 17-
year-old gravida 1, para 1 mother with no prenatal visits. His vital signs show a temperature of
39°C, heart rate of 130 beats/min, respiratory rate of 28 breaths/min, blood pressure of 90/62 mm
Hg, and oxygen saturation of 97% by pulse oximetry on room air. A physical examination shows
mild dehydration and a prominent suprapubic area that is dull on percussion.
A. intravesical level
D. ureteropelvic level
E. ureterovesical level
The patient described in the vignette has UTI (fever, pyuria, leukocytosis), enlarged bladder
(prominent suprapubic area dull on percussion), and renal failure (serum creatinine of 1 mg/dL
[88 µmol/L]). The ultrasonographic findings are suggestive of PUV as the underlying cause of
the patient’s symptoms.
Patients with PUV develop chronic renal failure caused by associated renal dysplasia and
acquired renal injury caused by poor bladder function or infection. Therefore, such patients are
regularly followed to monitor their renal function, blood pressure, and growth. Long term
management is aimed at management of bladder dysfunction (to minimize increased urinary tract
pressures), avoiding UTI and complications associated with chronic kidney injury.
The membranous urethra is the shortest, least dilatable, and the narrowest part of the urethral
canal (except the external urethral orifice). Patients with PUV develop chronic renal failure
caused by associated renal dysplasia and acquired renal injury caused by poor bladder function
or infection. Therefore, such patients are regularly followed to monitor their renal function,
blood pressure, and growth. Long term management is aimed at management of bladder
dysfunction (to minimize increased urinary tract pressures), avoiding UTI and complications
associated with chronic kidney injury. The membranous urethra is the shortest, least dilatable,
American academy of pediatrics 148
American Academy of Pediatrics PREP 2016
and the narrowest part of the urethral canal (except the external urethral orifice). It extends from
the apex of the prostate to the urethral bulb and perforates the urogenital diaphragm behind the
pubic symphysis. The membranous urethra is not obstructed in patients with PUV.
PREP Pearls
• Posterior urethral valves (PUV) are identified by prenatal ultrasonography in the majority of
cases.
• Postnatally, patients with PUV usually present with urinary tract infections, failure to thrive,
abdominal distension, and poor urinary stream.
• Ultrasonographic findings of bilateral hydronephrosis, dilated bladder, thickened bladder
wall, and a dilated posterior urethra in male patients are highly suggestive of underlying
PUV.
• Voiding cystourethrogram demonstrates the characteristic findings of a dilated and elongated
posterior urethra during the voiding phase (after catheter removal).
• Ablation of urethral valves during cystoscopy is the preferred initial surgical approach of
PUV ablation.
Suggested Readings
• Elder JS. Obstruction of the urinary tract. In: Kleigman RM, Stanton BF, St Geme JW III,
Schor NJ, Behrman RE, eds.Nelson Textbook of Pediatrics. 20th ed. Philadelphia, PA:
Saunders Elsevier; 2015:2567-2575.
Question 47
A 9 month-old infant who attends childcare presents with a 2-day history of vomiting and
copious, nonbloody, watery diarrhea. He is breastfed and receives puréed food 3 times per day.
He has not had any new food exposures in the week prior to developing the symptoms. Several
other children in his childcare have similar symptoms. On admission to the hospital, his vital
signs are a temperature of 380C, heart rate of 140 beats/min, and respiratory rate of 30
breaths/min. He is pale and has decreased energy. Abdominal examination demonstrates
hyperactive bowel sounds. His mother is concerned by his decreasing interest in eating.
Of the following, the laboratory stool study MOST sensitive to diagnose the cause of this infant's
diarrhea is
Rotavirus infection begins with acute onset of vomiting and fever, followed 1 to 2 days later with
watery diarrhea. Diarrhea is typically mild to moderate. Symptoms last 3 to 8 days. Severe cases
can develop significant dehydration, acidosis, and electrolyte abnormalities. A small number of
children experience neurologic issues including seizures and less commonly, encephalitis,
encephalopathy, and cerebellitis.
Rotaviruses are from the Reoviridae family and are segmented, double-stranded RNA viruses
with at least 7 different antigenic groups (A through G). Group A is the most common
worldwide. Testing is rarely completed, as it has no influence on the course of the disease. The
virus can be detected by enzyme-linked immunosorbent assay (ELISA), latex agglutination,
electron microscopy, electrophoresis, and reverse transcriptase polymerase chain reaction (RT-
PCR). Polymerase chain reaction techniques have been shown to be more sensitive in detection
of the virus at all stages of infection and are the primary tool used in research. Despite this,
ELISA vs used most often in clinical practice because of the ease of testing and affordability, in
addition to the high sensitivity and specificity. Reducing substances would be nonspecific as to
the cause of the diarrhea. Viral cultures are time consuming and are not generally used to
diagnose the cause of the acute vomiting and diarrhea seen in the patient described in the
vignette.
PREP pearls
• Rotavirus is leading cause of diarrhea worldwide, with decreasing prevalence due to
vaccinations.
• Vaccinations are safe and effective.
• Enzyme-linked immunosorbent assay testing is available clinically, is affordable, and has
high sensitivity and specificity, although polymerase chain reaction is the most sensitive
method to detect rotavirus, it is primarily a research tool.
Suggested Readings
• gernstein Di. Rotavirus overview. Pediatr Infect DuisJ. 2009 ,
http:/dx.doi.org/10.1097/INF.0b013e3181957bee.
Question 48
A 14-year-old adolescent presents to your office accompanied by her mother who is concerned
about behavioral difficulties and mild intellectual disability. The mother reports that her child has
attention problems, depression, impulsiveness, and occasional delusional thinking. She states that
her daughter is paranoid that "people are always talking about her." The patient's medical history
is significant for a large ventricular septal defect repaired at 4 months of age and recurrent
infections. She had significant hypocalcemia and low parathyroid hormone with seizures in
infancy. She has mild bilateral sensorineural hearing loss. Physical examination is remarkable for
a thin teenager with slender, hyperextensible hands and fingers. She has a long narrow face with
a narrow nose with a squared nasal root, short upward-slanting palpebral fissures, a bifid uvula,
and a high arch palate. She has a mild pectus excavatum present with a linear vertical scar to her
chest. She has a short, thick webbed neck with reduced range of motion and mild scoliosis. She
does have unusual mannerisms and affect.
The heart disease often manifests as conotruncal defects including tetralogy of Fallot,
perimembranous ventricular septal defect, truncus arteriosus, or interrupted aortic arch. The
palatal deformities can range from velopharyngeal incompetence to cleft palate. Additional
clinical findings can include feeding and swallowing problems, gastrointestinal and
laryngotracheoesophageal anomalies, hearing loss, growth hormone deficiency, seizures, central
nervous system anomalies, skeletal abnormalities (club feet, scoliosis, vertebral anomalies), renal
abnormalities, ophthalmologic problems, thyroid problems, psychiatric disorders, autism, and
enamel hypoplasia.
Diagnosis of 22q11.2 deletion syndrome can be made with fluorescence in situ hybridization
(FISH) analysis for the submicroscopic deletion of chromosome 22 or a chromosomal
microarray. 22011.2 deletion syndrome is a contiguous gene deletion syndrome. A contiguous
gene deletion syndrome is caused by a microdeletion that encompasses 2 or more genes in
tandem position along a chromosome. By virtue of the fact that several genes are involved,
contiguous gene syndromes often impact multiple systems of the body. Thus, one must assess the
function of the specific genes involved within the deletion and thoroughly examine the patient
for involvement for those specified regions (heart, kidney, brain, etc). Other common contiguous
gene deletions include Williams syndrome, Ip36 deletion, Smith-Magenis syndrome, and Cri-du-
chat syndrome.
Treatment for 22q11.2 deletion syndrome entails a multidisciplinary approach specific to the
individual's needs by a combination of specialists that may include allergy, audiology,
cardiology, cardiac surgery, developmental/behavioral specialists, otolaryngology,
endocrinology, pediatric dentistry, gastroenterology, immunology, medical genetics, neurology,
ophthalmology, orthopedics, plastic surgery, psychiatry, pulmonology, rheumatology,
hematology, urology, speech pathology, and early intervention.
CHARGE syndrome is an autosomal dominant disorder often caused by CHD7 gene mutations
that can present with a combination of the following findings: coloboma, heart defects, choanal
atresia, retarded growth and development, and genital and ear anomalies. This patient lacks the
coloboma or choanal atresia findings commonly seen.
PREP Pearls
• 22q11.2 deletion presents with a combination of clinical findings including congenital heart
disease, palatal abnormalities, characteristic facial dysmorphology, learning difficulties,
hypocalcemia, and immune deficiency.
Question 49
The parents of a previously healthy 9-month-old male infant bring him to your office for follow-
up after a 3-day hospitalization for bronchiolitis. In the hospital, the infant received oxygen and
intravenous hydration. He is much better now, but still having a cough. On physical examination,
you note diffuse rales, but no respiratory distress.
Infants and young children with bronchiolitis are at increased risk for recurrent wheezing,
particularly during the first decade of life. Studies have found this risk to range from 2 to 4 times
that of controls. Also, some studies have shown a correlation between bronchiolitis, specifically
RSV infection, in infancy or early childhood and the subsequent diagnosis of asthma. The nature
of this association is uncertain and causality cannot be determined. This correlation may be
reflective of the multifactorial nature of risk for asthma: genetic predisposition, environmental
aeropollutants, atopy, immunologic of the multifactorial nature of risk for asthma: genetic
predisposition, environmental aeropollutants, atopy, immunologic mechanisms, airway growth
affected by prior infection, and inflammation, In one study, non-RSV etiology compared with
RSV etiology of bronchiolitis before 2 years of age significantly increased asthma risk in
adulthood.
PREP pearls
• Infants who have an episode of bronchiolitis have a 2 to 4 times greater risk for recurrent
wheezing than controls, particularly during the first decade of life,
• Some studies have shown a correlation between bronchiolitis in infancy or early childhood
and the subsequent development of asthma.
Suggested Readings
• Piippo-savolainen E, Korppi M, Korhonen K, Remes S. Adult asthma after non-respiratory
syncytial virus bronchiolitis in infancy: subgroup analysis of the 20-year prospective follow-
up study. Pediatr Int. 2007 001:
Question 50
A 3-month-old infant presents to your practice with a physical examination remarkable for the
finding shown in Item Q50.
Although hemangiomas can regress with time, it can take many months or even years. As this
lesion covers the eye and may obstruct the light pathway, waiting for spontaneous resolution
may lead to permanent vision loss in that eye. Reassurance alone is therefore not the correct
management.
PREP Pearls
• An obstruction of the pathway of light from the pupil to the retina in a young child can lead
to permanent vision loss in that eye, called amblyopia.
• Strabismus occurs when a child develops exotropia or esotropia and can lead to amblyopia.
• Any time there is concern for the development of amblyopia, an immediate referral to an
ophthalmologist should be considered.
Suggested Readings
• Gunton KB. Advances in amblyopia: what have we learned from PEDIG trials? Pediatrics.
2013;131(3):540-547. DOI:http://dx.doi.org/10.1542/peds.2012-1622.
Question 51
You are seeing an 11-year-old girl in the sixth grade who has been having recent trouble in
school. Her school performance was described as “okay” until she started fourth grade. Since
that time, her grades are described as having been “poor.” Her early motor skill development was
normal, although the mother states that she was “a little slow” in acquiring language in the first 2
years after birth. Her mother adds that she has not been concerned about her speech since the girl
started kindergarten. Last year, her parents arranged for the girl to have an IQ test, which
revealed intelligence in the average range. She does not have many friends and is not a
troublemaker. There are frequent conflicts around a rule that she completes her homework before
she is allowed to watch television at night. In the office, the girl’s speech sounds normal, she
smiles, is pleasant, and seems to interact normally for her age.
Of the following, the evaluation MOST likely to reveal the cause of her poor school performance
is
A. audiology testing
Children with learning disabilities can have normal range or high overall IQ. In the past, a lower
academic performance relative to what one might predict from IQ was considered to be the
definition of a learning disability, but that is no longer the case. Response to intervention (RTI) is
now the standard approach because this helps to screen out those with poor performance due to a
lack of adequate instruction. The RTI involves a psychoeducational assessment, followed by
increasing levels of instruction in areas of deficit then reassessment. If a child’s performance
normalizes with minimal intervention, this would argue against having a learning disability.
Another common cause of schooling difficulties that may not appear until after academic
demands increase would be inattentive type attention-deficit/hyperactivity disorder, but it is a bit
less likely here because of a lack of parental reports of attention problems. It is also not one of
the options listed.
An audiology test would be key to perform had her speech failed to normalize after the initial
reported delay in speech development, and such testing may still be reasonable as part of health
maintenance. However, it is not very likely to be the cause of her current poor school
performance.
Serum ferritin level would be helpful in assessing for iron deficiency. Chronic iron deficiency
does produce neuropsychiatric effects that include lower average math scores and impaired
psychomotor development. Ten years of age is not a peak time for this diagnosis, however, and it
is less likely to be found in this scenario than a learning disability. A serum lead level would also
be appropriate to assess in a young child with developmental concerns because of its potential
neurotoxicities. However, as with iron deficiency, 10 years of age is older than when one would
typically expect to find this as an abnormality in the absence of persistent pica. A Wood lamp
skin examination can help to detect ash leaf spots that are frequently found in tuberous sclerosis.
While tuberous sclerosis is associated with intellectual disabilities, it is not a likely diagnosis.
PREP Pearls
• Children with learning disabilities can have normal range or high overall IQ.
• Response to intervention is now the standard approach for learning disability assessment
because this helps to screen out poor performance due to a lack of adequate instruction.
Suggested Readings
• Dworkin PH. School learning problems and developmental differences. In: McInerny TK,
Adam HM, Campbell DE, Kamat DM, Kelleher KJ, eds. American Academy of Pediatrics
Textbook of Pediatric Care. Elk Grove Village, IL: American Academy of Pediatrics; 2009.
Question 52
A 13-year-old adolescent with trisomy 21 who underwent surgery at 4 months of age for an
atrioventricular canal defect is brought to your office for a school physical examination. He has
not been seen by you for 4 years. He has not been on any medications. He is in special education
class and would like to participate in the Special Olympics. On review of symptoms, his mother
states that he has not had the same level of energy for the last 2 weeks. He has been sleeping
during the day on the weekend and has been too tired to do his school work.
On physical examination, his heart rate is 56 beats/min, his respiratory rate is 32 breaths/min,
and his blood pressure is 92/45 mm Hg. He has mild intercostal retractions. His pulse oximetry is
90% on room air. He is alert but not talkative today. His chest examination shows decreased
breath sounds bilaterally with crackles. His cardiac examination reveals a slow regular heart rate
with a 2/4 diastolic murmur and a 2/6 systolic murmur at the left midclavicular line at the fourth
intercostal space. No hepatosplenomegaly or lymphadenopathy is noted.
Given the combination of history and physical findings, the MOST likely etiology of his current
fatigue is
A. hyperthyroidism
B. leukemia
C. mitral stenosis
D. pericardial effusion
Patients with trisomy 21 have a 40% to 50% risk of congenital heart disease (CHD). Of the
patients with CHD, 40% to 50% will have an endocardial cushion defect or atrioventricular
septal defect (AVSD), commonly called atrioventricular (AV) canal defects. There are many
variations on the anatomy of AV canal defects. Some children will have a large atrial septal
defect (ASD) component (called a primum defect, where the lowest portion of the atrial septum
is missing) and almost no ventricular component, with a cleft mitral valve. Other children will
have a huge ventricular septal defect (VSD), as well as an ASD. The ASD is not the type that can
close spontaneously. The VSD in an AV canal defect is classified as an inlet VSD and will not
close spontaneously. The degree of flow or shunt across the defect can be very large or minimal,
depending on the exact architecture of the AV valve or valves.
The natural history of infants with AVSDs is to develop symptoms of pulmonary overcirculation
or high output heart failure within the first few months of life. The timing will be dependent on
the size of the defect and the pulmonary vascular resistance. If no heart failure develops at
several months of age, this would raise concern for pulmonary hypertension. Another
confounding issue for patients with trisomy 21 is their increased risk for hypothyroidism.
Follow-up with genetics and early intervention is very important for these patients. After initial
repair of an AVSD, children require ongoing monitoring for development of atrioventricular
valve dysfunction. The mitral valve often becomes regurgitant and may become stenotic,
requiring subsequent repair or replacement later in life.
Children born with trisomy 21 may also have increased risk for leukemia and hypothyroidism. In
this patient, there is no bruising, hepatosplenomegaly, or lymphadenopathy, making leukemia
less likely as a cause of the child’s fatigue. There is an increased risk of pericardial effusion,
especially if there is hypothyroidism in patients with trisomy 21, but the physical examination is
not consistent with that. The murmur suggests atrioventricular valve insufficiency and stenosis,
rather than tricuspid valve findings.
Children born with congenital heart disease, even when initial repairs are done and are
successful, require ongoing monitoring over their lifetime. Patients with AVSD repairs often
require further surgery to address their mitral valve. Patients with tetralogy of Fallot after initial
repair will require ongoing monitoring for pulmonary insufficiency and need for pulmonary
valve replacement as they age.
Patients with single ventricle physiology such as hypoplastic left heart syndrome will require a
minimum of 3 surgeries to complete palliation. The initial surgery for hypoplastic left heart
syndrome is called the Norwood procedure to establish a systemic outflow tract utilizing the
American academy of pediatrics 165
American Academy of Pediatrics PREP 2016
pulmonary artery and then creating a source of pulmonary blood flow. The second surgery is the
Glenn anastomosis, which creates an attachment of the superior vena cava to the right pulmonary
artery. The third surgery is called the Fontan procedure, which creates (passive) circulation from
the inferior vena cava to the pulmonary artery. This circulation is passive, therefore any disease
process that increases the pulmonary vascular resistance, such as pneumonia, can cause a severe
decrease in cardiac output in these patients.
PREP Pearls
• Atrioventricular canal defects are common in trisomy 21 patients.
• The natural history of infants with atrioventricular septal defect is to develop symptoms of
pulmonary overcirculation or high output heart failure within the first few months of life.
• Ongoing monitoring is essential in any child who has undergone repair of congenital heart
disease, especially for complex lesions. Additional procedures may be needed in later
childhood or adolescence.
Suggested Readings
• American Academy of Pediatrics. Health supervision for children with Down syndrome.
Pediatrics. 2001;107(2):442-449. DOI: http://dx.doi.org/10.1542/peds.107.2.442.
Question 53
You are seeing a 7-year-old boy in your office for a health supervision visit. The boy has asthma
that is poorly controlled with his maintenance medication regimen of inhaled fluticasone and
montelukast, with albuterol as a rescue medication. He has seasonal allergic rhinitis to grass that
partially responded to nasal steroids last spring. The boy has no other medical problems. The
allergist has recommended subcutaneous immunotherapy.
Of the following, the MOST appropriate advice regarding this recommendation is that the boy
should
Specific immunotherapy (SIT) is useful for allergic rhinitis, and is most effective in a narrow
range of allergens. Patients should be selected for SIT based on history and allergy testing, and
after other causes of nasal symptoms have been ruled out. The effectiveness of SIT for allergic
rhinitis in drug-resistant cases is well documented. Three years of treatment can provide long-
lasting effects, whereas the benefits of only a single year of therapy diminish quickly. The risks
and cost effectiveness of SIT for allergic rhinitis should be assessed on a case-by-case basis.
Some patients with atopic dermatitis may benefit from SIT, with a decrease in the number and
severity of atopic dermatitis events.
Specific immunotherapy affects the natural history of allergic disease. Allergic children start
with a limited range of allergen sensitivities that progress over time. SIT may limit the tendency
to acquire new sensitivities. A proportion of persons with allergic rhinitis will develop asthma
every year, and SIT may be able to decrease this number.
Although current asthma treatments suppress inflammation and relieve bronchospasm, they are
not curative. Allergen avoidance can be of benefit in allergic asthma, but is not always possible.
SIT can be beneficial in allergic asthma, reducing symptom scores, the number of exacerbations,
and the need to increase medications. However, it is important to recognize that the majority of
fatal reactions with SIT have occurred in patients with asthma. Although asthma is not an
absolute contraindication, SIT should be used with caution in patients with uncontrolled asthma
or those with reduced peak flow rates.
Specific immunotherapy is contraindicated in patients with cardiac disease. Other conditions that
should induce caution when using SIT include autoimmune conditions, immunodeficiency
syndromes, and malignant disease. There is no direct evidence that SIT is harmful in these
conditions, but manipulating the immune system in these patients could theoretically cause
exacerbations or worsening of their underlying disease. ACE-inhibitors and β-blockers are
contraindications to using SIT. β-blockers decrease the response to epinephrine that may be
needed to combat adverse reactions to SIT. Angiotensin converting enzyme blockers can
accentuate angioedema. A large local reaction may occur during SIT, but is not a
contraindication to therapy. The most common intervention for a large local reaction is
observation.
The medical indications for immunotherapy include venom hypersensitivity, atopic dermatitis,
allergic rhinitis, and asthma. Approximately 40 deaths occur each year in the United States
because of anaphylactic reactions to Hymenoptera venom. Although it is common to have some
venom-specific IgE antibodies a few months after a sting, a few patients will have high levels of
American academy of pediatrics 168
American Academy of Pediatrics PREP 2016
IgE for years. This group of individuals, with persistently high IgE levels, is at risk for
anaphylaxis with future envenomation. In addition, patients with systemic symptoms after a
Hymenoptera sting are at higher risk for future systemic reactions, with this risk decreasing over
time. The risk of systemic reaction is over 17% after 10 years in individuals with a previous
systemic response compared with a 2% to 3% risk of systemic response in the general
population. Desensitization with venom will reduce the risk of systemic reaction to
approximately 10% after completion of therapy.
Patients should be observed for 30 minutes after each injection, longer observation is not
required. All patients receiving SIT should have injectable epinephrine for home use in case of a
delayed reaction, regardless of how long they are observed. Patients begin SIT with weekly
injections until a maintenance dose is achieved, and then receive monthly or 6-week maintenance
injections for 3 to 5 years. Use of SIT should not be limited to the allergy season.
PREP Pearls
• Specific Immunotherapy (SIT) is effective in treating allergic rhinitis, asthma, and venom
hypersensitivity.
• All patients receiving SIT should have injectable epinephrine available for home use in case
of a delayed hypersensitivity reaction.
• Although SIT is not contraindicated in asthma, it has been associated with fatal reactions in
patients with asthma and should not be used in patients with poorly controlled asthma.
Suggested Readings
• Frew AJ. Allergen immunotherapy. J Allergy Clin Immunol. 2010;125(2):S306-S312.
DOI:http://dx.doi.org/10.1016/j.jaci.2009.10.064.
Question 54
A 6-year-old boy presents to your office for evaluation of a limp. His parents have noted a left-
sided limp for the past 3 to 4 months. At times, the boy reports activity-related anterior thigh
pain. There is no history of antecedent infection, trauma, recurrent fevers, change in appetite, or
unusual rashes. On physical examination, you note decreased internal rotation and abduction of
the left hip.
B. Legg-Calvé-Perthes disease
C. septic arthritis
Legg-Calvé-Perthes disease occurs when an inadequate blood supply to the femoral head
epiphysis leads to synovitis and early necrosis (initial stage), resulting in collapse of the femoral
head (fragmentation stage). The femoral head reossifies in the healing phase, but may not retain
a spherical shape and therefore may not fit well in the acetabulum.
Legg-Calvé-Perthes disease has a male predominance and 90% of cases are unilateral. The peak
incidence occurs in children aged 4 to 8 years. Children typically present with a history of limp
and mild activity-related pain. On physical examination, there is decreased internal rotation and
abduction of the affected hip(s).
In patients with LCP, anterior-posterior (AP) and frog-leg lateral radiographs show collapse of
the femoral head. The degree of collapse along the lateral aspect of the femoral head is used to
grade LCP; this is known as the lateral pillar classification. For patients with negative
radiography findings, in the setting of a high degree of clinical suspicion for LCP, a technetium
bone scan can be used to detect a deficient blood supply to the femoral head.
The treatment of LCP depends on a patient’s age and the severity of radiographic findings.
Children who present with symptoms before 6 years of age tend to have a better prognosis and
treatment is often nonsurgical. Physical therapy can help maintain adequate hip range of motion,
and nonsteroidal anti-inflammatory medications can be taken as needed for pain. Children older
than 8 years who develop LCP tend to do poorly and generally require surgical intervention.
Children with LCP should avoid high-impact physical activities.
Juvenile idiopathic arthritis (JIA) is an unlikely diagnosis for this boy, because he has no
morning stiffness and his pain worsens with activity. Additionally, solitary hip involvement is
uncommon with JIA. Septic arthritis is rapidly progressive and typically causes severe pain,
fever, and refusal to bear weight. Boys with slipped capital femoral epiphysis typically present
between the ages of 11 and 15 years, and the majority are overweight or obese. Transient
synovitis has a short duration of symptoms, typically less than 1 week.
PREP Pearls
• Children with Legg-Calvé-Perthes disease generally present with chronic limp and may have
activity-related pain.
• Pain or a decrease in range of motion with hip internal rotation and/or abduction generally
indicates intraarticular hip pathology.
Suggested Readings
• Sarwark JF, LaBella C. Pediatric Orthopaedics and Sports Injuries: A Quick Reference
Guide. Elk Grove Village, IL: American Academy of Pediatrics; 2014.
Question 55
A previously healthy 13-year-old adolescent presents to your office with a facial rash that began
2 days after the onset of fever and myalgias. He reports that the involved skin is swollen and
painful. On physical examination, his temperature is 38°C, heart rate is 100 beats/min,
respiratory rate is 20 breaths/min, and blood pressure is 115/68 mm Hg. He is ill appearing, but
does not appear toxic. You note a large, raised, tender area of nonfluctuant induration and
erythema with clear borders over the left cheek. His mucous membranes are moist. The
remainder of the physical examination is unremarkable.
Of the following, the MOST appropriate antimicrobial therapy for this patient is
A. intravenous ceftriaxone
B. intravenous vancomycin
C. oral amoxicillin
D. oral doxycycline
E. oral trimethoprim/sulfamethoxazole
Group A Streptococcus is transmitted through respiratory tract secretions or direct contact with
the affected area (eg, impetigo). Environments with close crowding facilitate spread of the
organism. Group A Streptococcus pharyngitis is more common in the late fall, winter, and spring
in temperate climates, whereas pyoderma occurs more commonly in tropical regions and during
warm seasons. The incidence of invasive GAS diseases is highest in infants and the elderly. In
the United States, the development of acute rheumatic fever is rare, presumably because of the
decreased circulation of rheumatogenic strains. The incubation period for GAS pharyngitis is 2
to 5 days, whereas impetigo develops 7 to 10 days after acquisition of the organism.
A swab of both tonsils and the posterior oropharynx for rapid antigen testing and bacterial
culture are recommended for the diagnosis of GAS pharyngitis. The bacterial culture swab
should be sent in all cases of negative rapid antigen testing, because of the possibility of false-
negative results. For nonpharyngitis infections, routine bacterial cultures of blood and other
sterile sites or tissues will isolate the organism. The diagnosis of acute rheumatic fever should be
made using the Jones criteria (Item C55). Streptococcal toxic shock syndrome requires the
isolation of GAS and a myriad of clinical findings to include hypotension and a minimum of two
of the following additional presentations: an erythematous macular rash, severe soft tissue
infection, adult respiratory distress syndrome, a coagulopathy, elevated liver enzymes or
increased bilirubin, and increased creatinine levels.
Penicillin is the drug of choice for the treatment of GAS infection; amoxicillin is an appropriate
oral alternative given its palatability. An infection caused by a strain of GAS resistant to either
penicillin or a cephalosporin has never been documented. In patients with severe penicillin
allergy, alternative agents for the treatment of GAS infection include clindamycin, azithromycin,
erythromycin, and clarithromycin. Clindamycin is useful for severe infections because it has a
long postantibiotic effect and inhibits the production of bacterial toxins; however, it should never
be used alone in life-threatening infections because of the possibility of GAS-resistant strains.
Vancomycin is appropriate for the empirical treatment of invasive infection caused by gram-
positive organisms and for definitive therapy for severe infection (eg, endocarditis) caused by
methicillin-resistant S aureus, but is unnecessary for the treatment of erysipelas or other
infections caused by GAS. Tetracyclines (eg, doxycycline) and sulfonamides (eg, trimethoprim-
sulfamethoxazole) are not useful for the treatment of GAS disease.
PREP Pearls
• Acute pharyngotonsillitis is the most common presentation of group A streptococcal disease.
• Erysipelas can present as a red, raised “butterfly” appearing rash on the face with a clear line
of demarcation between involved and uninvolved areas. Ceftriaxone is used to treat
erysipelas in moderately ill patients.
• Penicillin is the drug of choice for the treatment of infection caused by group A
Streptococcus.
Suggested Readings
• American Academy of Pediatrics. Group A streptococcal infections. In: Kimberlin DW,
Brady MT, Jackson, MA, Long SS, eds. Red Book: 2015 Report of the Committee on
Infectious Diseases. 30th ed. Elk Grove Village, IL: American Academy of Pediatrics;
2015:732-734.
Question 56
A 17-year-old, previously healthy adolescent is brought to the emergency department with
complaint of an acute change in mental status and abdominal pain. His father heard him entering
the house at approximately 4 AM. Upon speaking with him, the father suspected that his son
“was high on drugs.”
Physical examination shows a diaphoretic, delirious, and agitated teenager, who is repetitively
wringing his hands and writhing in bed. The patient can tell you his name, but when asked where
he is, he states he is “at the concert.” He does not consistently follow commands. His vital signs
include a heart rate of 134 beats/min, respiratory rate of 24 breaths/min, blood pressure of 152/96
mm Hg, axillary temperature of 38.5°C, and pulse oximetry of 98% on room air. There are no
obvious signs of traumatic injury. The boy’s pupils are 5 mm in diameter with sluggish reactivity
bilaterally. His mucous membranes are moist. His lungs are clear with no signs of respiratory
distress. His cardiac examination reveals tachycardia with a regular rhythm. His pulses are
bounding and symmetric. His abdomen is soft and nontender with no bruising and hyperactive
bowel sounds. Neurologic examination reveals normal strength and brisk reflexes in all
extremities. The patient is agitated, pulling at his intravenous tubing, and will not lie still,
ignoring your attempts to calm and redirect his behavior verbally.
Of the following, the BEST option for treating this patient’s symptoms is
A. intramuscular haloperidol
B. intravenous dantrolene
C. intravenous diazepam
E. oral propranolol
Illicit cocaine use by adolescents and adults is relatively widespread, and small children may also
be exposed inadvertently (or even intentionally) to cocaine by others. Pediatric providers must be
able to recognize the clinical findings associated with acute cocaine intoxication and manage
these cases appropriately.
Signs and symptoms in patients with cocaine toxicity arise from stimulation of the sympathetic
nervous system. Characteristic findings include central nervous system (CNS) excitation (which
may manifest as euphoria, agitation, delirium, hyperactivity), dilated pupils, tachycardia,
hypertension, and diaphoresis. Patients may experience headache, myalgias, and abdominal pain;
in the most severe cases, focal neurologic symptoms, intracranial hemorrhage, myocardial
ischemia, seizures, and even coma may result.
Patients with mild toxicity from cocaine abuse or exposure generally require supportive therapy
only. For patients with moderate or severe agitation, as well as for those with mild to moderate
hypertension, benzodiazepines (including diazepam) are the agents of choice. Benzodiazepines
would also be the first-line agents for initial treatment of seizures related to cocaine toxicity.
Benzodiazepines are the first line treatment for both the CNS overstimulation and cardiovascular
sequelae arising from cocaine’s toxic effects. However, additional antihypertensive medications,
such as phentolamine or sodium nitroprusside may be required in patients with severe or
refractory hypertension. Active cooling measures should be initiated for those with severe
hyperthermia. Fluid therapy with urine alkalinization may be necessary for patients with
rhabdomyolysis precipitated by cocaine abuse. Activated charcoal should be strongly considered
for gastric decontamination in cases involving oral ingestion.
Haloperidol is an antipsychotic medication that has long been used for sedation of patients with
acute agitation or delirium, especially those with a history of psychiatric disorders including
schizophrenia. Although haloperidol may effectively decrease agitation in the patient in the
vignette, it can reduce the body’s heat-dissipating capacity and lower the seizure threshold.
Therefore, haloperidol would not be the safest agent for treating agitation in a patient with
cocaine toxicity, which can also be complicated by hyperthermia and seizures. Benzodiazepines
are generally the preferred agents for treating agitation in patients with drug overdose.
Dantrolene is used to treat muscular rigidity caused by malignant hyperthermia and neuroleptic
malignant syndrome. It does not have an established role in the treatment of acute cocaine
intoxication.
In the clinical scenario of drug intoxication, sodium bicarbonate is used to treat cardiovascular
toxicity secondary to tricyclic antidepressant overdose. It may also be added to intravenous fluids
to achieve urine alkalinization, which can increase the rate of elimination of many poisons from
American academy of pediatrics 178
American Academy of Pediatrics PREP 2016
the body. Sodium bicarbonate would not be the initial agent of choice to treat agitation and
hypertension in a child with cocaine toxicity.
The use of β-blockers such as propranolol is generally not recommended in the treatment of
cocaine-related cardiovascular sequelae, including hypertension. These agents may cause
unopposed stimulation of alpha-adrenergic receptors, which could contribute to increased
coronary vasoconstriction and end-organ ischemia.
PREP Pearls
• The predominant clinical effects seen in children and adolescents with cocaine intoxication
include central nervous system excitation, hypertension, tachycardia, hyperthermia,
diaphoresis, and dilated pupils.
• Benzodiazepines are the agents of choice for patients with mild to moderate cocaine toxicity,
as well as for those with seizures related to cocaine overdose.
• For children and adolescents with acute cocaine intoxication, initial management must focus
on immediate stabilization of the airway, adequate ventilation, and prompt establishment of
vascular access to support circulatory function.
Suggested Readings
• Gummin DD, Aks SE. Cocaine. In: Erickson TB, Ahrens WR, Aks SE, Baum CR, Ling LJ,
eds. Pediatric Toxicology: Diagnosis and Mangement of the Poisoned Child. New York, NY:
McGraw Hill; 2005;chap 57:392-397.
Question 57
A mother who received no prenatal care has just delivered a male newborn who weighs 2,100 g.
Although her menses were irregular, the gestational age is estimated to be 39 weeks by her last
menstrual period. The newborn’s physical examination includes pink skin with some cracking
pale areas with rare veins, thinning lanugo on the back, well-curved pinnae with soft but ready
recoil, a stippled areola with a 1 to 2 mm breast bud, testes just below the inguinal ring, and an
anterior crease on the plantar surface of the foot.
Of the following, the MOST appropriate description of this newborn is
The Dubowitz scoring system was developed in 1970 to estimate gestational age based on
neuromuscular and physical findings. This scoring system required modification, with the
increased survival of extremely premature infants, thus leading to the development of the New
Ballard Score (NBS) (Item C57). The NBS has been validated in newborns between 20 and 44
weeks’ gestational age and found to be accurate to within 2 weeks. Neurologic examination of
posture and physical examination of the genitalia have the highest reliability.
Accurate assessment of gestational age and size for gestational age are important, because this
information will guide the provision of safe and appropriate newborn care. Newborns born
between 34 0/7 weeks and 36 6/7 weeks’ gestational age are considered late preterm and require
close monitoring for issues of respiratory control, feeding skills, and temperature regulation.
Full-term small-for-gestational age newborns have an increased risk of hypoglycemia and
hypothermia, whereas newborns who are large for gestational age should be monitored for
hypoglycemia.
The infant in the vignette has the neuromuscular and physical findings of a late-preterm infant,
including testes just below the inguinal ring, a single plantar crease, and immature posture with
decreased flexion. His weight of 2,100 g is consistent with an appropriate-for–gestational age
late preterm infant.
PREP Pearls
• Accurate assessment of gestational age and size for gestational age are important to guide
safe and appropriate newborn care.
• The New Ballard Score is used to estimate gestational age based on neuromuscular and
physical examination findings in newborns between 20 and 44 weeks’ gestational age and is
accurate to within 2 weeks.
• The neuromuscular examination of posture and physical examination of the genitalia have
the highest reliability for estimation of gestational age.
Suggested Readings
• Ballard JL, Khoury JC, Wedig K, et al. New Ballard score, expanded to include extremely
premature infants. J Pediatr. 1991;119:417. Abstract available at
http://www.ncbi.nlm.nih.gov/pubmed/1880657.
Question 58
A 16-year-old adolescent is brought to the emergency department by her friends after she had a
generalized tonic-clonic seizure at a house party where no adults were present. Her friends drop
her off and leave immediately. She is arousal, but keeps falling asleep and cannot provide a
coherent history. Her physical examination reveals a temperature of 36.9°C, blood pressure of
106/68 mm Hg, heart rate of 92 beats/min, and respiratory rate of 20 breaths/min. Her physical
examination is unremarkable. Her neurologic examination is unremarkable, except for altered
mental status. Her extraocular movements are normal and she can move all extremities equally.
There is no arm dysmetria when she reaches for objects and she can sit up independently in bed.
Gait examination is deferred because of her mental status. She has another generalized tonic-
clonic seizure in the emergency department that stops before any medication is administered. To
prevent further seizures, fosphenytoin (1,500 mg) is administered intravenously. A urine
pregnancy test is negative and a computed tomography of the head is normal. Toxicology screen,
complete blood cell count, and electrolytes are normal. She is admitted to the hospital for further
monitoring. The next morning, her neurological examination shows bilateral nystagmus, brisk
reflexes, and truncal ataxia, with a wide-based and unsteady gait.
Of the following, the MOST likely cause of the change in her neurological examination findings
seen on the morning examination is
A. cerebellitis
B. fosphenytoin
C. intoxication
D. postictal state
E. subdural hematoma
Acute cerebellitis due to infection can cause ataxia and nystagmus. It is usually associated with
other infectious symptoms, such as fever or meningismus. In toddlers, acute cerebellar ataxia is a
common neurologic cause of acute ataxia. This condition is thought to be postviral and therefore
infectious symptoms can be absent. The girl in the vignette has no infectious symptoms of acute
cerebellitis and is much older than the typical patient with postviral acute cerebellar ataxia.
Alcohol intoxication causes acute cerebellar dysfunction including acute ataxia. It is unknown
whether the girl in the vignette ingested alcohol or other substances before her arrival in the
emergency department; however, her neurologic examination in the emergency department did
not show ataxia originally. A postictal state can cause ataxia, especially gait ataxia, but lateral
nystagmus is not a common postictal phenomenon. Cerebellar stroke can cause ataxia and
nystagmus, but typically the nystagmus is 1-sided (ipsilateral to the cerebellar stroke).
Development of a subdural hematoma while in the hospital overnight is very unlikely, and
presentation with cerebellar signs is uncommon. Hemiparesis, headache, or signs of increased
intracranial pressure are more common signs of a new subdural hematoma. Although each of
these disorders is a possible cause, the most likely cause of acute ataxia for the girl in the
vignette is the intravenous fosphenytoin.
PREP Pearls
• Acute ataxia can be caused by medications, including phenytoin.
• In toddlers, a common neurologic cause of acute ataxia is acute cerebellar ataxia.
Suggested Readings
• Dinolfo EA. Evaluation of ataxia. Pediatr Rev. 2001;22(5):177-178. DOI:
http://dx.doi.org/10.1542/pir.22-5-177.
• Ramierz-Montealegr D, Mink, JW. Ataxias. In: Kliegman RM, Stanton BF, St. Geme JW III,
Schor NF, Behrman RE. Nelson Textbook of Pediatrics. 20th ed. Philadelphia, PA: Saunders
Elsevier; 2015: 2883-2888.
Question 59
You are called to the newborn nursery to evaluate a term newborn with ambiguous genitalia. On
physical examination, you find a well-appearing newborn in no acute distress. The physical
examination is unremarkable, except for the genital examination. The phallus is under 1 cm in
length. There is hypospadias with the urethral meatus visible at the base of the penis, a bifid
scrotum, and gonads palpable in the scrotum bilaterally (Item Q59). Family history reveals
multiple maternal aunts who are infertile.
Genitalia of the infant showing small phallus, hypospadias with bifid scrotum, and scrotal
gonads. Courtesy of Jordan Pinsker, MD.
Of the following, the MOST common cause of ambiguous genitalia in a newborn with this
clinical presentation is
B. 21-hydroxylase deficiency
C. Denys-Drash syndrome
E. Turner syndrome
The infant in the vignette has descended testes that are palpable in the scrotum. This implies that
he is an undervirilized male with a “46,XY DSD.” In addition, there is a family history of
maternal aunts who are infertile, implying the disorder is inherited in an X-linked fashion. Of the
answer choices listed that are compatible with 46,XY DSD, only androgen insensitivity causes
ambiguous genitalia and is inherited in this manner. Androgen insensitivity is the most common
underlying cause of 46,XY DSD. When a child has partial androgen insensitivity due to
incomplete expression of the mutation, the child can have ambiguous genitalia. When the
mutation causes complete androgen insensitivity, the child will appear completely female, but
will have a blind vaginal pouch and no Müllerian structures, resulting in infertility. Thus, the
maternal aunts in this family who are infertile have complete androgen insensitivity and a 46,XY
karyotype.
5-α reductase deficiency results in impairment of the conversion of testosterone to the more
potent dihydrotestosterone. This disorder can also be a cause of 46,XY DSD. However, it is not
inherited in an X-linked pattern, and occurs far less commonly than partial androgen
insensitivity.
21-hydroxylase deficiency, the most common form of congenital adrenal hyperplasia (CAH), can
affect both boys and girls, but would only cause ambiguous genitalia in a girl (46,XX DSD).
Thus, in a child with palpable gonads in the scrotum (implying the presence of Y chromosome
material leading to testicular formation), this would not be the underlying cause.
Denys-Drash syndrome results from a mutation in the WT1 gene. These children can have
ambiguous genitalia (46,XY DSD) or in some cases appear completely female. However, in boys
the testes will remain undescended (unlike the child in this vignette), and both boys and girls
develop severe kidney disease. This syndrome occurs much less commonly than partial androgen
insensitivity.
Turner syndrome, in which there is loss of all or part of 1 sex chromosome in a phenotypic
female, is characterized by growth failure and pubertal delay, amongst other findings. It is not a
cause of ambiguous genitalia.
PREP Pearls
• Ambiguous genitalia can be classified into disorders of sex development (DSD), with the
terms 46,XY DSD representing an undervirilized male and 46,XX DSD representing an
overvirilized female.
• The most common cause of 46,XY DSD is androgen insensitivity, which is inherited in an X-
linked pattern.
Suggested Readings
• Nabhan ZM, Lee PA. Disorders of sex development. Curr Opin Obstet Gynecol.
2007;19:440-445. Abstract available athttp://www.ncbi.nlm.nih.gov/pubmed/17885459.
Question 60
A 6-year-old child presents to your office for follow-up with a chief parental concern of
persistent cough. The cough started 6 weeks ago in association with a viral respiratory illness.
Associated fever and rhinorrhea have resolved, but the cough has persisted and worsened. The
cough is mucousy and worse at night. The parents believe that sputum has been produced but
swallowed. They report occasional vomiting after coughing, which has appeared mucousy. A
chest radiograph series (posteroanterior and lateral views) was obtained 10 days ago and was
read as normal without infiltrate, effusion, or air trapping.
The child is in preschool with multiple sick contacts. There is no prior history of chronic cough
or exertional intolerance. The child’s weight and height have consistently measured at the 75th
percentile. Stools are normal without grease or malodor. There is no history of recurrent ear or
skin infections. There is no history of headache or vomiting, other than as noted with the current
cough. There have been no witnessed choking episodes.
On physical examination, the child is well developed and well nourished. The patient’s
respiratory rate is comfortable and nonlabored at 20 breaths/min. Tympanic membranes are clear
without erythema. There is mucoid drainage in the posterior oropharynx. Inferior nasal turbinates
are edematous with scant mucus. Lungs are clear bilaterally without wheezing, differential
aeration, crackles, or rhonchi. Abdomen is soft and nontender. Extremities are without clubbing.
Gait is normal.
A. bronchiectasis
B. cystic fibrosis
C. immunodeficiency syndrome
D. Kartagener syndrome
E. sinusitis
The evaluation of the child with chronic cough is complex, but a complete history and physical
examination are likely to assist with reaching the proper diagnosis in a timely manner without
the need for extensive testing. The majority of coughs that last less than or equal to 8 weeks are
postviral in nature and otherwise uncomplicated.
Most children experience multiple episodes of nasal congestion and cough each year; the most
common etiology for these symptoms is an acute or resolving viral illness. Children may be
expected to have 6 to 8 viral upper respiratory infections each year. Only 5% to 13% of these
will be complicated by a bacterial superinfection. In an effort to prevent excessive antibiotic use
for viral illness, the Infectious Diseases Society of America in 2012 provided guidelines to
distinguish which clinical presentations best identify patients with acute bacterial, as opposed to
viral, rhinosinusitis (RS):
1. Onset with persistent symptoms or signs compatible with acute RS, lasting greater
than or equal to 10 days without evidence of improvement
2. Onset with severe symptoms/signs of high fever (≥ 39°C) and purulent nasal
discharge or facial pain for at least 3 to 4 days (consecutive) at beginning of illness
3. Worsening symptoms or signs characterized by new onset fever, headache, or
increase in nasal discharge following a typical upper respiratory infection that lasted 5
to 6 days and were initially improving
The young child with sinusitis may present only with a chronic cough and not with the more
classic symptoms of nasal congestion, nasal drainage, postnasal drip, or fetid breath.
A child with recurrent or chronic sinusitis should be evaluated for conditions that predispose to
recurrent infections or sinopulmonary disease, such as immune deficiency, cystic fibrosis, or
ciliary dysfunction.
The patient in this vignette has experienced 6 weeks of cough, but has otherwise been well and
thriving. The most recognizable etiologies for chronic bronchiectasis include suppurative lung
diseases (cystic fibrosis, ciliary dyskinesia), immunodeficiency, chronic dysphagia, and
aspiration. This child has a normal chest radiograph and has not demonstrated the signs or
symptoms that would suggest a comorbid condition. A child with cystic fibrosis would likely
have a chronic cough of longer duration. Failure to thrive and other symptoms of fat
malabsorption, including malodorous or greasy stools, may predominate. The child with cystic
fibrosis may also demonstrate digital clubbing.
A provider should suspect immunodeficiency in a child with recurrent skin, ear, sinus, or
pulmonary infections. A child with immune defects may also present with poor weight gain or
growth.
American academy of pediatrics 189
American Academy of Pediatrics PREP 2016
The child with Kartagener syndrome not only has ciliary dysfunction, but also situs inversus
totalis. Dextrocardia, as well as a right-sided stomach “bubble,” will likely be noted on chest
radiograph.
PREP Pearls
• Prior to initiating antimicrobials, a child with acute nasal congestion and cough should be
evaluated by existing clinical criteria to differentiate viral from bacterial rhinosinusitis.
• Children with chronic cough should undergo a thorough clinical history and physical
examination to evaluate for signs and symptoms of a chronic underlying condition.
Suggested Readings
• Chow AW, Benninger MS, Brook I, et al. IDSA clinical practice guideline for acute bacterial
rhinosinusitis in children and adults. Clin Infect Dis. 2012;54(8):e72-e112. DOI:
http://dx.doi.org/10.1093/cid/cir1043.
Question 61
A 14-year-old adolescent was recently evaluated in the emergency department for vomiting,
which has now resolved. As part of his evaluation, he had a comprehensive metabolic panel,
which was normal except for an alanine aminotransferase of 54 U/L and aspartate
aminotransferase of 70 U/L. Abdominal ultrasonography obtained at that time showed fatty
deposition in the liver. On physical examination today, you note a body mass index greater than
95th percentile for age and acanthosis nigricans. His parents ask how his condition can be most
effectively treated.
Of the following, the MOST effective initial treatment approach for this adolescent is
A. bariatric surgery
D. Metformin
E. ursodeoxycholic acid
The differential diagnosis of obesity in children includes exogenous obesity, genetic conditions,
and endocrine disorders. Exogenous obesity accounts for more than 95% of cases of excess
weight. Height velocity and pubertal development are among the most important factors to
differentiate exogenous obesity from other causes. Most children with exogenous obesity have
normal to increased height velocity and a normal pubertal progression, although often at an
earlier age than normal weight peers. The presence of short stature, dysmorphic features, and
delayed puberty should cause the pediatrician to pursue other, much rarer, causes of obesity.
Nonalcoholic fatty liver disease is a common complication of exogenous obesity and represents a
range of liver pathology from deposition of large fat droplets within hepatocytes all the way to
cirrhosis and end stage liver disease, requiring transplantation. Inflammation in the presence of
fatty deposition, with or without fibrosis, is called nonalcoholic steatohepatitis (NASH). The vast
majority (90%) of patients with NAFLD are obese, and the condition is present in up to 80% of
obese children. Boys are more commonly affected than girls, and the highest prevalence is
among Hispanic children and youth, with Hispanic adolescents more likely to develop liver
fibrosis than other ethnic groups. Despite higher rates of insulin resistance, African-American
children have low rates of NAFLD compared to other racial and ethnic groups.
Current evidence suggests that the primary metabolic dysfunction in MetS is insulin resistance.
Insulin resistance in the liver is fundamental to MetS pathogenesis. Two critical insulin-mediated
liver pathways respond differentially in the face of insulin resistance. When presented with a
glucose load, glucose homeostasis in the liver is impaired (ie, increased gluconeogenesis) and
increased de novo lipogenesis (DNL) is seen. Increased DNL leads to increased intrahepatic lipid
storage and increased release of triglyceride and apoprotein B as very low density lipoprotein,
which in turn leads to MetS-associated dyslipidemia. At the same time in the liver, lipid
metabolism is shifted toward mitochondrial β oxidation, which converts fatty acids into
American academy of pediatrics 192
American Academy of Pediatrics PREP 2016
adenosine triphosphate and ketone bodies. This also leads to elevated production of free radicals,
and the oxidative stress produced may promote inflammation characteristic of NASH.
Clinical findings in patients with NAFLD include acanthosis nigricans in 50% of children and
hepatomegaly in 40% to 50%, although this is often difficult to detect. Laboratory studies show
hyperinsulinemia and dyslipidemia, particularly hypertriglyceridemia. Elevated aminotransferase
levels, particularly alanine aminotransferase, are commonly seen and may be twice the upper
limits of normal. Ultrasonography of the liver reveals homogenously increased echogenicity, but
does not differentiate fatty infiltration from inflammation or fibrosis. Other imaging modalities
tend to be nonspecific and are not currently recommended. These biochemical and radiographic
abnormalities are neither consistently present nor predictive of disease severity, therefore further
evaluation may be required. The definitive diagnostic test for NAFLD is a liver biopsy, but the
risks of the procedure often outweigh the benefits in otherwise well children. Research is
underway to validate more specific and sensitive tests such as the pediatric NAFLD fibrosis
index (PNFI) combined with the enhanced liver fibrosis test (ELF). The PNFI is determined
from age, waist circumference, and triglyceride levels, and the ELF is made up of hyaluronic
acid, aminoterminal propeptide of type III procollagen, and inhibitor of metalloproteinase 1
levels. A measure of hepatocyte apoptosis (caspase-generated cytokeratin-18 fragments) has
been useful in adults with NASH; further work is needed to establish its validity in children. In
children with persistently elevated transaminases, further evaluation to rule out other causes of
liver disease should be undertaken, and consultation with a pediatric gastroenterologist is often
recommended.
PREP Pearls
• The primary treatment of nonalcoholic fatty liver disease (NAFLD) is gradual weight loss.
Studies show that the most effective method to achieve this is through family-based
behavioral treatment.
• Findings of NAFLD include obesity, acanthosis nigricans, hepatomegaly, dyslipidemia
(particularly hypertriglyceridemia), and elevated aminotransferase levels.
• Insulin resistance in the liver is critical to the development of metabolic syndrome.
• Nonalcoholic steatohepatitis is defined as liver inflammation in the presence of fatty
deposition.
• The imaging study of choice for NAFLD is ultrasonography, although it does not
differentiate fatty infiltration from inflammation or fibrosis.
Suggested Readings
• Bremer AA, Mietus-Snyder M, Lustig RH. Toward a unifying hypothesis of metabolic
syndrome. Pediatrics. 2012;129(3):557-570. DOI: http://dx.doi.org/10.1542/peds.2011-2912.
Question 62
You are seeing a 5-year-old girl in your office. Her mother reports that she first noticed a small
bump in the child’s “private area” about 2 months ago, and now the patient has several small
bumps in the area. Her mother reports that the child has not complained of pain. The mother
denies erythema or drainage from the lesions. Upon physical examination, the child is shy, but
cooperative. A photo of the lesions in her perianal area is shown in Item Q62.
Condylomata acuminata is primarily a clinical diagnosis; biopsy is not typically indicated. Many
cases self-resolve. Medical management includes the use of topical products such as imiquimod
or podophyllotoxin. Surgical resection or laser therapy may be needed for lesions that do not
resolve or cause symptoms. Genital warts are caused by low risk HPV types; however, in adults,
they can be a marker for carriage of high risk types. The oncogenic potential of high risk types
acquired prior to puberty is unknown. There are no current recommendations for Papanicolaou
testing or high resolution anoscopy in this scenario.
While there are recommendations that all children with condylomata acuminata be tested for
HIV, there is no recommendation for an assessment of immunologic function.
PREP Pearls
• Sexual abuse should be considered for any prepubescent child with condylomata acuminata.
• Condylomata acuminata (genital warts) is a result of human papillomavirus infection.
• Medical management includes the use of topical products such as imiquimod or
podophyllotoxin.
Suggested Readings
• Culton D, Morrell DS, Burkhart CN. The management of condyloma acuminata in the
pediatric population. Pediatr Ann. 2009;38(7):368-372.
Question 63
You are evaluating a 15-year-old adolescent for management of her acne. She has used an over-
the-counter salicylic acid wash without benefit. She has been in good health and is taking no
medications. Her physical examination reveals 5 to 6 active inflammatory lesions on each cheek
and the chin. There are several open comedones (Item Q63). There is no scarring. The forehead,
chest, and back are free of acne lesions.
Adolescent described in the vignette. Reprinted with permission from Krowchuk DP, Mancini AJ, eds.
Pediatric Dermatology. A Quick Reference Guide. Elk Grove Village, IL: American Academy of
Pediatrics; 2012.
Of the following, the MOST appropriate treatment for this adolescent’s acne is
E. a retinoid topically
Moderate inflammatory and comedonal acne on the face. There are inflammatory papules and pustules
(red arrow), open comedones (yellow arrow), and resolving inflammatory lesions (blue arrow). Reprinted
with permission from Krowchuk DP, Mancini AJ, eds. Pediatric Dermatology: A Quick Reference Guide.
2nd ed. Elk Grove Village, IL: American Academy of Pediatrics; 2012.
Several factors contribute to the development of acne. Key among these are disordered
keratinization (leading to obstruction within pilosebaceous follicles), increased sebum production
(which contributes to obstruction), and inflammation (due, in large part, to activation of the
immune system by the bacterium, Propionibacterium acnes). As obstruction increases, follicles
may rupture, contributing to the inflammatory process. In some patients, the inflammatory
process results in scarring. On the face, scars appear as small pits, while on the trunk, they are
hypopigmented macules.
Treatment plans, especially for those who have moderate or severe disease, should be designed
to impact as many aspects of the disease pathophysiology as possible. In such cases, it is
important to note that follicular obstruction is present, even if blackheads (open comedones) and
whiteheads (closed comedones) are not observed. Accordingly, for the adolescent in the vignette,
the most appropriate treatment is benzoyl peroxide (BPO)/clindamycin applied to the face each
American academy of pediatrics 197
American Academy of Pediatrics PREP 2016
morning and a retinoid (eg, tretinoin or adapalene) applied at bedtime. Therapy with BPO,
BPO/clindamycin, or doxycycline orally combined with BPO would not address the obstructive
component of her disease. In addition, because the adolescent has moderate acne limited to the
face, has no scarring, and is using no medication, an attempt to manage the inflammatory
component of her disease with topical agents is reasonable. Although topical retinoids have some
anti-inflammatory activity, the number of papules and pustules exhibited by the adolescent in the
vignette indicates the need for specific treatment of this component of her disease.
Guidelines for acne management exist, but treatment plans should be individualized based on the
patient’s perception of disease severity, past experiences with medications, and the ability to
adhere to therapy. Suggested treatment plans for mild, moderate, and severe acne are presented
in Item C63B, Item C63C, and Item Item C63D. For more detailed information, refer to the
suggested readings.
PREP Pearls
• A topical retinoid should be included in the management of adolescents with moderate or
severe acne. Obstruction within follicles is present and should be addressed, even if
blackheads and whiteheads are not observed.
• Extensive inflammatory acne (ie, involving the trunk, as well as the face) requires treatment
with an oral antibiotic.
Suggested Readings
• Eichenfield LA, Krakowski AC, Piggott C, et al. Evidence-based recommendations for the
diagnosis and treatment of pediatric acne. Pediatrics. 2013;131(Suppl 3):S163-S186. DOI:
http://dx.doi.org/10.1542/peds.2013-0490B.
Question 64
A 2-year-old girl was found face down on the surface of the family's above ground pool. She was
out of her caregiver's sight for 10 min before she was found. When she was pulled from the
water, she was motionless, blue, and not breathing. Emergency medical services (EMS) were
immediately called and when they arrived 5 min later, she was still not breathing. The EMS
personnel performed cardiopulmonary resuscitation consisting of chest compressions and rescue
breathing, with the return of spontaneous circulation within 5 min. As she was not spontaneously
breathing in the emergency department, the girl was endotracheally intubated. Upon arrival in
the intensive care unit, the girl’s vital signs were a temperature of 36°C, heart rate of 150
beats/min, respiratory rate of 20 breaths/min, and blood pressure of 100/60 mm Hg. Pulse
oximetry was 95% on 50% oxygen via mechanical ventilation. On physical examination, there
was no spontaneous movement. On painful stimulus, the girl exhibited extensor posturing, but
did not open her eyes. Her pupils were 4 mm, equal, and sluggishly reactive. There were no
external signs of trauma. Her abdomen was soft, nontender, non-distended, with no
organomegaly. Her extremities were cool with capillary refill time of approximately 3 seconds.
Of the following, the MOST appropriate next step in the management of this patient is
Drowning is an important cause of childhood morbidity and mortality. According to the Centers
for Disease Control and Prevention, more than 3,500 fatal, unintentional, non–boating related
drowning deaths occur in the United States annually. Approximately one in five people who die
from drowning are age 14 years or younger. Children ages 1 to 4 years have the highest
drowning rates, most occurring in home swimming pools. In that age group, drowning is the
second most common cause of death (the first being congenital anomalies). Risk factors for
drowning include poor swimming ability, inadequate barriers around the pool, lack of
supervision, and alcohol use (for older children and adults). Pediatric health supervision visits
should include anticipatory guidance that emphasize supervision, swimming skills, avoidance of
alcohol, and installation of appropriate barriers and alarms around home pools.
When water enters the airway, the diving reflex is stimulated, causing apnea, bradycardia, and
laryngospasm. Although laryngospasm can prevent further aspiration of water, it impairs
oxygenation and ventilation. Water or aspirated vomitus in the airspaces can cause abnormal
surfactant production and hypoxia from ventilation-perfusion mismatch, leading to
intrapulmonary shunting, poor lung compliance, and acute respiratory distress syndrome.
Ventilator management should be targeted toward recruitment of lung volume and maintenance
of oxygenation and ventilation.
Hypoxia, hypercarbia, acidosis, and the resultant decreased myocardial contractility can lead to
asphyxial cardiopulmonary arrest. Asphyxial cardiac arrest can cause death or long-term
encephalopathy, resulting from both hypoxic-ischemic and reperfusion injury. Supportive critical
care ensuring adequate oxygenation, ventilation, hemodynamics, and nutrition is recommended.
Extensive clinical trials investigating various treatments for cardiac arrest after drowning were
undertaken in the 1970s and 1980s including therapeutic hypothermia, hyperventilation,
osmotherapy, and goal-directed therapy to limit intracranial pressure. However, no evidence of
therapeutic benefit was proven. Although hyperventilation can lower intracranial pressure, it is
not recommended after cardiac arrest because it can exacerbate cerebral ischemia. There are
ongoing multicenter clinical trials investigating therapeutic modalities after pediatric cardiac
arrest.
PREP Pearls
• Drowning is most common in children ages 1 to 4 years, and usually occurs in home pools.
• Hypoxia after drowning can occur from laryngospasm, aspiration of water or vomitus, and
ventilation-perfusion mismatch.
• Neurologic outcome after cardiac arrest is dependent on the timing, quality, and duration of
cardiopulmonary resuscitation.
Suggested Readings
• Lieh-Lai MW, Sarnaik AA, Sarnaik AP. Drowning. In: Fuhrman BP, Zimmerman JJ, eds.
Pediatric Critical Care. 4th ed. Philadelphia, PA: Saunders Elsevier: 2011:1480-1488.
Question 65
An 18-month-old girl with chronic malnutrition caused by intestinal malabsorption presents to
the emergency department with fever, irritability, and left arm weakness. Vital signs show a
temperature of 38.2°C, respiratory rate of 36 breaths/min, heart rate of 138 beats/min, blood
pressure of 110/50 mm Hg, and a weight of 9 kg. On physical examination, she is irritable, has a
facial droop, and left-sided weakness and tremor.
The purified protein derivative skin test result is 14 mm of swelling. You suspect tuberculous
meningitis.
Of the following, the immune defect that BEST explains this child’s increased susceptibility to
this infection is
Overall, both lack of adequate macro- and micronutrients can be associated with immune
dysfunction and infections. Protein-calorie malnutrition has been associated with varied immune
dysfunction, including atrophy of lymphoid tissue, decreased cell-mediated immunity, decreased
immunoglobulin and complement levels, and diminished phagocytosis. Vitamin D and zinc
deficiencies have also been linked to impaired immune responses.
While malnutrition can be associated with altered innate immunity, such as decreased phagocytic
cell function, adaptive immunity is felt to be more critical in responding to intracellular
pathogens, such as mycobacteria.
Malnutrition can also alter antibody-specific responses. However, antibodies are not critical in
the control of tuberculosis infection.
Natural killer cells are a component of the innate immune system and are critical in immunity
against viral infections. Deficiency of natural killer cells is associated with increased
susceptibility to infection, especially Herpesviridae.
Immune tolerance is mediated by regulatory T cells. Decreased regulatory T-cell function can be
associated with increased autoimmune and atopic disease. Upregulation of regulatory T cells is
not associated with malnutrition.
PREP Pearls
• Malnutrition can impair both innate and adaptive immune responses, increasing susceptibility
to infections.
• Malnourished children are at high risk for progression from latent tuberculosis infection to
tuberculosis disease.
• Malnourished children can have increased lymphocyte anergy, which can limit the use of
purified protein derivatives (PPD) for screening for tuberculosis infection.
Suggested Readings
• Jaganath D, Mupere E. Childhood tuberculosis and malnutrition. J Infect Dis.
2012;206(12):1809-1815. DOI:http://dx.doi.org/10.1093/infdis/jis608.
Question 66
A 14-year-old adolescent presents to the emergency room with the chief complaint of having
cola-colored urine for 1 day. He had a mild sore throat without fever 4 weeks ago. Vital signs
show a respiratory rate of 26 breaths/min, heart rate of 110 beats/min, and blood pressure of
138/90 mm Hg. He has normal growth parameters. On physical examination, he has facial
puffiness, but the remainder of the examination is unremarkable. A urine test strip analysis
demonstrates a specific gravity of 1.015, pH of 5.5, 3+ blood, 2+ leukocyte esterase, and no
protein or nitrites.
Of the following, the MOST appropriate anti-hypertensive agent for this patient is
A. chlorothiazide
B. enalapril
C. furosemide
D. hydralazine
E. nifedipine
Facial puffiness, respiratory distress, and high blood pressure, as present in the patient in the
vignette, are indicative of volume overload. Such patients are managed with volume restriction
(two-thirds maintenance) and intravenous furosemide for achieving diuresis and net negative
fluid balance. Loop diuretics (furosemide, bumetanide, torsemide) inhibit sodium absorption via
the Na-K-2Cl channels in the medullary and cortical aspects of the thick ascending limb, leading
to excretion of up to 20% to 25% of tubular sodium. All diuretics inhibit sodium reabsorption at
different sites in the nephron, thereby increasing sodium and water losses in urine. Intravenous
furosemide (onset of action: oral, sub-lingual: 30-60 minutes; intramuscular: 30 minutes;
intravenous: approximately 5 minutes) has a rapid onset of action, and in patients with
pulmonary edema symptomatic improvement, in 15 to 20 minutes prior to the onset of the
diuretic effect has been reported.
The thiazide diuretics (chlorothiazide) have a decreased natriuretic and diuretic effect compared
to loop diuretics and inhibit the reabsorption of 3% to 5% of filtered sodium in the distal tubule.
Thiazide diuretics inhibit sodium entry via the Na-Cl cotransporter in the distal nephron.
Thiazides are not the preferred diuretics for the patient in the vignette, in view of the decreased
diuresis in comparison to loop diuretics and slower onset of action (oral, within 2 hours;
intravenous, 15 minutes). However, thiazide diuretics are preferred over loop diuretics for
chronic antihypertensive therapy and have been commonly used for management of primary
hypertension, especially in adults. The antihypertensive effects of thiazide diuretics are
incompletely understood. The initial decrease in systemic blood pressure (BP) is associated with
the reduction in plasma volume and cardiac output secondary to the diuretic effect. However, the
volume depletion is blunted because of the activation of the renin-angiotensin system in response
to hypovolemia. Long term decrease in BP with chronic thiazide therapy is associated with
partial reversal of the initial decrease in plasma volume and vasodilation, leading to decrease in
systemic vascular resistance. The factors responsible for the chronic vasodilation with prolonged
thiazide treatment remain unclear.
All calcium channel blockers inhibit the L-type calcium channels, leading to decreased influx of
calcium into cells during depolarization. The dihydropyridine calcium channel blockers act on
the vascular smooth muscles and are potent vasodilators with minimal (or no) negative effect on
cardiac contractility. Angiotensin-converting enzyme inhibitors or long acting calcium channel
blockers can be used for management of persistently elevated BP despite edema resolution, as in
high BP associated with prednisone or other immunosuppressive therapy (tacrolimus,
cyclosporine) or in chronic kidney disease. Intravenous hydralazine is most frequently used for
hypertensive emergencies in the emergency department, intensive care unit, or inpatient hospital
settings. In patients with severe acute renal failure and oliguria or anuria resistant to aggressive
diuretic therapy, intravenous hydralazine or oral nifedipine may be used for hypertensive
emergencies. Oral hydralazine is used as a third or fourth antihypertensive choice (after ACE,
calcium channel blocker, diuretics, or β-blockers) in patients with resistant chronic hypertension
requiring multiple antihypertensives for BP control.
PREP Pearls
• Edema and hypertension in patients with acute glomerulonephritis (GN) is caused by sodium
and water retention with associated renal failure.
• Loop diuretics are the treatment of choice for initially managing volume overload and
hypertension in patients with acute GN.
• Angiotensin-converting enzyme inhibitors, dihydropyridine calcium channel blockers, and
hydralazine are vasodilators, leading to decrease in systemic blood pressure.
Suggested Readings
• Brady TM. Hypertension. Pediatr Rev. 2012;33(12):541-552. DOI:
http://dx.doi.org/10.1542/pir.33-12-541.
Question 67
A 4-year-old girl presents to your office because of nose bleeds on and off for the last few
months. The child was diagnosed with extrahepatic biliary atresia at 7 weeks of age and
underwent a hepatoportoenterostomy (Kasai procedure). She is currently awaiting evaluation at a
regional liver transplant center.
The child consumes a regular diet for age, including a daily multivitamin supplement. Her height
and weight are at the 20th and 25th percentile, respectively, and a review of her growth chart
demonstrates both previously at the 25th percentile. Physical examination shows a small, active
child in no distress, icteric sclera, small amounts of crusted blood in the nares, a mildly
protuberant abdomen, a firm liver edge palpated 2 cm below the right costal margin, and a spleen
tip palpated 3 cm below the left costal margin. Blood vessels are prominently visible on her
abdomen. Laboratory studies include:
• Electrolytes, normal
• Prothrombin time, 18.0 seconds (reference 11-14 seconds)
• Partial thromboplastin time, 45 seconds (reference 23-40 seconds)
• Platelet count 150 ×103/µL (150 x 109/L)
• Hemoglobin, 10 g/dL (100 g/L)
• Calcium, 7.8 mg/dL (1.95 mmol/L)
• Bilirubin (total), 6.5 mg/dL (111.2 µmol/L)
• Bilirubin (direct) 4.0 mg/dL (68.4 µmol/L)
• Aspartate aminotransferase, 90 U/L (reference range 20-60 U/L)
• Alanine aminotransferase, 110 U/L (reference range 20-50 U/L)
• Alkaline phosphatase, 650 U/L (reference range 150-350 U/L)
Based upon these findings, the MOST likely cause of this child’s symptoms is a deficiency of
A. vitamin A
B. vitamin C
C. vitamin D
D. vitamin E
E. vitamin K
Chronic liver disease is often associated with malnutrition. This places the child at increased risk
for complications and poor outcomes following liver transplantation. The malnutrition is
multifactorial, including inadequate intake caused by anorexia associated with chronic liver
disease and malabsorption of nutrients.
Chronic liver disease can result in an impairment of the production and secretion of bile.
Malabsorption of fat in cholestatic liver disease is caused by decreased bile salts in the small
intestine. Bile is required for fat emulsification and micelle formation. Without adequate bile
salts, the digestion and absorption of fats is inadequate, resulting in gastrointestinal losses of fats.
Portal hypertension-associated vascular congestion can result in gastropathy and decreased
nutrient absorption. Small bowel bacterial overgrowth in the Roux-en-Y loop created by the
surgical procedure can be associated with bile salt deconjugation, resulting in additional fat
malabsorption.
Fat malabsorption also results in fat soluble vitamin deficiencies, each with a classic
presentation. Vitamin E deficiency is the most common, presenting with peripheral neuropathy
and hemolysis in severe cases. Vitamin D deficiency results in osteomalacia and rickets. Vitamin
K deficiency causes coagulopathy. Vitamin A deficiency is less common and is typically
associated with night blindness.
It is important to monitor the nutritional status of children with chronic liver disease. A thorough
history and physical examination to include a complete nutritional history should be completed
at every clinic visit. Growth parameters should be plotted and anthropometric measurements
serially monitored. To evaluate for fat malabsorption, a spot stool fat may identify elevated fecal
fat. Additional laboratory tests to investigate deficiencies seen in chronic liver disease are shown
in Item C67.
Fat malabsorption can be treated with supplementation of medium chain triglycerides, which
have better absorption than other fat sources and can be used to increase absorption of fats for
added calories. Vitamins A, D, E, and K are typically supplemented to avoid fat soluble vitamin
deficiencies; however, levels should be monitored to avoid toxicity.
The child in the vignette has a coagulopathy caused by vitamin K deficiency. She is also at risk
for vitamin A, D, and E deficiency; however, these are not contributing to her epistaxis. Vitamin
C is not a fat soluble vitamin and although severe deficiency can be associated with bruising or
bleeding, it is not associated with a prolongation of the prothrombin or partial thromboplastin
time.
PREP Pearls
• Chronic liver disease is often associated with malnutrition.
• Fat malabsorption is multifactorial and includes inadequate bile, malabsorption caused by
vascular congestion, and small bowel bacterial overgrowth, resulting in bile deconjugation.
• The fat soluble vitamins are A, D, E, and K. Each has a unique presentation when deficient.
Suggested Readings
• Nightingale S, Ng VL. Optimizing nutritional management in children with chronic liver
disease. Pediatr Clin North Am.2009;56(5):1161-1183. DOI:
http://dx.doi.org/10.1016/j.pcl.2009.06.005.
Question 68
You are called to the nursery to evaluate a baby with multiple anomalies and respiratory distress.
The family is from a rural community, but was told to deliver at a tertiary care center because of
suspected congenital defects. The nurse reports that the mother’s prenatal ultrasonography
revealed that the baby had an absent right kidney, a single umbilical artery, and polyhydramnios,
as well as absence of a fluid-filled stomach, a small abdomen, and intrauterine growth retardation
suggestive of a swallowing dysfunction caused by obstruction. Shortly after birth, the baby had
copious oral secretions, cough, vomiting, and intermittent respiratory distress. Physical
examination shows a cardiac murmur, imperforate anus, tachypnea, grunting, and mild subcostal
retractions. While consulting a pediatric surgeon to manage the obstructive anomaly, an
echocardiogram demonstrated a moderate ventricular septal defect.
Of the following studies, the BEST next step to establish the diagnostic cause for the multiple
anomalies is
B. chromosomal microarray
D. spine radiograph
E. TORCH titers
Most cases are sporadic with no family history of the condition. Its frequency is 1 in 10,000 to
40,000 newborns. There has been some association with maternal hyperglycemia.
Since the constellation of findings is highly suggestive of VACTERL association, a brain
magnetic resonance image and dilated eye examination would not be the best next tests. Some
VACTERL patients can have associated hydrocephalus, which should be clinically suggestive on
physical examination and would best be assessed by a head ultrasound in the neonatal period.
The first steps in evaluation of a patient should involve a thorough clinical workup to determine
the extent and type of congenital malformations. Additionally, a detailed family and medical
history is necessary. TORCH infections do not typically manifest with this combination of
findings. A chromosomal microarray and karyotype would be indicated in this situation, but
would not be a first line test in assessing the degree of systemic involvement that would be most
useful in this newborn at initial assessment for clinical management.
PREP Pearls
• VACTERL is the mnemonic for a complex nonrandom association of vertebral, anal, cardiac,
tracheoesophageal (TE) fistula, renal and limb defects that concomitantly occur in a patient,
but are not currently known to be caused by a unifying genetic defect.
• In order to be diagnosed with VACTERL association, a patient must have at least 3 of the
congenital malformations listed in the mnemonic.
• The first steps in evaluation of a suspected VACTERL patient should involve a thorough
clinical workup to determine the number and type of congenital malformations, as well as a
detailed family and medical history.
• A prenatal high risk sonography or a genetic sonogram is utilized to take a comprehensive
look at the structures of the developing fetus, including the brain, lips, face, nose, spine,
heart, abdomen, kidneys, bladder, extremities, and umbilical cord to identify birth defects
during pregnancy, ideally performed between 15- to 22-weeks of gestation.
Suggested Readings
• Shaw-Smith CJ. Oesophageal atresia, tracheo-oesophageal fistula, and the VACTERL
association: review of genetics and epidemiology. J Med Genet. 2006;43(7):545-554. DOI:
http://dx.doi.org/10.1136/jmg.2005.038158.
Question 69
You are asked to speak to a group of new mothers about the importance of breastfeeding. You
want to highlight that there are only a few absolute medical contraindications to breastfeeding. In
the case of the rare maternal infection that requires temporary discontinuation of breastfeeding,
expressed breast milk from the mother may be offered until feeding at the breast can be resumed.
Of the following, an example of this situation would include a mother who
E. receives the live attenuated rubella virus vaccine immediately after delivery
Mothers who have untreated brucellosis or those who live in the industrialized world and are
positive for human immunodeficiency virus (HIV), human T-lymphotropic virus (HTLV) 1, or
HTLV type 2 should not breastfeed or provide expressed breast milk to their infants.
Breastfeeding is not recommended for HIV-infected women in the United States where safe and
affordable nutritional alternatives are available, even when adequate treatment with antiretroviral
medications is assured. In developing countries, where mortality is increased in non-
breastfeeding infants because of malnutrition or infectious diseases, the benefits of breastfeeding
may outweigh the risk of acquiring HIV or HTLV infection.
Mothers who receive the live attenuated rubella virus vaccine after delivery may continue to
breastfeed. Although wild type strains from natural disease and vaccine strains of rubella virus
have been isolated from human milk, neither situation has been associated with significant
disease in infants.
PREP Pearls
• Feeding at the breast should be temporarily interrupted and expressed breast milk offered in
cases of perinatal maternal infection with varicella, untreated active infectious tuberculosis,
or active herpes simplex lesions on the breast.
• Mothers with untreated brucellosis should not breastfeed or feed their infant expressed breast
milk.
• In the industrialized world, mothers with HIV, HTLV 1, or HTLV 2 should not breastfeed or
feed their infant expressed breast milk.
Suggested Readings
• AAP Section on Breastfeeding. Breastfeeding and the use of human milk. Pediatrics.
2012;129(3):e827-41. DOI:http://dx.doi.org/10.1542/peds.2011-3552.
Question 70
A 2-year-old girl presents to your office for a follow-up visit after being seen in the local
emergency department. One week ago, the girl fell down a flight of 5 stairs onto a tiled floor and
hit her forehead. Her mother reported that her daughter cried immediately and was taken to the
local emergency room. She had an unremarkable neurological examination, was observed for
several hours without incident, and was discharged without any further workup.
At this visit, of the following physical examination findings, the one that MOST suggests the
need for further evaluation with a complete blood cell count is
Palatal purpura or petechiae are typically seen in the context of thrombocytopenia. Thus, any
presentation of palatal petechiae or purpura requires a complete blood cell count to assess
platelet number. If bruising occurs in excess to what would be expected in the context of a given
trauma or if bruising occurs in the absence of trauma, an assessment of the entire coagulation
system, including platelet number and function and the components of the coagulation pathways
leading to fibrin formation should be undertaken. In the assessment of nonaccidental trauma, it is
important to demonstrate that bruising or bleeding that has occurred is not a result of abnormal
coagulation, either through platelet (number or function) or fibrin deficiency or dysfunction.
The glabella is the area between the eyebrows and above the nose. It has a very loose
subcutaneous tissue and very little anchored tissue constricting the compartment. Any bleeding
into that area can lead to a significant accumulation of blood, as there is little pressure to stop the
bleeding. It would be expected that if there is a significant impact to the glabella, a large,
palpable hematoma may develop. Thus, this would be an expected finding in this scenario.
Given the large potential space of the glabella with little to constrict the movement of blood, any
accumulation of blood would follow gravity to the lowest sealed location and would be expected
to seep beneath the eyes over time. As the blood degrades, it would go through the color changes
consistent with the degradation of hemoglobin, so purple-green discoloration under the eyes
would be expected several days after the injury.
As stated previously, it is likely that the girl in the vignette would have instinctively raised her
arms and legs, so there would be expected impact to the elbows and legs, making bruising in
those locations explainable by the trauma.
PREP Pearls
• An assessment of bruising requires an understanding of the nature of the trauma, and the
parts of the body that would have been expected to experience impact.
• The glabella is the area between the eyebrows and above the nose and has very loose
subcutaneous tissue unable to easily staunch bleeding
• Any trauma to the glabella would be expected to result in a sizable hematoma.
Suggested Readings
• Anderst JD, Carpenter SL, Abshire TC. Evaluation for bleeding disorders in suspected child
abuse. Pediatrics. 2013;131(4):e1314-e1322. DOI: http://dx.doi.org/10.1542/peds.2013-0195.
Question 71
A 4-year-old girl with bilateral congenital visual impairment comes to your office for a health
supervision visit. Other than her vision, she has not had any other significant health problems.
Her hearing screens and her growth are normal. Some of her motor milestones have been
delayed, such as acquisition of a pincer grasp, head raising, crawling, and walking. She started
early intervention services at 2 years of age. At 3 years of age, she received services through the
local school district’s developmental services program. These services have focused on physical
therapy to assist her with fine and gross motor skills. She is legally blind and can only identify
light, dark, and large shapes and symbols while using corrective lenses. Her parents report
getting mixed advice about whether she should be taught to read braille and would like your
opinion.
A. academic and employment success is closely linked to the ability to read braille
B. audio reading machines of printed language allow one to read just as quickly as with
using braille
C. braille is a unique language of its own that would take her many years to learn
D. literacy is more readily achieved through the use of audio reading machines than
learning braille
E. reading magnified print will be a preferred skill for her to learn over braille
Braille is not a language, but rather a tactile coding system for letters, numbers, and punctuation.
Standard braille print takes nearly 5 times as many pages to convey the same information as
regular print, so there is also a braille contraction system called “braille 2” that is used to reduce
paper volume. The benefit of reading braille is that it is significantly faster to use than an audio-
or video-magnifying assistive technology. This reflects the difference in the time it takes for
listening to a book on tape versus reading a book.
Unemployment in the visually impaired community is a major problem. Unemployment rates for
visually impaired people who do not use braille is 77% compared to 44% for those who can read
braille. Those who can read braille are also 3 times as likely to achieve an advanced degree.
Technology has made it easy to actively convert printed or electronic information into a
refreshable tactile braille output that can be read more quickly than what is possible through
visual- or audio-assistive devices. Therefore, learning to read braille is important to promote as a
way to improve future employment and academic success.
While learning to read with a braille code takes at least as much time with skilled instruction as it
takes for anyone else to learn and read visually, it would not be correct to call braille a language
of its own. This is because the same braille system is used to encode many different written
languages. Audio reading machines translate text into spoken words, which is the skill of
listening rather than reading (reading = literacy). For those with sufficient visual ability, reading
magnified print is a helpful option, but again, people can read much more text in a unit of time
by reading braille.
PREP Pearls
• Braille is a coding system for letters and numbers rather than a language of its own.
• Visually impaired people who can read braille have more academic success and employment
success than those without that skill.
Suggested Readings
• Massof RW. The role of braille in the literacy of blind and visually impaired children. Arch
Ophthalmol. 2009;127(11):1530-1531. DOI:
http://dx.doi.org/10.1001/archophthalmol.2009.295.
Question 72
A 7-year-old boy is seen for the first time in your office. He has just come from overseas to live
with an aunt and uncle. On the flight, he became very dizzy and almost passed out. He has a
history of a heart murmur as an infant and was thought to need surgery, but was not able to have
that done. The murmur resolved by 4 years of age. He is not able to participate in gym class in
his new school because he gets too short of breath. He is very drowsy in the morning.
On physical examination, you note that he is cyanotic, but not in any acute distress. His weight is
19 kg (sixth percentile) and his height is 122 cm (50th percentile). His heart rate is 100
beats/min, his respiratory rate is 30 breaths/min, his blood pressure is 85/45 mm Hg, and his
oxygen saturation is 78% on room air. His jugular veins are distended. His chest examination
shows intercostal retractions. The breath sounds are clear. The cardiac examination is notable for
a loud P2. There is a 3/6 systolic murmur at the right midclavicular line. The liver is 2 cm below
the right costal margin. Screening laboratory test results are significant for a hemoglobin of 17
g/dL (170 g/L) and a hematocrit of 51%.
Of the following, the MOST likely cause of the patient’s symptoms and physical findings is
This child in the vignette has pulmonary hypertension and the murmur in this case is a result of
tricuspid regurgitation. The right ventricle (RV) has systemic pressure, so there will not be any
gradient across the defect. The airplane flight made this child worse because airplanes are
pressurized only to what would be encountered at 8,000 ft elevation. This causes worsening
hypoxic vasoconstriction in a patient with PHTN. If there were a gradient of 90 mm Hg across
his VSD, there would be a loud systolic murmur.
On physical examination, a newborn with a VSD may have a loud murmur or no murmur. A
small muscular VSD will cause a short systolic high-pitched murmur at the left mid-sternal
border as soon as the RV pressure drops below the left ventricular pressure. Since it is small, the
speed of the blood across the defect will be rapid and the murmur will be loud. This murmur may
be heard within the first week of life. These small defects usually close spontaneously.
A perimembranous VSD, which may be large or small (or be physiologically small if there is
tricuspid valve tissue within it), may or may not cause a murmur early in life (Item C72). Most
will cause a murmur by 3 to 4 weeks of age as the pulmonary vascular resistance drops. A very
large VSD that is part of an atrioventricular canal will often not cause a murmur because the
defect is so large that there is no difference in pressure between the right and left ventricles.
Blood may be flowing from the left to right ventricle, but it is not across a pressure gradient, so
the speed of the flow is not rapid or audible.
If there is large flow across a VSD for a prolonged period of time, and therefore a large increase
in blood flow to the lungs, damage may occur to the pulmonary vasculature with development of
PHTN. Once this occurs, the RV pressure may increase to greater than the systemic pressure and
the flow across the ventricular defect will reverse and become right to left. This is the
characteristic of Eisenmenger syndrome. Repair of an isolated perimembranous VSD by 12 to 24
months of age has been shown to protect against this phenomenon. Results of surgery have
become so reliable at younger ages that there is not usually any need to wait until after a year of
age if closure is indicated.
PREP Pearls
• The clinical course of ventricular septal defects (VSD) can vary depending on the size of the
defect and the size or degree of left to right shunt over time.
• If there is large blood flow across a VSD for prolonged period of time, and therefore a large
increase in flow to the lungs, damage may occur to the pulmonary vasculature with
development of pulmonary hypertension.
• If right ventricle pressure increases to greater than the systemic pressure, then the flow across
a ventricular septal defect will reverse and become right to left, resulting in Eisenmenger
syndrome.
Suggested Readings
• Engelfriet PM, Duffels MJ, Möller T, et al. Pulmonary arterial hypertension in adults born
with a heart septal defect: the Euro Heart Survey on adult congenital heart disease. Heart.
2007;93(6):682–687.
Question 73
You are seeing a 7-year-old girl in your office for a health supervision visit. She has juvenile
idiopathic arthritis that affects her right knee and ankle. The girl’s parents state that she took
steroids for 6 months when she was initially diagnosed. Her disease is now well controlled with
methotrexate, folic acid, and etanercept. Her parents are concerned about the girl’s bone health,
specifically her risk of developing osteoporosis.
C. she should avoid exercise, as her knee and ankle are affected with arthritis
Osteoporosis is defined as a decrease in bone mineral mass per volume of bone tissue without
evidence of mineralization defects (ie osteomalacia or rickets). Primary forms of osteoporosis are
rare in young patients. This can occur in idiopathic juvenile osteoporosis and genetic connective
tissue diseases such as Ehlers-Danlos syndrome, Marfan syndrome, and osteogenesis imperfecta.
More common in the pediatric population is secondary osteoporosis. Secondary osteoporosis
may occur with neuromuscular disorders, inborn errors of metabolism, endocrine diseases,
various chronic diseases, gastroenterologic and nutritional disorders, and certain medications or
treatments (Item C73).
The best initial step in the management of osteoporosis is control of the underlying condition,
and reduction of other risk factors for bone loss. Risk factors include low body mass,
undernutrition, low levels of physical activity and/or weight bearing, hypogonadism, minimal
sun exposure, and other lifestyle choices such as smoking or significant soft drink intake.
Patients at risk for osteoporosis should be counseled regarding nutritional and lifestyle factors,
including the importance of appropriate calcium, phosphorous, and vitamin D intake. Fruit and
vegetable intake positively correlate with bone mineral density. Malnutrition and undernutrition
are common in many chronic conditions and should be directly addressed. Physical activity
including weight-bearing exercises can increase bone mineral density and is recommended for
the patients who are able. When appropriate, the female athlete triad that includes malnutrition,
intense exercise, and hypogonadism should be discussed. Calcium supplementation is
recommended for deficient individuals and children at high-risk for osteoporosis. The Institute of
Medicine recommended daily dietary allowance is 700 mg for children age 1 to 3 years, 1,000
mg for age 4 to 8 years, and 1,300 mg for age 9 years and older. Children with malabsorption or
on medications that induce osteoporosis may need higher doses. The recommended daily dose of
vitamin D for children at low risk for osteoporosis is 400 IU.
Vitamin D supplementation is recommended for those at risk for osteoporosis, because low
vitamin D levels affect bone quality. However, research on the effect of supplementation on bone
mineral densities has not shown consistent results. Patients age 1 year and older who are
deficient in vitamin D should be supplemented with 2000 IU daily or 50,000 IU weekly for at
least 6 weeks. Children at risk for osteoporosis should be treated with 400 to 1,000 IU daily on a
long-term basis. Patients with malabsorption may need higher doses. Either vitamin D2 or D3
can be used to treat vitamin deficiency, but some studies have demonstrated that the D3 form
may be more beneficial in increasing bone mass and strength.
Juvenile idiopathic arthritis (JIA) is a risk factor for osteoporosis. Weight-bearing exercise can
increase bone mineral density and is not contraindicated in JIA. Although methotrexate has been
associated with decreased bone mineral density, the girl in the vignette should not discontinue
the methotrexate to avoid osteoporosis, because uncontrolled arthritis can itself do significant
bone and joint damage. The girl should not start magnesium supplementation, because this
would have little effect on her bone density.
PREP Pearls
• Weight-bearing exercise is osteogenic.
• The recommended daily dose of vitamin D for children at low risk for osteoporosis is 400 IU.
• The recommended daily dietary allowance for calcium is 700 mg for children age 1 to 3
years, 1,000 mg for age 4 to 8 years, and 1,300 mg for children 9 years and older.
Suggested Readings
• Ma N, Gordon CM. Pediatric osteoporosis: where are we now? J Pediatr. 2012;161(6):983-
990. DOI:http://dx.doi.org/10.1016/j.jpeds.2012.07.057.
Question 74
A 9-year-old girl presents to your office for the evaluation of a 3-month history of activity-
related bilateral heel pain. The pain worsens during and after soccer practice and games. She
denies swelling, decreased ankle motion, or constitutional symptoms. Physical examination is
remarkable for tenderness with medial and lateral compression (squeezing) of the heel.
The girl in the vignette has Sever disease, inflammation of the calcaneal apophysis. The
diagnosis of Sever disease is based on history and physical examination findings. Patients often
report insidious onset of activity-related heel pain. On physical examination, pain with
simultaneous medial and lateral compression of the heel is the classic finding.
Sever disease typically affects patients between 8 and 13 years of age. The Achilles tendon, an
extension of the gastrocnemius and soleus muscles, attaches to the calcaneus adjacent to the
apophysis. With contraction of the calf muscles, the Achilles tendon puts tension on the
apophysis, causing mechanical irritation. Direct force applied by high impact activities such as
running and jumping also irritate the apophysis. Use of a soft heel cup in the shoe can blunt the
force applied to the apophysis and appears to relieve symptoms. Ice and nonsteroidal anti-
inflammatory drugs may help diminish pain. Activity modification or restriction is indicated for
patients with significant pain or alteration in gait despite the use of symptomatic treatment. The
symptoms of Sever disease typically abate when the apophysis closes, within 1 to 2 years of
onset. This condition does not appear to have any long-term sequelae.
The diagnosis of Sever disease is made clinically. Radiography may be useful to rule out other
conditions and may show fragmentation or sclerosis of the calcaneal epiphysis. Magnetic
resonance imaging may be indicated if stress fracture or osteomyelitis of the calcaneus is
suspected. Elevation of the erythrocyte sedimentation rate may be seen with infection or an
inflammatory condition such as juvenile idiopathic arthritis. Computed tomography is not
indicated for Sever disease.
PREP Pearls
• The diagnosis of Sever disease is based on history and physical examination findings;
radiographic studies are generally not needed.
• Most children and adolescents with Sever disease may participate in sports and physical
activities as pain allows.
Suggested Readings
• Sarwark JF, LaBella C. Pediatric Orthopaedics and Sports Injuries: A Quick Reference
Guide. 2nd ed. Elk Grove Village, IL: American Academy of Pediatrics; 2014:744 pp.
Question 75
A 3-week-old, previously healthy full-term male newborn is brought to your office for evaluation
of a rash on the bottom of his feet that began a few days ago. The mother has no other concerns.
On physical examination, the newborn is afebrile with normal vital signs. The anterior fontanel is
soft, open, and flat. There is minimal clear rhinorrhea. The liver and spleen are palpable 2 cm
and 1 cm below the right and left costal margins, respectively. There are numerous copper-
colored, circular lesions, each less than 0.5 cm in diameter on the plantar surfaces of both feet
(Item Q75). The remainder of the physical examination is unremarkable.
A. Candida albicans
B. coxsackievirus
C. cytomegalovirus
D. Streptococcus agalactiae
E. Treponema pallidum
Acquired syphilis occurs in 3 stages: primary, secondary, and tertiary. The primary stage of
infection is characterized by 1 or more painless ulcers (chancres) on the skin or mucous
membranes at the initial site of inoculation that develop approximately 3 weeks after exposure.
These lesions will spontaneously heal after a few weeks and can go undetected. The secondary
stage of syphilis is characterized by lymphadenopathy, mucocutaneous lesions, and rash. The
rash is generalized and typically involves the palms and soles. Patients may experience flulike
symptoms such as fever, headache, sore throat, arthralgias, and malaise. This stage will
spontaneously resolve in 1 to 4 months without treatment. The period following the secondary
stage is called the latent period during which time patients are asymptomatic and seroreactive but
may suffer recurrences of secondary stage symptoms. The tertiary stage of syphilis occurs 15 to
30 years after initial infection and can include neurosyphilis, cardiovascular symptoms, and
gumma formation.
Penicillin is the treatment of choice for congenital or acquired infection. For patients with
penicillin allergy and neurosyphilis, congenital syphilis, syphilis during pregnancy, or human
immunodeficiency virus infection, desensitization is recommended. The recommended
evaluation and treatment of neonates exposed to mothers infected with T pallidum is outlined in
Item C75A. The recommended treatment for syphilis in patients older than 1 month of age is
displayed in Item C75B.
Mucocutaneous infection caused by Candida in the neonate can involve the oropharynx (eg,
thrush) or vagina, digits and nails, and the intertriginous areas in the groin, axillae, and neck. It
also can cause a mild but diffuse cutaneous infection (Item C75C). Cutaneous infection caused
by Candida typically is described as an erythematous rash with satellite lesions; it typically
would not be isolated to the feet as described for the patient in the vignette. Coxsackievirus can
American academy of pediatrics 236
American Academy of Pediatrics PREP 2016
cause a nonspecific viral exanthem or hand, foot, and mouth disease characterized by painful red
blisters on the affected areas (Item C75D); it would not cause macular copper-colored lesions as
described for the infant in the vignette. Similarly, acquired cytomegalovirus infection could
present with a diffuse, nonspecific rash but not with localized findings. The classic “blueberry
muffin” rash of congenital cytomegalovirus infection (Item C75E) would be present at or shortly
after birth and not develop in the third week after birth as described for the patient in the
vignette. Lastly, late-onset infection caused by group B Streptococcus (S agalactiae) may present
as cellulitis but discrete copper-colored macular lesions are not characteristic findings.
Item C75C
Item C75D
Item C75E
PREP Pearls
• Intrauterine infection with Treponema pallidum can result in stillbirth, preterm birth, hydrops
fetalis, or asymptomatic infection.
• Manifestations of congenital syphilis include
• hepatosplenomegaly
• copious nasal secretions (snuffles)
• cutaneous lesions
• edema olymphadenopathy
• steochondritis
• pneumonia
• pseudoparalysis
• hemolytic anemia
• thrombocytopenia
• Untreated intrauterine infection can affect the
• central nervous system (eighth cranial nerve deafness)
• eyes (interstitial keratitis)
• teeth (peg-shaped incisors [Hutchinson teeth], mulberry molars)
• bones (frontal bossing, saddle nose, tibial bowing)
• joints (swelling of knees [Clutton joints])
• skin (ulceration, desquamation, palpable lesions)
• The treatment of choice for all stages of syphilis is parenteral penicillin.
Suggested Readings
• American Academy of Pediatrics. Syphilis. In: Kimberlin DW, Brady MT, Jackson, MA,
Long SS, eds. Red Book: 2015Report of the Committee on Infectious Diseases. 30th ed. Elk
Grove Village, IL: American Academy of Pediatrics; 2015:755-768.
Question 76
A 30-month-old girl is brought to the pediatric urgent care center for evaluation of fever, cough,
and rhinorrhea. Her mother is concerned that her daughter has an ear infection because she has
been crying frequently. The girl is a well-appearing, well-developed, active child in no distress.
Her vital signs include a temperature of 38.4°C, heart rate of 115 beats/min, respiratory rate of
22 breaths/min, and pulse oximetry of 100% on room air. Physical examination is remarkable
only for clear rhinorrhea from both nares and a swollen, tender right wrist with bruising over the
dorsal aspect.
When you inquire about any history of trauma, the mother explains that the girl is extremely
active and “very accident prone,” but that she recalls no specific injury. You order a right wrist
radiograph (Item Q76). A review of the urgent care records show that she was seen for a left
humerus fracture at 9 months of age. The mother denies any family history of musculoskeletal
disorders.
Of the following, the MOST likely explanation for her current radiologic finding is
A. child abuse
B. leukemia
C. osteogenesis imperfecta
D. osteomyelitis
E. rickets
Metaphyseal corner fractures result almost exclusively from inflicted injury. These typically
occur in young children (usually ≤3 years of age) and are often identified in the absence of a
history of trauma. This injury typically results from traction or torsion forces that occur when a
child’s extremities are pulled or swung forcefully. For the girl in the vignette, in addition to the
current injury, the history of a left humerus fracture during her first year after birth is a “red flag”
for ongoing child abuse.
American academy of pediatrics 241
American Academy of Pediatrics PREP 2016
No fracture, when considered in isolation, can definitively be said to result from an abusive
versus a nonabusive cause. During the evaluation of individual bony abnormalities, the lesion
site and type, age and developmental stage of the child, and associated reported history can help
clinicians to determine the likelihood of inflicted injury. A number of studies, including a recent
systematic review of the child abuse literature, have found that abusive fractures are detected
most commonly in infants younger than 1 year and toddlers (1-3 years of age). When any infant
or toddler presents with a fracture in the absence of a confirmed accidental cause, inflicted injury
should be considered as a potential etiology. Specific fracture patterns that should further raise
suspicion for child abuse include posterior rib fractures, metaphyseal corner fractures (also
known as “bucket-handle” fractures, Item C76), humeral and femur fractures (especially those
occurring in nonambulatory children), and complex skull fractures.
The presence of multiple fractures is also suggestive of abusive trauma, though this can certainly
occur in accidental trauma and as a result of underlying musculoskeletal disorders. In addition,
recurrent fractures at an unusual location for repeat injury, fractures in radiographic stages of
healing that do not correspond to the reported clinical history, or multiple fractures in various
stages of healing should raise suspicion of abuse. When suspicion of abuse is high, a
radiographic skeletal survey should be obtained to evaluate the entire skeletal system for
unrecognized injuries (new or old). Whenever possible, in cases of suspected nonaccidental
injury, clinicians should consult with providers possessing specific expertise and experience in
the diagnosis and management of child abuse. Physicians are mandated to report suspected child
abuse to the appropriate child protective and/or law enforcement agencies.
Children with various forms of leukemia may present with fever or bone pain, or may even limp.
However, the diagnosis of leukemia does not explain the radiographic finding of a metaphyseal
corner fracture in the child in the vignette.
Osteogenesis imperfecta (OI) is a musculoskeletal disorder resulting in fractures that are most
frequently mistaken for abusive fractures. OI is a heterogeneous group of disorders, usually
caused by mutations in the gene involved in the production of type I collagen, a protein integral
to the structural framework of bone. Although OI is an inherited disorder, the presentation of the
disease can be quite variable even within the same family. Obtaining a thorough medical and
family history, performing a physical examination, and interpreting the results of appropriate
laboratory and radiographic studies can usually lead to a diagnosis of OI. Many children with OI
American academy of pediatrics 242
American Academy of Pediatrics PREP 2016
will have the classic diagnostic signs such as osteopenia, bony deformities, and “wormian bones”
of the skull on radiography. Findings such as blue sclera, discolored teeth, limb deformities, and
a triangular-shaped face may be apparent on physical examination in some children with OI, but
these features may be subtle or absent. The girl in the vignette has no reported family history of
OI or physical examination findings, other than the fracture, that would suggest the diagnosis of
OI. This child’s history of a humerus fracture at 9 months of age raises suspicion for child abuse,
especially given that her newly identified fracture is a type that almost always arises from an
abuse mechanism.
Osteomyelitis is an infection of the bone that typically presents with fever and localized
musculoskeletal pain and tenderness. Although these symptoms are present in the girl in the
vignette, metaphyseal corner fractures are not typically associated with osteomyelitis. Early
radiographic findings suggestive of osteomyelitis include deep soft-tissue swelling with elevation
of the muscle planes from adjacent bone, which can be seen 3 to 4 days after the onset of
symptoms. Lytic bone changes and periosteal elevation may also be observed; however, these
changes are generally not visible on plain radiography until at least 10 to 14 days after the onset
of infection.
Rickets is an uncommon disorder that can occur in exclusively breastfed infants who are not
receiving vitamin D supplementation, or in dark-skinned children who do not get adequate
exposure to sunlight because of lifestyle or geographic location. Rickets can be diagnosed by
characteristic changes on plain radiography, which include cupping and fraying of the
costochondral junctions and epiphyses, demineralization, widened epiphyses, and cortical
thinning. Children affected by rickets will have decreased serum concentrations of vitamin D
metabolites, and serum alkaline phosphatase concentration is usually elevated. The radiographic
finding in this child is not consistent with the diagnosis of rickets.
PREP Pearls
• Metaphyseal corner fractures result almost exclusively from inflicted injury. These fractures
typically occur in young children, secondary to traction forces that occur when extremities
are pulled or swung forcefully.
• In evaluating fractures, the site and type, age and developmental stage of the child, and
associated reported history can help clinicians to determine the likelihood of inflicted injury.
• When child abuse is suspected, a radiographic skeletal survey should be obtained to evaluate
the entire skeletal system for injuries. Providers with experience in the evaluation and
management of child abuse should be consulted, and the suspected abuse must be promptly
reported to appropriate child protection and law enforcement agencies.
• Differentiate the cutaneous findings associated with physical abuse from those of non-
abusive skin conditions
Suggested Readings
• Jenny C. Evaluating infants and young children with multiple fractures. Pediatrics.
2006;118;1299. DOI:http://dx.doi.org/10.1542/peds.2006-1795.
Question 77
You are examining a small-for–gestational age newborn 8 hours after birth. She was delivered at
37 weeks of gestation because of worsening maternal pregnancy-induced hypertension. The
pregnancy was unremarkable, except for increasing maternal blood pressure and proteinuria that
were first noted 1 week before delivery.
Of the following, the MOST likely finding on physical examination of this newborn is
A. clitoral prominence
B. hepatosplenomegaly
D. jaundice
E. microcephaly
Hepatosplenomegaly may be found in SGA newborns with congenital infections, but the small
size of the infant in the vignette is attributed to PIH rather than infection. SGA newborns
typically have decreased breast tissue because of their undernourished state. Jaundice may
develop in these infants due to polycythemia, but it is not expected to be present 8 hours after
birth. The head circumference of a newborn who is SGA due to PIH is typically in the same or
higher percentile as the weight, not lower as would be seen with microcephaly.
PREP Pearls
• Small-for–gestational newborns have decreased muscle mass and subcutaneous tissue, dry
peeling skin, increased plantar creasing, diminished breast tissue, underdeveloped ear
cartilage, and clitoral prominence.
• Small-for–gestational age newborns are at risk for hypothermia, hypoglycemia, and
polycythemia.
Suggested Readings
• AAP Committee on Fetus and Newborn. Postnatal glucose homeostasis in late-preterm and
term infants. Pediatrics. 2011;127:575-579. DOI: http://dx.doi.org/10.1542/peds.2010-3851.
Question 78
The father of a 3-year-old boy in your practice calls on Friday evening. The boy has previously
been healthy, growing and developing well. The father reports that the boy has had nonbloody,
nonbilious vomiting intermittently for 3 days. He is still drinking fluids, urinating his usual
amount, and has no diarrhea, fever, or rash. He is not lethargic. There are no sick contacts. You
advise supportive care. On Monday, the mother brings the boy in to your office for evaluation.
He continues to vomit, is now drinking less, and continues to urinate. His mother reports the
boy’s eyes are sometimes “crossed,” which is new for him. On physical examination, he has a
temperature of 36.5°C, blood pressure of 96/50 mm Hg, heart rate of 91 beats/min, and
respiratory rate of 36 breaths/min. He is tired appearing, but arouses with the examination. His
abdominal examination is unremarkable. There is no rash. On neurologic examination, the boy is
able to visually fixate on your face with both eyes, but when tracking to the left, his left eye
cannot fully abduct.
Of the following, the BEST recommendation for this child is referral to the
A. emergency department
B. neurology clinic
C. neurosurgery clinic
D. ophthalmology clinic
E. urgent care
When acute increased intracranial pressure is suspected, even when vital signs are normal,
immediate emergency department evaluation is the most appropriate referral. This child is at risk
for rapid decompensation. The boy in the vignette will require urgent imaging and neurosurgical
consultation and intervention. A neurologist or ophthalmologist may be consulted in the hospital,
but referral to an outpatient clinic or urgent care is not appropriate in this situation.
PREP Pearls
• A child with new-onset signs of increased intracranial pressure must be immediately
transported to the emergency department.
• New ocular malalignment, (crossed eyes), should prompt evaluation for other signs of
increased intracranial pressure.
Suggested Readings
• Ansell P, Johnston T, Simpson J, Crouch S, Roman E, Picton S. Brain tumor signs and
symptoms: analysis of primary health care records from the UKCCS. Pediatrics.
2010;125:112-119. DOI: http://dx.doi.org/10.1542/peds.2009-0254.
• Crawford J. Childhood brain tumors. Pediatr Rev. 2013;34(2): 63-78. DOI:
http://dx.doi.org/10.1542/pir.34-2-63.
Question 79
A 10-year-old boy has had poor growth over the past few years (Item Q79). He also complains of
fatigue, muscle weakness, and worsening acne. On physical examination, the boy has a
prominent fat pad on the back of his neck, central obesity, and stretch marks (striae) on his
abdomen.
Of the following, the test MOST likely to determine the underlying cause of this patient’s poor
growth is a(n)
Evaluation for Cushing syndrome can be performed by measurement of 24-hour urine free
cortisol, overnight dexamethasone suppression, late night salivary cortisol, and in some cases,
assessment of diurnal variation in cortisol. Thus, of the choices offered, a 24-hour urine-free
cortisol would be appropriate for this patient.
An AM GH level is not recommended for evaluation of growth, even during evaluation for GH
deficiency, because GH levels peak in deep sleep and are often very low during the day. This is,
in part, the rationale for growth hormone stimulation testing, and why insulin like growth factor
1 (IGF-1) or insulin like growth factor binding protein 3 (IGF-BP3) levels are measured instead.
Obtaining a prolactin level can help rule out prolactinoma as the cause of poor growth. Thyroid-
stimulating hormone and free thyroxine levels are used when evaluating for hypothyroidism.
PREP Pearls
• An endocrine cause of poor growth is likely when linear growth slows down or arrests but
weight gain is either normal or increasing.
• Cushing syndrome in children is characterized by linear growth failure, weight gain, pubertal
changes from excess androgen production, as well as signs and symptoms similar to adults
(easy bruising, facial plethora, myopathy, striae).
Suggested Readings
• Nieman LK, Biller BM, Findling JW, et al. The diagnosis of Cushing's syndrome: an
Endocrine Society Clinical Practice Guideline. J Clin Endocrinol Metab. 2008;93:1526-1540.
DOI: http://dx.doi.org/10.1210/jc.2008-0125.
• Weintraub B. Growth. Pediatr Rev. 2011;32:404-406. DOI: http://dx.doi.org/10.1542/pir.32-
9-404.
Question 80
A 14-year-old adolescent is brought to the emergency department (ED) with 2 months of
intermittent wet cough. More recently, she has had 2 to 3 episodes of blood-tinged sputum. She
has experienced 1 month of increasing fatigue and pallor. She has been increasingly intolerant of
physical activity and now experiences shortness of breath when climbing 1 flight of stairs. Today
while at school, she experienced a fainting episode and was brought to the ED via emergency
medical services. On review of systems, her mother reports a several month history of a rash on
the nose and cheeks, as well as decreased range of motion with swelling in her daughter’s fingers
and wrists. Most recently, her urine has looked like “tea.”
On physical examination, the patient has regained consciousness, but is ill appearing with
moderate respiratory distress. Her heart rate is 152 beats/min, and her respiratory rate is 40
breaths/min and labored. Her blood pressure is 90/60 mm Hg. Oxygen saturation is 92% in room
air. Oropharynx and nares are clear without blood. Her mucous membranes are pale but moist.
Auscultation of the lungs reveals diffuse polyphonic wheezing in upper and mid lung fields with
decreased breath sounds at both bases. Scattered fine crackles are also audible throughout. The
cardiac and abdominal examinations are unremarkable, with the exception of tachycardia.
Capillary refill is brisk. Her extremities are warm and well perfused, and there is no digital
clubbing. Her hemoglobin is 5.9 g/dL (59 g/L).
B. cystic fibrosis
C. pulmonary hypertension
D. tuberculosis
E. vasculitis
Bleeding from the lungs or airway may occur in a focal or in a diffuse pattern. The most
common etiology for focal pulmonary hemorrhage is chronic infection or inflammation; classic
examples are tuberculosis and the endobronchial infections that cause bronchiectasis in patients
with cystic fibrosis.
The etiologies of diffuse alveolar hemorrhage (DAH) syndromes are multiple and varied, and
may be categorized by whether there is pathologic evidence of pulmonary capillaritis. The
finding of pulmonary capillaritis denotes inflammation in the pulmonary capillary bed and is a
poor prognostic finding in individuals with DAH; those with systemic lupus erythematosus,
Goodpasture syndrome, granulomatosis with polyangiitis (previously Wegener granulomatosis),
and Henoch-Schönlein purpura, among others, may be affected.
In the absence of capillaritis, the origins of DAH are divided into cardiac and noncardiac causes.
Cardiovascular associations include arteriovenous malformations and pulmonary hypertension.
Noncardiac etiologies include celiac disease, coagulation disorders, and acute idiopathic
pulmonary hemorrhage of infancy. When an exhaustive search for an etiology of diffuse
pulmonary hemorrhage is unrevealing, patients may be designated as having idiopathic
pulmonary hemosiderosis. Some of these patients will later be diagnosed with an associated
condition.
In patients with DAH, the rate of bleeding is typically slow and insidious. Patients may never
expectorate blood and instead are likely to present with fatigue, pallor, tachycardia, or exercise
intolerance. Initial symptoms may suggest the underlying condition and the diagnosis of DAH
may be a secondary one. The classic laboratory finding is a microcytic anemia. Radiographs are
often nonspecific, but may demonstrate bilateral alveolar opacities with lower lobe
predominance as in the patient in this critique. The diagnosis of DAH is confirmed by the
presence of hemosiderin laden macrophages on sputum analysis. Therapy is dependent on
underlying condition, but may include systemic steroids and immunosuppressive agents.
The patient in this vignette is not in the age group classically associated with foreign body
aspiration. In addition, there is no asymmetry or air trapping on chest radiograph to suggest an
inhaled foreign body. Similarly, the radiograph does not reveal nodularity or lymphadenopathy
suggestive of tuberculosis. Furthermore, the bleeding in both of these conditions would be
expected to be more brisk with notable bright red hemoptysis.
An adolescent may be diagnosed with cystic fibrosis if they have atypical or mild disease.
Bleeding from the airways in patients with cystic fibrosis, however, occurs from bronchiectasis,
which is a late manifestation of disease. An individual with pulmonary hypertension may present
American academy of pediatrics 256
American Academy of Pediatrics PREP 2016
with syncope, exertional dyspnea, and fatigue. However, the joint, skin, and urinary symptoms
found in the patient in this vignette would not be expected.
PREP Pearls
• he most common causes of focal pulmonary hemorrhage are infection and inflammation.
• Diffuse alveolar hemorrhage syndromes may be classified by the presence or absence of
pulmonary capillaritis.
• Bleeding in diffuse alveolar hemorrhage is likely to be slow and insidious; patients may
present with symptoms of anemia rather than with hemoptysis.
Suggested Readings
• Godfrey S. Pulmonary hemorrhage/hemoptysis in children. Pediatr Pulmonol.
2004;37(6):476-484. DOI:http://dx.doi.org/10.1002/ppul.20020.
Question 81
You are evaluating a 2-month-old infant who was born at 28 weeks of gestation and went home
from the neonatal intensive care unit 3 days ago. The baby’s birth weight was 990 g and her
current weight is 1,900 g. The formula-fed infant was discharged on a diet of 22 cal/oz premature
formula. The parents have had difficulty finding the formula and ask if the baby could be fed a
different type of milk while still maintaining the benefits of premature formula.
Of the following, the MOST accurate statement about feeding options for this infant is
A. banked human milk supplies sufficient minerals and calories to support growth and
prevent metabolic bone disease in low birth weight infants
B. compared to formula made for term infants, premature formulas contain higher levels
of calcium and phosphorus
C. formula made for term infants would provide iron in excess of this infant’s needs
While BMD is primarily dependent on intake and absorption of calcium and phosphorus, vitamin
D intake also plays a role. There are currently no studies of the clinical impact of 25-
hydroxyvitamin D concentrations in preterm newborns, so deficiency and sufficiency is based on
extrapolation from adult and pediatric populations. Current recommendations are that preterm
newborns with a birth weight of more than 1,500 g receive 400 IU of vitamin D daily just as term
infants do. For very low birth weight newborns, 200 to 400 IU daily is recommended, with an
increase to 400 IU/day once they have achieved 1,500 g in weight. The maximum dose of
vitamin D for term infants is 1,000 IU/day and preterm newborns should not exceed this dose.
Iron stores are also laid down during the third trimester, and preterm and low birth weight infants
are at risk for iron deficiency that can contribute to neurodevelopmental issues later in the child’s
life. Repeated blood draws or transfusions also impact the infant’s iron homeostasis. Current
recommendations are that low birth weight infants receive 2 to 3 mg/kg per day of iron
beginning at 1 to 2 months of age. Although the iron concentrations in formula or human milk
plus fortifier are quite variable, this route can supply at least part of this iron supplementation.
For the infant in the vignette, she may be able to consume enough iron from term or preterm
formula.
PREP Pearls
• Most mineral accumulation occurs during the third trimester, therefore premature newborns
are at risk for developing deficiencies of calcium, phosphorus, iron, copper, and zinc.
• The primary cause of decreased bone mineral density in premature infants is low levels of
calcium and phosphorus caused by either low intake or low absorption.
• Fortified human breast milk is the preferred energy source for preterm infants and supplies
adequate minerals for their growth needs. Formula made for preterm infants also supplies
adequate calcium and phosphorus.
• Vitamin D supplementation for healthy preterm infants is 200 to 400 IU/day for weight less
than 1,500 g and 400 IU/day for weight greater than 1,500 g.
ABP Content Specifications(s)
• Understand the dietary mineral requirements of patients of various ages, including those born
prematurely, and the circumstances in which those requirements may change
Suggested Readings
• Bhatia J, Griffin I, Anderson D, Kler N, Domellöf M. Selected Macro/micronutrient needs of
the routine preterm infant. J Pediatr. 2013;162(3 Suppl):S48-S55. DOI:
http://dx.doi.org/10.1016/j.jpeds.2012.11.053.
Question 82
A 13-year-old adolescent is admitted to the hospital for a tonsillectomy. As part of his pre-
operative evaluation, a metabolic panel was ordered that revealed an alkaline phosphatase of 325
U/L (upper limit of normal = 116 U/L). You are asked to evaluate him for this abnormal
laboratory result. His past medical history is significant only for tonsillar hypertrophy and related
obstructive sleep apnea. His review of systems is negative except for snoring. He does not take
any medications. Aside from having grade 3 tonsils, his physical examination is unremarkable.
His sexual maturity rating is 3.
Of the following, the MOST likely reason for his abnormal laboratory result is
A. bone tumor
B. gallbladder disease
C. laboratory error
D. pubertal growth
E. vitamin C deficiency
Item C82A and Item C82B show the SMR for boys and girls.
The first signs of puberty are breast development (girls) and testicular enlargement (boys). The
typical age of the onset of puberty can vary by ethnicity, particularly among girls. A recent study
by Susman and colleagues looked at the longitudinal development of secondary sexual
development in a multiracial population and found the mean age for each stage of sexual
development (see suggested reading 5).
While biliary disease can cause elevated ALP, typically it is 3 to 4 times the upper limit of
normal. Bone tumors can also cause elevated ALP, but would be an unlikely diagnosis in the
adolescent in this vignette with a review of systems significant only for snoring. Lower than
expected ALP values can be seen with vitamin C deficiency. Laboratory error is a possibility;
however, it is developmentally appropriate for this patient to have an elevated ALP, making
pubertal growth a better explanation for this finding.
PREP Pearls
• Elevated alkaline phosphatase can be seen during linear growth associated with adolescence.
• Increases in luteinizing hormone and follicle stimulating hormone are seen in puberty.
Luteinizing hormone increases throughout puberty. Follicle-stimulating hormone increases,
but can plateau when sexual maturity rating 3 is achieved.
• All of the sex hormones, including estradiol and testosterone, increase during puberty.
• Insulin secretion increases during adolescence.
Suggested Readings
• Corathers SD. Focus on diagnosis: the alkaline phosphatase level: nuances of a familiar test.
Pediatr Rev. 2006;27(10):382-384. DOI: http://dx.doi.org/10.1542/pir.27-10-382.
Question 83
A 16-year-old adolescent girl presents to your office for evaluation of a 2-month history of hair
loss. Her physical examination demonstrates an area of incomplete alopecia at the vertex. Within
the affected area are hairs of differing lengths and 2 areas of hemorrhage (Item Q83).
Hair loss as described for the girl in the vignette. Reprinted with permission from Krowchuk DP,
Mancini AJ, eds. Pediatric Dermatology. A Quick Reference Guide. Elk Grove Village, IL:
American Academy of Pediatrics; 2012.
A. alopecia areata
B. friction alopecia
C. tinea capitis
D. traction alopecia
E. trichotillomania
Item C83A
The girl described in the vignette has an area of hair loss within which hairs of differing lengths may be
seen. Two areas hemorrhage (arrows) are present at sites where hairs were pulled. Adapted and
reprinted with permission from Krowchuk DP, Mancini AJ, eds. Pediatric Dermatology: A Quick
Reference Guide. Elk Grove Village, IL: American Academy of Pediatrics; 2012.
Trichotillomania usually involves the scalp, but any hair-bearing area can be affected (eg,
eyebrows, eyelashes). In young children, trichotillomania often represents a habit similar to
thumb sucking. For example, children unconsciously twirl or pull hair while watching television.
In such cases, parents may be advised to offer a gentle reminder when the behavior is observed.
In older children and adolescents, trichotillomania often represents a compulsion, and is
considered among the obsessive-compulsive-related disorders. Comorbid depression and anxiety
are not uncommon and therapy is more challenging. Cognitive behavioral therapy, especially
habit reversal training (HRT), can be highly successful in reducing hair pulling, but treatment is
intensive and finding skilled therapists can be difficult. In HRT, patients are taught to monitor
American academy of pediatrics 266
American Academy of Pediatrics PREP 2016
Several other disorders may produce circumscribed areas of hair loss without scalp scarring and
therefore may mimic trichotillomania. These include:
• Alopecia areata: round to oval patches of complete hair loss; the scalp appears normal (Item
C83B)
• Friction alopecia: round to oval patch of hair thinning located at the occiput in young infants
who spend much of their time in the supine position (Item C83C)
• Tinea capitis: in the most common form, 1 or more patches of alopecia are present, as well as
scale and “black-dot” hairs (the remnants of broken hairs within follicles) (Item C83D)
• Traction alopecia: thinning of hair in areas where the hair is being stretched as the result of
braiding or creating ponytails (often seen at temporal-parietal hairline) (Item C83E)
Item C83C
In alopecia areata, round or oval patches of complete hair loss are observed. Courtesy of D. Krowchuk.
Item C83C
Friction alopecia appears as thinning of the hair most often at the occiput in young infants. Courtesy of
D. Krowchuk.
Item C83D
In the most common form of tinea capitis, there are one or more patches of hair loss within
which one may see scale, “black-dot” hairs (yellow arrows), or pustules (red arrows). Adapted
and reprinted with permission from Krowchuk DP, Mancini AJ, eds. Pediatric Dermatology: A
Quick Reference Guide. 2nd ed. Elk Grove Village, IL: American Academy of Pediatrics; 2012.
Item C83E
Thinning of the hair at sites of tight braiding (often the temporal-occipital region) is characteristic of
traction alopecia. Adapted and reprinted with permission from Krowchuk DP, Mancini AJ, eds. Pediatric
Dermatology: A Quick Reference Guide. 2nd ed. Elk Grove Village, IL: American Academy of Pediatrics;
2012.
PREP Pearls
• When evaluating hair loss, it is useful to consider: (1) is the hair loss localized or generalized,
and (2) is there scarring of the scalp? Most hair loss in children is localized and scarring is
absent.
• The leading causes of localized hair loss are tinea capitis, traction, friction, alopecia areata,
and trichotillomania.
• In trichotillomania, the hair loss is incomplete (unlike in alopecia areata or tinea capitis) and,
within the affected area, hairs are of differing lengths.
Suggested Readings
• Bloch MH, Panza KE, Grant JE, Pittenger C, Leckman JF. N-acetylcysteine in the treatment
of pediatric trichotillomania: a randomized, double-blind, placebo-controlled add-on trial. J
Am Acad Child Adolesc Psychiatry. 2013;52(3):231-240.
DOI:http://dx.doi.org/10.1016/j.jaac.2012.12.020.
Question 84
A 10-year-old boy presents to the emergency department after he was hit in the head with a
baseball. His vital signs show a temperature of 37°C, blood pressure of 120/80 mm Hg, pulse of
60 beats/min, and respiratory rate of 15 breaths/min. He is awake and alert, complaining of a
headache and blurry vision. He denies any other pain, nausea, vomiting, difficulty breathing, or
dizziness. There is a boggy, tender area over his right temporal area with no other signs of
trauma. Pupils are 3 mm, equal, and reactive. When he looks straight ahead, his right eye
deviates medially. Extraocular movements and cranial nerves are otherwise intact. Mental status
examination and the remainder of the neurologic examination are unremarkable. A computed
tomography of the head confirms your suspected diagnosis.
D. neurosurgery consultation
E. ophthalmology consultation
The presence of bleeding between the skull and the dura mater is known as EDH. The most
common cause of EDH is blunt trauma, and the most frequent source of bleeding is arterial. A
common mechanism of such trauma involves being struck in the temporal region of the skull,
causing temporal skull fracture and injury to the middle meningeal artery. Characteristic
appearance on computed tomography is a lenticular-shaped density, as shown in Item C84A. The
classic progression of EDH is an initial loss of consciousness at the scene, followed by a lucid
interval that can last several hours, followed by neurologic deterioration caused by expanding
hemorrhage and cerebral edema. This can lead, in some cases, to increased intracranial pressure,
herniation syndrome, and death. It is important to note that only approximately 20% of patients
with EDH experience the classic initial loss of consciousness. In many cases, the neurologic
examination is normal, so a high index of suspicion should be maintained in patients with
persistent symptoms of vomiting, headache, and signs of skull fracture. In such patients,
computed tomography of the head should be obtained. Although the child in this vignette is
lucid, the presence of extraocular motion abnormality is ominous, and could represent uncal
herniation. In a unilateral brain hemorrhage, downward pressure on the temporal lobe could
cause the uncus to herniate through the tentorium, exerting pressure on the midbrain structures.
An early sign of this is a lateral rectus palsy caused by dysfunction of the sixth cranial nerve.
Item C84A
Bleeding between the dura mater and arachnoid mater is known as subdural hematoma (SDH).
Similar to EDH, SDH is most commonly caused by blunt trauma, though in contrast, it is not
usually associated with an overlying skull fracture. Other than blunt trauma, SDH can be caused
by acceleration-deceleration injury, such as occurs in motor vehicle collisions or abusive shaking
of an infant, leading to tearing of bridging veins between the dura and arachnoid maters. Classic
appearance on head computed tomography is crescentic (Item C84B). SDH is most often
associated with underlying brain edema, injury, and ischemia. Thus, patients are likely to present
with significant depressed consciousness or signs of elevated intracranial pressure. In previously
unrecognized SDH, or in cases of abuse, SDH may be chronic and of varying ages. If SDH is
suspected, computed tomography should be obtained, and emergent neurosurgical consultation
should be sought.
Item C84B
For any patient with traumatic brain injury, the clinician should frequently assess the patient’s
degree of neurologic dysfunction and coma. A Glasgow Coma Score (GCS) should be obtained
(Item C84C). Emergent treatment for increased intracranial pressure and severe herniation
syndrome may be necessary in cases of severe coma (GCS < 8). This may include endotracheal
intubation and mechanical ventilation for airway protection, brief hyperventilation, and
osmotherapy including hypertonic saline or mannitol.
For the boy in the vignette, observation alone is not the best course of action, because he will
likely require evacuation of the hemorrhage. Intubation and mechanical ventilation is not
necessary, because he is awake, alert, and protecting his airway. Magnetic resonance imaging is
a sensitive and specific modality for intracranial injury. However, the diagnosis of epidural
hematoma is already evident from the computed tomography study. Ophthalmology consultation
is not necessary because the origin of his extraocular abnormality is neurologic.
PREP Pearls
• Intracranial epidural hematoma (EDH) is usually associated with a skull fracture, is
commonly caused by disruption of a meningeal artery, and usually does not include
underlying cerebral edema or contusion
• Intracranial subdural hematoma (SDH) is usually not associated with a skull fracture, is
commonly caused by disruption of bridging veins, and usually includes underlying edema
and contusion.
• An emergent computed tomography of the head and neurosurgical consultation should be
sought in cases of suspected or confirmed SDH or EDH.
Suggested Readings
• Atabaki SM. Pediatric head injury. Pediatr Rev. 2007;28(215):215-223.
DOI:http://pedsinreview.aappublications.org/content/28/6/215.
Question 85
A 13-year-old adolescent is hospitalized with fever, bloody stools, and weight loss. She has a
history of multiple infections. Vital signs show a temperature of 38.3°C, respiratory rate of 24
breaths/min, heart rate of 130 beats/min, blood pressure of 101/70 mm Hg, and a weight of 32.5
kg. On physical examination, she is cachectic, has abdominal tenderness over the left lower
quadrant, and hepatosplenomegaly. Laboratory data shows:
• White blood cells, 4,600/µL (4.6 x 109/L)
• Hemoglobin, 8.1 g/dL (81 g/L)
• Platelets, 185 x 103/µL (185 x 109/L)
• Differential, 63% segmented neutrophils, 23% bands, 8% lymphocytes, 4% monocytes, 2%
eosinophils
• HIV enzyme-linked immunosorbent assay, positive
• HIV viral load, 312,729 copies/mL
A resident working with you asks about HIV testing in the adolescent.
B. the US Centers for Disease Control and Prevention mandates pre-test counseling be
provided prior to testing for HIV
The US Centers for Disease Control and Prevention (CDC) recommends universal testing for
HIV starting at 13 years of age, based on prevalence instead of risk-based testing. While the
CDC recommends that patients should be informed that testing is planned, it should be
performed unless patients opt out. Separate written informed consent or pre-test counseling are
not recommended, as these can be barriers to evaluation. The CDC policy is influenced by an
increasing number of HIV cases, missed opportunities for testing, and missed cases when only
risk-based testing is performed. For example, in 2006, 5% of the over 1 million HIV-infected
individuals living in the United States were adolescents and young adults, and nearly half of
these were unaware of their infection. However, individual state laws vary and can inhibit
implementing the CDC recommendation. Clinicians can reference the Compendium of State HIV
Testing Laws from the National HIV/AIDS Clinician’s Consultation Center
(http://nccc.ucsf.edu/) to determine how local laws and the CDC recommendations apply in their
setting.
While there may be hesitation to disclose status because of the stigma associated with the
diagnosis of HIV, withholding the diagnosis can have negative consequences such as preventing
access to social support, which is known to buffer psychological distress in those with HIV. In
some studies, disclosure of HIV status is related to greater quality of social support, greater self-
esteem and lower levels of depression. The American Academy of Pediatrics supports disclosure
of information that is developmentally appropriate. Most perinatal infections are disclosed by 10
years of age.
The laws that govern consent and confidentiality for HIV care vary by state. However, public
health statutes and legal precedent support medical evaluation and treatment of HIV without
parental knowledge or consent. Confidential community-based HIV testing is available if it
cannot be achieved in a particular practice for concerns of insurance billing or other reasons.
Local testing sites can be identified through the CDC web site (http://gettested.cdc.gov/).
PREP Pearls
• The American Academy of Pediatrics supports routine HIV testing in adolescents at least
once when the prevalence of HIV in the population is more than 0.1%.
• Although local laws may differ, the US Centers for Disease Control and Prevention
recommends universal HIV screening without separate written informed consent or pre-test
counseling.
• Disclosure of status congruent with the patient’s developmental stage is supported by the
American Academy of Pediatrics.
Suggested Readings
• American Academy of Pediatrics. Adolescents and HIV infection: the pediatrician’s role in
promoting routing testing.Pediatrics. 2011;128(5):1023-1029. DOI:
http://dx.doi.org/10.1542/peds.2011-1761.
Question 86
You are evaluating a 10-hour-old term female neonate. The neonate was born to a 34-year-old
gravida 2, para 2 woman by normal vaginal delivery. Antenatal history was significant for
bilateral hydronephrosis. A third trimester ultrasonography reported the anteroposterior renal
pelvic diameter was dilated at 15 mm bilaterally.
The neonate’s current vital signs show a weight of 3.0 kg, temperature of 37°C, heart rate of 140
beats/min, respiratory rate of 40 breaths/min, blood pressure of 80/46 mm Hg, and oxygen
saturation of 97% by pulse oximetry on room air. Physical examination reveals a comfortable
pink newborn, with clear and equal breath sounds bilaterally, no murmur, and no abdominal
mass.
Of the following, you are MOST likely to inform the parents that the risk for congenital anomaly
of the kidneys and urinary tract is
Transient antenatal hydronephrosis, not associated with clinically significant renal dysplasia or
risk for renal injury, is common; therefore, testing of these newborns is not indicated or helpful.
Neonatal serum creatinine concentration (usually < 1.0 mg/dL [88.4 μmol/L]) is reflective of
maternal serum creatinine concentration. In a full term neonate, the serum creatinine
concentration normalizes in 7 to 10 days, whereas in a preterm infant, it may take up to 1 month
to normalize. Therefore, elevated creatinine in a 10-hour-old neonate is not a good indicator of
underlying kidney function.
Maternal oligohydramnios is indicative of decreased fetal urine and impaired renal function. It is
associated with severe renal dysplasia or obstruction of the urinary tract (as in posterior urethral
valves). Maternal history of severe oligohydramnios is associated with increased neonatal risk
for pulmonary hypoplasia and these patients may present with respiratory problems immediately
after birth. The 2 most common congenital anomalies associated with prenatal hydronephrosis;
American academy of pediatrics 280
American Academy of Pediatrics PREP 2016
ureteropelvic junction (UPJ) obstruction and VUR are not usually associated with maternal
oligohydramnios.
In the neonatal period, UPJ obstruction and VUR rarely present with elevated blood pressure. In
patients with congenital renal dysplasia, blood pressure is normal at initial diagnosis, as these
patients have high urine output caused by tubular injury and increased sodium losses from
damaged or dysplastic tubules. Hypertension develops later in association with progressive renal
failure.
PREP Pearls
• Antenatal hydronephrosis can be transient, secondary to urinary tract obstruction or
vesicoureteral reflux (VUR).
• Transient antenatal hydronephrosis is not associated with clinically significant renal
dysplasia or risk for renal injury; therefore, testing of these newborns is not indicated or
helpful.
• Hydronephrosis associated with urinary tract obstruction or VUR may be associated with
abnormal renal development (renal dysplasia) or increased risk for renal injury.
• Severe hydronephrosis (> 10 mm renal-pelvic diameter) and bilateral hydronephrosis are
associated with increased risk for congenital anomalies of the kidneys and urinary tract.
Suggested Readings
• Carmody JB, Carmody RB. Question from the clinician: management of prenatal
hydronephrosis. Pediatr Rev. 2011;32(12):e110-e112. DOI: http://dx.doi.org/10.1542/pir.32-
12-e110.
Question 87
You are caring for a newborn delivered at a gestational age of 26 weeks with complex congenital
cardiac disease. A resident is assisting you in the nursery. On physical examination, you find an
intrauterine growth-restricted newborn with a 3/6 systolic ejection murmur. He is on nasal
cannula oxygen with a pulmonary examination that is unremarkable. His mother prefers not to
breastfeed and you are discussing his nutritional needs with your resident.
Of the following, the BEST choice for protein intake in this newborn is
Protein accounts for 10% to 15% of energy intake. The typical Western diet has increased
protein intake, with an average of 70 g to 100 g per day. Protein requirement in adults is 0.75 to
1 g/kg per day. Proteins are digested to amino acids, which are then used for protein synthesis
and molecular signaling. Premature infants have increased needs because of increased protein
synthesis and elevated enteral losses. Many children with chronic medical conditions have
increased metabolic needs, including increased requirements of protein. These conditions include
cancer, cystic fibrosis, failure to thrive, and other scenarios in which children require additional
calories. There are also conditions that require lower protein intake, including chronic renal and
liver disease.
PREP Pearls
• Premature infants have increased protein needs.
• Many children with chronic medical issues have increased protein needs.
• A term infant requires 2 to 3 g/kg per day of protein.
• An adult requires 0.75 to 1g/kg per day of protein.
Suggested Readings
• Kleinman RE, Greer FR. Pediatric Nutrition. 7th ed. Elk Grove Village, IL: American
Academy of Pediatrics; 2013: 1431 pp.
Question 88
A 17-month-old boy presents to your clinic with developmental delay, poor growth, hypotonia,
and microcephaly. He has very little speech beyond repetitive sounds. The mother reports that he
rolled over at 5 months of age, sat at 9 months of age, and began crawling at 16 months of age.
He had a full endocrinology workup for his slow growth, which was unremarkable. His birth
history is significant for mild intrauterine growth retardation and an unusual high-pitched cry
that has persisted. The remainder of his past medical history is notable for gastroesophageal
reflux and hypospadias. His height and weight are at the first percentile; his head circumference
is at the second percentile. Dysmorphologic examination shows microcephaly, mild
hypertelorism, broad nasal bridge, short palpebral fissures, bilateral epicanthal folds, and
micrognathia (Item Q88). No cardiac murmur is noted. A high resolution chromosome analysis
is ordered and reveals the diagnosis.
A. Angelman syndrome
B. cri-du-chat syndrome
C. fragile X syndrome
D. Russell-Silver syndrome
E. Williams syndrome
Angelman syndrome manifests with severe developmental delay, significant speech impairment,
gait ataxia, microcephaly, seizures, and a happy demeanor with inappropriate laughing and
excitability. They lack the classic facial dysmorphology and cat-like cry seen in cri-du-chat
syndrome. It is diagnosed with DNA methylation analysis of the 15q11.2-q13 chromosome
region (78%) or UBE3A sequence analysis (11%).
Fragile X syndrome is a disorder caused by a FMR1 full gene mutation (> 200 CGG repeats) or
other loss-of-function mutations characterized by moderate intellectual disability in affected
males and mild intellectual disability in affected females. Males with an FMR1 full gene
mutation develop a classic dysmorphologic facial appearance as they age, which includes
macrocephaly, long face, prognathism, and large, outwardly-rotated ears. Patients also have joint
laxity and macroorchidism after puberty. Behavioral abnormalities are common.
PREP Pearls
• Cri-du-chat syndrome is caused by a deletion of the short arm of chromosome 5
characterized by a high pitched or cat-like cry (> 95%), small head, epicanthal folds,
micrognathia, broad nasal bridge, hypertelorism, downward-slanting palpebral fissures, and
moderate to severe intellectual disability.
• Cri-du-chat syndrome can be diagnosed based on clinical presentation along with a high
resolution karyotype, fluorescent in situ hybridization (FISH) for 5p-, or a chromosomal
microarray.
• The American College of Medical Genetics and the American Academy of Pediatrics
recommend all children presenting with developmental delays have a chromosomal
microarray, which would not only pick up a gross chromosomal deletion, but also
microdeletions that could be easily missed on a high resolution karyotype.
Suggested Readings
• Mainardi CP. Cri-du-Chat syndrome. Orphanet J Rare Dis. 2006;1:33. DOI:
http://dx.doi.org/10.1186/1750-1172-1-33.
• Rodríguez-Caballero A, Torres-Lagares D, Rodríguez-Pérez A, Serrera-Figallo MA,
Hernández-Guisado JM, Machuca-Portillo G. Cri du chat syndrome: a critical review. Med
Oral Patol Oral Cir Bucal. 2010;15(3):e473-e478. DOI:
Question 89
The mother of a 3-year-old boy brings him to the office for evaluation of penile discharge. She
became concerned when she noticed some white matter while bathing him. The boy has been
otherwise well. He is toilet trained and has not been complaining of pain with urination. On
physical examination, you notice a partially retractable foreskin with a small amount of pasty,
cheesy material under the foreskin. There is no swelling or erythema of the shaft or glans penis.
There are no penile lesions and no discharge from the meatus.
A. evaluate for possible sexual abuse and refer to child protective services
B. instruct the mother to retract the foreskin fully by applying increasing force daily
C. obtain a culture and sensitivity of the material and begin antibiotic treatment
The foreskin typically extends about 1 cm beyond the glans penis and provides protection to the
urethral meatus and glans. Phimosis is the inability to retract the foreskin or prepuce to expose
the glans penis, and is caused by either a small foreskin opening or adhesions. In young boys,
this is normal or physiologic and the ability to fully retract the foreskin increases with age.
Physiologic phimosis is present in almost all newborns, because of normal adhesions that exist
between the foreskin and glans penis. After birth and during childhood, penile growth and
physiologic erections aid in the natural process that loosens the adhesions and leads to foreskin
retraction. The degree of normal retractability by age varies a great deal, and reports of norms
have differed. There is a high likelihood that physiologic phimosis will resolve spontaneously.
Partial phimosis is quite common and may still be present in one-third of 5- to 7-year-old boys.
The foreskin can be retracted fully, or enough to visualize part of the glans penis, in most boys
by the age of 12 to 13 years.
The uncircumcised penis should be washed with soap and water regularly when bathing. As the
foreskin naturally begins to retract, proper hygiene that includes cleaning and drying underneath
should be emphasized. Patients or parents can be taught to perform gentle stretching exercises,
but forcible retraction should be avoided. A 1- to 2-month course of topical corticosteroids
applied to the preputial outlet may be used, in addition to stretching exercises, to speed up the
natural process of obtaining full retraction of the foreskin.
Pathologic phimosis is a truly nonretractable foreskin secondary to distal scarring of the prepuce.
This may be the result of previous trauma or inflammation. It is important for primary care
clinicians to distinguish pathologic cases, because they will need referral to a pediatric urologist
or surgeon.
Paraphimosis occurs when the foreskin is retracted and gets trapped behind the coronal sulcus of
the glans penis, leading to venous and lymphatic congestion. Painful swelling of the shaft and
glans penis distal to the constriction progresses, which further hinders return of the foreskin to its
natural position, and promotes more congestion. Eventually, this can impede blood flow to the
glans penis with the potential for permanent damage or gangrene. Paraphimosis is a urologic
emergency. Treatment requires adequate pain control and timely reduction of the foreskin back
to the normal position. If the general practitioner is not able to perform the reduction, or if signs
American academy of pediatrics 289
American Academy of Pediatrics PREP 2016
PREP Pearls
• Smegma, desquamated epithelial cells trapped under the foreskin, is a common physiologic
finding in uncircumcised males.
Question 90
A 2-year-old boy presents to the emergency department having fallen down a flight of 5 stairs
and hitting his head on a ceramic floor. On further history, his mother reports that, since birth, he
has had excessive bruising in response to seemingly minor trauma, bruising in unusual places
such as the small of the back and the dorsal aspects of the proximal arms, frequent gum bleeding
with tooth brushing, and having had excessive bleeding following circumcision. There is no
known family history of abnormal bleeding symptoms. Given this history, a head computed
tomography is obtained that demonstrates an intracranial bleed. Laboratory evaluations are
shown:
The formation of a functional clot requires 2 components, fibrin and platelets. Fibrin is the end
product of the coagulation cascade. The absence of any of the factors in the coagulation cascade
can result in failure to form a clot and is associated with a prolonged PT or PTT. The PT and the
PTT are effective measures of the functionality or presence of the components of the coagulation
cascade, except for the conversion of fibrinogen to fibrin. While the absence of functional
fibrinogen could explain the scenario presented in the vignette, none of the answers provided
would correct a disorder of fibrin, which is treated with infusion of cryoprecipitate.
Von Willebrand factor is a linking factor that allows functional platelets to bind to fibrin to form
a clot. Von Willebrand disease (VWD) is the result of decreased function or absence of von
Willebrand factor. Von Willebrand disease has multiple phenotypes ranging from mild to severe
bleeding disorders that mirror the degree of dysfunction or absence of the von Willebrand factor.
There are several different types of VWD, including types 1, 2A, 2B, 2M, 2N, and 3. Von
Willebrand disease is the most common heritable bleeding disorder. While severe VWD could
explain the scenario in the vignette, the most common subtypes have mild to moderate mucosal
bleeding and patients typically present later in life. In addition, none of the answers provided
would have corrected for VWD, which would have required the infusion of von Willebrand
factor concentrates or cryoprecipitate.
As the platelet number was normal in the child in this vignette, this is not idiopathic
thrombocytopenic purpura requiring intravenous gamma globulin. The normal PTT rules out
types A and B hemophilia, so infusion of factor VIII or IX is not indicated. The severity of the
reported bleeding, especially with circumcision and gum bleeding, makes child abuse unlikely in
the absence of additional concerning history, therefore child protective services does not need to
be contacted.
PREP Pearls
• The formation of a clot requires normal platelet numbers and function, as well as the ability
to form fibrin.
• The treatment for bleeding in a patient with a platelet function disorder is the infusion of
platelets with normal function.
• Two of the most common heritable platelet function disorders are Glanzmann
thrombasthenia and Bernard-Soulier syndrome.
Suggested Readings
• Carpenter SL, Abshire TC, Anderst JD. Evaluating for suspected child abuse: conditions that
predispose to bleeding. Pediatrics. 2013;131(4):e1357-e1373.DOI:
http://dx.doi.org/10.1542/peds.2013-0196.
• Journeycake JM, Buchanan GR. Coagulation disorders. Pediatr Rev. 2003;24(3):83-91. DOI:
http://dx.doi.org/10.1542/pir.24-3-83.
• Sharathkumar AA, Pipe SW. Bleeding disorders. Pediatr Rev. 2008;29(4):121-130. DOI:
http://dx.doi.org/10.1542/pir.29-4-121.
Question 91
You see an 8-year-old boy who has been struggling in school for the past several years. He was
recently tested and identified by his school as having significant learning disabilities in both
reading and writing. Neither of his parents had any personal experience with this kind of learning
difficulty; in fact, they had found school to be relatively easy. They express worries about how
their son will cope and adjust to this problem.
A. ask the school to adjust his curriculum such that he almost never experiences "failure"
on academic assignments
B. emphasize that parents should uphold firm or "tough love" academic expectations for
him at home
D. encourage him to view that his disability is something to cope with rather than
something in his power to control
Promoting resilience and empowerment for a child with a learning disability can improve their
self-esteem, reduce social isolation, and improve academic performance. There is no single path
for generating resilience given the multiple overall contributions. However, elements for
addressing self-esteem should include:
1. Providing the child with resources and opportunities to become competent and
capable (ie, special education instruction)
2. Encouraging the child to become a self-advocacy expert in their own best path toward
accomplishments
3. Ensuring that the child has a social network that supports stress mediation and
increases their sense of competence
Having at least 1 caring adult within the school that the student knows they can turn to and who
will be their advocate is thought to be a critical element to academic success. Therefore,
encouraging a nurturing relationship with an educator at the school is likely to improve a child’s
self-esteem and promote resilience.
Removing all possibilities for experiencing “failure” would not be a productive strategy because
learning requires being challenged and thus occasionally experiencing failure. Also, developing
resilience to adversity is a very important life skill, which cannot be learned without actually
managing stress and adversity. Parents enforcing rigid academic expectations at home (such as
negative consequences for receiving a poor grade) will be unlikely to promote coping if done
without regard to the child’s own learning style and abilities. Instead, it is more helpful if parents
expect their child to exert an effort in learning, effort in homework completion, and achievement
commensurate to their abilities. Viewing a learning disability just as a burden to be coped with is
a less adaptive and self-effective strategy than developing self-effectiveness and mastery over it.
For instance, it should become natural for the child to tell an educator in a self-assertive fashion,
“I learn best when I….” Parents celebrating a child’s accomplishments rather than only focusing
on their child’s struggles or failures can enhance self-esteem. Thus, if parents only talk about and
discuss what the child is struggling to do, a cumulative message of inadequacy would be
delivered.
PREP Pearls
• A learning disability creates self-esteem challenges because of the child experiencing
repeated academic “failures,” and some degree of social isolation from peers.
• Having at least 1 caring adult within the school that the student knows they can turn to and
who will be their advocate may be critical to academic success for a child with a learning
disability.
American academy of pediatrics 295
American Academy of Pediatrics PREP 2016
Suggested Readings
• Brooks RB. Children at risk: fostering resilience and hope. Am J Orthopsychiatry.
1994;64(4):545-553.
• Roer-Strier D. University students with learning disabilities advocating for change.
Disabil Rehab. 2002;24(17):914-924. DOI:
http://dx.doi.org/10.1080/09638280210148611.
Question 92
A 13-year-old adolescent is transferred into your practice and comes for an initial visit and
physical examination. He was full term, has never been hospitalized or had surgery, and takes no
medications. He has a history of a heart murmur and the family brings you a copy of his last
cardiac evaluation done 5 years ago. The report states that he has a bicuspid aortic valve. He
plays baseball and soccer without any symptoms. On physical examination today, he has a heart
rate of 72 beats/min, respiratory rate of 18 breaths/min, and blood pressure of 115/65 mm Hg in
the right arm and 105/65 mm Hg in the right leg. He has no jugular venous distension. His lungs
are clear to auscultation. His cardiac examination reveals a regular rhythm. His S1 and S2 are
normal. There is a 1/6 systolic murmur at the left mid sternal border, which does not radiate into
the neck. There is no rub or gallop. His abdominal examination is normal. His femoral pulses are
brisk. He has been invited to participate in a rugged 2-week survival course and his parents want
your opinion as to whether it is safe for him to participate and what his long term prognosis is.
Of the following, the MOST accurate advice or assessment for them at this time is
The presence of a murmur does not necessarily mean that the patient will need exercise
restriction, but the BP does suggest coarctation of the aorta, which would require restriction.
Although the murmur is soft and the gradient across the aortic valve is likely to be mild, this
finding is not the most important in this patient because of concern for concomitant coarctation.
This patient will have a 30% risk of developing complications of BAV, rather than 75%.
Endocarditis prophylaxis is no longer recommended for patients with BAV. Endocarditis
prophylaxis is recommended only for those patients with unrepaired cyanotic congenital heart
disease, repaired congenital heart disease which utilize prosthetic material, or those with residual
defects or cardiac transplant with valvular disease.
PREP Pearls
• Bicuspid aortic valves may not be symptomatic in childhood, but frequently become so
later in life.
• Bicuspid aortic valves are associated with coarctation of the aorta.
Suggested Readings
• Beekman RH III. Coarctation of the aorta. Moss and Adams’ Heart Disease in Infants,
Children, and Adolescents: Including the Fetus and Young Adult. 8th ed. Philadelphia,
PA: Lippincott Williams & Wilkins; 2012: 1044-1060.
Question 93
You are seeing a 4-year-old girl in your office for a sick visit. Her parents are concerned that the
girl is having difficulty walking up stairs and has been falling more than usual. She has been
complaining of pain in her thighs, especially when walking long distances. The girl had coldlike
symptoms 2 weeks ago and developed a rash on her hands and elbows 2 days ago. The parents
deny any sick contacts, travel history, or animal exposures. The remainder of the review of
systems is unremarkable. On physical examination, you note erythematous, hyperkeratotic, flat
papules overlying her metacarpophalangeal joints and elbows bilaterally. Strength testing is 3/5
with hip flexion and 4/5 with knee flexion and extension bilaterally. There is no upper extremity
weakness. Her reflexes and sensation are normal.
Of the following, the BEST next step in evaluating this patient is
B. edrophonium testing
C. electromyography
The clinical presentation of JDM affects several different systems (Item C93). Children often
present with constitutional signs including fever, fatigue, malaise, anorexia, and weight loss.
Musculoskeletal involvement usually presents as proximal muscle weakness and pain, with
weakness occurring predominantly in the lower limb girdle and lower extremities making it
difficult to climb stairs or walk. Gowers and Trendelenburg signs are often present. Weakness
can affect the anterior neck flexor and back muscles, making holding up the head and overhead
arm use difficult. Dysphonia and nasal speech can be an ominous sign, indicating pharyngeal,
hypopharyngeal, and palatal muscle weakness that places the patient at risk for aspiration.
Children may have arthralgia or even a mild arthritis, and some develop contractures because of
myofascial inflammation.
Most patients will develop the pathognomonic Gottron papules and heliotrope discoloration of
the eyelids. Gottron papules are erythematous to violaceous, sometimes scaling, flat-topped
lesions located over the extensor surfaces of interphalangeal or metacarpophalangeal joints. The
heliotrope rash is often accompanied by eyelid swelling.
Some patients present with a malar type rash. There is often dilation of the capillary loops of the
nailfold with corresponding cuticle overgrowth. Lipodystrophy may occur. Cutaneous vasculitic
ulcerations are commonly seen at the corners of the eyes, in the axillae and at pressure points.
Calcinosis may occur in the muscle or subcutaneous layer, and can be debilitating. Some patients
experience visceral vasculopathy, which can cause severe abdominal pain, melena, and
hematemesis, and can be life threatening. Although serious cardiac involvement is rare,
conduction defects have been associated with mortality. Respiratory muscle weakness is also
rare, but can cause restrictive pulmonary disease. Osteopenia and osteoporosis are associated
with JDM independent of treatment with corticosteroids.
Laboratory evaluation for suspected JDM should include markers of serum sarcoplasmic muscle
enzymes including creatine phosphokinase (CPK), aspartate transaminase (AST), alanine
aminotransferase (ALT), aldolase, and lactic acid dehydrogenase (LDH) levels. Elevated levels
of these enzymes are evidence of cellular breakdown. Because AST and ALT elevations can be
caused by muscle or liver disorders, simultaneously obtaining CPK and gamma-glutamyl
transferase (GGT) levels can help determine the origin. CPK and AST are usually elevated 20 to
40 times normal early in the disease, though they will occasionally be normal in the acute phase.
Generally CPK rises first, and aldolase and LDH are last to increase. LDH level can correlate
with disease activity. Additional useful laboratory tests include C-reactive protein (CRP) level
and erythrocyte sedimentation rate (ESR), markers for inflammation that can help differentiate
an inflammatory myopathy, like JDM, from other myopathies. The presence of factor VIII–
related antigen or von Willebrand factor antigen would be evidence of endothelial cell damage
Several autoantibodies are associated with JDM. Rheumatoid factor is usually negative. The
evidence regarding antinuclear antibody (ANA) positivity is widely variable, with studies
demonstrating between 10% and 85% positivity in JDM. Myositis-specific autoantibodies
(MSAs) and myositis-associated antibodies (MAAs) are more commonly positive in adult JDM
patients.
Muscle biopsy is the gold standard in diagnosis, and the typical findings in dermatomyositis are
necrosis, phagocytosis, perifascicular atrophy, fiber size variation, and perivascular
inflammation. However, muscle involvement is patchy and pathology can be missed if a good
biopsy site is not identified.
Edrophonium testing is used for diagnosing myasthenia gravis. This diagnosis is unlikely for the
girl in the vignette, who has a distinctive rash and does not have progressive weakness or signs
of muscle involvement other than her legs.
Electromyography (EMG) can be useful in the diagnosis of JDM, but is invasive and can be
difficult in children. Although EMG will help to localize the problem at the nerve or muscle
level, it would not be the next step in testing. The dystrophin gene is used to test for muscular
dystrophy. The patient in the vignette has an inflammatory muscle disease with rash, weakness,
and abnormal inflammatory markers that would not be seen in muscular dystrophy.
Magnetic resonance imaging (MRI) is often used to identify areas of myositis or to determine a
biopsy site. MRI of the musculature has largely replaced electromyography, because it is less
invasive. MRI of the spine would be appropriate if there were a concern for spinal cord
pathology. The girl in the vignette did not have any changes in sensation or sensory level
consistent with spinal cord compression, and her family did not report urinary or stool
incontinence.
PREP Pearls
• Creatine phosphokinase, aspartate transaminase, alanine aminotransferase, aldolase, and
lactic acid dehydrogenase levels are usually elevated in juvenile dermatomyositis.
• The gold standard for diagnosing juvenile dermatomyositis is muscle biopsy.
• Proximal muscle weakness is the pattern of muscle involvement associated with juvenile
dermatomyositis.
Suggested Readings
• Khan S, Christopher-Stine L. Polymyositis, dermatomyositis, and autoimmune
necrotizing myopathy: Clinical Features.Rheum Dis Clin North Am. 2011; 37:143- 158.
DOI: http://dx.doi.org/10.1016/j.rdc.2011.01.001.
Question 94
A 13-year-old obese adolescent presents to your office for evaluation of left anterior knee pain.
The pain began 2 or 3 weeks before this visit and had an insidious onset. You observe that the
patient has a slight limp and out-toed gait on the left side. He denies history of trauma, knee
swelling, knee instability, and catching or locking of the knee joint.
Of the following, the MOST important element to include in this patient’s physical examination
is
D. to assess anterior translation of the tibia relative to the femur with the patient supine
and the knee flexed to 30 degrees (Lachman maneuver)
E. to load the tibia onto the femur and rotate the lower leg with the patient in a prone
position and the knee flexed to 90 degrees (Apley grind test)
With SCFE, the femoral head epiphysis “slips” away from the femoral neck. The peak incidence
of SCFE is at ages 12 to 15 years in boys and ages 10 to 13 years in girls. Although
approximately 70% of affected children are above the 80th percentile for weight, pediatric
providers should bear in mind that 30% of patients with SCFE are not overweight. SCFE is
bilateral in about 40% of cases. In some cases, patients present with bilateral symptoms whereas
in others, SCFE develops in the contralateral hip subsequently. SCFE is classified as acute in
patients with symptoms of less than 3 weeks’ duration and chronic in patients with symptoms
lasting 3 weeks or more. SCFE can be further classified as stable (when patients can bear weight)
or unstable. Children with SCFE typically present with a painful limp but some children will not
have pain. Because comorbid endocrine abnormalities (eg, hypothyroidism, panhypopituitarism)
exist in 5% of patients with SCFE, patients should be asked about systemic symptoms. On
physical examination, obesity and decreased hip internal rotation are the most common findings.
Hip radiographs typically show widening of the physis on anterior-posterior view. On the lateral
view, a “Klein line” drawn along the superior border of the femoral neck should intersect the
femoral head, and should be symmetric with comparison of both hips. The absence of these
features is diagnostic of SCFE. Magnetic resonance imaging (MRI) should be considered for
patients with normal radiographs and a high index of suspicion for SCFE. This may help detect a
“preslip” in which the MRI may demonstrate bone marrow edema adjacent to the proximal
femoral physis or slight separation at the physis.
Patients with SCFE should not bear weight and should be referred to an orthopedic specialist for
urgent surgery. Internal fixation of the femoral head with screws is the most common treatment.
Bilateral surgery is generally performed in patients with bilateral symptoms or in the presence of
risk factors (eg, body mass index >35, young age) for a subsequent contralateral slip. Patients
with SCFE have an increased risk of developing osteoarthritis; the risk increases with a higher
degree of slippage.
Although complete physical examination of the knee is important in patients presenting with
knee pain, the boy in the vignette has classic signs and symptoms of SCFE, therefore assessing
hip rotation would be the most pressing option. This boy has anterior knee pain of insidious
onset. Palpation of the medial joint line would be most useful to assess for a medial meniscal
tear, which typically results from an acute injury in a young patient. Tenderness noted on patellar
facet palpation could indicate patellofemoral pain syndrome (PFPS). Although PFPS would be in
the differential diagnosis of anterior knee pain, this syndrome would not result in an out-toed
gait; detection of PFPS is less urgent than detection of SCFE. The Apley grind test is used to
assess for a meniscal tear and the Lachman test is used for an anterior collateral ligament (ACL)
American academy of pediatrics 304
American Academy of Pediatrics PREP 2016
injury. Meniscal and ACL tears occur as a result of an acute mechanism of injury in this age
range, but the patient in the vignette reports insidious onset of pain.
PREP Pearls
• Hip pathology can present as pain referred to the knee.
• Obligate external hip rotation is a classic finding with slipped capital femoral epiphysis
(SCFE) and can present as an out-toed gait.
• Thirty percent of patients with SCFE are not overweight.
• A child with suspected SCFE should not bear weight on the affected extremity and
should be referred for urgent evaluation.
Question 95
A previously healthy 12-year-old boy complains of worsening vision in the left eye over the last
several weeks without other symptoms. His visual acuity is 20/80 in the left eye and 20/20 in the
right eye. Fundoscopic examination of the left eye reveals a chorioretinal scar with surrounding
retinitis nasal to the disk and mild overlying vitritis (Item Q95). The right eye is normal.
Item Q95
Fundoscopic examination of the left eye for the boy described in the vignette. Courtesy of V. Chen. Reprinted with
permission from Pickering LK, et al. Red Book® Online. Elk Grove Village, IL: American Academy of Pediatrics.
Of the following, the test MOST likely to establish the cause of the patient’s findings is
Isolated ocular toxoplasmosis develops in up to 85% of adolescents and young adults after
untreated congenital infection. It also can become reactivated years after initial infection in both
immunocompetent and immunocompromised individuals. Patients with ocular toxoplasmosis
may complain of blurred vision, decreased acuity, epiphora (excessive tear formation), eye pain,
floaters, photophobia, or scotoma. Chorioretinitis caused by Toxoplasma infection may lead to
vision loss.
Toxoplasma gondii is found worldwide, and the seroprevalence (11% in the United States) of
infection varies by geographic area and socioeconomic status. Felines are definitive hosts.
Intermediate hosts such as pigs, sheep, and cattle can have tissue cysts in multiple organs.
Humans become infected by ingestion of oocysts from soil (or cat feces) or contaminated food or
water. Congenital infection results from primary maternal infection during gestation and is
estimated to occur in 1 in 1,000 to 1 in 10,000 live births.
The duration of therapy for congenital toxoplasmosis is prolonged, usually lasting approximately
1 year. For immunocompromised patients, long-term antibiotic suppression often is indicated
after the initial course of therapy to prevent recurrence.
Ocular manifestations of infection with T pallidum (syphilis) can include uveitis (anterior or
posterior) that usually is granulomatous. Posterior uveitis is more common and manifests as
chorioretinitis that is multifocal, retinal necrosis, or optic neuritis. Patients usually have
decreased visual acuity, often in association with syphilitic meningitis. The diagnosis is made
serologically and not with blood culture.
Acanthamoeba causes keratitis that is indolent and can resemble keratitis caused by herpes
simplex virus (HSV) or bacteria. Patients typically present with severe pain, photophobia,
tearing, and the sensation of a foreign body in the eye. Classic findings include a stromal ring
infiltrate and radial keratoneuritis. Diagnosis is made with corneal scrapings to identify the
organism.
PREP Pearls
• In most immunocompetent individuals, Toxoplasma infection either is asymptomatic or
results in a self-limited and benign flulike illness.
• In ocular toxoplasmosis, the chorioretinal lesion is described as white focal retinitis with
overlying vitreous inflammation that has the appearance of a “headlight in the fog.”
• Isolated ocular toxoplasmosis develops in up to 85% of adolescents and young adults
after untreated congenital infection.
• Most cases of acquired toxoplasmosis in immunocompetent hosts do not require
antimicrobial therapy.
• Pyrimethamine plus sulfadiazine are the drugs of choice for treating toxoplasmosis.
Suggested Readings
• American Academy of Pediatrics. Amebic meningoencephalitis and keratitis. In:
Kimberlin DW, Brady MT, Jackson, MA, Long SS, eds. Red Book: 2015 Report of the
Committee on Infectious Diseases. 30th ed. Elk Grove Village, IL: American Academy
of Pediatrics; 2015:231-234.
Question 96
A 16-year-old high school football player presents to the pediatric emergency department with
pain in his chest and left shoulder after an injury that he sustained during a football game,
approximately 1 hour ago. While running and holding the football in his left arm, he was tackled
from behind. The patient fell forward and landed forcefully on his chest, with the weight of the
opposing player on top of him. At that time, he heard a “cracking” sound and immediately felt
pain in his chest and left shoulder.
In the emergency department, the boy is alert and fully oriented. He is very uncomfortable and is
holding his hand over the left side of his chest. He tells you that it is difficult to breathe. His vital
signs include a heart rate of 100 beats/min, respiratory rate of 24 breaths/min, blood pressure of
130/80 mm Hg, temperature of 37°C, and pulse oximetry of 96% on room air. On physical
examination, his breath sounds are clear and equal bilaterally. The patient is taking shallow
breaths because of pain, but is not in respiratory distress. He is tender to palpation over the left
sternoclavicular junction, as well as over his left first rib, and you note bruising over these areas.
He has no focal tenderness on examination of his left shoulder, but he refuses to move his left
shoulder due to pain. You find no evidence of trauma to his head and his cervical spine is
nontender to palpation. The remainder of his physical examination findings, including a full
neurologic examination, is unremarkable.
An electrocardiogram reveals sinus tachycardia with no other abnormality. You administer an
intravenous analgesic and order plain radiographs of the patient’s chest and left shoulder, which
reveal a non-displaced fracture of his left first rib. His left shoulder radiograph reveals no
fracture or dislocation. He continues to complain of severe pain over his left sternoclavicular
joint and subjective dyspnea, and refuses to move his left shoulder due to pain. His vital signs are
unchanged.
A. bone scan
C. echocardiography
D. no further imaging
Physical examination and/or plain radiography are sufficient to identify many chest wall injuries
in children; however, CT of the chest may be needed to evaluate for bony injuries that are not
apparent on plain radiography (eg, posterior sternoclavicular fracture or dislocation) and to
investigate for associated intrathoracic injuries (eg, injury to the airway or great vessels). In
general, stable children with fractures to the first rib and sternum should be evaluated with CT of
the chest because of the high risk for associated intrathoracic injury.
It is important for all pediatric providers to know how to appropriately evaluate children who
have sustained chest wall trauma. Pediatric chest wall injuries generally arise from blunt trauma
to the thorax. The forceful mechanism of the trauma, such as a motor vehicle accident, can cause
additional serious injuries. All children with chest wall injuries should undergo a full physical
examination, beginning with assessment of their airway, breathing, circulatory, and neurologic
status, and then progressing to a head-to-toe secondary survey after any life-threatening
conditions have been addressed. It is important for clinicians to keep in mind that, because the
rib cages of children are generally more pliable than those of adults, traumatic forces may be
transmitted to their intrathoracic organs; thus, significant intrathoracic injuries such as
pulmonary contusion may occur even in the absence of injury to the chest wall structures.
Chest wall injuries may include fractures of the ribs, sternum, clavicles, and scapulae, with rib
fractures being the most common in children. Multiple rib fractures may result in flail chest, an
uncommon but very serious injury that can cause respiratory insufficiency from a compromise to
the structural integrity of the chest wall. Fractures of the upper ribs (especially the first and
second), sternum, and scapulae should lead clinicians to consider more serious intrathoracic
injuries such as pulmonary contusion, injury to the intrathoracic vessels, cervical spine injuries,
and injuries to the trachea or esophagus. This concern is due to the large amount of force
generally required to fracture these bones, given their protected anatomic positions.
A bone scan may identify fractures that are not apparent on plain radiography, particularly stress
fractures, but would not be helpful in excluding injury to intrathoracic structures.
Echocardiography certainly has a role in the evaluation of children who have sustained chest
wall trauma, as a means of excluding cardiac injury. Echocardiography is indicated for those
with abnormal electrocardiograms, ectopy, abnormal cardiac enzyme levels, or other evidence of
cardiac injury after thoracic trauma. The boy in the vignette does not meet any of these criteria.
Many children with uncomplicated chest wall injuries, including those with simple clavicle
fractures and fractures to the middle or lower ribs will not require further evaluation beyond
plain radiography. However, the boy in the vignette has a clinical picture suggestive of a
posterior sternoclavicular joint dislocation, and CT is indicated to evaluate for this injury.
Furthermore, CT of the chest would be indicated in this patient because of the presence of the rib
fracture.
Finally, plain radiography of the ribs would not aid in the evaluation for associated intrathoracic
injuries or dislocation of the posterior sternoclavicular joint, which is indicated for the boy in the
vignette. Standard anterior-posterior and lateral radiographs of the chest are sufficient for
detecting the majority of rib fractures, therefore dedicated plain radiographs of the ribs are not
needed in the evaluation of most patients.
PREP Pearls
• The thoracic cages of children are more pliable than those of adults, resulting in the
transmission of traumatic forces to the intrathoracic organs; significant intrathoracic
injuries may occur even in the absence of injury to the chest wall.
• Computed tomography (CT) of the chest may be indicated to evaluate for injuries to chest
wall structures that are not apparent on plain radiography (eg, posterior sternoclavicular
fracture dislocations) and to investigate for associated intrathoracic injuries (eg, airway or
great vessels).
• In general, stable children with fractures to the first rib or sternum should be evaluated
with CT because of the high risk for associated intrathoracic injury.
Suggested Readings
• Garg S, Alshameeri ZA, Wallace WA. Posterior sternoclavicular joint dislocation in a
child: a case report with review of literature. J Shoulder Elbow Surg. 2012;21:e11-e16.
DOI: http://dx.doi.org/10.1016/j.jse.2011.07.007.
Question 97
You are caring for a 600 g, 25-week gestation male in the intensive care unit. The neonate
remains on a ventilator for respiratory distress syndrome 2 days after delivery. He is receiving
light-emitting diode phototherapy for indirect hyperbilirubinemia (total bilirubin 6 mg/dL [102.6
µmol/L] and direct bilirubin 0.5 mg/dL [8.6 µmol/L]) while on an open warmer. Over the past 24
hours, the intravenous fluids have been increased to 140 mL/kg per day of intravenous dextrose
10% in water and his urine output has been 4 mL/kg per hour. Ultrasonography of the head
reveals a right grade III intraventricular hemorrhage. The laboratory value trend over 48 hours is
shown in Item Q97.
Of the following, the BEST next step in care for this neonate is to
B. discontinue phototherapy
D. initiate desmopressin
Insensible water loss rates up to 200 mL/kg per day may be seen in extremely premature infants.
If tape or topical agents do not compromise the integrity of the premature skin further, the skin
will mature rapidly over the first week after birth and approach the insensible water loss rate of
20 mL/kg per day seen in a full-term infant. A premature infant on an open radiant warmer may
have an increase of up to 50% in evaporative water loss. Placing the infant in a humidified
isolette can markedly decrease these losses.
Full-term and late preterm infants typically require 60 to 80 mL/kg of fluid per day to meet their
maintenance requirements and replace ongoing losses. The fluid requirement increases to 90 to
120 mL/kg of fluid per day for infants born before 28 weeks’ gestational age, primarily because
of the increased insensible losses from evaporative water loss. Caution must be used to avoid
fluid overload in premature infants, as noted in some studies that demonstrated increased rates of
bronchopulmonary dysplasia in infants with greater fluid intake.
The premature infant in the vignette has a complicated medical course. The increasing serum
sodium, chloride, blood urea nitrogen, and creatinine all suggest excessive free water loss that
would be improved by placement in a humidified isolette. Changing the intravenous fluid to
dextrose 5% in water would drop the serum glucose but not provide additional free water. The
infant requires treatment for hyperbilirubinemia and the light emitting diode phototherapy does
not increase insensible water loss. Increasing the intravenous fluid rate of the dextrose 10% in
water would give more free water, but worsen the infant’s hyperglycemia and lead to an osmotic
diuresis. Because diabetes insipidus is rarely seen with grade III intraventricular hemorrhage in
the premature infant and the urine output is not excessive, initiation of desmopressin is not
indicated.
PREP Pearls
• The high body surface–to-mass ratio and immaturity of the skin found in infants of less
than 28 weeks’ gestational age leads to high transepidermal water loss through
evaporation.
• Evaporative water loss can be minimized in the premature infant by placement in a
humidified isolette.
• Light-emitting diode phototherapy does not increase transepidermal water loss.
• Understand how prematurity and the use of radiant warmers affect insensible water loss,
especially in preterm infants
Suggested Readings
• Dell KM. Fluids, electrolytes, and acid-base homeostasis. In: Martin RJ, Fanaroff AA,
Walsh MC, eds. Fanaroff and Martin’s Neonatal-Perinatal Medicine. 9th ed. St. Louis,
MO: Elsevier Mosby; 2011:669-677 .
Question 98
A 10-year-old girl is brought to your office for possible seizures. her parents report that several
times, as she started to laugh at something, the girl suddenly leaned over and her eyelids sagged
for a few seconds. There was no loss or alteration in consciousness and no twitching or limb
jerking. She has been falling asleep at school and her parents report that, on the weekends, the
girl naps all afternoon. Recently, she chose to take a nap rather than attend a friend's birthday
party. The girl appears to get good sleep at night, her hair and skin seem normal, she has not had
any recent illnesses, and she does not have headaches. Her parents are not aware of any recent
stressors or toxic exposures, and no one else in the family has similar symptoms. On physical
examination, the girl weighs 51 kg (8 kg more than at her health supervision visit 3 months
previously) her blood pressure is 98/74 mm Ho, heart rate is 92 beats/min, and respiratory rate is
32 breaths/min. Her physical examination including neurological examination is unremarkable
Of the following, the test MOST likely to yield the correct diagnosis is
A. Depression screening
B. Electroencephalogram
C. Magnetic resonance imaging of the brain
D. Multiple sleep latency test
E. Thyroid stimulating hormone
In narcolepsy, a sleep study is diagnostic. Overnight polysomnography will confirm the absence
of sleep apnea or other sleep disorders causing excessive daytime sleepiness, and the multiple
sleep latency test conducted the next day as part of the sleep study will show the presence of
rapid eye movement (REM) sleep during at least 2 daytime naps. Normally, naps are too short to
allow the emergence of REM sleep.
The differential diagnosis for excessive daytime sleepiness includes depression, disorders of
attention, absence seizures, hypothyroidism, conversion disorder, Epstein-Barr virus infection,
hypothalamic or pituitary tumors, and other more rare disorders. The girl in the vignette does not
have signs or symptoms of any of these disorders. In addition, none of these disorders are
characterized by cataplexy, which she does have.
Symptoms of cataplexy can be differentiated from atonic seizures (“drop attacks”) and absence
seizures because cataplexy is provoked by strong emotion such as laughter or sudden fright.
Absence seizures can be provoked by prolonged hyperventilation but not by sudden emotion, and
atonic seizures are unprovoked.
The presence in a child of excessive daytime sleepiness, weight gain, and cataplexy should
prompt the clinician to consider narcolepsy and order polysomnography with a multiple sleep
latency test to make the diagnosis.
PREP Pearls
• The presence of excessive daytime sleepiness, weight gain, and cataplexy should prompt
the clinician to consider narcolepsy.
• Cataplexy, the sudden loss of muscle tone provoked by strong emotion such as laughter
or fright, suggests a diagnosis of narcolepsy.
• Narcolepsy is diagnosed by polysomnography with a multiple sleep latency test.
Suggested Readings
• Martin S. Index of suspicion. Pediatr Rev. 2009;30(3):107-113. DOI:
http://dx.doi.org/10.1542/pir.30-3-107.
• Rosen GM, Bendel AE, Neglia JP, Moertel CL, Mahowald M. Sleep in children with
neoplasms of the central nervous system: case review of 14 children. Pediatrics.
2002;112(1)e46-e54. Available
athttp://pediatrics.aappublications.org/content/112/1/e46.full.
Question 99
An 11-year-old boy has been growing at the third percentile in the last 4 years of age (Item Q99).
His parents are concerned because both of their heights are at the 25th percentile and they are
worried he will not catch up. The boy is otherwise healthy. He denies headaches or abdominal
pain. The boy’s bone age is 12 months delayed from his chronologic age. His parents request
further evaluation to identify the cause of his poor growth.
Of the following, the BEST next step in the evaluation of this child is to
In cases where laboratory evaluation for short stature is warranted, measurement of insulin like
growth factor 1 (IGF-1) and/or insulin like growth factor binding protein 3 (IGF-BP 3) could be
used to assess the GH axis. Random or AM GH levels are generally not helpful, because GH
levels peak in deep sleep and are often very low during the day. Thus IGF levels are a better test,
and GH stimulation testing can be used if IGF levels are low. Measurement of thyroid-
stimulating hormone and free thyroxine may be used to evaluate for hypothyroidism, a common
cause of poor linear growth.
If GH deficiency is suspected, or there is concern for another pituitary finding such as a tumor,
cranial magnetic resonance imaging (MRI) can contribute significantly to diagnosis. In some
studies, more than 25% of children with GH deficiency will have an abnormal finding on MRI,
including pituitary hypoplasia, empty sella, hypoplastic anterior pituitary, missing pituitary stalk,
and ectopic posterior pituitary. Children with abnormal MRI findings tend to have more severe
characteristics of GH deficiency. However, in children such as the boy in the vignette, linear
growth is normal with a normal predicted final height, so no further laboratory or radiologic
assessment is needed.
PREP Pearls
• An endocrine cause of poor growth is likely when linear growth slows down or arrests
while weight gain is either normal or increasing.
• In children with normal growth velocity and normal final height prediction, reassurance
is appropriate and no further evaluation is needed.
Suggested Readings
• Maghnie M, Lindberg A, Koltowska-Häggström M, Ranke MB. Magnetic resonance
imaging of CNS in 15,043 children with GH deficiency in KIGS (Pfizer International
Question 100
A 15-year-old adolescent is brought to your office with a complaint of chronic cough. He has
had a cough for "as long as he can remember." He has been treated with short courses of
antibiotics for bronchitis and with systemic steroids, inhaled corticosteroid, and short acting β-
agonist therapy for asthma without reported improvement. His cough is wet and productive of
greenish sputum without blood. He is moderately short of breath with exertion. He has had poor
weight gain with malodorous stools and has been frustrated by his inability to gain muscle mass
with weight lifting.
There is no history of recurrent ear or sinus infections. The patient was born at term without
perinatal or neonatal respiratory complications.
On physical examination, you note a thin, alert adolescent in no acute distress. His respiratory
rate is 24 breaths/min. Auscultation of lungs reveals bilateral and diffuse coarse crackles without
wheezing. A chest radiograph shows a diffuse bilateral increase in interstitial markings with
notable peribronchial thickening. There is no focal infiltrate or effusion. Pulmonary function
testing is obtained, which confirms moderate obstructive lung disease with a forced expiratory
volume in 1 second of 58% predicted.
A computed tomography of the chest is obtained and a representative image is shown (Item
Q100).
Of the following, the MOST likely etiology of this patient’s respiratory complaints is
B. cystic fibrosis
C. hypogammaglobulinemia
D. Kartagener syndrome
E. sarcoidosis
PREP Pearls
• Bronchiectasis generally denotes irreversible airway damage from chronic inflammation
or infection.
• Bronchiectasis may occur early in life in children with cystic fibrosis.
• Bronchiectasis causes decrements in lung function and is associated with decreased
survival.
Suggested Readings
• Redding GJ. Bronchiectasis in children. Pediatr Clin North Am. 2009;56(1):157-171.
doi:http://dx.doi.org/10.1016/j.pcl.2008.10.014.
• Sly PD, Gangell CL, Chen L, et al. Risk factors for bronchiectasis in children with cystic
fibrosis. N Engl J Med. 2013;368(21):1963-1970. doi:
http://dx.doi.org/10.1056/NEJMoa1301725.
Question 101
A 4-day-old newborn presents for his newborn check. His mother, who has not breastfed before,
reports that she is having a lot of pain with breastfeeding despite assistance from a lactation
consultant. She has heard that tongue-tie can cause breastfeeding problems and wonders whether
the baby needs to have his frenulum clipped.
Of the following, the MOST appropriate information to provide this mother is that
C. strong evidence exists that newborn frenulotomy improves the baby’s long term
speech outcome
D. there are reliable tools for determining which infants may benefit from frenulotomy
One of the great difficulties in assessing the effects of frenulotomy is in determining which
patients have clinically significant issues related to ankyloglossia. Tongue-tie is an anatomic
variant that affects approximately 2% to 5% of infants with a male predominance. However,
probably no more than half of affected infants have difficulty with breastfeeding, and even fewer
will have future problems with speech development. Some lactation consultants and researchers
have used the Hazelbaker Assessment Tool for Lingual Frenulum Function to evaluate tongue
and frenulum appearance and tongue function with good inter-rater reliability for appearance, but
not for all of the function factors. Other researchers have used ultrasonography to assess tongue
function pre- and postfrenulotomy. Neither tool is universally available or utilized. The need for
intervention is based on more subjective criteria. Most authors recommend a maximal trial of
conservative interventions to support breastfeeding before considering frenulotomy, but there are
no accepted guidelines as to if or when intervention should occur.
To date, there is not strong evidence to support a relationship between ankyloglossia and later
speech problems. In one study, there was no statistically significant difference in speech
articulation between controls and children with ankyloglossia who did or did not have a
frenulotomy during infancy.
PREP Pearls
• Ankyloglossia is an anatomic variant present in 2% to 5% of newborns with male
predominance.
• Frenulotomy can facilitate breastfeeding, improve milk transfer, and decrease maternal
nipple pain for many affected infants and mothers.
• There is no widely accepted method to assure detection of those infants with
ankyloglossia who will benefit from frenulotomy.
Suggested Readings
• Buryk M, Bloom D, Shope T. Efficacy of neonatal release of ankyloglossia: a
randomized trial. Pediatrics. 2011;128:280-288. DOI:
http://dx.doi.org/10.1542/peds.2011-0077.
Question 102
You are seeing a 15-year-old adolescent for her annual health supervision visit. During the
assessment, she reveals that she is sexually active. She reports 2 lifetime sexual partners. Both
partners were male. She reports unprotected vaginal and anal sex. Her last menstrual period was
2 weeks ago. Her physical examination is unremarkable. Her urine pregnancy test is negative and
screens for sexually transmitted infections are pending.
A. discuss the importance of dual protection with hormonal contraceptives and barrier
methods
B. discuss the need for an initial Papanicolaou test to screen for cervical dysplasia
C. discuss the need for the patient to have her partners screened for sexually transmitted
infections
D. explain to the patient that the age of sexual consent is 16 years of age
E. explain to the patient that unprotected anal sex does not place her at risk for HIV
acquisition
Sexually active adolescents should receive screening and counseling for sexually transmitted
infections and pregnancy prevention. In July 2014, the AAP Committee on Adolescence and the
Society for Adolescent Health and Medicine issued the policy statement Screening for Nonviral
Sexually Transmitted Infections in Adolescents and Young Adults, which recommends
gonorrhea and chlamydia screening for all sexually active female adolescents and young adults,
sexually active adolescent and young adult males who have sex with males, and other males
from high prevalence areas. The United States Preventive Services Task Force (USPSTF)
recommends HIV testing for individuals 15 years of age and older, with subsequent testing based
on risk.
Sexually active adolescents should be counseled about contraceptive options. The American
College of Obstetricians and Gynecologists and the AAP recommend that adolescents are
counseled on the first line use of long acting, reversible contraceptives (eg, intrauterine
contraceptives, contraceptive implants) for pregnancy prevention. Additionally, it is imperative
to counsel adolescents on the dual use of hormonal contraceptives with barrier methods for the
prevention of pregnancy and STIs, and is recommended for the adolescent in this vignette.
The USPSTF recommends cervical cancer screening for women ages 21 to 65 years with
cytology (Papanicolaou smear) every 3 years or every 5 years for women ages 30 to 65 years,
with a combination of cytology and human papillomavirus testing. The USPSTF specifically
recommends against screening for women younger than 21 years of age.
While screening for STIs in sexual partners may be important, it does not negate the need for use
of dual protection for the prevention of STIs and unintended pregnancy. Unprotected anal sex is
a significant risk factor for HIV acquisition. The age of sexual consent varies by state.
PREP Pearls
• Sexuality should be addressed during the health supervision visit at each period of
adolescence, starting at the early adolescent visit.
• Taking a sexual history allows the clinician to identify individuals at risk for acquisition
of sexually transmitted infections or unintended pregnancy, as well as sites for anatomic
testing and guide risk reduction counseling.
Suggested Readings
• American Academy of Pediatrics, Society For Adolescent Health And Medicine.
Screening for nonviral sexually transmitted infections in adolescents and young adults.
Pediatrics. 2014;134(1):e302-e311. DOI:http://dx.doi.org/10.1542/peds.2014-1024.
Question 103
During the health maintenance examination of a 6-month-old female infant, you note a diaper
rash. There is erythema involving the convexities of the thighs and suprapubic area; the skin
folds are not involved (Item Q103). Her physical examination is otherwise unremarkable.
Of the following, the MOST appropriate treatment for this infant’s rash is topical
A. clindamycin
B. clotrimazole
C. nystatin
D. triamcinolone/nystatin
E. zinc oxide
Folliculitis represents infection of hair follicles, usually with Staphylococcus aureus. In the
diaper area, it is characterized by erythematous papules and pustules centered about follicles.
The buttocks are commonly involved (Item C103A). In mild or localized forms of the infection,
clindamycin may be applied topically. More extensive cases require an oral antistaphylococcal
antibiotic.
Infection of the diaper area with Candida albicans may occur primarily or complicate an existing
ICD. The eruption is composed of “beefy-red” erythematous patches that involve the convexities
and inguinal creases (Item C103B). Scaling at the periphery of patches, or satellite papules and
pustules may be observed. Treatment of diaper candidiasis is with topical nystatin or an
imidazole (eg, clotrimazole, ketoconazole, miconazole).
PREP Pearls
• The most common forms of diaper rash are irritant contact dermatitis, candidiasis, and
seborrheic dermatitis. Irritant contact dermatitis involves the convexities, but spares the
folds, while candidiasis and seborrheic dermatitis involve the convexities and folds.
Candidiasis often is bright red, while seborrheic dermatitis appears orange-pink.
• Consider referral to a pediatric dermatologist if a diaper rash does not resolve with
appropriate therapy.
• Refer to a pediatric dermatologist infant who have atypical findings (eg, petechiae,
papules with hemorrhagic crust), as these may be signs of Langerhans cell histiocytosis.
Suggested Readings
• Krowchuk DP, Mancini AJ. Diaper dermatitis. Pediatric Dermatology: A Quick
Reference Guide. 2nd ed. Elk Grove Village, IL: American Academy of Pediatrics;
2012:501-509.
Question 104
A 12-year-old boy is brought to the emergency department after being involved in a house fire.
He is currently awake, alert, and oriented. He is complaining of throat and chest pain. His
temperature is 37°C, pulse is 110 beats/min, respiratory rate is 24 breaths/min, and blood
pressure is 115/70 mm Hg. His oxygen saturation by pulse oximetry is 100% on room air.
Physical examination shows an emotionally upset child in no apparent physical distress. There is
no apparent trauma to the head, trunk, or extremities. There is soot around his mouth, nose, and
the exposed skin on his neck, hands, and feet. When the soot is washed away, there is no redness
or burn to any area of skin. The boy’s oropharynx is erythematous. He is breathing comfortably.
On auscultation, his lungs are clear with transmitted upper airway sounds, and his heart has a
regular rhythm. The boy’s extremities are warm and well-perfused. His abdomen is soft,
nontender, and non-distended. His pupils are equal, round, and reactive. The remainder of his
neurologic examination is unremarkable.
A. dexamethasone, 10 mg intravenously
B. endotracheal intubation
Inhaled heated air alone rarely causes sufficient physical burn to cause airway compromise.
However, smoke has a higher heat capacity because of suspended particulate material, and can
thus distribute thermal injury to the upper airway. Significant airway injury can occur without
evidence of burns to the skin because the airway is relatively less resistant to thermal injury.
Because airway resistance is inversely proportional to the radius of the airway to the fourth
power, there is exponentially more airway resistance in younger children or with decreasing
inner diameter of the airway as the disease process worsens. Thus, if a child has a history and
any sign of smoke exposure, the airway should be evaluated immediately. This holds true even if
the child looks well, because if the airway edema worsens while the disease process declares
itself, endotracheal intubation becomes more immediately lifesaving but also more technically
difficult. Although the child in this vignette looks clinically well, he displays signs of smoke
inhalation that could portend progression of airway edema, such as soot around the mouth and
nose, transmitted upper airway sounds, and an erythematous oropharynx. Other signs of smoke
inhalation not present in this child include singed nasal hairs, soot-stained secretions, and
hoarseness.
The best modality for airway evaluation is direct laryngoscopy and bronchoscopy by an airway
expert. However, if personnel are not readily available, endotracheal intubation is recommended.
Dexamethasone can help reduce airway inflammation and swelling, but is not likely to prevent
injury. Although controversial, hyperbaric oxygen therapy can be used to mitigate hypoxic injury
in carbon monoxide poisoning from house fires, but is not recommended in a child with a normal
neurologic status. Methylene blue can be used to treat methemoglobinemia, which can occur in
victims of house fires, but will not be helpful in treating airway injury. Noninvasive positive
pressure ventilation can be helpful in cases of upper airway obstruction caused by decreased
airway tone, but not from airway swelling.
PREP Pearls
• A child with smoke inhalation may be at a higher risk for airway edema than predicted by
clinical appearance.
• The airway of a child with smoke inhalation should be evaluated immediately because
airway edema can worsen rapidly and also lead to a technically difficult intubation.
• Signs of smoke inhalation in children include soot around the mouth and nose,
pharyngeal erythema, singed nasal hairs, soot-stained secretions, and hoarseness.
Suggested Readings
• Fidowski CW, Fuzaylov G, Sheridan RL, Coté CJ. Inhalation burn injury in children.
Pediatr Anaesth. 2009;19(suppl 1):147-154. DOI: http://dx.doi.org/10.1111/j.1460-
9592.2008.02884.x
Question 105
A 13-month-old girl is hospitalized with fever and pancytopenia. She has a history of an
orthotopic liver transplant for biliary atresia. She currently is on tacrolimus and prednisone for
rejection prophylaxis and trimethoprim-sulfamethoxazole for pneumocystis prophylaxis. Vital
signs show a temperature of 38.8°C, respiratory rate of 44 breaths/min, heart rate of 130
beats/min, blood pressure of 121/67 mm Hg, and weight of 9.85 kg. On physical examination,
she is irritable, mottled and has several erythematous macules over her face. Her abdomen is
distended and you note hepatosplenomegaly. Laboratory data shows:
• White blood cells, 1,110/µL (1.1 x 109/L)
• Hemoglobin, 7.5 g/dL (75 g/L)
• Platelets, 42 x 103/µL (42 x 109/L)
• Differential, 1% segmented neutrophils, 18% bands, 78% lymphocytes, 2% monocytes,
1% eosinophils
• Aspartate aminotransferase, 90 U/L
• Alanine aminotransferase, 61 U/L
• Human herpesvirus 6 polymerase chain reaction, 261,000 copies/mL
• Tacrolimus level 11.5 ng/mL
A. human herpesvirus 6
B. liver transplant
C. prednisone
D. tacrolimus
E. trimethoprim-sulfamethoxazole
By 2 years of age, nearly all children have been exposed to HHV-6. The seroprevalence in adults
is as high as 95%. Human herpesvirus 6 establishes latency and can reactivate in the setting of
immune suppression, such as after solid organ transplantation. The incidence of reactivation
peaks at 2 to 4 weeks after transplantation. While reactivation of HHV-6 is common, clinical
illness ascribable to HHV-6 is not, occurring in only 1% of solid organ transplant recipients.
Prednisone would not be expected to cause pancytopenia. Tacrolimus can cause leukopenia,
anemia, and thrombocytopenia. However, the child in this vignette has therapeutic levels and
additional symptoms that cannot be explained by receipt of tacrolimus. Similarly, while
trimethoprim-sulfamethoxazole can cause bone marrow suppression, it cannot explain all of the
findings manifested by the patient in the vignette.
PREP Pearls
• Human herpesvirus 6 (HHV-6) is the causative agent of roseola (exanthema subitum), a
classic childhood exanthem characterized by a period of high fevers followed by a
generalized maculopapular rash.
• Like other Herpesviridae, HHV-6 establishes latency after primary infection and can
reactivate, especially in the setting of immune suppression.
• The manifestations of HHV-6 infections in transplant recipients can include fever, rash,
hepatitis, bone marrow suppression, pneumonia, encephalitis, and graft rejection.
Suggested Readings
• American Academy of Pediatrics. Human herpesvirus 6 (including roseola) and 7. In:
Kimberlin DW, Brady MT, Jackson, MA, Long SS, eds. Red Book: 2015 Report of the
Committee on Infectious Diseases. 30th ed. Elk Grove Village, IL: American Academy
of Pediatrics; 2015: 449-451.
Question 106
You are evaluating a 4-month-old infant in the emergency room for vomiting and decreased oral
intake. The baby was born to a 29-year-old primigravida woman at 39 weeks of gestation via
normal vaginal delivery. The baby’s birth weight was 3.6 kg (50th percentile) and had a length of
50 cm (50th percentile). The mother mentions that his pediatrician has been concerned about his
weight gain for the last month. His vital signs show a weight of 5.0 kg (less than the third
percentile), length is 55.2 cm (less than the third percentile), temperature of 38.0°C, heart rate of
126 beats/min, respiratory rate of 30 breaths/min, and a blood pressure of 70/60 mm Hg. His
physical examination is significant for mild dehydration, excessive watering of the eyes,
photophobia, frontal bossing, and wrist widening.
His urinalysis today demonstrates a specific gravity of 1.005, a pH of 7.0, 3+ glucose, and no
blood, protein, leukocyte esterase, or nitrites.
A. Bartter syndrome
B. Gitelman syndrome
The presence of glycosuria, rickets, and photophobia, along with excessive watering of the eyes,
is indicative of Fanconi syndrome in a patient with RTA and is most commonly associated with
proximal (type 2) RTA, as demonstrated in this infant. Fanconi syndrome, characterized by
generalized proximal tubular dysfunction as evidenced by rickets (phosphaturia leading to
hypophosphatemic rickets), glycosuria (dipstick positive glycosuria with normal plasma glucose
concentration), and aminoaciduria or tubular proteinuria (urine dipstick negative for protein and
quantitative urine tests positive for amino acids and protein). Other RTAs (type 1 or type 4) or
Bartter and Gitelman syndromes are not associated with Fanconi syndrome.
Renal tubular acidosis occurs from the inability of the kidneys to keep up with the excretion of
the daily endogenous acid production arising from dietary proteins and amino acids. The renal
response to endogenous acids leads to maintenance of normal plasma HCO3-, the extracellular
buffer. Urinary ammonia (NH3) and monohydrogen phosphate (HPO42-) are the 2 principal
buffers binding the endogenously produced H+ ions. Of these, increased urinary ammonia
excretion is the main adaptive response (can be increased by 10-fold) to increased acid excretion,
as urinary phosphate excretion is fixed. Therefore, all forms of RTA are associated with
metabolic acidosis (low plasma HCO3-) and decreased renal ammonium (NH4+) excretion.
Ammonia excretion by the kidneys can be explained as a 4-step process, each of which is
implicated in different types of RTA. Ammonium (NH4+) is produced by the proximal tubule
from glutamine and secreted by the proximal tubule into the tubular fluid via Na+-H+ exchanger
by binding to the H+ site. This step primarily generates the NH4+, which is subsequently
processed in the renal tubule to regeneration of new HCO3- and also contributes to restoration of
the bicarbonate pool. The next step is the absorption of NH4+ by the thick ascending loop of
Henle via the Na-K-2Cl channel by occupying the K site. Ammonium in the cell dissociates into
NH3 and H+. The apical membrane of thick ascending loop of Henle is impermeable to NH3,
while the basolateral membrane is highly permeable. This leads to increased movement of NH3
across the basolateral membrane into the medullary interstitium and a gradient of increasing
concentration of NH3 in the deeper medulla similar to the countercurrent system. This is the
third step in normal ammonium secretion by the kidney. The fourth step involves diffusion of
American academy of pediatrics 343
American Academy of Pediatrics PREP 2016
NH3 into the cortical collecting duct, wherein it reacts with H+ ions secreted by the intercalated
A cells. This H+ secretion (proton donated by H2CO3, which dissociates into HCO3- and H+)
leads to the addition of a new HCO3- molecule to the circulation (Item C106A).
The kidneys maintain acid base balance in vivo by: 1) reclaiming the filtered preformed HCO3-
in the tubular fluid 2) regeneration of new HCO3- to account for HCO3- lost in buffering of the
endogenous acids
Eighty percent to 90% of the filtered bicarbonate is reclaimed by the proximal tubule and the
remaining by the thick ascending loop of Henle. Reclamation is an appropriate term to describe
the processes of the proximal tubule, as the HCO3- molecule returning to the circulation is
different from the filtered HCO3- (Item C106B). Proximal RTA is a defect in the ability of the
proximal renal tubules to reclaim filtered HCO3- from the tubular fluid. Fanconi syndrome, a
form of proximal RTA, has been associated with cystinosis, galactosemia, tyrosinemia,
oculocerebrorenal (Lowe) syndrome, and hereditary fructose intolerance. The presence of
excessive eye watering and photophobia is indicative of cystinosis as the cause of the patient’s
symptoms and RTA. A slit lamp examination will reveal the characteristic cysteine crystals and
confirm the diagnosis. In proximal RTA, once the serum bicarbonate has dropped to a level
(around 12-15 mEq/L [12-15 mmol/L]) wherein the distal renal tubules can absorb the filtered
bicarbonate load, the urine can be appropriately acidified and the presenting urine pH may be
less than 5.5.
Type 3 RTA is rare and associated with marble bone disease. It has findings of both type 1
(impaired regeneration of HCO3-) and type 2 (bicarbonate wasting) RTA. Treatment of RTA
involves correction of metabolic acidosis via sodium or potassium citrate solutions. Potassium
citrate solutions are avoided in hyperkalemic RTA. As the distal tubules are responsible for
regulating only 5% to 10% of the acid load, the alkali dose is higher for proximal (10-20 mEq/kg
per day), as compared to distal or hyperkalemic (5-8 mEq/kg per day) RTA.
Bartter syndrome and Gitelman syndrome are characterized by hypokalemia and metabolic
alkalosis. Bartter syndrome often presents in childhood with growth retardation, hypokalemia,
metabolic alkalosis, and polyuria or polydipsia. Bartter syndrome is caused by a primary defect
in sodium chloride reabsorption in the medullary thick ascending limb of the loop of Henle,
similar to chronic furosemide therapy. Gitelman syndrome generally presents in late childhood
or adulthood with muscle cramps (hypokalemia), polyuria, or polydipsia. Mutations in the gene
encoding for the thiazide-sensitive Na-Cl transporter in the distal tubule have been identified in
patients with Gitelman syndrome. In contrast to Bartter syndrome, patients with Gitelman
syndrome have reduced urinary calcium and hypomagnesemia (more common).
PREP Pearls
• The diagnosis of renal tubular acidosis (RTA) should be considered in a young infant
with failure to thrive, recurrent vomiting, and episodes of dehydration.
• Proximal RTA is most commonly associated with Fanconi syndrome, characterized by
generalized proximal tubular dysfunction, as evidenced by rickets, glycosuria, and
aminoaciduria.
• Patients with cystinosis develop cystine deposits in the cornea and the conjunctiva,
leading to photophobia, watering, and blepharospasm.
• Bartter syndrome and Gitelman syndrome are characterized by hypokalemia and
metabolic alkalosis.
Suggested Readings
American academy of pediatrics 346
American Academy of Pediatrics PREP 2016
• Chan JCM, Scheinman JI, Roth KS. Consultation with the specialist: renal tubular
acidosis. Pediatr Rev. 2001;22(8):277-287. DOI: http://dx.doi.org/10.1542/pir.22-8-277.
Question 107
You are caring for a 13-year-old adolescent with a 4-month history of periumbilical pain and
intermittent nausea with a normal, soft, nonbloody bowel movement daily. She denies a history
of emesis, headaches, joint pain, and weight loss. She has had an extensive evaluation including:
• Normal abdominal radiograph without evidence of constipation
• Normal complete blood cell count, electrolytes, and liver panel
• Stool heme test is negative
• Celiac and Helicobacter pylori serum screens are negative
A. acid blockade
B. laxatives
C. no medications
D. probiotics
E. tricyclic antidepressant
The Rome criteria are useful in establishing the diagnosis of FAP. Childhood functional
abdominal pain is defined by the 2006 Rome III criteria as episodic or continuous abdominal
pain with some loss of daily function and no or only occasional relationship to psychological
stress. There must be insufficient criteria to meet the diagnosis of any other functional
gastrointestinal disorder. The evaluation must be negative for evidence of inflammatory,
anatomic, metabolic, or neoplastic process to explain the symptoms. The pain must occur at least
weekly for 2 months. If the patient demonstrates any of the red flags for pathologic chronic
abdominal pain (Item C107B), additional evaluation is required.
Based upon the Rome III criteria and current literature, the evaluation for children with chronic
abdominal pain should include a thorough history and physical examination looking for possible
red flags. If no red flags are identified and the child meets the described diagnostic criteria, the
American academy of pediatrics 350
American Academy of Pediatrics PREP 2016
diagnosis of FAP is made. There is no longer a need to “rule out” other disease processes. If
there are red flags or continued concerns for an organic etiology, the evaluation may include
complete blood cell count, inflammatory markers, liver and pancreatic enzymes, infectious
evaluation, and possible endoscopy with biopsy as clinically indicated.
The role of pharmacotherapy is limited in FAP, with placebo success rates being quite high in
functional gastrointestinal disorders. Randomized controlled trials in pediatrics are lacking,
therefore limiting recommendations for medications.
Psychological therapies are increasingly recommended for children with chronic abdominal pain
and functional abdominal pain. This may include cognitive behavioral therapy, relaxation
training, and hypnotherapy. Studies show that children utilizing cognitive behavioral therapy
have a higher rate of resolution of pain.
The adolescent in this vignette has chronic periumbilical pain with intermittent nausea and no
evidence of constipation. There are no red flags to raise concern for an organic etiology. The
literature shows no consistent benefit to treatment with tricyclic antidepressants, probiotics, or
acid blockade. With no evidence of constipation, the use of laxatives is not appropriate, however,
this should be considered in a child with irritable bowel syndrome – constipation type.
PREP Pearls
• Cognitive behavioral therapy has been shown to be effective in children with functional
gastrointestinal diseases to include functional abdominal pain.
• Pharmacotherapy has not been shown to have significant benefit in children with
functional abdominal pain, although studies are very limited.
• If a child lacks any red flag symptoms, no significant workup is needed for diagnosis.
Suggested Readings
• Gottsegen D. Complementary, holistic, and integrative medicine: recurrent abdominal
pain. Pediatr Rev. 2010;31(5):e36-e39. DOI: http://dx.doi.org/10.1542/pir.31-5-e36.
Question 108
A 16-year-old adolescent presents for a health supervision visit. The father reports his son is an
endearing, shy teenager with a mild reading disability. The adolescent's height is at the 85th
percentile and his weight is at the 35th percentile. The physical examination is remarkable for an
adolescent with a thin build, disproportionately long arms and legs, gynecomastia, and small
testes.
Of the following, the MOST appropriate next step for his diagnosis is
B. karyotype analysis
C. serum prolactin
D. serum testosterone
E. testicular ultrasonography
A multidisciplinary approach is best at managing the overall care of a patient with Klinefelter
syndrome and could include an endocrinologist, urologist, geneticist, primary care physician,
psychologist, psychiatrist, and a fertility specialist. Medication regimens during transition into
adulthood focus on testosterone replacement therapy. Patients with Klinefelter syndrome are also
prone to thyroid dysfunction, autoimmune diseases, dental caries, metabolic syndrome, varicose
veins, thrombosis, and malignancy (breast cancer, lung cancer, lymphoma, and
nonseminomatous germ cell tumors). They are not at increased risk for testicular cancer or
prostate cancer compared to the general population.
From a behavioral standpoint, many patients with Klinefelter syndrome have a higher incidence
of anxiety, depression, attention-deficit/hyperactivity disorder, autism spectrum disorders,
substance abuse, or other psychiatric disorders. They tend to be introspective, shy, and sensitive.
Infertility can lead to psychological concerns as well. A fertility counselor is quite helpful in
discussing the options of testicular sperm extraction, artificial insemination with donor sperm, or
adoption. From an educational standpoint, they tend to experience delayed verbal development
and learning difficulties (75%).
The best diagnostic test to determine the etiology of a patient presenting with the clinical
constellation described in this vignette is a karyotype. A low serum testosterone level and a
testicular ultrasound showing testicular fibrosis would be informative, but not diagnostic. A
serum prolactin would be helpful in a patient with galactorrhea, not gynecomastia, because of
concerns for a secretory pituitary adenoma. Klinefelter patients do not have structural anomalies
of the brain, so a magnetic resonance image of the brain would not be helpful.
PREP Pearls
• Klinefelter syndrome, or 47,XXY syndrome, is the most frequent chromosomal disorder
in males affecting 1 to 2 patients per 1,000. Although diagnosis is easy, given the
gynecomastia and small testes in affected males, Klinefelter syndrome is often missed
and untreated until adulthood.
• The best diagnostic test for Klinefelter syndrome is a karyotype.
• Testosterone replacement therapy, as well as identification and management of the
comorbid disorders, remain important determinants in positive outcomes for patients with
Klinefelter syndrome.
Suggested Readings
• Aksglaede L, Juul A. Testicular function and fertility in men with Klinefelter syndrome:
a review. Eur J Endocrinol. 2013;168(4):R67-R76. doi: http://dx.doi.org/10.1530/EJE-12-
0934 .
Question 109
The parents of a 2-day-old, 35-week gestation, male newborn request to speak with the
pediatrician about routine circumcision. They are interested in having the procedure performed
before discharge. However, they want to discuss whether current methods of analgesia are safe
and effective.
Of the following, the MOST accurate statement to include when counseling this family is that
C. topical analgesic creams are as effective as a dorsal nerve or subcutaneous ring block
D. topical lidocaine cream has a slower onset of action than topical lidocaine-prilocaine
cream
E. topical lidocaine-prilocaine can cause skin irritation, especially in low birth weight
infants
Circumcision is the surgical removal of some, or all, of the foreskin (or prepuce) from the penis.
This procedure yields specific health benefits that include prevention of urinary tract infections,
decreased acquisition of human immunodeficiency virus, decreased transmission of sexually
transmitted diseases, and a lower risk of penile cancer. Analgesia that is safe and effective in
reducing the pain associated with newborn circumcision is available and should always be
provided.
Both the dorsal penile nerve block and subcutaneous ring block are effective options for
analgesia. Onset of the anesthestic effect occurs after approximately 7 minutes for both
procedures. Nonpharmacologic techniques are not sufficient when used alone to manage the pain
associated with circumcision. Comfortable positioning and oral sucrose may be used as adjunct
therapies, but neither should be used as the sole method of analgesia. Topical lidocaine-
prilocaine cream does attenuate circumcision pain, but is less effective than either the dorsal
penile nerve block or subcutaneous ring block. In addition, there are case reports that suggest a
rare risk of methemoglobinemia with lidocaine use. Topical 4% lidocaine has a faster onset of
action than lidocaine-prilocaine cream (20-30 versus 60-90 minutes, respectively).
PREP Pearls
• Safe and effective analgesia should always be provided to reduce the pain associated with
newborn circumcision.
• When used for circumcision, the adverse effects of topical anesthetic creams are
uncommon (though more common in low-birthweight infants) and are usually limited to
mild skin irritation.
• Both the dorsal penile nerve block and subcutaneous ring block are effective options for
analgesia.
• Neither nonpharmacologic techniques nor topical analgesics are sufficient to manage the
pain associated with circumcision.
Suggested Readings
• American Academy of Pediatrics Task Force on Circumcision. Male circumcision.
Pediatrics. 2012;130(3):e756-e785. doi:http://dx.doi.org/10.1542/peds.2012-1990.
Question 110
You are caring for a 4-year-old girl hospitalized for newly diagnosed acute lymphoblastic
leukemia. The child has a hemoglobin level of 6 g/dL (60 g/L) and has been tachycardic and
fatigued. You order a transfusion of packed red blood cells. One hour into the transfusion, she
develops a temperature to 38.5°C.
Of the following, the procedure that is MOST likely to have prevented this reaction is
E. using only blood products that have had extended antigen phenotyping
The infusion of even small numbers of granulocytes can lead to the release of pro-inflammatory
cytokines, thereby increasing the risk of a febrile transfusion reaction. In order to reduce this
risk, granulocytes are removed from blood products at the time of initial processing or
immediately prior to transfusion via filtration, a process called leukodepletion. For the girl in the
vignette, leukodepletion of the red blood cells (RBC) prior to transfusion would have had the
greatest likelihood of reducing her risk of fever.
Diphenhydramine reduces the risk of an allergic transfusion reaction through the blockade of
histamine, but will not influence the risk of fever. While bacterial infections can certainly be
transmitted through blood product transfusions and can lead to fevers, culturing the packed cells
prior to transfusion is neither practical nor feasible. Once thawed, blood products need to be
transfused prior to culture results being available.
Although granulocytes are removed through filtration, many lymphocytes are the size of RBC
and are therefore not as effectively removed by filtration. While RBCs are typed by ABO typing,
they are not human leukocyte antigen typed or matched to the recipient, and lymphocytes infused
with a transfused product may recognize the recipient as foreign. In immune compromised hosts,
the host immune system is unable to clear the infused donor lymphocytes, which may then
undergo expansion in response to recognition of a foreign antigen and cause a severe, typically
fatal, graft-versus-host reaction. In immune competent hosts, this does not occur, as the host
immune system recognizes the donor lymphocytes as foreign and successfully eliminates them.
Irradiation of blood products prior to transfusion renders donor lymphocytes replication
incompetent, eliminating the threat of transfusion-associated graft-versus-host disease.
There is a marked discrepancy in the surface antigens commonly found on the RBCs of different
subpopulations or ethnic groups and those commonly found on the RBCs in the general donor
pool. These differences can lead to the development of alloantibodies to RBC antigens when
transfused with RBCs, with surface antigens not routinely evaluated in routine RBC typing
(minor RBC antigens). Patients with alloantibodies to minor antigens are at higher risk of
hemolytic transfusion reactions when infused again with RBCs that have had a standard typing
for only ABO and Rh antigens. It is important to perform extended spectrum RBC typing,
especially in patients receiving chronic transfusions, to obtain the best possible match of RBC
surface antigens between donor and recipient in order to reduce the risk of a hemolytic
transfusion reaction. This is especially important in children with sickle cell disease.
PREP Pearls
• Leukodepletion removes granulocytes from blood products through filtration, thereby
reducing the risk of a febrile transfusion reaction.
• Irradiation of blood products renders donor lymphocyte replication incompetent, thereby
eliminating the risk of transfusion-associated graft-versus-host disease.
• Extended spectrum typing can improve the minor surface antigen match on red blood
cells between the donor unit and the recipient, thereby reducing the risk of a hemolytic
transfusion reaction.
Suggested Readings
• Bell MD. Consultation with the specialist: red blood cell transfusions. Pediatr Rev.
2007;28(8):299-304. doi:http://dx.doi.org/10.1542/pir.28-8-299.
Question 111
You are seeing a 1-year-old boy who had an episode earlier today that greatly concerned his
parents. While playing with a toy, his older sister came by and took it away, which made him
start to cry vigorously. After a couple of crying exhalations, his mother saw him get very quiet
and "turn blue." Then his body became limp and he seemed to become "unconscious." The
mother said that although it seemed longer at the time, he may have remained this way for about
20 to 30 seconds before he woke up, took a few big breaths, and seemed to "act like his usual
self again."
Less commonly, a breath holding spell will involve a sudden pallid or pale appearance and loss
of muscle tone, typically after a painful experience, which looks like fainting. These pallid breath
holding spells are thought to come from abnormal vagal responses to a sudden emotion.
Occasionally, some tonic-clonic movements may occur, which if they do, may make it more
difficult to differentiate from a seizure based on history alone. Breath holding spells are not a
choice of the child, as in a child making a threat like "I'm going to hold my breath till I turn
blue!", but rather they represent a sudden involuntary reaction of an immature nervous system.
The hallmarks of a breath holding spell are that there is no loss of bowel or bladder control, no
post-ictal state, and a sequence of events that include crying from a trigger of anger, frustration,
or pain right before the incident happens. Normal consciousness and normal breathing are
rapidly restored (within 1 minute). It is unusual, but not impossible, to see tonic-clonic
movements. Diagnosis is based on the clinical history and reassurance can be provided because
of their common and self-resolving nature. If these spells are happening frequently, a parent can
be encouraged to video record an episode so that it can be reviewed with their provider to assure
that the clinical assessment is correct.
There are no recommended treatments for breath holding spells. There have been occasional
associations of breath holding spells with the presence of anemia or iron deficiency without
anemia, therefore checking for the presence of anemia and obtaining iron studies is reasonable.
Some have suggested that an electrocardiogram (EKG) be performed with all of these children
because of the very rare risk of long QT syndrome. However, this is not necessary unless the
reported symptom pattern is atypical (ie, no precipitating intense cry or emotional trigger, lasts
longer than a minute). An EKG could be ordered for an extra degree of reassurance, however,
referral to a cardiologist would only be appropriate if there were any abnormal findings on that
study or on the physical examination.
An electroencephalogram (EEG) would be appropriate if the story of the child's symptoms were
more consistent with a seizure disorder, such as appearing groggy or sluggish when the episode
American academy of pediatrics 363
American Academy of Pediatrics PREP 2016
ends. With the exception of an acute head injury, an EEG would not be appropriate. Performing
a lumbar puncture to evaluate for a central nervous system infection would be appropriate if
there was a fever, or persisting symptoms of illness such as listlessness or irritability. Given that
the child was immediately "acting like his usual self again," a central nervous system infection is
highly unlikely. Referral to a neurologist would also not be necessary, given the lack of any
seizure hallmarks such as post-ictal states.
PREP Pearls
• Breath holding spells are nonvolitionally triggered by the child's sudden experience of
anger, frustration, or pain
• Nearly 5% of children will experience a breath holding spell.
• Breath holding spells most commonly appear around 1 year of age, and if present, they
should disappear entirely by 5 years of age.
Suggested Readings
• Narchi H. The child who passes out. Pediatr Rev. 2000;21(11):384-
388.http://pedsinreview.aappublications.org/content/21/11/384.
Question 112
A 7-week-old infant was noted to have a short, soft 2/6 systolic murmur at the left upper sternal
border at 2 weeks of age and comes today for a follow-up visit. His mother has noticed that he
has been breathing rapidly for the last few days. The baby has been breastfeeding less than usual
for the last 2 days, taking feedings every 3 hours, and only for 5 min. When you enter the
examination room, you see an infant breathing at 90 breaths/min. The saturation is 68% in room
air taken on the right hand. The baby becomes fussy when you do your examination, but the
saturations are stable even with crying. The heart rate is 160 beats/min and the blood pressure is
80/60 mm Hg in the left leg. Physical examination shows the chest is clear, the respirations are
shallow but there are no retractions, and there is a long 4/6 systolic murmur at the left upper
sternal border that begins at S1 and is present for all of systole. You do not appreciate an ejection
click or any split to S2. There is no hepatosplenomegaly and the femoral pulses are 2+
bilaterally. You start oxygen in the office and the saturation increases to 75%. You call
emergency medical services and warn the emergency department that the baby is on the way.
You speak to the cardiologist on call.
Based on the clinical presentation and physical examination, the finding the echocardiogram is
MOST likely to show is
C. pulmonic stenosis
D. tetralogy of Fallot
The natural history of PS varies greatly. Patients may have stable and mild PS and remain
asymptomatic during childhood. Pulmonic stenosis severity is differentiated by the gradient
across the pulmonary valve on echocardiogram, as well as the estimated right ventricular
pressure compared to the pressure in the left ventricle. If there is no left ventricular outflow tract
obstruction, the left ventricular systolic pressure is estimated by the systolic blood pressure.
Right ventricular pressure can be estimated if there is adequate tricuspid regurgitation to measure
the difference between the right atrial and ventricular pressures. The tricuspid regurgitation
velocity allows us to calculate the difference in the pressure of the right ventricle and right
atrium. Pulmonary stenosis severity is categorized as:
• Mild PS is defined as a gradient up to 30 to 40 mm Hg or right ventricular (RV) pressure
less than 50% of systemic or left ventricular (LV) pressure.
• Moderate PS is defined as a gradient of 40 to 70 mm Hg or RV pressure 50% to 75% of
the LV pressure.
• Severe PS is a gradient greater than 70 mm Hg or an RV pressure greater than 75% of the
LV pressure. The RV pressure correlates with the degree of outflow tract obstruction.
The more severe form of PS can worsen quickly over the first few weeks of life if the newborn
does not present immediately after birth. The newborns with this form of PS usually have thick
and dysplastic valves and require close monitoring and referral for balloon angioplasty if they
begin to show desaturation or respiratory distress. Valve dysplasia is present in 10% to 20% of
patients with PS. Balloon angioplasty of critical PS in infancy has a 95% probability of relieving
American academy of pediatrics 366
American Academy of Pediatrics PREP 2016
the obstruction and not requiring re-intervention for up to 5 years. Mild pulmonary insufficiency
is common after the procedure.
PREP Pearls
• The murmur of pulmonic stenosis prolongs as the gradient across the valve increases.
• Severe pulmonic stenosis may be urgently treated with balloon angioplasty.
• The murmur in tetralogy of Fallot will decrease during a hypercyanotic spell.
Suggested Readings
• Park MK. Obstructive lesions: pulmonary stenosis. Park’s Pediatric Cardiology for
Practitioners. 6th ed. Philadelphia, PA: Saunders Elsevier; 2014:184-205.
Question 113
A 15-year-old adolescent girl presents to your office for a health supervision visit. She has a
history of migraine headaches that have improved with sumatriptan and naproxen as needed, but
she continues to have severe headaches that affect her daily activity. The patient is concerned
about feeling fatigued when she takes sumatriptan and is exploring more natural treatments for
her migraines. She has done research on the internet and has questions about herbal supplements,
such as butterbur and biofeedback therapy.
Of the following, your BEST response to her regarding these therapies would be that
D. they are not recommended as they are not approved by the US Food and Drug
Administration
Many patients with chronic conditions will pursue CAM as an option, especially when a
conventional treatment method has significant side effects or is perceived to be ineffective.
Patients use CAM for many reasons, and most will use CAM in conjunction with traditional
medical treatment. A study on the reasons and associations for CAM use reported that higher
education, poorer health status, anxiety, chronic pain, back problems, transformational
experiences, or a personal world view were associated with higher rates of CAM use. No
association was found between dissatisfaction with conventional therapy and CAM use. In recent
studies, up to 70% of patients seen in subspecialty pediatric clinics report CAM use. In
neurology clinics, although 60% of patients had tried CAM along with conventional therapy,
only 50% of those discussed the use of CAM with their physician. CAM use is widespread and
inadequately reported to their physicians, therefore it should be discussed as a routine part of
each patient visit.
PREP Pearls
• Up to 70% of families use complementary and alternative medicine (CAM) including
nutritional supplements.
• Patients often use CAM along with traditional medical therapy.
• A nonjudgmental approach to the use of CAM can prevent harm resulting from
interactions with traditional medical therapy.
American academy of pediatrics 369
American Academy of Pediatrics PREP 2016
Suggested Readings
• Adams D, Dagenais S, Clifford T, et al. Complementary and alternative medicine use by
pediatric specialty outpatients.Pediatrics. 2013;131:225-232. DOI:
http://dx.doi.org/10.1542/peds.2012-1220.
Question 114
The parents of an 8-year-old boy with trisomy 21 would like to enroll their child in a soccer
league. The child’s medical history is significant for ventricular septal defect repair at 6 months
of age. The boy does not report any neck pain. His parents deny gait disturbance, bowel or
bladder changes, or weakness. On physical examination, the child has low-normal muscle tone
and normal reflexes.
A. allow the child to participate after discussing his increased risk for atlantoaxial
instability with his parents
B. allow the child to participate as long as flexion and extension radiographs of the
cervical spine are normal
C. allow the child to participate as long as magnetic resonance imaging of the cervical
spine is normal
D. recommend the child not participate, given his history of congenital heart disease
E. recommend the child not participate, given his increased risk of atlantoaxial
instability
Atlantoaxial instability refers to excessive motion at the junction of the spinal atlas (C1) and the
axis (C2). After age 3 years, the bones of the cervical spine are adequately ossified to be
evaluated on plain radiography. The atlantodens interval (ADI), the distance from the atlas to the
anterior aspect of the dens on lateral radiograph of the cervical spine, has historically been used
to assess for atlantoaxial instability. Approximately 20% of children with trisomy 21 meet the
radiographic criteria for atlantoaxial instability; most of these individuals are asymptomatic. In
the absence of symptoms, a large ADI has not been associated with injury to the cervical spine.
In addition, a child with normal radiographs can subsequently develop atlantoaxial instability.
Because ADI measurements on radiography are neither sensitive nor specific for the risk of
cervical spine injury, routine screening is not recommended for asymptomatic children. The
American Academy of Pediatrics recommends counseling parents that children with trisomy 21
who participate in certain contact or collision sports (eg, soccer, football, and basketball) and
sports that require cervical spine flexion/extension (eg, gymnastics) are at increased risk for
cervical spine injury. In addition, parents should be advised that trampoline use is not
recommended for children with trisomy 21 who are younger than 6 years. Children age 6 years
or older should only be allowed to use trampolines with careful supervision.
Children who experience symptoms of spinal cord compression (eg, weakness, neck pain,
radicular symptoms, bowel or bladder symptoms) should undergo urgent evaluation of the
cervical spine. For these patients, providers should obtain anterior-posterior and lateral cervical
spine radiographs with the neck in a neutral position. If findings on these films are normal,
flexion and extension films should then be obtained. During health supervision visits, parents
should be counseled about the signs and symptoms of cervical myelopathy and the urgent need
to seek evaluation and treatment if symptoms develop; physical examination should include a
neurologic evaluation to look for weakness and hyperreflexia.
Despite the lack of evidence to support screening, the Special Olympics continues to require
cervical spine screening for participation in certain sports. The exact requirements for Special
Olympics screening vary by state, therefore providers who care for patients with trisomy 21
should be familiar with their state’s requirements.
Screening radiography and magnetic resonance imaging are not indicated for the boy in the
vignette because he is asymptomatic. This child had a ventricular septal defect that had been
repaired during infancy, which, in the absence of other congenital heart disease and pulmonary
hypertension, is not a contraindication to sports participation.
PREP Pearls
• Asymptomatic children with trisomy 21 do not need routine radiographic screening for
atlantoaxial instability.
• Children with cervical spine compression symptoms should have prompt radiographic
evaluation of the cervical spine, with anterior-posterior and lateral views in the neutral
position. Flexion and extension films should only be obtained once films obtained with
the cervical spine in neutral position are normal.
• Children with signs or symptoms of cervical spine compression should be urgently
referred to a pediatric orthopedic or neurosurgical specialist.
Suggested Readings
• Bull MJ and the Committee on Genetics. Health supervision for children with Down
syndrome. Pediatrics. 2011;128(2):393-406. DOI: http://dx.doi.org/10.1542/peds.2011-
1605.
Question 115
A 5-year-old girl, visiting the United States from India, is brought to the emergency department
with a complaint of fever, abdominal pain, back pain, and lethargy. On physical examination, her
temperature is 39.5°C, heart rate is 135 beats/min, respiratory rate is 30 breaths/min, and blood
pressure is 95/65 mm Hg. She is thin and ill appearing. Her conjunctivae are pale and mucous
membranes are tacky. Capillary refill is 2 seconds. There is an II/VI vibratory systolic ejection
murmur. Lungs are clear to auscultation. Her abdomen is diffusely tender to palpation, and the
liver and spleen are palpated 4 cm and 5 cm, respectively, below the costal margins. Examination
of the spine and paraspinal muscles is normal.
The most common malaria species are P vivax (most prevalent in the Indian subcontinent and
Central America) and P falciparum (worldwide but most prevalent in Africa, Papua New Guinea,
Haiti, and the Dominican Republic). Less common are P ovale (most prevalent in West Africa)
and P malariae (worldwide distribution). P vivax and P ovale can cause persistent hepatic
infection, which may result in reactivation of disease months to years later. Persistent, low-
concentration parasitemia caused by P falciparum or P malariae also can result in recurrence of
symptoms. Chronic infection caused by P malariae can lead to nephrotic syndrome. P knowlesi,
American academy of pediatrics 375
American Academy of Pediatrics PREP 2016
found in Southeast Asia, is a primate parasite that can infect humans and cause severe disease
due to hyperparasitemia.
Patients with malaria typically present with fever, chills, rigors, and headache, which may be
paroxysmal. Nausea, vomiting, diarrhea, abdominal pain, arthralgias, myalgias, and respiratory
symptoms can occur. Hepatosplenomegaly is common. Typical laboratory findings include
anemia and thrombocytopenia. P falciparum infection can be severe or even fatal, presenting
with neurologic manifestations (eg, seizure, confusion, coma), renal failure (acute tubular
necrosis), respiratory failure (pulmonary edema), severe anemia (from splenic sequestration or
hemolysis), metabolic derangements (hypoglycemia, metabolic acidosis), vascular collapse, and
shock. The treatment of malaria depends on the severity of disease, the species, and the
likelihood of drug resistance (See suggested reading no. 5 for recommended treatment).
Babesia microti causes babesiosis, characterized by mild, nonspecific symptoms such as malaise,
anorexia, and flulike symptoms. Clinical signs usually are minimal, unlike those described for
the girl in the vignette. Babesiosis is a tick-borne zoonosis most commonly acquired in the
Northeastern United States; the primary reservoir is the white-footed mouse, and the primary
vector is the tick Ixodes scapularis, which also can transmit Borrelia species. The organism is
difficult to distinguish from P falciparum on stained blood smears, but the clinical presentation
generally distinguishes one infection from the other.
Infection with Borrelia species can cause relapsing fevers, either tick borne or louse borne (B
recurrentis), as well as Lyme disease (B burgdorferi). Relapsing fever is characterized by high
fever, shaking chills, myalgias, arthralgias, headache, and nausea. A fleeting rash may occur.
Louse-borne disease occurs in Ethiopia, Eritrea, Somalia, and the Sudan. Tick-borne relapsing
fever occurs worldwide. The diagnosis is made by observing spirochetes (not ring-form parasites
as seen in malaria) on dark field microscopy or stained preparations of peripheral blood smears.
The symptoms of Lyme disease typically are indolent and occur in stages (early localized, early
disseminated, and late disease). Fever, arthralgias, myalgias, headache, and fatigue are common
during the early disseminated stage. The diagnosis of Lyme disease can be made clinically or
serologically.
Manifestations of Ehrlichia infection include fever, chills, malaise, headache, and myalgias.
Vomiting and diarrhea may also occur. Rash is present in 50% of children. Severe manifestations
of disease include disseminated intravascular coagulation, respiratory distress syndrome, renal
failure, and encephalopathy. Most cases of ehrlichiosis occur in California, Texas, and in the
southeast, northeast, and north central regions of the United States. The diagnosis typically is
made serologically or with commercially available polymerase chain reaction assays.
Examination of peripheral blood smears to detect morulae (inclusion bodies in the cytoplasm of
monocytes) is insensitive.
Most patients infected with the parasite Trypanosoma cruzi (Chagas disease) are asymptomatic.
Young children are more likely than adults to be symptomatic. The infection is transmitted by
the “kissing bug” (triatomine insects), and occurs primarily in Mexico and Central and South
America. A red, indurated nodule may form at the inoculation site on the face or arms. If the site
American academy of pediatrics 376
American Academy of Pediatrics PREP 2016
of entry is the conjunctiva, a Romana sign (unilateral eyelid edema) may occur. Fever, malaise,
hepatomegaly, and generalized lymphadenopathy may develop acutely, but resolve without
treatment within 3 months. Chagas cardiomyopathy is a late complication that occurs years to
decades after the initial infection. The diagnosis is established serologically, but parasites may be
observed in blood smears during the acute phase of infection.
PREP Pearls
• The treatment of malaria depends on the severity of disease, the infecting parasite, and
the likelihood of drug resistance.
• The most common malaria species in the world are P vivax (most prevalent in the Indian
subcontinent and Central America) and P falciparum (worldwide but most prevalent in
Africa, Papua New Guinea, Haiti, and the Dominican Republic).
• P vivax and P ovale can cause persistent hepatic infection which may result in
reactivation of disease months to years later.
Suggested Readings
• American Academy of Pediatrics. American trypanosomiasis. In: Kimberlin DW, Brady
MT, Jackson, MA, Long SS, eds. Red Book: 2015 Report of the Committee on Infectious
Diseases. 30th ed. Elk Grove Village, IL: American Academy of Pediatrics; 2015:803-
805.
Question 116
A 16-year-old adolescent girl is transported to the emergency department by paramedics
following a motor vehicle collision. Thirty minutes earlier, she was a passenger in the front seat
of a car that struck a tree. She was not wearing a seatbelt and the right side of her head hit the
passenger side window. She does not recall any further details about the collision.
The patient was awake when paramedics arrived at the scene, but then had a 1-min generalized
seizure, followed by several minutes of decreased responsiveness. Her level of consciousness has
improved gradually since the seizure occurred. She is now following commands and answering
questions, although she intermittently seems confused. She is maintaining her own airway, has
clear and equal breath sounds, and her vital signs are within normal limits. She has had no
episodes of vomiting.
On your physical examination, the patient is lying flat with her entire spine immobilized. She is
awake, follows your commands, and answers your questions, although she seems confused at
times. Her Glasgow Coma Scale score is 14. You detect the scent of alcohol on her breath. She
has a 4 x 6 cm boggy hematoma over her right temple, which is very tender to palpation. Her
pupils are 4 mm in diameter bilaterally and equally reactive. She has full strength and normal
reflexes in all extremities. The remainder of your physical examination is unremarkable.
B. electroencephalogram
Among trauma patients, head injury is the leading cause of death and disability. Although most
children sustaining head trauma have only minor injuries, a small number will have more serious
injuries with the potential for clinical deterioration and significant sequelae. Providers of
pediatric care should understand how to appropriately evaluate and initially manage children and
adolescents who present with trauma to the central nervous system.
Pediatric head injuries account for more than 600,000 emergency department (ED) visits and an
even larger number of visits and calls to primary care providers annually. Most pediatric head
trauma results from falls, motor vehicle collisions, automobile-pedestrian accidents, bicycle-
related injuries, and sports-related injuries. Most pediatric head injuries are minor, but some can
be extremely serious; more than 3,000 deaths related to head trauma occur in US children each
year. A challenge for pediatric providers is to differentiate the relatively small number of
children at high risk for intracranial complications and clinical deterioration after head trauma
from the many who are at very low risk. Clinical symptoms are neither completely sensitive nor
specific for significant injury.
Computed tomography of the brain is a rapid and accurate way to identify intracranial injuries in
children after head trauma. There is consensus that patients identified as being at high risk for
intracranial injury should undergo early noncontrast CT of the brain for evidence of intracranial
hemorrhage, midline shift, or increased intracranial pressure. Widespread use of this diagnostic
modality has its downsides, however, including exposure of the brains of developing children to
ionizing radiation, identification of minor lesions or incidental findings with unclear clinical
importance, the need for sedation for younger or uncooperative pediatric patients, and a
significant increase in health care costs. The goal should be to identify children with clinically
important intracranial injury after head trauma to prevent deterioration and secondary brain
injury, while limiting unneeded radiographic imaging in children at very low risk.
Recent studies, including a very large multicenter study conducted through the Pediatric
Emergency Care Applied Research Network (PECARN), have yielded validated clinical decision
guidelines that provide a useful clinical framework for determining which children are at higher
(as well as at very low) risk for clinically important brain injuries after head trauma. Based on
the PECARN study, the risk of clinically important traumatic brain injury is estimated at more
than 4% for children with any of the following: Glasgow Coma Score of 14 or other signs of
altered mental status, a palpable skull fracture (under age 2 years), or signs of basilar skull
fracture (≥2 years of age). A CT scan of the brain is thus recommended for these children. The
girl in the vignette would fall into this category.
American academy of pediatrics 379
American Academy of Pediatrics PREP 2016
Brain CT is not recommended for children who are found to be at extremely low risk (< 0.05%)
for clinically important traumatic brain injury. This includes children meeting all of the
following criteria: normal neurologic examination findings, normal mental status, normal
behavior as noted by a caregiver, no loss of consciousness, no vomiting, no severe headache, no
evidence of skull fracture (for children younger than 2 years, and no or frontal only scalp
hematoma), no signs of basilar skull fracture, no high-risk mechanism of injury, and no concern
for inflicted injury.
Children with no alteration of mental status or signs of skull fracture fall into the “intermediate”
risk category if they have a history of isolated loss of consciousness, headache, vomiting, or
certain scalp hematomas. For these children the PECARN guidelines recommend that the
decision about whether brain CT or observation alone is needed should be based on additional
clinical factors that include: the presence of multiple intermediate risk factors, age younger than
3 months, worsening symptoms or signs after ED observation, physician experience, and
parental preference. Observing a child in the ED when emergent imaging is not indicated
provides the opportunity to watch for symptom improvement or worsening. Children who
clinically worsen during a period of observation should undergo CT.
Initial management of cases of central nervous system trauma must focus on stabilizing the
airway, maintaining adequate ventilation and oxygenation, and maintaining adequate perfusion
to the brain and other vital organs, while ensuring continuous cervical spine precautions until a
cervical spine injury can be excluded.
Although the patient in the vignette did have a self-limited post impact seizure after sustaining
head trauma, she is displaying no indication of ongoing seizure activity in the emergency
department, therefore electroencephalography would not be useful at this time.
Plain radiography of the skull may identify a skull fracture in this patient, but this diagnostic
study would provide no direct information about the presence of intracranial injury.
The patient's history and clinical presentation suggest that she was likely drinking alcohol before
the motor vehicle collision. Although intoxication with ethanol could indeed explain her
intermittent confusion, this symptom could also be evidence of a life-threatening intracranial
injury. Therefore, a brain CT should not be delayed in obtaining a serum ethanol level in this
patient.
Similarly, although the girl may have abused other drugs before the motor vehicle collision,
ruling out a serious intracranial injury must take greater priority in her evaluation and
management than obtaining a urine toxicology screen.
PREP Pearls
• Initial management of cases of central nervous system trauma must focus on stabilizing
the airway, maintaining adequate ventilation and oxygenation, ensuring perfusion to the
brain and other vital organs, and continuing cervical spine precautions until cervical spine
injury is excluded.
• Computed tomography (CT) of the brain is a rapid and accurate way to identify
intracranial injuries in children after head trauma. Risks associated with CT include
exposure to ionizing radiation, identification of minor lesions or incidental findings,
potential need for sedation, and significant increases in health care costs.
• Recent studies have yielded validated clinical decision guidelines that provide a useful
framework for determining which children are at higher risk for clinically important brain
injuries after head trauma, while limiting unnecessary imaging in children at low risk.
Suggested Readings
• Kuppermann N, Holmes JF, Dayan PS, et al. Identification of children at very low risk of
clinically important brain injuries after head trauma: a prospective cohort study. Lancet.
2009;374(9696):1160-1170. DOI:http://dx.doi.org/10.1016/S0140-6736(09)61558-0.
Question 117
A full-term newborn is delivered by repeat caesarean delivery. The mother requires general
anesthesia, with the time from abdominal incision to delivery 10 minutes because of multiple
adhesions. Artificial rupture of the membranes occurs at delivery and reveals clear amniotic
fluid. The newborn emerges apneic and limp. Upon arrival at the warmer, the newborn is dried
and stimulated. Assessment at 30 seconds after delivery reveals an apneic, cyanotic newborn
with a heart rate of 30 beats/min.
B. initiate bag mask ventilation with 100% oxygen and begin compressions
D. initiate bag mask ventilation with room air and begin compressions
After 30 seconds of effective assisted ventilation, the newborn should be evaluated again. If the
heart rate is below 60 beats/min, compressions should be initiated, coordinated with positive
pressure ventilation using a 3:1 ratio, with 90 compressions and 30 breaths occurring in a 1-
minute period. Chest compressions may be performed using the thumb technique (Item C117B)
or 2-finger technique (Item C117C), compressing to one-third the depth of the anterior-posterior
chest. Compressions should be discontinued when the heart rate is greater than 60 beats/min,
with the decision to discontinue assisted ventilation being made when the heart rate is greater
than 100 beats/min.
The 2010 American Heart Association Guidelines for Neonatal Resuscitation recommend that
the initial resuscitation of a full-term newborn with positive pressure ventilation be performed
using room air. An oximeter probe should be placed on the newborn’s right hand (preductal)
when assisted ventilation is begun to allow adjustment of oxygen to attain targeted goal
saturations during the resuscitation. Although hypoxia may cause multiorgan damage, even brief
episodes of hyperoxia may be associated with adverse outcomes.
The full-term newborn in the vignette is found to have apnea 30 seconds after birth and requires
bag mask ventilation with room air. Oxygen should not be used until the oxygenation status is
determined with a pulse oximeter. Compressions are not indicated until reassessment after 30
seconds of effective assisted ventilation. Intubation and administration of intratracheal
epinephrine may be considered if the newborn fails to respond to the initial resuscitation,
including compressions and effective assisted ventilation.
PREP Pearls
• Assisted ventilation is the most effective action in the resuscitation of a compromised
newborn.
• If the heart rate is below 60 beats/min after 30 seconds of effective positive pressure
ventilation, compressions should be initiated.
• Compressions are coordinated with positive pressure ventilation using a 3:1 ratio, with 90
compressions and 30 breaths occurring in a 1-minute period.
Suggested Readings
• Kattwinkel J, ed. Textbook of Neonatal Resuscitation. 6th ed. Elk Grove Village, IL:
American Academy of Pediatrics; 2011:328 pp.
Question 118
A 12-year-old boy complains of limb weakness that began 3 months ago. He cannot keep up
with peers when playing sports and he has difficulty going up or down more than 1 flight of
stairs. The boy can still open jar lids with his hands and his handwriting remains normal. He has
no eyelid drooping, double vision, facial weakness, or difficulty swallowing. The limb weakness
remains the same throughout the day and does not improve with rest. There is no family history
of limb weakness.
On physical examination, the boy’s vital signs are normal and his weight is stable from his health
supervision visit 9 months previously. He does not have any rashes, skin changes in his hands or
knuckles, and there are no striae. He does not have elbow contractures. His neurological
examination shows normal cranial nerve function with symmetric weakness of his deltoid,
biceps, triceps, quadriceps, and hamstring muscles. His muscle bulk and tone are normal. He has
mild tenderness to palpation of his quadriceps muscles and they have a rubbery feel. His reflexes
are normal and symmetric. Sensation of light touch is normal in his arms and legs. The boy
cannot rise from a chair without using his hands and he uses a Gower maneuver to rise from the
floor. An electrocardiogram is normal. You order laboratory tests and the results are shown:
Result Normal Range
Of the following, the test MOST likely to identify the correct diagnosis is
E. muscle biopsy
The differential diagnosis of muscle disorders presenting in childhood can be divided into 2
categories: genetic and acquired. Genetic muscle disorders typically present in the preschool
years and have a slowly progressive course. Pediatricians should recognize the more common
genetic muscle disorders such Duchenne muscular dystrophy and myotonic dystrophy. Duchenne
muscular dystrophy typically presents in boys younger than 6 years with progressive proximal
weakness, calf hypertrophy, and creatine kinase of 10,000 U/L or greater. Childhood onset
myotonic dystrophy presents with distal weakness (weak ankle dorsiflexion and weak hand
muscles), a long, narrow face, and tapering forearms. Acquired muscle disorders include
myopathies caused by endocrine or rheumatologic diseases or inflammatory muscle disorders.
Acquired muscle disorders have a more abrupt onset than genetic muscle disorders. Signs of an
endocrine or rheumatologic disease may be present. Characteristic dermatologic findings of
heliotrope rash and Gottron papules suggest a diagnosis of dermatomyositits. If there are no
elements to suggest a particular acquired muscle disease, muscle biopsy is the next best
diagnostic test.
PREP Pearls
• Typical findings in boys with Duchenne or Becker muscular dystrophy include proximal
muscle weakness, calf hypertrophy, and creatine kinase levels of 10,000 U/L or greater.
• Patients with distal muscle weakness involving the hands often notice that they have
difficulty opening jar lids.
Suggested Readings
• Andersson PB, Rando TA. Neuromuscular disorders of childhood. Curr Opin Pediatr.
1999;11(6):497-503. Abstract available at
http://www.ncbi.nlm.nih.gov/pubmed/10590906.
Question 119
A 9-year-old girl presents to your office for evaluation of poor growth. Her height is significantly
below the first percentile, but has been at the same percentile for many years. Her weight has
always been at the fifth percentile (Item Q119). Midparental height is 152.4 cm (fifth percentile).
There is no family history of delayed puberty. You obtain bone age radiographs. Based on her
concordant bone age of 9 years, her predicted adult height is 147.3 cm (significantly below the
first percentile). The girl’s family requests that she be started on growth hormone treatment.
Of the following, the MOST accurate statement regarding this request is that
In 2003 the US Food and Drug Administration (FDA) approved GH as a treatment for idiopathic
short stature (ISS), defining the condition as height of an individual greater than or equal to 2.25
standard deviation scores below the corresponding mean height for a given age, sex, and
population group. This approximates the 1.2nd percentile. Predicted height (based on bone age)
in ISS is less than 5’3” for males and 4’11” for females, thus this definition does not include
children with constitutional delay of growth and maturation who would be expected to have
significant catch-up growth. Children with ISS are GH-sufficient, have normal birthweight and -
length, and do not have any other clearly identifiable diagnosis predisposing them to have poor
growth. Often bone age is not delayed.
Children with GHD can have significant catch-up growth with GH treatment. Although there is
less of an impact, the expected mean gain in height for children with ISS averages 3 inches from
predicted height. Thus, even with a normal growth velocity, this very short child could benefit
from GH treatment. Referral to a pediatric endocrinologist for further evaluation and possible
treatment for this FDA-approved indication is appropriate. Specific height velocity charts have
been published to show expected changes in height growth under various conditions, including
ISS, and can be used to guide treatment.
Evaluation of the growth chart of the child in the vignette shows that she is growing 5 cm/year, a
normal prepubertal growth velocity, therefore identification of an underlying cause of poor
growth is unlikely. Similarly, there is no indication that she has any underlying syndrome that
would make her response to GH treatment above average. With GH treatment, she would be
expected to grow approximately 3 inches beyond her predicted height. The child’s weight
percentiles are above the height percentiles, effectively ruling out a nutritional cause of poor
growth.
PREP Pearls
• Children with growth hormone deficiency (GHD) typically have short stature, abnormal
linear growth velocity, and delayed osseous maturation.
• Children with idiopathic short stature (ISS) have height at or below the 1.2 percentile,
with a predicted height (based on bone age) of less than 5’3” for men and 4’11” for
women, and no clearly identifiable diagnosis predisposing them to have poor growth.
Suggested Readings
• Allen DB, Cuttler L. Short stature in childhood: challenges and choices. N Engl J Med.
2013;368(13);1220-1228. DOI:http://dx.doi.org/10.1056/NEJMcp1213178.
Question 120
You are called to evaluate a newborn in the delivery room. The newborn was born to a 26-year-
old with 1 prior healthy child. No prenatal care or imaging was obtained with the current
pregnancy, but the previous pregnancy was notable for maternal group B streptococcal positive
cultures. The newborn’s Apgar scores are 1 at 1 min and 4 at 5 min. You arrive to find a
newborn in acute respiratory distress. Bag-mask ventilation is in progress. The newborn is
cyanotic with poor tone and little spontaneous respiratory effort. You place an endotracheal tube
and provide positive pressure ventilation. On auscultation, you note asymmetric aeration with
primarily right-sided breath sounds. A needle thoracostomy of the left chest returns no air. The
abdomen is distended in appearance and tympanitic to percussion. An orogastric tube is placed to
decompress the stomach.
Of the following, the MOST likely etiology of this newborn’s respiratory distress is
C. pneumomediastinum
D. septic shock
The frequency of CDH has been reported to be 1 in 3,000 to 5,000 live births. However, the true
incidence may approximate 1 in 2,000 births when premature fetal terminations and cases of
neonatal demise are considered. The most common associated anatomic diaphragmatic defect
(95%) is in the foramen of Bochdalek; these defects are posterolateral in location and 80% are
found on the left side. Far less frequently, herniation may occur through the foramen of
Morgagni; these defects are classically retrosternal in location.
Historically, persistent pulmonary symptomatology and lung hypoplasia have been attributed to
compression of the developing ipsilateral lung by abdominal contents displaced into the thoracic
cavity during uterine development. However, recent models suggest that lung hypoplasia occurs
prior to diaphragm development and that early defects in lung morphogenesis may only be
exacerbated by the later presence of intrathoracic compression by abdominal viscera. This data
provides some explanation for the fact that pulmonary hypoplasia can be seen bilaterally and not
only on the side of the diaphragmatic defect.
Prenatal diagnosis via ultrasonography has advanced in recent decades, but perinatal morbidity
and mortality remain high. Right-sided and bilateral defects carry a poor prognosis with
mortality rates that approximate 80% and 100%, respectively. Prenatal ultrasonography detects
up to 60% of defects, but regional and institutional variability in detection rate exist. Congenital
diaphragmatic hernia may be diagnosed by the presence of abdominal organs in the thoracic
cavity. Additional associated findings may include displacement of the cardiac axis and
mediastinal structures or polyhydramnios. Right-sided defects are difficult to detect on
ultrasonography, as the echogenicity of the liver approximates that of the lung. The finding of
hydrops carries a poor prognosis.
While respiratory distress is commonly encountered in an infant with hyaline membrane disease
(HMD), the infant with HMD is more likely to be premature. In addition, while the low
compliance of the lung with HMD may result in pneumothorax from positive pressure
ventilation and asymmetric aeration, recovery of air on needle thoracostomy would be expected.
A severe pneumomediastinum may result in a secondary pneumothorax and related asymmetric
aeration, but asymmetry is not expected with a pneumomediastinum alone. Associated findings
with pneumomediastinum include subcutaneous air and related crepitus.
The newborn in the vignette may be at risk for sepsis with a history of group B streptococcal
infection in the mother and the lack of prenatal care. However, the current presentation is
atypical for septic shock or for a congenital cyanotic cardiac defect.
PREP Pearls
• The majority of congenital diaphragmatic hernias (CDH) occur on the left,
posterolaterally, and through the foramen of Bochdalek.
• Even with left-sided defects and optimal imaging quality, CDH may not be recognized on
prenatal ultrasonography.
• Pulmonary hypertension is a frequent complication for survivors of CDH; infants and
children should be screened accordingly.
Suggested Readings
• Benachi A, Cordier AG, Cannie M, Jani J. Advances in prenatal diagnosis of congenital
diaphragmatic hernia. Semin Fatal Neonatal Med. 2014;19(6):331-337. DOI:
http://dx.doi.org/10.1016/j.siny.2014.09.005.
Question 121
A 17-month-old boy presents for his health supervision visit. You last saw him at 12 months of
age. The parents have no concerns, but remark that his left eyelid has looked droopy for “awhile”
(Item Q121). Reviewing his old chart, you find that he was born at term by repeat cesarean
delivery, had an uncomplicated perinatal course, and a normal examination without ptosis at 12
months of age. His growth and development are unremarkable. His physical examination is
notable for left-sided ptosis, anisocoria, and a paler pupil on the left than the right. You also
palpate a 3 cm left-sided lower cervical lymph node. He has normal movement of his upper and
lower extremities.
Item C121A
Horner syndrome is one form of ptosis, a common eyelid disorder resulting from dysfunction of
the muscles that elevate the upper eyelid. The frequency of ptosis in the United States is not
defined; a recent study over a 40-year period in 1 county in Minnesota found an incidence of 7.9
American academy of pediatrics 396
American Academy of Pediatrics PREP 2016
per 100,000 people younger than 19 years of age. Of those, nearly 90% were classified as
congenital, ie, occurring before 1 year of age, and three-quarters of those with early onset were
classified as having simple congenital ptosis (called in some studies myogenic developmental
abnormality). In patients who do not have simple congenital ptosis, one must consider genetic,
mechanical, mitochondrial, developmental or embryologic, traumatic, neoplastic, and
neurological causes (Item C121B).
In addition to addressing the underlying condition, the most important consideration for the
pediatrician and ophthalmologist dealing with a child with ptosis is to recognize the risk of
developing amblyopia because of obstruction of the visual axis. Treatment of the underlying
condition may lead to resolution, and mild cases may not require treatment otherwise. More
severely affected patients require corrective surgery because most ptosis will not improve over
time.
An edrophonium test would be used to diagnose myasthenia gravis, another rare cause of ptosis.
A key characteristic of myasthenia is the worsening of ptosis as the muscle fatigues. The patient
in the vignette has persistent ptosis and additional symptoms (lymph node enlargement) that
would make myasthenia a less likely diagnosis. While ophthalmology consult may eventually be
needed for this patient if the ptosis persists, ruling out a neoplasm is the most immediate concern.
Age of onset and additional symptoms make brachial plexus injury unlikely and therefore
physical therapy input is not currently required. Given the constellation of symptoms, the
noninflamed lymph node is more likely to represent neoplasm than infection, so antibiotic
treatment is not warranted.
PREP Pearls
• Ptosis is a common condition resulting from dysfunction of the muscles that elevate the
eyelid.
• Congenital ptosis is the most common form and presents during the first year of life.
• Horner syndrome consists of ptosis, miosis, anisocoria, facial anhidrosis, and
heterochromia. It can be the result of trauma, infection, neoplasm, migraine, or carotid
artery abnormality.
• In children, the most common neoplasm causing Horner syndrome is neuroblastoma.
Suggested Readings
• Griepentrog GJ, Diehl NN, Mohney BG. Incidence and demographics of childhood
ptosis. Ophthalmology. 2011;118(6):1180-1183. DOI:
http://dx.doi.org/10.1016/j.ophtha.2010.10.026.
Question 122
The father of a 14-year-old adolescent calls your office because of concerns about his son’s
behavior. The father says that his son seems withdrawn and paranoid. The adolescent rarely
wants to spend time with the family. Whenever the parents plan a family event, the son finds an
excuse to not attend. He constantly asks his mother how he looks and spends 30 min selecting his
clothes each day. He frequently argues with his 13-year-old sister and insists that she is always
looking at him. The father reports that his son has friends at school and that he is an honor roll
student.
Of the following, you advise the father that
Adolescents progress from concrete thinking to abstract thinking, which means that the early
adolescent has limited capacity to recognize the consequences of risk-taking behaviors.
Emotional separation from parents is a characteristic task of adolescent development; therefore,
an adolescent’s desire to spend less time with their parents should be expected. Additionally, as
adolescents adjust to their changing physical appearance and develop a mature self-image, they
are often very concerned about their physical appearance and appear very self-absorbed. The
behavior of the adolescent in this vignette is appropriate for his developmental age.
The prevalence of depression in adolescents 14 to 18 years of age ranges from 4% to 7%. The
American Academy of Pediatrics recommends routine screening of adolescents for depression. It
is important to recognize that many adolescents may experience transient depressive symptoms,
but an adolescent with clinical depression has depressed or irritable mood, loss of interest or
pleasure, and at least 3 of the following symptoms for a 2-week period:
1. Significant weight loss or decrease in appetite (more than 5% of body weight in a
month)
2. Insomnia or hypersomnia
3. Psychomotor agitation or retardation
4. Fatigue or lack of energy
5. Feelings of worthlessness or guilt
6. Decreased concentration or indecisiveness
7. Recurrent thoughts of death or suicide
The adolescent described in this vignette is exhibiting normal developmental tasks with his
desired separation from family and an interest in self-appearance.
PREP Pearls
• Adolescent development has 3 main components: physical, psychosocial, and cognitive
development.
• Adolescents progress from concrete thinking to abstract thinking, which means that the
early adolescent has limited capacity to recognize the consequences of risk-taking
behaviors.
Suggested Readings
• American Psychiatric Association. Diagnostic and Statistical Manual of Mental
Disorders, Fifth Edition. Washington, DC: American Psychiatric Association; 2013: 991
pp.
Question 123
You are seeing a 10-year-old boy in your office with complaint of joint pain and swelling. He
has significant right knee and ankle swelling that started 2 days ago, which is very painful. He
also complains of right heel and second toe pain. A review of systems is significant for a scaly
rash on the soles of the feet, low-grade fevers (38.3°C), myalgia, painless oral ulcers, and
conjunctivitis of 2-day duration. Two weeks ago, the boy had an episode of diarrhea, which self-
resolved. While other family members had diarrhea as well, no family member shares his current
symptoms. On physical examination, the boy has erythema overlying the right knee and ankle
with significant swelling and pain, and a limited range of motion. He is tender to palpation along
the right posterior heel and has dactylitis of the second toe. He has 2 shallow ulcers on the buccal
mucosa and bilateral conjunctivitis. There are hyperkeratotic skin lesions on his soles bilaterally.
The remainder of the physical examination is unremarkable.
A. Bechet disease
B. Crohn disease
C. mononucleosis
D. psoriatic arthritis
E. reactive arthritis
Reactive arthritis most frequently presents as an oligoarticular arthritis of the lower extremity,
However, multiple joint and upper extremity involvement can occur in some cases. Other
features include dactylitis, enthesitis, and bursitis. Some patients with reactive arthritis can have
inflammatory back pain. Extra-articular manifestations of reactive arthritis include conjunctivitis
or uveitis. Reactive arthritis associated with urogenital infections can be associated with
urethritis and cervicitis. Skin lesions can occur with small hard nodules on the feet or hands
called keratoderma blennorrhagicum. Penile lesions such as balanitis circinata can develop.
Some patients demonstrate painless ulcers in the mouth. Rarely aortic regurgitation, pericarditis,
and conduction abnormalities can occur in prolonged cases.
Although no laboratory studies are diagnostic and there are no specific diagnostic criteria, HLA-
B27 positivity is associated with the development of reactive arthritis. Laboratory studies
looking for the triggering infections (testing for Chlamydia trachomatis, stool cultures, or
evidence of synovial infection by immunohistology) can support the diagnosis. Nonspecific
inflammatory markers are frequently elevated.
Treatment of reactive arthritis includes treatment of the preceding infection, when appropriate,
and nonsteroidal anti-inflammatory agents. Steroid injections can occasionally be used in
monoarticular disease. If the arthritis becomes chronic (lasting >6 months), other treatments such
as immunosuppressive agents may be used.
Bechet disease can present with arthritis, gastrointestinal involvement and oral ulcers, however
the boy in this vignette has diarrhea that is most likely infectious because the rest of the family
had similar findings. In addition, Bechet disease is rare. Crohn disease does present with diarrhea
and arthritis, but the diarrhea is noninfectious, chronic and often associated with bloody stools.
Mononucleosis can present with arthritis, but in the setting of an infectious diarrhea followed by
arthritis, reactive arthritis is most likely. The rash described as hyperkeratotic skin lesions on his
soles is consistent with keratoderma blennorrhagicum not psoriasis; therefore, psoriatic arthritis
is incorrect.
PREP Pearls
• Reactive arthritis typically occurs 1 to 3 weeks after an infection of the gastrointestinal or
urogenital tract.
• Reactive arthritis most frequently presents as an oligoarticular arthritis of the lower
extremity.
• Initial treatment of reactive arthritis includes treatment of the preceding infection when
appropriate and nonsteroidal anti-inflammatory agents.
Suggested Readings
• Selmi C, Gershwin ME. Diagnosis and classification of reactive arthritis. Autoimmun
Rev. 2014;13:546-549. DOI:http://dx.doi.org/10.1016/j.autrev.2014.01.005.
Question 124
A 4-year-old, previously healthy boy presents to the emergency department with a facial
laceration that requires 4 simple, interrupted sutures. His vital signs are normal for age, and other
than the laceration, his physical examination is unremarkable. The boy has no allergies and there
is no family history of adverse reaction to anesthetic or sedative medications. Despite local
anesthesia, distraction, and reassurance, the boy is uncooperative and sedation is required to
perform the repair. He receives a dose of intranasal midazolam, 0.5 mg/kg. Five minutes later, he
is calm and has a normal response to verbal stimuli.
Of the following, while this boy is sedated, it would be MOST appropriate to continuously
monitor his
B. electrocardiogram
D. pulse oximetry
E. pupillary response
The principal adverse effects of benzodiazepines and narcotics include hypotension, bradycardia,
and respiratory depression. Neurologic status and treatment effect can be assessed with verbal
and mild tactile stimuli. Monitoring of a patient receiving these medications for pharmacologic
anxiolysis should include continuous pulse oximetry, visual assessment of ventilation, and
noninvasive blood pressure measurement every 5 minutes. Continuous monitoring of arterial
blood pressure is too invasive for this setting. Because these medications generally have minimal
effect on cardiac conduction, electrocardiographic monitoring is not required in the absence of
significant preexisting cardiovascular disease. Bradycardia can be detected on pulse oximetry.
Hypoventilation can be detected visually or on pulse oximetry. Noninvasive capnography by
nasal cannula is sometimes used in conscious sedation for procedures involving decreased access
for visual monitoring, such as magnetic resonance imaging. Monitoring of exhaled tidal volume
would require endotracheal intubation, which is usually not necessary in sedations short of
general anesthesia. Monitoring of the pupillary response is not necessary unless an overdose is
suspected.
PREP Pearls
• Sedation in children is represented by a continuum ranging from completely awake to
anxiolysis, moderate sedation/analgesia, deep sedation, and finally general anesthesia (in
which the child is completely unarousable).
• Children undergoing conscious procedural sedation exhibit purposeful reaction to verbal
stimulation.
• Children undergoing conscious/moderate procedural sedation require continuous pulse
oximetry, visual assessment of breathing, and noninvasive blood pressure monitoring.
Suggested Readings
• American Academy of Pediatrics Committee on Drugs. Guidelines for monitoring and
management of pediatric patients during and after sedation for diagnostic and therapeutic
procedures: addendum. Pediatrics. 2002;110:836-838.
DOI:http://dx.doi.org/10.1542/peds.110.4.836.
Question 125
A 2-year-old boy presents to the emergency department with fever, vomiting, and behavioral
changes. He has had increased agitation and repeated episodes of facial twitching. Vital signs
show a temperature of 39.5°C, respiratory rate of 38 breaths/min, heart rate of 131 beats/minute,
blood pressure of 124/76 mm Hg, and a weight of 12.8 kg. On physical examination, he is
irritable, has nuchal rigidity, and several insect bites are noted over his legs. Laboratory data
shows:
• White blood cells, 20,200/µL (20.2 x 109/L)
• Hemoglobin, 11.3 g/dL (113 g/L)
• Platelets, 341 x 103/µL (341 x 109/L)
• Differential, 62% segmented neutrophils, 26% lymphocytes, 12% monocytes
• Cerebrospinal fluid (CSF) white blood cells, 145/µL (23% segmented neutrophils, 45%
lymphocytes, 32% monocytes)
• CSF red blood cells, 2/µL
• CSF glucose, 49 mg/dL (2.7 mmol/L)
• CSF protein, 15 mg/dL
• CSF West Nile immunoglobulin M, 1:64
The resident working with you asks about the epidemiology of West Nile virus.
A. avian deaths from West Nile virus have been predictive of human cases
B. horses serve as a reservoir for avian infections with West Nile virus
D. the mosquito vector for West Nile virus tends to bite during the day
E. peak case numbers for West Nile virus usually occur in late spring
The term arbovirus is an abbreviation of arthropod-borne viruses, which alludes to the vectors of
transmission (mosquitoes, ticks, sand flies, and midges). Transmission usually occurs between
birds or small mammals and the arthropod vectors. Infections in humans and domestic animals
do not maintain transmission for most arboviruses. For West Nile virus, birds serve as a
reservoir, and avian-mosquito-avian cycles maintain transmission. West Nile virus infections in
humans and horses are considered dead end infections. However, there are exceptions to this. In
dengue, chikungunya, and yellow fever, arthropods that feed on infected humans can then infect
other humans.
Culex mosquitoes are the principal vectors for West Nile virus. Culex mosquitoes preferentially
have blood meals at dusk or after dark. Most infections occur in the summer or early fall, which
coincides with peak mosquito activity.
West Nile infections are asymptomatic in 80% of cases. Although most infections caused by
arboviruses are subclinical, they can manifest as an acute febrile illness or neurologic disease.
Manifestations during the acute febrile illness can be nonspecific and include headaches,
myalgias, arthralgias, and exanthems. Some arboviruses have a predilection for causing
particular symptoms, such as the severe joint pain seen in chikungunya virus infection, and bone
pain and retro-orbital headache seen in dengue fever. Neuroinvasive disease occurs in less than
1% of those that are infected with West Nile virus. Manifestations can range from aseptic
meningitis to encephalitis and even flaccid paralysis. Although most individuals have
asymptomatic infection, La Crosse virus can cause severe neurologic disease, especially in
children. In general, severe manifestations of arbovirus infections tend to occur more commonly
in adults compared to children. Dengue and yellow fever can cause hemorrhagic disease
manifestations.
PREP Pearls
• For West Nile virus, birds serve as a reservoir and avian-mosquito-avian cycles maintain
transmission.
• Most West Nile virus infections occur in the summer or early fall, which coincides with
peak mosquito activity.
• Although most infections caused by arboviruses are subclinical, they can manifest as an
acute febrile illness or neurologic disease, including aseptic meningitis, encephalitis, and
flaccid paralysis.
Suggested Readings
• American Academy of Pediatrics. Arboviruses. In: Kimberlin DW, Brady MT, Jackson,
MA, Long SS, eds. Red Book: 2015 Report of the Committee on Infectious Diseases.
30th ed. Elk Grove Village, IL: American Academy of Pediatrics; 2015:240-246.
Question 126
The parents of a 7-year-old girl present to your office for evaluation. The girl was diagnosed
with Henoch-Schönlein purpura (HSP) by your associate 2 months ago. Since then, she has had
multiple urinalyses showing 2-3+ blood by urine test strips and 10 to 50 red blood cells per high
power field (HPF). Her record shows physical examinations on previous encounters remarkable
only for palpable purpuric macules on the legs. Further studies include renal ultrasonography and
serum chemistry, which were normal when last checked 6 weeks ago.
Her vital signs today show a temperature of 37.9°C, heart rate of 72 beats/min, respiratory rate of
18 breaths/min, and blood pressure of 110/60 mm Hg. Her physical examination is significant for
palpable purpuric macules on the legs with occasional confluence. According to the parents, her
rash is much improved.
Her urinalysis today demonstrates a specific gravity of 1.035, a pH of 6.0, 3+ blood, 1+
proteinuria, and no leukocyte esterase or nitrites. Her urine microscopy shows 50 to 100 red
blood cells/HPF, less than 5 white blood cells/HPF, and no crystals or bacteria.
Of the following, you are MOST likely to inform the parents that
A. children have poorer prognosis than adults with renal involvement because of HSP
D. nephrotic range proteinuria would increase her risk for progressive renal disease
E. rash persisting for more than a month indicates severe renal involvement
Renal involvement is reported in 20% to 50% of the patients with HSP and accounts for the long
term morbidity with this vasculitis. Renal manifestations of HSP include transient or persistent
(lasting greater than 6 months) hematuria, recurrent gross hematuria (along with recurrent skin
rash), proteinuria, nephritic syndrome (hematuria, hypertension, and azotemia), nephrotic
syndrome (nephrotic range proteinuria, hypoalbuminemia, edema), or nephritic-nephrotic
syndrome. Microscopic hematuria is the most common renal manifestation of HSP vasculitis.
Proteinuria (mild and non-nephrotic), with and without hematuria, is more common than
nephrotic range proteinuria. Nephrotic syndrome (proteinuria, hypoalbuminemia, and edema) is
present in only a minority of the patients. The presence of nephrotic syndrome, renal
insufficiency (elevated blood urea nitrogen and serum creatinine), and hypertension (nephritic
syndrome) is associated with poorer renal prognosis in patients with HSP vasculitis. Adults with
HSP vasculitis have similar clinical manifestations to children, but are at increased risk for
progressive renal disease. Risks and severity of renal involvement caused by HSP vasculitis
cannot be predicted from the severity of extrarenal involvement such as skin rash, joint, or
abdominal pain.
Renal manifestations in patients with HSP appear commonly in the first month after diagnosis
(around 80%) and in the majority of patients (95%) by 6 months. Therefore, patients presenting
with extrarenal manifestations of HSP should have a urinalysis at presentation and frequently
thereafter to look for evidence of renal involvement secondary to HSP vasculitis. The frequency
of urinalysis after the initial onset of HSP rash is determined based on the urinary findings at
presentation. As the majority of the patients develop renal manifestations in the first month,
weekly urinalysis in the first 4 weeks will lead to early identification of renal involvement. It is
estimated that 2% to 5% of the patients with renal involvement caused by HSP progress to end
stage renal disease. Patients with nephritic or nephrotic syndrome have a higher risk of
developing chronic kidney disease (10%-20%), compared to patients with persistent hematuria or
proteinuria that does not fall into the nephrotic or nephritic range (around 2%). Patients with
renal manifestations associated with increased risk for chronic kidney disease merit frequent
urinalysis and should be managed in consultation with a pediatric nephrologist.
PREP Pearls
• Renal involvement is reported in 20% to 50% of the patients with Henoch-Schönlein
purpura (HSP) and accounts for the long term morbidity with this vasculitis.
• Microscopic hematuria is the most common renal manifestation of HSP vasculitis.
• The presence of nephrotic syndrome, renal insufficiency (elevated blood urea nitrogen
and serum creatinine), and hypertension (nephritic syndrome) is associated with poorer
long-term renal prognosis in patients with HSP.
American academy of pediatrics 412
American Academy of Pediatrics PREP 2016
• As the majority of patients develop renal manifestations in the first month, weekly
urinalysis in the first 4 weeks will lead to early identification of renal involvement.
Suggested Readings
• Adrogue HE, Flores FX. Henoch-Schönlein purpura. In: McInerny TK, Adam HM,
Campbell DE, Kamat DM, Kelleher KJ, eds.American Academy of Pediatrics Textbook
of Pediatric Care. Elk Grove Village, IL: American Academy of Pediatrics; 2009:2112-
2115.
Question 127
You are preparing to discharge a 2-month-old preterm infant with a history of duodenal atresia,
requiring prolonged total parenteral nutrition exposure that resulted in cholestasis. Recent
laboratory work shows:
• Total bilirubin, 5.7 mg/dL (97.5 µmol/L)
• Direct bilirubin, 3.5 mg/dL (59.9 µmol/L)
In reviewing the growth chart with the dietician, the age-adjusted height is at the tenth percentile
and the weight is at the third percentile. The infant is currently receiving a partially hydrolyzed
formula concentrated to 24 kcal/oz. You would like to initiate a dietary supplement to maximize
growth.
Of the following, the supplement that is MOST likely to be well tolerated and result in improved
growth is
A. flaxseed oil
C. olive oil
E. soybean oil
Fat requirements vary by age, with higher needs in infancy that slowly decrease as children pass
2 years of age (Item C127). Premature infants and infants have fat malabsorption, resulting in
increased needs that begin to decrease by 6 months of age. Fats are needed in our diet to provide
essential fatty acids and to assist with the absorption of fat soluble vitamins (vitamins A, D, E,
and K).
Children have increased dietary fat requirements for several reasons, including fat
malabsorption, failure to thrive, and increased metabolic demand. Children with complex
congenital heart disease must frequently limit their daily volume intake, and additional calories
from fats allow for adequate calories while respecting fluid limitations. Fat malabsorption
occurs:
1. Normally in infancy because of immature bile synthesis
2. In chronic liver diseases due to decreased bile production and secretion
3. In pancreatic exocrine insufficiency as a result of decreased secretion of enzymes
necessary for the breakdown and absorption of fats
PREP Pearls
• Fat requirements in children decrease with age.
• Fat requirements in premature infants are higher than in term infants.
• Increased fat needs are found in children with fat malabsorption, failure to thrive, and
increased metabolic demand.
• Children with complex congenital heart disease may have increased fat needs due to
intake restricted by volume limitations.
Suggested Readings
• Uauy R, Dangour AD. Fat and fatty acid requirements and recommendations for infants
0-2 years and children of 2-18 years. Ann Nutr Metab. 2009;551(1-3):76-96. DOI:
http://dx.doi.org/10.1159/000228997.
Question 128
A 10-year-old boy presents as a new patient to your office for a routine health supervision visit.
You note that his right leg and arm are slightly larger than his left. A physical examination
reveals macrocephaly, numerous skin tags, 3 to 4 palpable soft nodules on his arms and legs, 2
capillary hemangiomas on his right leg, and unusual freckling to his penis. The mother reports
that she has a history of a thyroid goiter, breast cancer, and uterine fibroids.
A. Beckwith-Wiedemann syndrome
B. Klippel-Trénaunay syndrome
C. neurofibromatosis type 1
E. Sotos syndrome
A suspected diagnosis of PHTS is typically based on clinical signs, but by definition, the
diagnosis of PHTS is definitively made only when a PTEN pathogenic mutation is detected.
Consensus diagnostic criteria for Cowden syndrome have been developed by the National
Comprehensive Cancer Network. There is also a free online risk calculator, produced by the
Cleveland Clinical Foundation, for adult and pediatric patients suspected to have PTEN
mutations (http://www.lerner.ccf.org/gmi/ccscore/).
This patient has an overgrowth syndrome in the presence of hemihyperplasia. Other disorders
that must be considered in this scenario include Beckwith-Wiedemann syndrome, isolated
hemihyperplasia, Klippel–Trénaunay syndrome (formerly known as Klippel-Trénaunay-Weber
syndrome), and neurofibromatosis type 1.
Sotos syndrome is an autosomal dominant disorder caused by NSD1 gene mutations or deletions
that presents with cardinal features, which include a typical facial appearance, overgrowth, and
learning disability. Facial dysmorphology is characterized by sparse frontotemporal hair, high-
bossed forehead, downslanting palpebral fissures, a long narrow face, and a prominent narrow
jaw. It does not typically manifest as hemihyperplasia, but a generalized overgrowth of the body.
PREP Pearls
• Within the differential diagnosis of a child with hemihyperplasia, a physician should
consider Beckwith–Wiedemann syndrome, Proteus syndrome, PTEN hamartoma tumor
syndrome, Klippel–Trénauny syndrome, isolated hemihyperplasia, and neurofibromatosis
type 1.
• PTEN hamartoma tumor syndrome caused by a pathogenic variant in the PTEN gene can
present as Cowden syndrome, Bannayan-Riley-Ruvalcaba syndrome, PTEN-related
Proteus syndrome, or Proteus-like syndrome. This presentation can place patients or other
family members at high risk for particular forms of cancer, intellectual disability, and
autism.
• Bannayan-Riley-Ruvalcaba syndrome classically presents with macrocephaly, autism,
developmental delay (IQ < 75), intestinal hamartomatous polyposis, lipomas, and
pigmented macules of the glans penis.
• Cowden syndrome patients can present with macrocephaly, trichilemmomas (a benign
cutaneous neoplasm developing from hair follicles), papillomatous papules, uterine
fibroids, and early onset cancers of the breast, thyroid, or uterus.
Suggested Readings
• Cleveland Clinic Lerner Research Institute. Risk calculator for estimating a patient’s risk
for PTEN mutation. Cleveland Clinic Lerner Research Institute website.
http://www.lerner.ccf.org/gmi/ccscore/ .
Question 129
During a routine health supervision visit for a healthy 7-month-old male infant, you notice that
his right testis is easily palpable in the scrotum, but the left testis is not. The left scrotal sac is
smaller than the right and is empty. You are not able to palpate the left testis. The infant was
born at full term without complications and has no medical problems. He has been growing
appropriately and the remainder of his physical examination is unremarkable. You discuss with
his parents the implications of undescended testes.
B. early orchiopexy will eliminate his increased risk for testicular cancer
It is often difficult to distinguish UDT from retractile testis. Because infertility and malignancy
are associated with UDT, it important to make the correct diagnosis. A history of UDT noted at
birth, prematurity, and scrotal asymmetry on physical examination are all findings that
significantly increase the likelihood that the testis is truly undescended. On physical
examination, a retractile testis will be palpable outside the external inguinal ring and can be
gently manipulated into the scrotum by overcoming the cremasteric reflex. Ultrasonography does
not reliably distinguish an undescended from retractile testis.
The complications of UDT are many. It is estimated that as many as 90% of UDTs have an
associated patent processus vaginalis and therefore an inguinal hernia can present at any time.
Testes that remain in the inguinal canal are at greater risk for injury from blunt trauma because
they can be compressed against the pubic bone. Testicular torsion is more common in UDT than
in scrotal testes and the diagnosis is often delayed in cryptorchidism, resulting in a lower rate of
salvage of the torsed testis. Impaired spermatogenesis in adults with a history of UDT is
common. The contralateral descended testis is also at risk for germ cell loss, infertility, and
malignancy. Bilateral involvement and increased duration of suprascrotal location of the testis
increase the degree of germ cell dysfunction. After 1 year of age, the histology of the UDT
begins to deteriorate. Hence, orchiopexy before age 1 year is preferred, with the ideal time being
as soon as possible after 6 months of age. Surgical repositioning of the testis before puberty
decreases the risk of testicular cancer, but does not completely eliminate it.
PREP Pearls
• Early orchiopexy for cases of undescended testes is important to help preserve fertility,
but it is not a guarantee.
• Undescended testes are associated with an increased risk of infertility, malignancy, blunt
trauma (for inguinal testes), testicular torsion, and inguinal hernia.
• Exploratory surgery with orchiopexy between 6 months and 1 year of age yields the best
outcome in cases of undescended testis because the histology of an undescended testis
begins to deteriorate after 1 year of age.
Suggested Readings
• Callaghan P. Undescended testis. Pediatr Rev.
2000;21(11):395.http://pedsinreview.aappublications.org/content/21/11/395.
Question 130
A 15-year-old adolescent boy presents to your office with a 1-month history of progressive
fatigue and exercise intolerance. His physical examination is remarkable only for pallor. His
weight is 60 kg. He specifically denies any history of hematuria, hematochezia, epistaxis, or
unusual bruising. He has had a normal diet. The results of a complete blood cell count are shown
in Item 130.
Of the following, the MOST appropriate next step is to advise him to start
A. 325 mg of iron sulfate twice daily on an empty stomach with a glass of orange juice and
perform a stool guaiac
B. 325 mg of iron sulfate twice daily on an empty stomach with a glass of orange juice and
repeat the blood cell counts in 4 weeks
C. 325 mg of iron sulfate twice daily with meals and perform a stool guaiac
D. 325 mg of iron sulfate twice daily with meals and repeat the blood cell counts in 4 weeks
E. 325 mg of iron sulfate twice daily with milk and perform a stool guaiac
The size of red blood cells (RBC), as measured by the mean corpuscular volume (MCV) is in
large part determined by the content of hemoglobin within the cell. Any deficiency in the
components of hemoglobin will therefore result in a low MCV. The 2 primary components of
hemoglobin that can be deficient are iron (deficiency of which causes decreased heme
production) or the globin protein. Hemoglobin A, the normal adult hemoglobin variant, consists
of 2 ß- and 2 a-globin chains, with the ß-globin gene located on chromosome 11 and the a-globin
gene on chromosome 16. Mutations resulting in reduced production of either a- globin or ß-
globin result in various thalassemia phenotypes, and present with a microcytic anemia. In order
to form hemoglobin, the 4 globin subunits must bind to a molecule of heme, which is dependent
on iron. Iron deficiency will therefore also result in a microcytic anemia.
Although the patient in the vignette could theoretically have a variant of thalassemia, his acute
findings are more consistent with iron deficiency.
The human body has a tight regulatory system for the absorption of iron, but no mechanism of
iron excretion. It would be highly unusual for a male adolescent with a relatively normal diet to
develop iron deficiency from dietary restriction. It is therefore very important that any time a
male adolescent presents with iron deficiency anemia, a source of iron loss must be sought.
Causes of iron loss in this population would include gastrointestinal bleeding, paroxysmal
nocturnal hemoglobinuria, and pulmonary hemosiderosis. Paroxysmal nocturnal hemoglobinuria
can be evaluated through flow cytometry on a blood sample, and pulmonary hemosiderosis can
be initially screened through a chest radiograph. If suspicion of hemosiderosis is strong, sputum
analysis for hemosiderin-laden macrophages is recommended. While all 3 should be evaluated in
the patient in the vignette, the most common cause would be gastrointestinal bleeding. It would
therefore be most appropriate to perform a stool guaiac.
Iron therapy for iron deficiency should include 2 mg to 4 mg of elemental iron per kg of weight
daily. Elemental iron is 20% of iron sulfate. For the patient in the vignette, the most appropriate
dose of elemental iron would be 120 mg to 240 mg daily, which would be 600 mg to 1,200 mg of
iron sulfate daily. Iron is best absorbed on an empty stomach with an acidic or neutral fluid. It is
therefore most appropriate to take iron with water or orange juice. The high concentration of
casein and whey proteins in milk inhibits iron absorption, and tea contains chelators that will
bind the iron and prevent its absorption. Medications that decrease the acidic environment of the
upper gastrointestinal tract may also impair absorption of iron.
PREP Pearls
• Iron deficiency in a male adolescent is rarely dietary and needs further evaluation to rule
out occult bleeding.
• The nondietary causes of iron deficiency include blood loss (typically gastrointestinal),
paroxysmal nocturnal hemoglobinuria, and pulmonary hemosiderosis.
• Iron should be administered on an empty stomach with water or an acidic drink such as
orange juice.
Suggested Readings
• DeLoughery TG. Microcytic anemia. N Engl J Med. 2012;371:1324-1331.
DOI:http://dx.doi.org/10.1056/NEJMra1215361.
Question 131
You are seeing a 13-year-old adolescent who has developed significant school avoidance. She
has periodically missed school over the past year when she had physical complaints or reported
having severe anxious feelings before school. This has worsened recently over the past week
with complaints of headache, stomachache, and anxiety before school each morning, causing her
to miss school each day. The mother notes that these complaints are relieved when she stays at
home by herself or when she goes to work with her mother. The adolescent denies any bullying
occurring at her school. She has been a good student, except for missing assignments when she is
absent from school. She has a history of being “clingy” with her mother periodically over the
years.
Of the following, the management that would MOST likely produce a positive outcome is
B. arrange for the parent to remain with the child in the classroom for 2 hours each morning
D. prescribe lorazepam and arrange for her to use it as needed during the school day
E. set up a plan where the child knows the parent will visit her once a day at school
School avoidance can be a major problem when it occurs because it typically becomes
increasingly difficult to resolve the longer the child remains out of school. One reason why
prolonged avoidance is such a problem is that our brains interpret anxiety relief from avoidance
as proof that a fear was well founded, and thus future anxious reactions to the same situation
deepen. For children avoiding school, this means that their fears about school usually increase
the longer their duration of avoidance, and it becomes more and more difficult to get them to
return.
The hallmark of an effective school avoidance intervention involves getting the child back into
school immediately without their parent sitting next to them. Supports of many forms can be
provided as appropriate while the child is at school, such as homework or class work
modifications, a plan for how the child will receive support by school staff, schedule
modifications, etc. If any persisting anxiety is present, enrollment in psychotherapy would be
appropriate. If the trigger for the avoidance was a truly aversive situation such as school
bullying, then that will need to be addressed.
Temporary home tutoring is counter productive for anxiety driven school avoidance because it
makes it easier for the child and family to avoid a return to school. Arranging for a parent to
remain in the classroom is a strategy that parents might request for a young child with separation
anxiety, but this is likely to only delay the separation crisis, as it nonverbally communicates to
the child that they cannot handle the situation on their own, and it is distracting to child and
classroom function while the parent is present. Setting up a plan for the parent to visit the child
during the day at scheduled times effectively creates additional separation experiences for the
child each day, and a daily opportunity for a child to "build their case" as for why they need to
leave mid-day. A single separation at the start of school is usually easier on both parents and
children in this situation. Pharmacologic intervention is not indicated.
PREP Pearls
• School avoidance is often related to anxiety, which may manifest as headaches and
stomach aches before going to school.
• School avoidance becomes more intervention-resistant the longer the child remains out of
school. An unaccompanied return to school as soon as possible is needed for these
children.
Suggested Readings
• Bagnell AL. Anxiety and separation disorders. Pediatr Rev. 2011;32(10):440-446. DOI:
http://dx.doi.org/10.1542/pir.32-10-440.
Question 132
A 4-year-old girl is brought to the emergency department with decreased level of consciousness.
She is hypotensive and in a wide complex tachycardia on arrival. She is cardioverted with 2 J/kg
and regains sinus rhythm. Her electrocardiogram shows low voltages with ST segment elevations
(Item Q132). Her mother states that she was ill with a diarrheal illness 2 weeks ago and has been
tired since that time, but became acutely ill and drowsy today. On physical examination, you see
a lethargic child with pallor. Her heart rate is 180 beats/min, her respiratory rate is 40
breaths/min, and her breathing is shallow. Her blood pressure is 75/40 mm Hg. She is cool in her
extremities with a capillary refill time of 4 seconds. Her pulse oximetry is not picking up well.
You notice that her jugular veins are distended. Her chest examination is significant for rales
throughout and retractions. Her cardiac examination is significant for a difficult to palpate point
of maximal impulse that is displaced to the left and weak. She has a regular rhythm, but it is
rapid. You appreciate S1 and S2 and hear a third heart sound in early diastole but no murmur.
Her chest radiograph shows pulmonary edema and an enlarged heart. You are planning
admission to the intensive care unit and getting consultations arranged. You place the child on
100% oxygen by non-rebreather mask and establish intravenous access.
Of the following, the intervention which would be MOST likely to improve the patient’s status
now is
Intravenous immunoglobulin (IVIG) may be helpful later in the management of this patient, but
would deliver too great of a fluid load at this point in the resuscitation, as the patient is already
showing symptoms of pulmonary edema. Esmolol is a good treatment for suppression of VT, but
would be contraindicated in this patient’s management because of its negative inotropic effect.
Amiodarone is also a good medication for treatment of VT, but a large bolus of 5 mg/kg,
especially if given quickly, will very likely make the patient more hypotensive. A normal saline
bolus may make the patient’s pulmonary edema worse.
Given the patient’s decompensation and presentation with VT, she will likely need advanced
mechanical support, extracorporeal membrane oxygenation (ECMO), to support cardiac function
and allow for management of her arrhythmias safely. Before ECMO can be initiated, dopamine
can be used to help maintain cardiac output and treat cardiogenic shock. Careful monitoring
needs to be maintained when dopamine is used because although it may improve function, it may
also stimulate atrial or ventricular arrhythmias. Soon after initiation of inotropic support,
antiarrhythmics would be indicated, especially if any further ectopy was noted. Lidocaine is a
good option for ventricular ectopy in this setting, as there is less risk of hypotension.
Arrhythmias are common in myocarditis. Ventricular arrhythmias are most common, but there is
also risk of atrioventricular block. Many viral etiologies are possible, including enteroviruses,
adenovirus, parvovirus, cytomegalovirus, influenza A (H1N1), herpes, varicella, mumps,
measles, rabies, hepatitis, rubella, rubeola, respiratory syncytial virus, HIV, and Epstein-Barr
virus. Diagnostic evaluation is directed towards assessment of the degree of cardiac failure and
impending respiratory failure, as well as determining the underlying etiology. A chest radiograph
will show cardiomegaly and pulmonary edema. An ECG may show a pattern of myocardial
damage, which can mimic ischemia or low voltages throughout the leads if there is not an
arrhythmia on presentation. Intensive care will be needed to monitor for development of
ventricular arrhythmia or heart block. An echocardiogram will be needed to assess the degree of
systolic ventricular dysfunction and chamber size. Blood work, including cardiac enzymes and
B-type natriuretic peptide, helps to assess the degree of myocardial damage and heart failure and
will also aid in the overall evaluation and follow-up, but will not change the initial management.
The initial management will be dictated by the clinical assessment of cardiac output. Viral
polymerase chain reaction studies by serology or biopsy help establish the exact viral etiology,
but does not impact the initial clinical management. Magnetic resonance imaging is also helpful
to confirm the diagnosis of myocarditis.
PREP Pearls
• Support with inotropic agents such as dopamine is useful in the initial management of
patients with myocarditis.
• Initial presentation of myocarditis may be a ventricular arrhythmia.
• Cardiogenic shock and arrhythmias may require extracorporeal membrane oxygenation in
the initial management of myocarditis.
Suggested Readings
• Anderson BR, Silver ES, Richmond ME, Liberman L. Usefulness of arrhythmias as
predictors of death and resource utilization in children with myocarditis. Am J Cardiol.
2014;114(9):1400-1405. DOI:http://dx.doi.org/10.1016/j.amjcard.2014.07.074.
Question 133
A 2-year-old African American boy presents to your clinic for evaluation of fever. His mother
states that he has had fever up to 39°C that started 5 days ago and his eyes and lips are red. The
boy had 1 episode of vomiting yesterday and 1 watery stool today. He is eating and drinking less
than usual, but urinating normally. The boy has a sibling that is currently well. On physical
examination, the boy is irritable. His temperature is 39.2°C, heart rate is 120 beats/min, and his
blood pressure is 80/50 mm Hg. He has bilateral, nonpurulent conjunctivitis. His lips and tongue
are red with hypertrophied papillae. He has a morbilliform rash on his chest. The dorsum of his
hands and feet appear edematous. The remainder of his physical examination is unremarkable.
You order laboratory studies. His erythrocyte sedimentation rate is 40 mm/h and his white blood
cell count is 15,200/µL (15.2 x 109/L). His other laboratory results are normal.
Of the following, the MOST concerning feature of the boy’s presentation that warrants further
evaluation is his
A. duration of fever
C. tachycardia
The differential diagnosis of a child with this presentation is broad. Early in its course, KD
mimics several conditions, including several viral illnesses that can present similarly, such as
adenovirus, enterovirus, influenza, roseola infantum, and Epstein-Barr virus. In very young or
unimmunized patients the practitioner must consider measles. Bacterial infections that can mimic
KD include scarlet fever, cervical lymphadenitis, Rocky Mountain spotted fever, and
leptospirosis. The skin peeling associated with KD is also seen in toxin-mediated diseases such
as toxic shock syndrome or scalded skin syndrome. Drug hypersensitivity reactions, including
Stevens-Johnson syndrome, can mimic KD. More rare conditions such as systemic juvenile
idiopathic arthritis, and periodic fever syndromes can present with continuous fever of unknown
origin. Mercury poisoning can also present similarly to KD.
The diagnostic criteria for KD include fever for at least 5 days, with at least 4 of the following:
bilateral, non-exudative, bulbar conjunctivitis; oropharyngeal changes; cervical
lymphadenopathy; polymorphous rash; and peripheral extremity changes. The oropharyngeal
changes can include strawberry tongue, erythema of the oropharyngeal mucosa, or erythematous
or cracking lips. The extremity changes include erythema or edema of the palms and soles in the
acute phase or periungual desquamation of the fingertips in the later subacute phase of KD.
The diagnosis can be difficult in cases where the patient has fever for 5 or more days but does
not meet criteria for diagnosis. Incomplete KD is the term used for patients with fever for 5 or
more days, but with only 2 or 3 of the principal clinical features. In such cases, echocardiography
is recommended. KD can be diagnosed in patients when coronary artery disease is detected with
2-dimensional echocardiography. Other laboratory study abnormalities that support a diagnosis
of KD are low albumin, elevated alanine aminotransferase, significantly elevated platelets,
elevated white blood cell count, or sterile pyuria.
Risk factors for coronary artery aneurysms include male sex, age younger than 1 year, prolonged
fever, delayed diagnosis, fever that persists after treatment, low hemoglobin, high white blood
cell count, high absolute band count, very high erythrocyte sedimentation (ESR) or C-reactive
protein, low platelet count, or low albumin. It should be noted that after intravenous
immunoglobulin treatment, ESR is no longer helpful in determining the level of inflammation.
PREP Pearls
• The diagnostic criteria for Kawasaki disease (KD) include fever for at least 5 days and at
least 4 of the following: bilateral, nonexudative, bulbar conjunctivitis; oropharyngeal
changes; cervical lymphadenopathy; polymorphous rash; and peripheral extremity
changes.
• No laboratory study is diagnostic for KD, but elevated erythrocyte sedimentation rate,
elevated C-reactive protein, elevated platelets, anemia, low albumin, elevated alanine
aminotransferase, elevated white blood cell count, and sterile pyuria can support the
diagnosis.
• Echocardiography should be performed in cases of incomplete or suspected KD.
Suggested Readings
• Scuccimarri R. Kawasaki disease. Pediatr Clin North Am. 2012;59:425-445.
DOI:http://dx.doi.org/10.1016/j.pcl.2012.03.009.
Question 134
A 12-year-old boy presents to your office for evaluation of right elbow pain. He denies any
history of trauma, but describes a 2-month history of medial elbow pain with baseball pitching.
He pitches for 4 different teams throughout the year. On physical examination, the boy is lacking
10 degrees of extension of the right elbow. He is tender over the medial epicondyle. Radiographs
of the right elbow show slight widening of the medial epicondyle apophysis.
The treatment for Little League elbow is rest, followed by an exercise program, ideally under the
direction of a physical therapist or athletic trainer familiar with pediatric injuries. For throwing
athletes, the exercise program should include a gradual increase in throwing frequency, distance,
American academy of pediatrics 437
American Academy of Pediatrics PREP 2016
and intensity. For avulsion injuries, with more than 0.5 to 1 cm of separation, surgery may be
required.
Limiting repetitive overhead motion will decrease the risk of Little League elbow. Many youth
baseball programs (eg, Little League) have set an upper limit for the number of pitches a young
athlete can throw during games and practices.
Many overuse injuries in children affect the physes (major growth plates) and apophyses (minor
growth areas where tendons attach). In adults, tendons and ligaments tend to be the weakest part
of the skeleton. In children, the cartilaginous physes and apophyses are especially vulnerable to
injury. With repetitive activity, children may develop irritation, separation, or even avulsion
injuries involving the physes and apophyses. These overuse injuries typically occur when
repetitive activities apply traction or pressure on a growth area. Osgood-Schlatter syndrome,
irritation of the tibial tubercle apophysis, is probably the most well-known form of apophysitis.
In Osgood-Schlatter syndrome, the quadriceps muscle pulls indirectly on the patellar tendon,
which puts traction on the tibial tubercle. Sever disease refers to calcaneal apophysitis resulting
from both traction on the apophysis by the Achilles tendon and repetitive impact to the heel with
running and jumping. Little League shoulder is an irritation of the proximal humeral physis
commonly seen in young athletes who participate in sports with repetitive overhead activity.
There are multiple apophyses of the pelvis. Pelvic apophysitis is most often seen in adolescent
athletes, with the anterior superior iliac spine the most commonly irritated location. Avulsion
injuries can also occur at the pelvic apophyses.
The child in the vignette should not be treated with immobilization of the elbow, because this
can lead to stiffness and should be avoided whenever possible. Research evidence is mixed
regarding whether throwing certain pitches (eg, breaking or curveball pitches) increases injury
rates. In this child, throwing any type of pitch would not be appropriate given his history and
physical examination findings. Surgery is generally indicated in cases with 5 to 10 mm of
separation (depending on the type of sport played and the patient’s symptoms), a finding not seen
in this patient. A hinged elbow brace may relieve some of the force on the medial epicondyle,
but is not a substitute for rest.
PREP Pearls
• Little League elbow is an irritation of the medial epicondyle apophysis that occurs in
athletes who participate in sports with repetitive overhead motions, most commonly
between the ages of 10 and 14 years.
• The cartilage growth centers of the pediatric skeleton are vulnerable to repetitive stress
that can cause overuse injuries.
Suggested Readings
• Sarwark JF, LaBella C. Pediatric Orthopaedics and Sports Injuries: A Quick Reference
Guide. 2nd ed. Elk Grove Village, IL: American Academy of Pediatrics; 2014: 445 pp.
Question 135
A 15-month-old boy presents with sudden onset of fever, left knee swelling, and refusal to bear
weight. On physical examination, his temperature is 39.5°C, heart rate is 140 beats/min,
respiratory rate is 30 breaths/min., and blood pressure is 95/65 mm Hg. He is ill appearing, but
not toxic. His left knee is swollen with mild overlying erythema and is exquisitely painful with
any movement. A plain radiograph is normal, but ultrasonography of the knee reveals moderate
fluid in the joint space. Fluid obtained by arthrocentesis shows 75,580 white blood cells/µL with
95% neutrophils. The joint fluid culture is growing gram-positive cocci in pairs and chains that
are ß-hemolytic on sheep blood agar (Item Q135).
Of the following, the BEST antimicrobial choice for treating this patient’s infection is
A. amoxicillin/clavulanate
B. cefotaxime
C. clindamycin
D. penicillin
E. vancomycin
Unlike children with toxic synovitis or arthralgia, children with pyogenic arthritis typically are
ill-appearing with fever, significant joint pain and swelling, and pseudoparalysis of the affected
area. Inflammatory markers usually are markedly elevated, and there is widening of the joint
space with joint effusion on radiographic imaging. Synovial fluid characteristically reveals a
white blood cell count of more than 50,000/µL (50 × 109/L) with a predominance of neutrophils.
To prevent bony destruction, especially in infants and children with prolonged symptoms,
surgical drainage of pyogenic arthritis of the hips and shoulders should occur promptly. Needle
aspiration of other infected joints may be sufficient, but depends on the clinical presentation and
response to antimicrobial therapy. Most children with septic arthritis require approximately 3
weeks of antimicrobial therapy directed toward the isolated or most likely pathogen. Because
antibiotics penetrate readily into joint fluid, most clinicians transition from parenteral to oral
therapy as soon as the patient demonstrates some clinical improvement (more joint mobility, less
pain, resolving fever, inflammatory markers trending down).
The treatment of choice for pyogenic arthritis caused by group A Streptococcus is penicillin.
Vancomycin is appropriate therapy when a gram-positive pathogen resistant to other agents is
suspected or documented (eg, methicillin-resistant Staphylococcus aureus, cefotaxime-resistant S
pneumoniae). Clindamycin is useful for susceptible staphylococci and S pneumoniae. Typically,
pyogenic arthritis caused by susceptible strains of pneumococcus and gram-negative pathogens
is treated with a third-generation cephalosporin such as cefotaxime. Amoxicillin-clavulanate is
appropriate therapy for septic arthritis caused by Pasteurella, because it also has anaerobic
coverage, which often is desired for skin and skin structure infections after animal bites.
PREP Pearls
• Children with septic arthritis typically are ill-appearing with fever, significant joint pain
and swelling, and pseudoparalysis of the affected area.
• Most cases of septic arthritis occur in children younger than 10 years (peak age, 2 to 3
years) by hematogenous spread of bacteria to the joint space and synovium.
• Staphylococcus aureus (methicillin-susceptible and methicillin-resistant), followed by
group A Streptococcus, is the most common cause of septic arthritis in children outside
the neonatal period.
• To prevent bony destruction, especially in infants and children with prolonged symptoms,
surgical drainage of pyogenic arthritis of the hips and shoulders should occur promptly.
Suggested Readings
• American Academy of Pediatrics. Group A streptococcal infections. In: Kimberlin DW,
Brady MT, Jackson, MA, Long SS, eds. Red Book: 2015 Report of the Committee on
Infectious Diseases. 30th ed. Elk Grove Village, IL: American Academy of Pediatrics;
2015:732-734.
Question 136
While “wrestling” with an older cousin, a 10-year-old healthy boy fell sideways from standing,
landing with his weight on his right shoulder. He reports feeling a snapping sensation when his
shoulder hit the floor. Physical examination reveals a well-appearing boy with tenderness to
palpation and slight swelling over the middle third of his right clavicle without tenting of the
skin. The boy has good passive range of motion in his left shoulder, but he reports pain with
abduction of the right shoulder. He has full grip strength and normal radial pulses bilaterally. His
breath sounds are clear and equal bilaterally, with no respiratory distress. A plain radiograph of
his right clavicle, is shown (Item Q136).
Clavicle fractures are the most common fracture type in children. Most clavicle fractures involve
the midshaft, whereas fractures of the medial third are the least common type (accounting for
only 2%-3% of cases).
Clavicle fractures arise most commonly from direct trauma (typically a fall onto the shoulder),
but may also result from indirect forces (such as transmitted force from a fall onto an
outstretched hand). Most are greenstick-type fractures involving the clavicular midshaft, with the
thick clavicular periosteum preventing significant displacement or angulation in children
younger than 10 years.
Children with clavicle fractures typically report pain localized to the clavicle that worsens with
movement of the shoulder. Patients will often cradle the arm on the injured side with the
opposite one to prevent movement of the arm. Some children may report feeling a "snap" or
"crack" at the time of the injury. Physical examination may reveal localized swelling, a visible
bulge and/or bruising over the clavicle, or even tenting of the skin over the fracture site. In some
cases, the injury can go unnoticed and is later recognized when a lump appears as a bony callus
forms. Despite the clavicle's anatomic location close to the brachial plexus and subclavian
vessels, neurovascular complications from clavicle fractures rarely occur unless a powerful direct
traumatic force results in significant displacement of the fracture fragments. Despite the rare
occurrence of neurovascular complications, physical examination should include careful
assessment for neurovascular injury, as well as a careful lung examination to assess for
associated pulmonary complications such as pneumothorax or hemothorax.
Plain radiography is generally the diagnostic mode of choice for the initial evaluation of isolated
clavicular injuries. Several views may be needed to identify more subtle nondisplaced fractures.
Computed tomography may occasionally be needed to identify clavicle fractures (particularly
those involving the proximal third) that are not apparent on plain radiography and to assess for
associated serious intrathoracic injuries. The boy in the vignette has a clavicle fracture with
minimal displacement, and he displays no signs or symptoms of associated intrathoracic injuries.
Therefore, computed tomography of the clavicle would not be the best next step in his
management.
Magnetic resonance imaging of the shoulder would not be indicated in the boy's management, as
there are no clinical findings suggestive of an injury involving his right shoulder joint.
Orthopedics consultation for closed reduction of this patient's clavicle fracture would not be
indicated; the fracture is not significantly displaced, and the boy displays no signs of associated
neurovascular compromise. Emergent orthopedic consultation is rarely necessary for children
with clavicular fractures. Indications for emergent orthopedic consultation include open fractures
(including fractures with significant tenting of the overlying skin), fractures with associated
American academy of pediatrics 445
American Academy of Pediatrics PREP 2016
neurovascular compromise, midshaft fractures with significant displacement (>2 cm), medial
clavicle fractures with posterior or significant anterior displacement, or grossly unstable distal
injuries. Patients with more than 1.5 cm of clavicle shortening caused by the fracture should be
referred for prompt orthopedic follow-up.
Nonoperative management is the initial treatment of choice for the vast majority of
uncomplicated clavicle fractures. Arm immobilization in a sling for comfort for 3 to 4 weeks,
followed by 3 weeks of rest from contact sports is recommended in most cases. Healing occurs
rapidly, typically with formation of a callus. Parents should be advised that as a part of the
normal healing process a lump will likely become apparent as a callus forms, and may persist for
a year or longer. Adequate pain control, as well as close follow-up until symptoms have resolved
and shoulder function has normalized, is also essential in the management of clavicle fractures.
Placing a posterior long arm splint on the right upper extremity would not be useful for the boy
in the vignette, because this would not stabilize his injury. Restricting shoulder motion to less
than 30 degrees, with either an arm sling or figure-of-8 bandage, best stabilizes clavicle
fractures. Because slings are typically more comfortable and more widely available than figure-
of-eight bandages, with similar outcomes, they are commonly used as the initial treatment of
choice for children with clavicle fractures.
PREP Pearls
• Children presenting with clavicle fractures generally report pain that is localized to the
clavicle but worsens with movement of the shoulder. Patients often cradle their arm on
the injured side with the opposite one to prevent movement.
• Plain radiography is the diagnostic modality of choice for the initial evaluation of isolated
clavicular injuries.
• Emergent orthopedic consultation is rarely necessary for children with clavicular
fractures. Indications include fractures that are open, those with significant tenting of the
overlying skin, those with associated neurovascular compromise, midshaft fractures with
significant displacement (>2 cm), medial clavicle fractures with posterior or significant
anterior displacement, or grossly unstable distal injuries.
Suggested Readings
• Bachman D, Santora S. Musculoskeletal trauma. In: Fleisher GR, Ludwig S, eds.
Textbook of Pediatric Emergency Medicine. 6th ed. Philadelphia, PA: Lippincott
Williams & Wilkins; 2010: 1335-1375.
Question 137
A mother delivers a neonate precipitously, at 25 weeks’ gestation, in the labor and delivery unit
of a community hospital. The membranes spontaneously rupture at the time of delivery,
revealing clear amniotic fluid. The very-low-birth weight neonate has poor tone, no spontaneous
cry, and rare gasping respirations. The newborn is brought over to the warmer.
Of the following, the BEST next step in the management of the neonate is to
Multiple strategies have been used to enhance thermoregulation of the ELBW infant in the
delivery room, because both hypothermia and hyperthermia are associated with increased
morbidity. The ambient temperature of the delivery room should be increased to at least 26°C to
decrease radiative heat loss. To minimize conductive heat loss, a radiant warmer should be
operational at delivery with an activated heated mattress (always placed under the receiving
blanket to prevent thermal contact burns). The ELBW infant should immediately be placed up to
the neck in a polyethylene bag or wrap, leaving the head exposed for airway management (Item
C137A). The use of a polyethylene barrier, such as a bag or wrap minimizes evaporative and
convective heat loss. A hat may be placed on the head to further decrease radiative heat loss. The
skin of the ELBW newborn should not be wiped dry or stimulated because the fragile epidermal
layer is easily damaged by friction.
Additional aspects of delivery management in the first minutes after birth can significantly affect
the long-term outcome of the ELBW infant. The resuscitation table should be flat, because
placement of the infant head down (Trendelenburg) may lead to intraventricular hemorrhage.
Excessive positive pressure with assisted ventilation may cause a pneumothorax and can
contribute to the development of chronic lung disease. Oxygen therapy should be administered
through an oxygen blender guided by the use of a pulse oximeter. Hypoxia and hyperoxia are
both associated with increased morbidity in ELBW infants. Although no agreement has been
American academy of pediatrics 448
American Academy of Pediatrics PREP 2016
reached on the optimum oxygen saturation level for preterm infants, targeted preductal
saturations are recommended immediately after delivery based on time after birth (Item C137B),
as outlined in the sixth edition of the Neonatal Resuscitation Program of the American Academy
of Pediatrics and American Heart Association. Many neonatal providers begin assisted
ventilation of a premature newborn with 30% to 40% oxygen rather than room air because of the
high likelihood of surfactant deficiency, but the use of 100% oxygen is avoided unless supported
by pulse oximetry.
PREP Pearls
• Multiple strategies to enhance the thermoregulation of a premature infant in the delivery
room should include increasing the ambient temperature of the delivery room, the use of
a radiant warmer, a warming mattress, a polyethylene barrier, and a hat.
• The delivery of oxygen therapy to a premature infant in the delivery room should be
guided by the use of a pulse oximeter and administered via an oxygen blender.
Suggested Readings
• Kattwinkel J, ed. Textbook of Neonatal Resuscitation. 6th ed. Elk Grove Village, IL:
American Academy of Pediatrics; 2011:329 pp.
Question 138
A 14-year-old adolescent is brought to the emergency department by his parents. He has
complained of neck pain for 3 days since riding a roller coaster at a theme park. The day before
this visit, his right arm started to tingle and became weak, and this morning, his left arm started
to tingle. He has not had urinary or fecal incontinence, changes in vision, or headache. He has
not been sick recently, although his sister had an upper respiratory infection 3 weeks ago. He
does not recall any specific head or neck trauma.
On physical examination, the patient’s temperature is 36.8°C, blood pressure is 102/74 mm Hg,
and his heart rate is 92 beats/min. He has no rashes and no tenderness to palpation of his
vertebral spinous processes. There is a palpable mass in his lower abdomen that you suspect is a
distended bladder. His neurologic examination demonstrates flaccid paralysis of his right upper
extremity. His left upper extremity has only antigravity strength and he is unable to provide any
resistance. Reflexes are diminished in his biceps and triceps, with normal patellar reflexes. His
toes are flexor on plantar stimulation. His mental status, cranial nerves, sensation, coordination,
and gait are normal.
Of the following, the test MOST likely to yield the correct diagnosis is
C. lumbar puncture
For the boy in the vignette, who does not have signs or symptoms of active infection, the best
test to evaluate his symptoms is magnetic resonance imaging (MRI) of the cervical spinal cord.
MRI of the brain would not identify a lesion in the cervical spinal cord. Computed tomography
angiography of the neck would show abnormalities of the arteries but not the spinal cord itself.
Electromyography and nerve conduction studies test the muscle, nerve, and neuromuscular
junction, which would not be helpful in this case. Lumbar puncture would likely be performed
once the MRI of the cervical spinal cord shows transverse myelitis, but it is not the best test to
diagnose an acute cervical spinal cord lesion.
PREP Pearls
• Sudden onset of bilateral arm weakness with sensory loss is suggestive of a cervical spine
lesion.
• Magnetic resonance imaging of the spinal cord is the diagnostic test of choice when there
is suspicion of an acute, nontraumatic spinal cord lesion.
Suggested Readings
• Amador CD. Index of suspicion. Pediatr Rev. 2001;22(9):303-308. DOI:
http://dx.doi.org/10.1542/pir.22-9-303.
Question 139
A 23-month-old girl presents to your office for evaluation of breast enlargement. The bilateral
breast tissue was noted 4 months after birth and has not significantly changed in size over the last
2 years. Linear growth remains normal. Physical examination reveals sexual maturity rating 3
breast tissue bilaterally, with enhancement of the breast and areola. There is no pubic hair. The
child’s parents are concerned that the breast enlargement has not regressed and request further
evaluation.
C. premature adrenarche
D. premature thelarche
Other normal variants include early menarche (due to an underlying genitourinary cause) and
isolated pubic hair of infancy.
Central precocious puberty in girls is defined as breast budding before 8 years of age, but outside
the age group of premature thelarche. In central precocious puberty, early activation of the
normal pubertal system is manifested by pulsatile GnRH secretion with subsequent LH and FSH
signaling and increased estradiol levels. A hypothalamic hamartoma is a classic lesion associated
with premature activation of the central pubertal pathway. Other central nervous system (CNS)
lesions associated with neurogenic dysfunction leading to central precocious puberty include
pineal gland cysts or tumors, and optic gliomas associated with neurofibromatosis type 1.
However, any type of intracranial disturbance or underlying congenital CNS abnormality can
disrupt GnRH inhibition and lead to central precocious puberty. In boys, the first clinical sign of
central puberty is testicular enlargement. It is essential to recognize that human chorionic
gonadotropin (hCG) can bind to the LH receptor and lead to testicular activation in the absence
of pulsatile GnRH release. In boys who have signs of central precocious puberty, hCG levels
should be measured and CNS imaging performed.
Peripheral precocious puberty is the development of pubertal signs resulting from other than the
central GnRH-mediated pathway. An example of this is androgen secretion from the adrenal
glands (causing body odor, pubic hair, and signs of virilization), seen in congenital adrenal
hyperplasia (CAH) or adrenocortical cancer (ACC). With adrenal disease, there is usually no
breast development because the adrenal glands are secreting androgens only. Children showing
signs of significant early androgen exposure, such as pubic hair with acne, clitoromegaly, voice
deepening, and rapid height acceleration, should be tested for dehydroepiandrosterone sulfate
(DHEA-S) level (elevated in ACC), 17-hydroxyprogesterone level (abnormal in the most
common form of CAH), and a total testosterone level (to evaluate for an androgen-secreting
ovarian tumor). In the event that all laboratory testing is normal, or the symptoms are so minor
that testing is not indicated, the diagnosis of premature adrenarche can be made. With premature
adrenarche there is isolated pubic hair development without breast budding or other estrogen
effects.
American academy of pediatrics 453
American Academy of Pediatrics PREP 2016
PREP Pearls
• Premature thelarche occurs in girls younger than 2 years of age who show breast
development but have no other signs of puberty.
• Girls with premature thelarche will not have rapid height acceleration consistent with a
pubertal growth spurt.
• Generally, no workup or treatment is needed for premature thelarche.
Suggested Readings
• Muir A. Precocious puberty. Pediatr Rev. 2006;27(10):373-381. DOI:
http://dx.doi.org/10.1542/pir.27-10-373.
Question 140
You have been caring for an 8-year-old girl who was hospitalized with right lower lobe
pneumonia and pleural effusion. She presented to the emergency department 4 days ago with
fever to 40.2°C, cough, and moderate respiratory distress. She has continued to require 2 L of
oxygen via nasal cannula to maintain oxygen saturations greater than 91%. She has been
persistently febrile.
A chest tube was placed on admission and an exudative pleural fluid collection was confirmed
by laboratory analysis. Culture of this fluid grew Staphylococcus aureus. The patient has been
treated with intravenous vancomycin. Despite confirmed adequate placement of the chest tube,
radiographic imaging reveals a persistent dense opacity and fluid collection in the inferior aspect
of the right lung. Fibrinolytic therapy is instilled into the pleural space without resolution of the
airspace disease. Ultrasonography reveals a complex and loculated fluid collection.
Pneumonia causes an inflammatory response in the pleura and in the pulmonary parenchyma. A
pleural exudative response to this irritation may result in the development of a parapneumonic
effusion or an inflammatory fluid collection between the parietal and visceral pleura. Initially,
this pleural fluid is likely to be clear with a low number of white blood cells. With persistence
and transition to a complicated parapneumonic effusion, there is an influx of inflammatory
mediators, development of purulence, and deposition of fibrin. The fluid may become loculated
with the presence of fibrous strands. It is important to recognize the complicated parapneumonic
effusion prior to the development of fibrotic change with irreversible lung restriction.
As disease progresses, changes occur in the pleural fluid, which include a decrease in glucose
and pH, as well as an increase in lactate dehydrogenase (LDH). The Light criteria have been
used to characterize pleural fluid as exudative if at least 1 of the following criteria are met:
o The ratio of pleural fluid protein to serum protein is greater than 0.5
o The ratio of pleural fluid LDH to serum LDH is greater than 0.6
o LDH in pleural fluid is greater than two-thirds the normal upper limit value
The sensitivity and specificity of the Light criteria for determination of an exudative pleural
effusion have been reported as 98% and 80%, respectively. While the Light criteria are
commonly utilized in clinical care, they have not been validated in pediatric patients.
Of note, recent data have shown that serum C-reactive protein levels are significantly higher in
children with empyema, as compared to children with uncomplicated pneumonia. In the imaging
of pneumonia and parapneumonic effusions, it is often challenging to differentiate between lung
consolidation and pleural fluid with the use of plain radiographs. While a lateral decubitus
radiograph may demonstrate “layering” with free flowing pleural effusions, the same is not
expected with a complex or loculated parapneumonic fluid collection. Ultrasonography involves
no exposure to ionizing radiation and is a useful modality in differentiating between a pleural
and parenchymal process. It is likely superior to computed tomography (CT) in the
demonstration of loculations. The use of CT should be limited to complex cases where
parenchymal abscess or bronchopleural fistulae are being considered in the differential.
In general, small and moderate uncomplicated parapneumonic effusions can be treated with
antibiotics and do not routinely require chest tube drainage. Drainage with chest tube insertion is
warranted when parapneumonic effusions are large in size or when purulence has been
demonstrated; drainage is also indicated if the effusion is associated with respiratory compromise
or distress. Chest thoracotomy tube drainage is an effective intervention for free flowing
effusions. When effusions are complex or loculated, they may respond to instillation of
fibrinolytic agents (such as tissue plasminogen activator or urokinase) through the indwelling
thoracostomy tube. Video-assisted thoracotomy (VATS) should be performed when there is
American academy of pediatrics 456
American Academy of Pediatrics PREP 2016
The patient in the vignette has now failed antibiotic therapy, chest thoracotomy tube drainage,
and fibrinolysis. The next most appropriate step in management is a VATS with decortication of
the loculations.
Hypertonic saline draws fluid into the airway after nebulized inhalation and may provide
symptomatic relief in cystic fibrosis or bronchiolitis by rendering secretions less tenacious. The
fluid collection in this case is not in the airway and would not respond to this therapy.
Continued observation, repeat instillation of fibrinolytic therapy, and lung biopsy are not
indicated. Timely management of the complicated effusion is warranted in order to prevent late
complications, including restrictive lung disease.
PREP Pearls
• An elevated C-reactive protein on admission in a patient with radiographic pneumonia is
a useful marker of inflammation and may predict the presence of a complicated effusion.
• Computed tomography (CT) is not superior to ultrasonography in the evaluation of
complicated parapneumonic effusions and the use of CT should be limited because of the
risks of exposure to ionizing radiation.
• Small- and moderate-sized parapneumonic effusions may respond to intravenous
antibiotic therapy alone.
• Video-assisted thoracotomy should be performed when there is persistence of moderate
to large effusions and ongoing respiratory compromise, despite 2 to 3 days of
management with chest tube and completion of fibrinolytic therapy.
Suggested Readings
• Carter E, Waldhausen J, Zhang W, Hoffman L, Redding G. Management of children with
empyema: pleural drainage is not always necessary. Pediatr Pulmonol. 2010;45(5):475-
480. DOI: http://dx.doi.org/10.1002/ppul.21200.
Question 141
You are seeing first time parents for a prenatal visit. The mother is trying to determine which
feeding practices will fit with her lifestyle as a biology graduate student. She has heard that
breastfeeding promotes a healthier immune system compared to formula feeding and asks what
the mechanism of this protection is.
I. the intestinal flora of breast- and formula-fed infants are very similar and have little effect
on the baby’s immune response
J. most of the immunologically active components of human milk enhance the baby’s
immune response at the mucosal level
The American Academy of Pediatrics (AAP) strongly supports breastfeeding both for its
nutritional and non-nutritive benefits. The AAP recommends exclusive breastfeeding for the first
6 months of life and continued breastfeeding for at least 1 year with the addition of
complementary foods starting at 6 months of age. Breastfed infants experience decreased rates or
severity of infectious disease (upper and lower respiratory tract infections, respiratory syncytial
virus bronchiolitis, otitis media, nonspecific GI infection, and necrotizing enterocolitis), atopic
disease (asthma, atopic dermatitis), sudden infant death syndrome, GI diseases (Celiac disease,
inflammatory bowel disease), leukemia, obesity, and diabetes mellitus. While less clear, there is
evidence that breastfeeding results in enhanced neurodevelopment.
The bacterial microflora of the gut is also critical in the development of local immunity.
Oligosaccharides are important for creating a healthy microflora, and cow milk-based formula
contains much lower levels of oligosaccharides than human milk does. The microflora of breast
versus formula-fed infants are quite different. In human milk-fed infants, Lactobacillus bifidus
and Bifidobacterium are the predominant organisms, while for formula fed-infants, the most
common organisms are gram negative. This difference in microflora likely contributes to
differences in immune response. Adding probiotics to formula may improve the gastrointestinal
microflora, but does not provide the other immune benefits of human milk.
While breastfeeding helps protect the infant against various pathogens, breast milk can be the
vehicle to transmit some viruses from mother to infant. Notable among these viruses are HIV and
cytomegalovirus.
PREP Pearls
• The immunoactive components of human milk act primarily within the gastrointestinal
tract at the mucosal level.
• Immunologically active proteins in human milk, including secretory immunoglobulin A,
support immune function by inhibiting bacterial growth, preventing microbial mucosal
adhesion and penetration, supporting gastrointestinal tract development and repair, and
enhancing macrophage activity.
• Oligosaccharides in human milk promote growth of the gastrointestinal microflora, which
differs between human milk-fed and formula-fed infants.
Suggested Readings
• American Academy of Pediatrics. Breastfeeding and the use of human milk. Pediatrics.
2012;129(3):e827-e837. DOI:http://dx.doi.org/10.1542/peds.2011-3552.
Question 142
You are asked to present to the pediatric residents at your local training program on adolescent
health. You remind the residents of the top causes of adolescent and young adult (age 15-24
years) mortality and stress the importance of thorough psychosocial risk assessments during
adolescent preventive care visits.
Of the following, the top 3 leading causes of death for this age group are unintentional injury,
PREP Pearls
• The leading causes of adolescent death are unintentional injuries, followed by homicide
and suicide.
• Sex and racial disparities exist in the causes of adolescent mortality.
• Non-Hispanic, American Indian/Alaska Native adolescents had the highest rate of
mortality among both males and females.
• Bright Futures discusses provision of anticipatory guidance to adolescents.
Suggested Readings
• American Academy of Pediatrics. Bright Futures. Bright Futures website.
http://brightfutures.aap.org/.
Question 143
A 15-year-old adolescent presents to your office with right-sided knee pain 10 days after a
basketball injury. She was running down the court during a game, came to a sudden stop, and felt
“a pop” in her knee. The patient was seen at an urgent care clinic on the day of injury, where
radiographs were performed and were unremarkable. The urgent care clinician fitted her with a
knee immobilizer and recommended follow-up with her primary care physician. On physical
examination, you note a large right knee effusion. The patient is guarding, and therefore you are
unable to adequately assess for ligamentous laxity.
Approximately 75% of ACL tears are noncontact injuries, with female athletes at particularly
high risk. Common mechanisms of injury include sudden deceleration, landing from a jump, and
twisting or changing direction.
The diagnosis of ACL injury can be made by taking a careful history and demonstrating
ligamentous laxity with the Lachman maneuver. To perform a Lachman maneuver, the examiner
flexes the patient’s knee to 30 degrees, stabilizes the femur with 1 hand, and attempts to pull the
tibia anteriorly with the other hand. Increased anterior translation of the tibia relative to the
femur compared with the contralateral side is indicative of ACL injury. Although radiographs
should be obtained to assess for bony injuries, MRI is the preferred test to evaluate for
ligamentous injury, meniscal tear, or cartilage injury. Surgical reconstruction is preferred for
athletes who wish to return to sports involving jumping or direction change. The surgical
technique for skeletally mature individuals involves drilling through the bone in the area where
the physis is located. Therefore, young children should be referred to an orthopedic surgeon
experienced in ACL reconstruction using physeal sparing surgical procedures.
Because children and teens with a history of ACL injury are at high risk for early arthritis,
prevention of these injuries is extremely important. Exercise programs emphasizing balance,
strength, and landing mechanics have been shown to reduce rates of injury. Athletes in sports
with high injury rates, for example, soccer and basketball, appear to derive the largest benefits
from neuromuscular training programs.
For the athlete in the vignette, a return to sports is inappropriate because her history and physical
examination are suggestive of intra-articular injury. Aspiration of the knee would not be helpful
in this case, because the fluid would likely reaccumulate and there is a risk of infection.
Nonsteroidal anti-inflammatory medications can be used for pain control, but are unlikely to
affect the girl’s clinical course 10 days after the injury. A patella-stabilizing brace would be an
appropriate initial treatment for a patella subluxation or dislocation, without associated
osteochondral fracture. Although this injury is in this girl’s differential diagnosis, it would be
important to rule out an ACL injury before implementing this type of treatment.
PREP Pearls
• Magnetic resonance imaging (MRI) of the knee is the best test to evaluate the integrity of
the ligaments, articular cartilage, and menisci.
• Anterior cruciate ligament (ACL) injuries are typically noncontact injuries, and affect
female athletes disproportionately.
• An athlete with an acute injury mechanism and joint effusion has a high likelihood of
intra-articular injury.
Suggested Readings
• Labella C, Carl R. Preventing knee ligament injuries in young athletes. Pediatr Ann.
2010;39(11):714-720. DOI:http://dx.doi.org/10.3928/00904481-20101013-10.
Question 144
A 4-year-old girl is brought to the hospital after falling backwards into a campfire pit. She has
been intubated and sedated in anticipation of painful dressing changes. Her weight is 20 kg. Her
vital signs include a temperature of 37°C, pulse of 120 beats/min, respiratory rate of 20
breaths/min, and blood pressure of 90/50 mm Hg. The girl has second degree burns involving her
neck, posterior trunk, and buttocks, estimated to cover approximately 20% of her total body
surface area. Her pupils are 2 mm, equal, and reactive. On auscultation, her heart has a regular
rate and rhythm with no murmurs, and her lungs are clear bilaterally. Her abdomen is soft,
nontender, and nondistended. Her extremities are warm and well-perfused, with a capillary refill
time of 2 seconds. There are no other external signs of trauma.
Of the following, the MOST appropriate fluid management for this child’s first 24 hours of
treatment is
Children with burns—second degree or greater—involving more than 15% TBSA generally
require IV fluid replacement. The “Parkland formula” can be used to plan the fluid requirement
for the first 24 hours:
For the child in the vignette, the required fluid volume would equal 4 × 20 kg × 20 = 1,600 mL.
Because the purpose of fluid administration is to maintain intravascular volume, the choice
should be isotonic, such as normal saline or lactated Ringer solution. For children younger than 2
years, maintenance IV fluids containing dextrose should be added over and above the Parkland
formula fluid. Colloids should be avoided unless the serum albumin level is extremely low,
because capillary extravasation of albumin may exacerbate tissue edema. If the child is in acute
shock, characterized by hypotension or evidence of decreased end-organ perfusion, additional
boluses of isotonic fluid may be required. Care must be taken to not overhydrate the patient,
because increased intravascular hydrostatic pressure can exacerbate edema, and fluid overload
can lead to respiratory and multiple organ failure.
PREP Pearls
• The percentage of total body surface area of a burn can be estimated using the “Rule of
Nines.”
• The increased fluid requirement for a burn patient is because of increased evaporative
insensible losses from skin damage and extravasation of fluids from increased capillary
permeability.
• A pediatric burn patient requires isotonic fluid replacement in the first 24 hours, the
volume of which can be estimated with the Parkland formula.
Suggested Readings
• Hansbrough JF, Hansbrough W. Pediatric burns. Pediatr Rev. 1999;20:117. DOI:
http://dx.doi.org/10.1542/pir.20-4-117.
Question 145
A term male newborn is born to a 35-year-old woman known to have hepatitis C. The mother is
anxious to know if her newborn has acquired the infection.
Of the following, the test that would BEST identify early infection in the newborn is
Immunoglobulin G (IgG) antibody assays and nucleic acid amplification tests are the 2 major
means of diagnosing HCV infections. Immunoglobulin M assays for HCV are not available.
Neonates born to HCV-infected mothers are expected to be antibody-positive at birth, given
transplacental transfer of maternal IgG. Serologic tests prior to 18 months of age cannot
distinguish infection in the child from infection in the mother. Therefore, serology performed at
4 months of age, is not a correct option. Serology could be performed at 18 months of age, when
maternal IgG would be expected to be undetectable; however, this option would not best identify
infection early.
The principal means of transmission of HCV is exposure to contaminated blood or body fluids.
Certain risk factors warrant screening for HCV infection in children. These include having a
HCV-infected mother, recipients of blood products or transplants prior to 1992, international
adoptees and refugees, HIV-infected children, adolescents with any history of illicit intravenous
drug use, adolescents with multiple sexual partners, victims of sexual assault, and children with
unintentional needle stick injuries.
Vertical transmission accounts for the majority of cases of HCV infection in children.
Approximately 5% to 10% of neonates born to mothers with chronic infection will acquire HCV;
this incidence is about 750 cases per year in the United States. Young children can spontaneously
resolve HCV infection. Twenty-five percent to 75% of HCV-infected infants will not develop
chronic infection.
PREP Pearls
• Neonates born to hepatitis C virus (HCV)-infected mothers are expected to be antibody-
positive, given transplacental transfer of maternal anti-HCV antibodies.
• Polymerase chain reaction assays that can detect viremia are preferred over serology for
early diagnoses of vertically transmitted HCV infection and should be performed
between 2 and 6 months of age.
• Vertical transmission accounts for the majority of cases of HCV infection in children.
Suggested Readings
• American Academy of Pediatrics. Hepatitis C. In: Kimberlin DW, Brady MT, Jackson,
MA, Long SS, eds. Red Book: 2015 Report of the Committee on Infectious Diseases.
30th ed. Elk Grove Village, IL: American Academy of Pediatrics; 2015:423-430.
Question 146
You are seeing a 16-year-old adolescent in your office for a health supervision visit. The patient
received a renal transplant from his father 2 years ago and is on a steroid-free immune
suppressive protocol. According to the parents, he is doing well post-transplantation, with no
concerns from the transplant team. His current medications include tacrolimus, mycophenolate
mofetil, atenolol, and oral magnesium and phosphorus supplements. He is afebrile with a
respiratory rate of 18 breaths/min, heart rate of 78 beats/min, and blood pressure of 119/76 mm
Hg. His physical examination shows a well-healed postsurgical abdominal scar.
A. he is at increased risk for graft failure after renal transplant because of his age
B. his graft survival is the same as a patient who receives a deceased donor kidney
E. his immune suppressive protocol is associated with poorer growth than those
containing steroids
According to the United States Scientific Registry of Transplant Recipients, outcomes of renal
transplantation in children have improved over the last 25 years. There are various factors
affecting the outcome of renal transplantation in children. According to a North American
Pediatric Renal Trials and Collaborative Studies report, patients with increased risk for renal
allograft failure 10 years post-transplant are likely to be older, nonwhite, have a history of focal
segmental glomerulosclerosis, have received a deceased donor kidney, and are less likely to have
bladder dysfunction. Similar findings for children with renal transplant were reported from the
analysis of Organ Procurement and Transplant Network database from 1995 to 2000.
Patients aged 12 years and older, such as the adolescent in this vignette, are at increased risk for
allograft failure. This is attributed to nonadherence to immunosuppressive medications and
transition to adult healthcare services.
Renal allografts from a living donor are associated with higher survival rates, as compared to
those from a deceased donor because of better circumstances for donor organ removal, storage,
and engraftment. The potentially injurious effect on deceased donor allograft may lead to
increased graft immunogenicity, which may influence the development of chronic allograft
dysfunction.
Growth impairment is a common problem in children with chronic renal failure and is attributed
to inadequate nutrition, metabolic acidosis, renal osteodystrophy, and growth hormone
insensitivity. In the setting of an optimally functioning renal allograft, these factors are reversed,
and children show improved growth and growth velocity. To prevent graft rejection, patients
with functioning renal allograft are on immunosuppressive medications, which also increase the
risk for infection in comparison to the general population.
Daily glucocorticoid therapy following renal transplantation (more than 5 mg/m2 per day) is a
contributor to poor growth in children following renal transplantation. Strategies to reduce the
cumulative glucocorticoid dose have included alternate day steroids, late or early steroid
withdrawal, and immunosuppressive regimens avoiding steroids completely (steroid avoidance
transplant protocol). Steroid avoidance protocols have been used only in patients at low risk for
allograft rejection. Although studies are limited, improved growth outcomes without increased
adverse effects of allograft rejection are reported with steroid avoidance. The beneficial effects
of growth are most prominent in children younger than 5 years of age with steroid avoidance.
PREP Pearls
• Patients 12 years of age and older are at increased risk for renal transplant allograft
failure.
• Living donor renal allografts are associated with higher survival rates, as compared to
those from a deceased donor.
• Improved growth and growth velocity is seen in children post-renal transplantation.
• Immunosuppressive regimens with steroid avoidance have improved growth outcomes
without increased adverse effects of allograft rejection.
• Patients receiving immunosuppressive medications are at increased risk for infections.
Suggested Readings
• Butani L, Perez RV. Effect of pretransplant dialysis modality and duration on long-term
outcomes of children receiving renal transplants. Transplantation. 2011;91(4):447-451.
Question 147
You are called to evaluate a 1-hour-old newborn who is experiencing some coughing and
choking that improves with suctioning. He was delivered by spontaneous vaginal delivery
following a prenatal course complicated by polyhydramnios. Vital signs show a temperature of
37°C, heart rate of 120 beats/min, and a respiratory rate of 25 breaths/min. On physical
examination, the newborn is drooling with copious oral secretions. His examination is otherwise
unremarkable.
A chest radiograph is taken (Item Q147). Laboratory studies are remarkable for a normal
complete blood cell count and electrolytes. A blood culture is pending.
His symptoms worsen significantly
following the first attempt to breastfeed.
A. congenital diaphragmatic
hernia
B. congenital esophageal
stricture
C. esophageal atresia
E. pulmonary sling
Other esophageal anomalies include congenital esophageal stenosis, presenting with 3 variants:
fibromuscular stenosis, cartilaginous tracheobronchial remnants, and esophageal webs. These
rare anomalies affect 1 in 25,000 to 50,000 live births and, in 85% of patients, are an isolated
anomaly. They often present at the time of pureed food introduction or later with failure to
thrive.
Duplication cysts can occur anywhere along the gastrointestinal tract. Duplications occur in 1 out
of 4,000 to 8,000 live births, with about 33% affecting the foregut. The cysts usually have a
smooth muscle wall and are lined with either secretory, respiratory, or alimentary tract mucosa.
The cysts can increase in size, obstructing the lumen, resulting in intolerance of feeds.
Vascular rings are ligamentous and vascular structures that encircle the trachea and esophagus.
These are very rare, making up less than 1% of vascular anomalies. Presentation often includes
feeding difficulty or respiratory compromise. Vascular rings may be indirectly visualized with
radiographs and by barium esophogram, but are best visualized with angiography, computed
tomography, and magnetic resonance imaging.
For the newborn in this vignette, congenital diaphragmatic hernia would have a chest radiograph
demonstrating intestines above the diaphragm. Esophageal stricture and vascular ring would not
have this significant intolerance of secretions and, depending on the severity of the stricture, the
orogastric tube should have passed. Esophageal duplication cysts do not commonly obstruct the
entire lumen of the esophagus and a cystic structure should be seen on the chest radiograph.
PREP Pearls
• Esophageal atresia with a distal tracheoesophageal fistula is the most common form of
esophageal atresia.
• Chest radiograph after orogastric tube placement is used as the initial evaluation for
esophageal atresia.
• Esophageal strictures, duplication cysts, and vascular rings can also reduce the lumen of
the esophagus and impact swallowing.
Suggested Readings
• Achildi O, Grewal H. Congenital anomalies of the esophagus. Otolaryngol Clin North
Am. 2007;40(1):219-244. DOI:http://dx.doi.org/10.1016/j.otc.2006.10.010.
Question 148
You are called by the newborn nursery to evaluate a baby that is having difficulty feeding. On
physical examination, you find a small, receding mandible, a cleft palate, bifid uvula, midfacial
hypoplasia, and an abnormal red eye reflex consistent with a cataract. The tongue is posteriorly
positioned. The newborn’s father looks similar to the baby and reports a history of severe
myopia, mitral valve prolapse, and a cleft palate repaired as a child.
A. audiogram
C. echocardiogram
E. skeletal survey
of micrognathia, cleft palate, and glossoptosis in an individual that can lead to life-threatening
obstructive apnea and feeding difficulties in the neonatal period. The obvious physical
examination findings in this patient include the facial dysmorphology and abnormal red eye
reflex secondary to the cataract. Health care providers should be mindful of the high risk of
hearing loss as well because it could impact normal cognitive and speech development. Skeletal
findings can include femoral head dysplasia, scoliosis, spondylolisthesis, and joint laxity early in
life that progresses to early onset osteoarthritis, or kyphosis. It can be autosomal dominant or
autosomal recessive in its inheritance pattern and can present with variable phenotypic
expression, even within the same family.
An audiologic examination should be performed early on, as patients can have both conductive
and sensorineural hearing loss in the neonatal period or develop it overtime. Therefore, patients
with PRS need audiologic evaluations every 6 months through 5 years of age, and annually
thereafter.
Pierre Robin sequence (PRS) (Item C148) can present as an isolated finding, in association with
additional congenital malformations (PRS-Plus), or in association with a defined genetic disorder
(syndromic PRS). The most common genetic disorders associated with PRS are 22q11.2 deletion
or Stickler syndrome. In fact, 11% to 18% of babies with PRS will have Stickler syndrome.
Clinical testing exists for both of these disorders. It can also be seen with other disorders
including Treacher Collins syndrome, Nager syndrome, Miller syndrome, chromosomal
abnormalities, teratogenic exposure, and some skeletal dysplasias (Kniest dysplasia, campomelic
dysplasia). The respiratory problems with PRS require careful attention, as many patients will
require prolonged stays in the neonatal intensive care unit, or surgical interventions such as
tracheotomy or a gastrostomy tube. The respiratory and feeding difficulties can persist into
childhood. Obstructive sleep apnea, failure to thrive, and long term developmental difficulties
are common. A genetics workup is strongly recommended in the newborn period, as it can prove
American academy of pediatrics 480
American Academy of Pediatrics PREP 2016
valuable for accurate diagnosis and an appropriate care plan with specific surveillance guidelines
for the child. At the minimum, one should order a high resolution karyotype, microarray, and a
detailed ophthalmological assessment at birth and 6 months of age. A targeted family history
should be taken for the presence of clinical features suggestive of Stickler syndrome in the
family, such as osteoarthritis, myopia, and short stature. If Stickler syndrome, as in this case, is
highly suspected, then an audiologic examination should be ordered. Stickler syndrome is not
commonly associated with hepatic dysfunction, brain abnormalities, or congenital heart defects,
so liver function tests, brain magnetic resonance image, or an echocardiogram would not be
indicated in this scenario. A skeletal survey may show abnormalities, but would not take
immediate precedence over clinical assessment for hearing loss in the newborn period for the
patient in this vignette.
PREP Pearls
• Stickler syndrome is a connective tissue disorder that is associated with midfacial
hypoplasia, cleft palate, Pierre Robin sequence (PRS), hearing loss, and abnormalities of
the eye, including high grade myopia, cataracts, and increased risk for retinal detachment.
• Pierre Robin sequence is the clinical constellation of micrognathia, cleft palate, and
glossoptosis in an individual that can lead to life-threatening obstructive apnea and
feeding difficulties in the neonatal period.
• Pierre Robin sequence can present as an isolated finding, in association with additional
congenital malformations (PRS-Plus), or in association with a defined genetic disorder
(syndromic PRS). The most common genetic disorders associated with PRS are 22q11.2
deletion or Stickler syndrome.
American academy of pediatrics 481
American Academy of Pediatrics PREP 2016
Suggested Readings
• Evans KN, Sie KC, Hopper RA, Glass RP, Hing AV, Cunningham ML. Robin sequence:
from diagnosis to development of an effective management plan. Pediatrics.
2011;127(5):936-948. DOI: http://dx.doi.org/10.1542/peds.2010-2615.
Question 149
A 2-year-old, African-American boy presents to your office for a new patient health supervision
visit. His birth and past medical history are unremarkable. His growth and development are
appropriate for his age. On physical examination, you notice an extra digit adjacent to the fifth
toe bilaterally (Item Q149). His mother reports that several family members on the paternal side
had similar extra digits, but they are otherwise well.
Of the following, the MOST common pattern of inheritance for this condition is
A. autosomal dominant
B. autosomal recessive
D. X-linked dominant
E. X-linked recessive
Molecular defects seen in patients with polydactyly include chromosomal abnormalities as well
as single gene defects. Polydactyly is associated with a number of syndromes including trisomy
13, chondroectodermal dysplasia, Meckel-Gruber, otopalatodigital, Bardet-Biedl, and short rib
polydactyly syndrome.
Management of polydactyly depends on the digit location and formation. The rudimentary skin
tag form may be simply tied off, but cosmetic outcomes and patient care may improve with
excision. Another consideration is functionality, that is, proper fit of shoes. Well-formed digits
may have bone and neurovascular structures, therefore, referral to a surgeon is necessary and
radiologic evaluation may be helpful to define the anatomy. For patients with signs and
symptoms of an underlying genetic disorder, referral to a genetic specialist and evaluation for
comorbidities is warranted.
PREP Pearls
• The most common pattern of inheritance for isolated polydactyly is autosomal dominant.
• The location and degree of supernumerary digit formation determines the most
appropriate management in cases of polydactyly.
• Because polydactyly may occur as part of a syndrome, providers must be aware of
potential comorbidities and the indications for referral to a specialist.
Suggested Readings
• Carrigan RS. The upper limb: polydactyly. In: Kliegman RM, Stanton BF, St Geme JW
III, Schor NF, Behrman RE, eds.Nelson Textbook of Pediatrics. 20th ed. Philadelphia,
PA: Saunders; 2015: 3302-3310.
Question 150
A 15-year-old adolescent girl presents to your office with a 1-month history of progressive
fatigue and exercise intolerance. Her medical history is remarkable for autoimmune thyroiditis
for which she takes 112 µg of oral levothyroxine daily. She has recently had normal thyroid
stimulating hormone, thyroxine (T4), and free T4 levels. Her physical examination is remarkable
only for pallor. She specifically denies any history of hematuria, hematochezia, epistaxis, or
unusual bruising. She has had a normal diet for age. The results of a complete blood cell count
are shown:
• White blood cell count 11,000/µL (11.0 x 109/L)
• Hemoglobin 7.2 g/dL (72 g/L)
• Mean corpuscular volume 116 fL
• Platelet count 467 x 103/µL (467 x 109/L)
• Reticulocyte count 0.5%
Of the following, her fatigue and pallor are MOST likely a result of damage to the
A. adrenal cortex
B. adrenal medulla
D. hepatocytes
The patient in the vignette has a medical history remarkable for autoimmunity, suggesting that
her macrocytic anemia also has an autoimmune origin. Thus, it is likely that her macrocytic
anemia is caused by autoimmune damage to the gastric parietal cells.
Autoimmune damage to the adrenal cortex results in Addison disease, a deficiency of cortisol
(glucocorticoids) and often aldosterone (mineralocorticoids). It will not cause a macrocytic
anemia. The adrenal medulla produces catecholamines. It can be damaged and underproduce
catecholamines or have a tumor such as a pheochromocytoma and overproduce catecholamines.
Neither will result in a macrocytic anemia. Hepatic injury in the form of alcoholic cirrhosis can
coexist with vitamin B12 deficiency, but it is not, by itself, a cause of macrocytic anemia. Injury
to the pancreatic islet cells results in an insulin deficiency and diabetes, but not macrocytic
anemia.
PREP Pearls
• Vitamin B12 deficiency results in a macrocytic anemia and neurological changes as a
result of the degeneration of the posterior and lateral spinal columns.
• Intrinsic factor is produced in the gastric parietal cells and is necessary for the absorption
of vitamin B12.
• The most common cause of vitamin B12 deficiency in adolescents is autoimmune gastric
atrophy and achlorhydria, resulting in damage to the gastric parietal cells, with a resultant
deficiency of intrinsic factor.
• Vitamin B12 is absorbed in the terminal ileum.
Suggested Readings
• Kaferle J, Strzoda CE. Evaluation of macrocytosis. Am Fam Physician. 2009;79(3):203-
208.
Question 151
A 16-year-old adolescent is having problems with his sleep. He says he can never fall asleep
before midnight, and usually falls asleep around 1:00 AM or even 2:00 AM. He wakes at 6:00
AM for school, but only after his mother has to struggle to get him out of bed. He says he feels
sleepy throughout most of the school day and occasionally falls asleep in class. On most days, he
takes a nap right after getting home from school, sleeping for more than 1 hour. He says he has
always been an “evening person.” Upon questioning, his mother says he does not snore at night.
She says at home they all tend to be evening people. They have instructed him to turn off all of
his electronics at 11:00 PM, but this has not seemed to help him fall asleep any earlier. On
physical examination, his tonsils are 2+ and symmetric.
A. encourage that he catch up on sleep on the weekends by staying in bed past noon
C. recommend that he perform a 20-min workout about 1 hour before going to bed
Adolescents tend to have phase advanced sleep, meaning that there is a natural tendency while
progressing into adolescence of falling asleep later in the evening, followed by waking up later in
the morning. Some schools now try to match their schedules to that pattern, by scheduling high
school classes to start later in the morning than classes for younger children.
The total number of hours a day of sleep that humans need is fairly consistent, so when there is
chronically short duration sleep at night, a sleep deficit is created that requires payback to be able
to maintain normal level functioning. This experienced sleep deficit is what is most likely
leading this adolescent to take a nap every afternoon. A downside of taking naps lasting beyond
about 15 min in duration is that "sleep inertia" is created in which it is harder for someone to feel
fully awake for the subsequent hours after waking. Even after taking his afternoon nap of
approximately 1-hour duration, he is likely to experience sleep inertia rather than feeling truly
refreshed upon awakening. More importantly in this situation, the use of afternoon naps
decreases the need for sleep later that evening, and impairs the ability to easily fall asleep at
night. The most immediate need for the adolescent in this vignette is sleep hygiene management.
Both patient and parent will need to commit to not allowing the taking of naps after school, while
also providing a reasonable opportunity for him to fall asleep before or up to 11:00 PM (ie, the
home activity winds down, no personal electronics, etc). It may be hard for him to make himself
stay awake after school at first, given the routine that has been established, but doing so should
lead to a rapid resolution of the problem.
Extra catch-up sleep on the weekends can help to restore an accumulated sleep deficit and restore
clear cognition. However, a big downside of sleeping in until noon or later every weekend is that
this perpetuates a phase-advanced circadian rhythm, which does not help the adolescent's sleep
schedule during the following school week. Maintaining a relatively consistent sleep pattern
every day of the week is another key way to help maintain an appropriate-for-circumstances
biological clock.
Obtaining regular exercise has been shown to help improve sleep habits overall. However,
performing significant physical exercise within an hour of the intended bedtime typically impairs
being able to settle the mind and body down and be able to fall asleep. Exercise should happen
earlier in the day to be able to obtain sleep benefits at night.
Referring the patient to a surgeon to consider a tonsillectomy may be appropriate in the setting of
significant problems with snoring, nocturnal enuresis, or gasping respirations. If these symptoms
are present, a polysomnogram should be considered before a referral to a surgeon. Tonsils that
American academy of pediatrics 489
American Academy of Pediatrics PREP 2016
are 2+ sized, as in the patient in this scenario, would not be a very likely cause of sleep apnea
difficulties in someone with an otherwise normal oropharynx.
Polysomnography would be the appropriate next step to document apneic pauses if sleep apnea is
suspected. In this vignette, the patient’s history suggests that he is having sleep hygiene problems
and referral for a sleep study is not necessary, presuming an uncomplicated course with sleep
hygiene interventions.
Sometimes sleep can be disrupted because of nightmares, particularly for individuals with post-
traumatic stress disorder for whom repeated nightmares may decrease sleep quality and cause
sleep avoidance behaviors. Nightmares occur during rapid eye movement sleep when muscle
tone and movement is inhibited, and generally in the latter part of the morning. Nightmares are
different than sleep terrors in that children between 4 and 12 years of age may experience them.
Sleep terrors usually occur during the first third of sleep at night, cause abrupt apparent
awakening (though typically not full consciousness) with a loud scream, and physical agitation
that is unresponsive to parental calming.
PREP Pearls
• Chronic daytime sleepiness is often related to poor sleep habits.
• Sleep hygiene interventions are the first line treatment for most sleep problems in young
people.
• For someone with chronic insomnia, taking naps during the day is likely to interfere with
their ability to obtain sleep at night.
Suggested Readings
• Cespedes EM, Gillman MW, Kleinman K, Rifas-Shiman SL, Redline S, Taveras EM.
Television viewing, bedroom television, and sleep duration from infancy to mid-
childhood. Pediatrics. 2014;133(5):e1163-e1171.
DOI:http://dx.doi.org/10.1542/peds.2013-3998.
Question 152
An 11-year-old girl faints as she walks off the soccer field at sleep away camp. The temperature
at camp has been 95°F (35°C) all day and she has been outside and active. Emergency medical
services (EMS) was called and she was taken to the small emergency department near the camp.
By history, she was unresponsive on arrival with a heart rate of 190 beats/min, blood pressure of
65/45 mm Hg, and respiratory rate was 15 breaths/min. While she was being evaluated, she
regained consciousness, her heart rate dropped to 90 beats/min, and her blood pressure
normalized to 105/70 mm Hg. She was observed for several hours and discharged home. She
comes to see you in the office next week. The family has requested a copy of the EMS rhythm
strip, but does not have it with them. She is feeling well today and has an unremarkable physical
examination. She is anxious to get back to playing competitive soccer.
The patient faints again in your office and you call 911. You obtain an electrocardiogram in the
office (Item Q152).
E. propranolol, 40 mg orally
When faced with a patient in wide complex tachycardia, especially if unstable, it is safest to
assume that the rhythm is VT. Even if the rhythm is an SVT with bundle branch aberrancy, if the
patient is unstable, then synchronized cardioversion is the treatment of choice. A starting dose of
2 J/kg is recommended. If the patient were stable, there would be more time to differentiate SVT
from VT. Ventricular tachycardia will more classically show atrioventricular (AV) dissociation.
This feature is the most helpful in differentiating SVT with aberrancy from VT. Inspection of the
electrocardiogram for a regular P to P interval and a regular R to R interval will eventually show
these to stop matching up one to one if there is AV dissociation as would be expected in VT.
Adenosine may convert SVT by causing transient AV block, but can also cause at least a
momentary pause in VT if it involves the posterior fascicle of the left bundle. In a more stable
patient, who does not require immediate cardioversion, amiodarone may be used to treat VT.
Lidocaine and procainamide are other medication choices to consider.
There are several subtypes of VT. The most common type in the pediatric population arises from
the right ventricular outflow tract (RVOT) and may be benign. This form of ventricular
tachycardia will have an inferior axis (negative in V1) because it comes from the superior aspect
of the heart. There will also be left bundle branch morphology because the activation arrives in
the left ventricle after the right ventricle (Item C152A). This type of VT or ventricular ectopy
may be benign if it suppresses with the sinus tachycardia that occurs with exercise, is always
monomorphic, and the heart is structurally normal. Monomorphic VT is regular (all the QRS
complexes are the same) and is consistent with the VT originating from one area of the ventricle
and not changing its axis in the midst of the arrhythmia. It often is seen on stress tests during the
recovery phase when the heart rate is decreasing. This type of VT may cause hypotension if
rapid or prolonged, but may be well tolerated, especially if it is not sustained.
Ventricular tachycardia that is not from the RVOT, worsens with exercise or increases with
catecholamines, is polymorphic, is associated with congenital heart disease, and is more likely to
be dangerous. Polymorphic refers to the morphology of the VT; the QRS will differ beat to beat
and the VT may change axis during the arrhythmia. This suggests a more disorganized VT not
originating from 1 focus. This form of VT is of higher risk. Arrhythmogenic right ventricular
cardiomyopathy or dysplasia (ARVD), catecholamine-sensitive polymorphic VT (CPVT), and
long QT (LQT) are inheritable arrhythmia syndromes capable of causing polymorphic
ventricular tachycardia. Torsades de pointes is a specific form of polymorphic VT seen with
LQT syndrome (Item C152B). In each of these syndromes, the first presentation may be collapse
or sudden cardiac arrest.
Long term management of VT depends on the type of VT, any underlying genetic abnormality,
and cardiac anatomy. If the ectopy suppresses with exercise, this is reassuring. Polymorphic VT
associated with congenital heart disease, CPVT, LQT syndrome, or ARVD is much higher risk.
In patients with one of the inheritable arrhythmia syndromes, a combination of medication and
protection with an implantable cardiac defibrillator will be needed if VT has developed.
PREP Pearls
• Unstable wide complex tachycardia requires cardioversion.
• Monomorphic (premature ventricular contractions with the same morphology and axis
throughout) ectopy from the right ventricular outflow tract in a patient with a structurally
normal heart, which suppresses with exercise, is a lower risk type of ectopy.
• Several hereditary arrhythmia syndromes are associated with a more malignant form of
tachycardia.
Suggested Readings
• Cannon BC, Snyder CS. Disorders of the cardiac rhythm and conduction. In: Allen HD,
Driscoll DJ, Shaddy RE, Feltes TF, eds. Moss and Adams’ Heart Disease in Infants,
Children, and Adolescents: Including the Fetus and Young Adult. 8th ed. Philadelphia,
PA: Lippincott Williams & Wilkins; 2012: 441-472.
Question 153
You are seeing a 14-year-old adolescent girl in your office after her yearly ophthalmology
appointment, where she was found to have a cataract in her left eye. During your review of
systems, she describes bilateral wrist pain that started 5 months ago. She also reports pain in her
hands that interferes with writing and school work, which started at the same time as her wrist
pain. She has noticed bilateral knee swelling and pain over the last month. You are concerned
that the patient may have juvenile idiopathic (rheumatoid) arthritis.
Of the following, the MOST specific sign or symptom for this diagnosis is
A. fatigue
B. frequent diarrhea
The differential diagnosis when evaluating a child for JIA is broad. Arthritis may be a presenting
symptom for reactive processes such as poststreptococcal arthritis, rheumatic fever, serum
sickness, and postinfectious arthritis (these illnesses tend to be self-limiting or have other
symptoms); infections that can present with symptoms of arthritis include septic arthritis, discitis,
or osteomyelitis; mononucleosis, parvovirus, and Lyme disease. Other autoimmune conditions
with findings similar to JIA include lupus, mixed connective tissue disease, Sjögren syndrome,
sarcoidosis, and inflammatory bowel disease. Joint enlargement caused by conditions other than
arthritis can include trauma; benign tumors such as osteoid osteoma or osteoblastoma;
malignancies such as leukemia, neuroblastoma, osteosarcoma, Ewing sarcoma, and
rhabdomyosarcoma. Because there is no diagnostic laboratory study for JIA, it is often a
diagnosis of exclusion. Laboratory findings that support the diagnosis of JIA, are anemia,
leukocytosis, elevated acute phase reactants, and hypergammaglobulinemia.
Juvenile idiopathic arthritis consists of 6 types of arthritis with different presenting features (Item
C153A).
Uveitis, also known as iridocyclitis, is a serious complication of JIA resulting from chronic
nongranulomatous inflammation of the anterior eye chamber affecting the iris and ciliary body.
The uveitis associated with JIA is usually clinically silent with an insidious onset. Risk factors
for uveitis in patients with JIA include JIA subtype, age at onset of disease, and antinuclear
antibody (ANA) status. The highest risk group is female patients with pauciarticular JIA who are
ANA positive and diagnosed before the age of 4 years. Screening guidelines have been
developed for uveitis based on risk (Item C153B). The severity of uveitis does not correlate well
with arthritis activity; therefore the status of joint disease should not affect the frequency of
screening. While ANA is often positive in patients with JIA who have uveitis, it is important to
remember that ANA is positive about 57% of the time in all patients with JIA.
Cataracts are among the late complications of JIA, resulting from chronic uveitis or chronic
ophthalmic steroid use and are not associated with specific laboratory findings. Patients with JIA
can develop bone erosions, and these patients are more likely to be rheumatoid factor (RF)
positive. Osteoporosis may result from chronic steroid use, methotrexate use, or the JIA itself.
Vitamin D and calcium should be supplemented in patients with JIA to help prevent
osteoporosis. Leg length discrepancy, resulting from increased inflammation in affected joints
that increases blood flow and growth factors, occurs in patients with JIA with prolonged poor
disease control. Some of these patients may have increased acute phase reactants, such as
erythrocyte sedimentation rate, C-reactive protein, or thrombocytosis.
Current treatment recommendations for JIA with low disease activity include an initial
nonsteroidal anti-inflammatory drug (NSAID) trial (Item C153C). The NSAID trial should not
American academy of pediatrics 498
American Academy of Pediatrics PREP 2016
last longer than 2 months in a patient with active arthritis. In patients with 4 or fewer joints
involved, intra-articular steroid injection may be used alone to control the arthritis. If the arthritis
is not responsive to NSAID treatment, or if more than 4 joints are involved, then methotrexate
with or without systemic or intra-articular glucocorticoids is recommended. If the patient fails
methotrexate therapy, then a tumor necrosis factor-α (TNF-α) inhibitor, usually adalimumab or
etanercept, is added to therapy. If the patient fails these regimens, then abatacept (a T-cell
modulator) is recommended. In patients with 5 or more active joints, NSAIDs should not be used
as monotherapy. In patients with systemic JIA (fevers, rash, and arthritis) the recommendations
are different. The first-line drugs are NSAIDs, but systemic corticosteroids are used if there is
inadequate response. If there is still failure to control the arthritis then methotrexate is used in
patients without fever and rash. If fever and rash are present then abatacept, anakinra (an IL-1
blocker), or toculizumab (an IL-6 blocker) is recommended.
PREP Pearls
• Juvenile idiopathic arthritis is a clinical diagnosis. Emergent conditions such as leukemia
and septic arthritis must be excluded.
• Juvenile idiopathic arthritis can be managed initially with nonsteroidal anti-inflammatory
drugs.
• A serious and preventable complication of juvenile idiopathic arthritis is damage caused
by uveitis.
Suggested Readings
• Bernard R. Approach to the child with joint inflammation. Pediatr Clin North Am.
2012;59:245-262. DOI:http://dx.doi.org/10.1016/j.pcl.2012.03.003.
Question 154
You are volunteering as an event physician for a regional high school softball tournament. A 14-
year-old adolescent presents to the medical tent for evaluation of her left eye after being hit by a
pitch during a game. On physical examination, you note blood in the anterior chamber of the left
eye. The patient has normal pupillary reflexes and extraocular eye movements. She has no
periorbital tenderness and normal sensation around the eye. Examination of the cornea with a
cobalt blue light following fluorescein staining does not reveal any defects. You discuss this
injury with the adolescent and her parents.
Of the following, the MOST appropriate statement to include in the discussion is that this type of
injury
A. carries a risk of secondary hemorrhage that is highest 7 to 14 days after the injury
Hyphema usually results from direct trauma to the eye. Affected individuals typically report pain
and blurry vision. Physical examination reveals a collection of blood in the anterior chamber.
Treatment includes rest and cycloplegic medication. Nonsteroidal anti-inflammatory drugs
should be avoided because they may increase the risk of bleeding. Secondary hemorrhage occurs
in up to one-third of patients with hyphema, with the risk being highest 2 to 7 days after injury.
Evidence suggesting that rest prevents rebleeding is limited, but most ophthalmologists
recommend restricting physical activity until the hyphema resolves and the risk of rebleeding has
passed. Hyphema is more common in children than adults, with the highest incidence seen
between 10 and 20 years of age. Surgery may be indicated for large hyphemas that could
potentially cause optic nerve damage, but vision loss after hyphema is rare. There are no
published return-to-play guidelines following eye injuries; an ophthalmologist should provide
clearance before the child returns to sports.
Severe pain, lack of normal extraocular motion, disruption of the sclera or cornea, and decreased
visual acuity are signs and symptoms of globe rupture. Globe rupture is an emergency; these
patients should have an eye shield placed and be referred to the emergency department for
ophthalmologic evaluation.
Sports and recreational activities account for about one-quarter of the eye injuries seen in the
emergency department. Basketball, baseball, softball, and football are the sports with the highest
risk of eye injury. Common sports-related eye injuries include corneal abrasions and corneal
foreign body. Approximately 80% of eye injuries occur in individuals not wearing eye
protection; appropriate sports eyewear can reduce the risk of eye injury. The American Academy
of Pediatrics (AAP) and the American Academy of Ophthalmology (AAO) issued a joint
statement in which they “strongly recommend” appropriate protective eyewear for athletes in
moderate or high-risk sports. Safety glasses should have shatter-resistant polycarbonate lenses.
PREP Pearls
• The presence of a hyphema merits urgent referral to ophthalmology.
• Athletes who participate in sports with moderate or high risk of eye injury should wear
appropriate protective eyewear to reduce their risk of injury.
Suggested Readings
• American Academy of Pediatrics Committee on Sports Medicine and Fitness. Protective
eyewear for young athletes.Pediatrics. 2004;113(3 Pt 1):619-622.
http://pediatrics.aappublications.org/content/113/3/619.long.
Question 155
A mother brings her 4-month-old and 7-year-old daughters in for evaluation of rash. The 7-year-
old has had recurrent itching of the scalp and physical examination findings shown in Item Q155.
The 7-year-old has been previously treated for this finding with permethrin.
B. ivermectin lotion
C. lindane shampoo
D. malathion lotion
E. permethrin lotion
The female louse lives approximately 1 month and lays up to 10 eggs (nits) each day at the base
of a host’s hair shaft. After approximately 8 days, the egg capsules hatch nymphs that mature
over the next 8 days into adult lice. The adults have mouth parts used to suck blood and grasp
host hairs. They move about by crawling and are transmitted by close person-to-person contact.
Pubic lice usually are spread through sexual contact.
Clinical manifestations of head, body (pediculosis corporis), and pubic (pediculosis pubis) lice
include intense itching and small, erythematous maculopapular lesions with excoriations at the
site of bites. Body lice can survive away from a blood source for 5 to 7 days. Pubic lice typically
survive for up to 36 hours away from a host, but may live for 10 days under ideal conditions.
For the 4-month-old infant in the vignette, the best option for treating head lice is over-the-
counter permethrin because none of the other topical agents are recommended for young infants
(Item C155). Lindane shampoo no longer is recommended for treating children because of
neurologic adverse effects and widespread resistance. Pediculicides used to treat pediculosis
capitis and corporis can also be used to treat pediculosis pubis. After each treatment, the hair and
body should be checked for nits and lice with a nit comb. Bedding and clothing should be
washed in hot water, and close contacts should be monitored for lice and treated if infested.
PREP Pearls
• Topical pediculicides are the treatment of choice for pediculosis.
• Most topical pediculicides kill only live lice, so reapplication in 7 to 10 days is often
necessary to treat infestation.
• The only pediculicide approved for use in children younger than 6 months is 1%
permethrin lotion.
• The most common adverse effect of topical pediculicides is skin irritation.
Suggested Readings
• American Academy of Pediatrics. Pediculosis capitis. In: Kimberlin DW, Brady MT,
Jackson, MA, Long SS, eds. Red Book:2015 Report of the Committee on Infectious
Diseases. 30th ed. Elk Grove Village, IL: American Academy of Pediatrics; 2015:597-
601.
Question 156
A tearful mother brings her 2-year-old son to your office 30 minutes after he pulled on the
handle of a mug full of hot coffee left on the edge of the kitchen counter. The coffee spilled onto
the boy and burned his right upper arm and chest. He is crying loudly, without stridor or
respiratory difficulty. Physical examination reveals a 3 × 3–cm “ruptured” blister on the child’s
upper chest with an erythematous, tender base. There is also a 3 × 4–cm area of superficial
erythema without blister formation on the child’s right upper arm. The child’s mother states that
he has been very healthy and takes no medications. His immunizations are up to date. After
administration of an oral analgesic, you clean the boy’s burns with soap and water.
Burns are a relatively common type of injury in children and adolescents. Pediatric providers
should be familiar with the initial assessment and management of burns. Burns may arise from
contact with hot objects or liquids, sun exposure, radioactivity, electricity, chemical exposure, or
friction. Thermal injuries that occur commonly in children include scald and contact burns
sustained from cooking or spilling hot foods or liquids. Scald burns related to bathing are a
particular risk in infancy and the toddler years. Contact burns from hot objects such as space
heaters, grills, stoves, ovens, irons, hair appliances, campfires, and fireworks are also seen fairly
frequently. Fortunately, only a few burns in children require hospitalization; most can be
managed on an outpatient basis.
Burn injuries are classified into 3 main types, based on the depth of tissue injury:
• First-degree (ie, superficial) burns: These burns involve injury to the epidermis only,
without involvement of the dermis. Clinical features include erythema and mild
inflammation of the epidermis, without blister formation. Uncomplicated sunburn is a
commonly occurring example. These burns may be painful, but generally resolve within a
few days without scarring. Although the injured epidermis often peels within a few days
of a superficial burn injury, new epidermal cells will be regenerated. Superficial burn
injuries are not generally included in calculations of total body surface area (TBSA)
involvement in patients presenting with burns.
• Second-degree (ie, partial thickness) burns: These burns involve injury to both the
epidermal and dermal layers of skin. Blister formation typically occurs, as well as tissue
edema arising from increased capillary permeability. These burns are generally painful
because of exposure of intact sensory nerve receptors in the injured dermis. Superficial
partial thickness burns typically heal within 2 weeks without scarring. Deeper partial
thickness burns may involve damage to most of the dermis and may have a paler, drier
appearance than more superficial partial thickness burns. Healing may take several
weeks, and scarring may occur. Deeper partial thickness burns may be difficult to
differentiate from full thickness burns.
• Third-degree (ie, full thickness) burns: These burns involve destruction of the epidermis
and entire dermal layer of skin. Full thickness burns may appear pale and "waxy" or
charred, and often have a leathery appearance. Skin affected by full thickness burns is
nontender because of destruction of the cutaneous nerves, though surrounding partial
thickness burns may be very painful. Full thickness burns cannot re-epithelialize because
of destruction of the entire dermal layer, and skin grafting is often required.
Children presenting for care of superficial burns, or superficial partial thickness burns involving
a small percentage (<5%) of TBSA, can generally be treated on an outpatient basis. First-degree
(superficial) burns generally require only supportive therapy with emollients and oral analgesics
such as acetaminophen or ibuprofen for discomfort.
Minor partial thickness burns can be cleansed with mild soap and water, diluted povidone-iodine
solution, chlorhexidine gluconate, or saline. Devitalized tissue should be debrided, often by
wiping gently with moist gauze. The exposed underlying skin will generally be erythematous,
moist, and painful. The burn wound should be dressed promptly to help reduce pain associated
with convection of air across the wound. Minor partial thickness burns should be dressed with a
topical antimicrobial agent such as bacitracin ointment or silver sulfadiazine and covered with a
sterile gauze dressing. Caregivers should be advised to gently cleanse minor burn wounds daily
with a clean cloth or gauze in the shower or bathtub, and then to redress the wounds. Minor
partial thickness burn injuries generally heal within 1 to 2 weeks. Providing adequate analgesia is
an essential component in the outpatient care of minor burns. Pediatricians should administer
appropriate analgesia before performing the initial burn wound assessment and care, and
anxiolytic agents may even be required in some children.
Regarding the management of blisters associated with minor partial thickness burns, those that
are intact provide protection for the underlying tissue and should be left intact, as long as they do
not cross joints or otherwise limit activity. Topical antimicrobial ointment does not need to be
applied to intact blisters. Once blisters rupture, they should be debrided, because devitalized
tissue could serve as a nidus for infection.
A course of oral cephalexin would not be the best next step in management for the boy in the
vignette, who displays no signs of systemic infection. There is no role for the empiric
administration of systemic antibiotics after burn injuries. Systemic antibiotics should only be
administered to children with clear evidence of infection on physical examination or on culture
of the burn wound.
Referral to a burn center is not indicated for the boy in the vignette. The burned area on his right
arm is a superficial burn that requires supportive care only, and the burn on his upper chest is a
superficial partial thickness burn that is small in size (<5% TBSA). His burns can certainly be
managed on an outpatient basis. Children with partial thickness burns involving more than 5% to
10% TBSA should be considered for admission to a hospital. Indications for referral of children
presenting with burn injuries to a regional burn center include full thickness burns; partial
thickness burns exceeding 10% TBSA; electrical or chemical burns; burns associated with
inhalational injury; associated traumatic injuries or comorbid conditions; burns involving the
American academy of pediatrics 510
American Academy of Pediatrics PREP 2016
face, hands, feet, genitalia, or joints; burns suspected to arise from intentional injury; and burns
exceeding local specialist or institutional capacity.
Reporting of the injuries sustained by the boy in the vignette to child protective services is not
warranted, given that the reported mechanism of injury is consistent with his developmental
stage, and that his pattern of burns is not suspicious for inflicted injury. Pediatric burn injuries
associated with a delay in seeking care or isolated scald or contact burns to the hands, feet,
genitalia, or buttocks without a plausible mechanism should raise suspicion for inflicted injury.
Burns to the hands and feet with a “stocking and glove” pattern (clearly demarcated borders
without surrounding splash burns) can arise from intentional immersion of the hands or feet in
scalding liquid, and should raise suspicion for child abuse. Scald burns to the buttocks and thighs
in toddlers can result from forced submersion in a tub of hot water, often following a toilet-
training mishap. Suspicious patterns of contact burns with hot objects (such as burning
cigarettes, hot irons, cooking pans, hair appliances, or heaters) that do not seem to correlate with
the reported history should prompt a thorough evaluation for abuse. If concern for an inflicted
burn injury exists, reporting to child protective services and referral of the injured child to a
pediatric burn center are indicated.
PREP Pearls
• Children presenting for care of superficial burns or superficial partial thickness burns
involving a small percentage (<5%) of total body surface area can generally be managed
on an outpatient basis.
• First-degree (superficial) burns generally require only supportive therapy with emollients
and oral analgesics, such as acetaminophen or ibuprofen, for discomfort.
• Minor partial thickness burns should be cleansed and devitalized tissue should be
debrided. A topical antimicrobial agent such as bacitracin ointment should be applied,
and the wound should be covered with a gauze dressing. Intact blisters should be left
intact.
• Pediatricians should administer appropriate analgesia before performing the initial burn
wound assessment and care, and anxiolytic agents may be required in some children.
Suggested Readings
• Jamshidi R, Sato TT. Initial assessment and management of thermal burn injuries in
children. Pediatr Rev. 2013;34:395. DOI: http://dx.doi.org/10.1542/pir.34-9-395.
Question 157
During rounds in the newborn nursery, a mother tells you she is worried because her daughter
has had no wet diapers since birth, 18 hours ago. The neonate was born at 41 weeks’ gestation by
normal vaginal delivery. The pregnancy was unremarkable, including a normal anatomy scan at
18 weeks of gestation. The mother presented in labor, with artificial rupture of the membranes 3
hours before delivery, revealing clear amniotic fluid. The newborn emerged vigorous and has
been exclusively breastfeeding since delivery. Physical examination reveals a pink, well-
perfused newborn with a normal cardiac and pulmonary examination, an intact spine, and a soft,
nondistended abdomen without palpable masses.
Of the following, the MOST appropriate next step in management of this newborn is to
A well-appearing newborn may be monitored up to 24 hours after birth for urine output. If anuria
persists beyond that point, the history and physical examination should be carefully reviewed for
evidence of a pathologic cause (Item C157). Urinary tract abnormalities often are detectable on
prenatal ultrasonography. Because amniotic fluid reflects urine production in the late second and
third trimester, oligohydramnios is an additional clue to a urinary tract abnormality. On physical
examination, a distended bladder should prompt concerns of bladder outflow obstruction
whereas a lower back abnormality such as a dimple or hair tuft may suggest an underlying spinal
anomaly affecting micturition.
PREP Pearls
• A well-appearing newborn may be monitored for urine output up to 24 hours after birth.
Ninety-two percent to 97% of newborns will urinate within 24 hours of birth and nearly
all within 48 hours.
• Prenatal ultrasound screening, amniotic fluid volume, and the physical examination
findings should be reviewed when evaluating an anuric newborn.
• The initial evaluation of a newborn with no urine output in the first 24 hours after birth
should include bladder catheterization, bladder/renal ultrasonography, and serum
electrolyte, blood urea nitrogen, and creatinine levels.
Suggested Readings
• Abdelhamid A. Index of suspicion in the nursery: unusual cause of feeding intolerance in
a term infant. NeoReviews. 2012;13:e732-e735. DOI: http://dx.doi.org/10.1542/neo.13-
12-e732.
Question 158
A 6-month-old infant is brought to your office by her parents with the complaint of decreased
use of her left arm. She was born at full term to a 38-year-old woman who took fluoxetine during
pregnancy and did not take prenatal vitamins. There were no complications during the
pregnancy, labor, or delivery. The infant had torticollis noted at 2 weeks of age that resolved
with physical therapy. Her parents report that over the past month, the infant’s left arm seems to
hang by her side, and she is holding her left hand in a fist with increasing frequency. Her
physical examination demonstrates a head circumference of 42 cm (10th percentile), length of 67
cm (50th percentile), weight of 8.2 kg (60th percentile), blood pressure of 82/68 mm Hg, and a
heart rate of 102 beats/minute. Her anterior fontanel is open and flat. She has no dysmorphic
features. There is no torticollis. Her neurological examination demonstrates symmetric facial
movements, increased tone in the left upper and lower extremities, and decreased spontaneous
movement of the left arm. Deep tendon reflexes are brisker on the left side compared to the right.
You order magnetic resonance imaging of the brain (Item Q158).
B. female sex
E. neonatal torticollis
Cerebral palsy is defined as motor impairment caused by a nonprogressive brain injury that
occurred during fetal or infantile brain development and the associated disability and
impairments that may develop over time. Even though the injury may have occurred prenatally
or in the perinatal period, the first symptoms may not appear until 4 to 6 months of age, as
described in the girl in the vignette. CP is subdivided into categories based on the limbs that are
involved: spastic, hemiparetic CP affects either the right or left side of the body, spastic diplegic
or diparetic CP affects both legs, and spastic quadriplegic or quadriparetic CP affects all four
limbs. Less commonly seen CP subtypes are athetotic and dystonic. These subtypes are now
thought to have different underlying causes along with a different natural history and prognosis.
Traditionally, the formal diagnosis of CP has been delayed until age 3 or 4 years, when
metabolic or genetic causes of motor disturbance can be clinically ruled out. However, as seen in
the girl in the vignette, when a structural abnormality is seen on brain imaging, the diagnosis of
CP can be made earlier.
On neuroimaging, focal arterial infarction and brain malformation are the 2 most commonly
identified causes of CP in full-term infants. Other causes include periventricular leukomalacia
and generalized atrophy. In up to one third of cases, neuroimaging will not show a cause.
Risk factors for CP in full-term infants include advanced maternal age, black race, and
intrauterine growth retardation. Infant sex and multiple gestation are not risk factors for CP in
full-term infants. Fetal selective serotonin reuptake inhibitor exposure can cause excessive
jitteriness in the perinatal period, but not CP. Prenatal folate supplementation can prevent neural
tube defects such as spina bifida, but does not prevent CP. Neither neonatal torticollis nor its
treatment causes CP.
PREP Pearls
• Risk factors for cerebral palsy in full-term infants include advanced maternal age, black
race, and intrauterine growth retardation.
• Symptoms of spastic cerebral palsy may not appear until 4 to 6 months of age.
Suggested Readings
• Novak I, Hines M, Goldsmith S, Barclay R. Clinical prognostic messages from a
systematic review on cerebral palsy.Pediatrics. 2012;130:e1285. DOI:
http://dx.doi.org/10.1542/peds.2012-0924.
Question 159
You receive a call from your state newborn screening laboratory regarding an abnormal thyroid
screening test on a now 6-day-old newborn. In your office, the newborn is asymptomatic and has
an unremarkable physical examination. Maternal history is significant for long-term therapy with
levothyroxine and previous thyroid surgery. Review of the newborn screening test results show a
low thyroxine level and a thyroid stimulating hormone over 100 mIU/L. Repeat laboratory
testing confirms these findings. You order imaging studies to further evaluate the newborn’s
thyroid. Technetium 99m pertechnetate uptake scan shows absence of any uptake over the
thyroid, suggesting athyrosis (Item Q159A). Thyroid ultrasonography reveals a normally sized
and positioned thyroid gland (Item Q159B).
A. DiGeorge syndrome
B. levothyroxine use during pregnancy
C. maternal history of Graves disease
D. maternal history of Hashimoto disease
E. neonatal lupus with antiphospholipid antibodies
Imaging studies used in the evaluation of congenital hypothyroidism can aid in identifying a
cause, which can have important prognostic implications for the child. Imaging study results may
include:
• An ectopic thyroid gland (the most common anatomic cause of hypothyroidism) which
establishes a permanent form of thyroid disease.
• The absence of thyroid gland uptake, which is most often associated with thyroid aplasia
or hypoplasia, another permanent cause. However, when radioiodine uptake is absent but
ultrasonographic examination reveals a normal gland, as shown in the infant in this
vignette, a TSH-receptor defect, iodine-transport defect, or maternal transfer of thyroid
antibodies may be present.
• A normal scan may indicate a transient form of hypothyroidism. These infants should
undergo a careful follow-up evaluation after 3 years of age, when it is safe to discontinue
levothyroxine treatment with certain precautions.
The infant in this vignette has a normal thyroid ultrasound but negative uptake on 99mTc uptake
imaging. A common reason for this discrepancy in imaging findings is maternal transfer of
thyroid-blocking antibodies, which block the TSH receptor. Mothers with Graves’ disease (either
active disease or treated with surgery or radioactive iodine) can have thyroid antibodies present.
Antibodies to the TSH receptor (stimulating or blocking) freely cross the placenta and can act in
the fetal thyroid gland during the second half of pregnancy and after delivery, causing either
neonatal Graves’ disease or congenital hypothyroidism. This child had very high thyroid-binding
inhibitory immunoglobulins that blocked the functionality of the TSH receptor, resulting in CH.
Thus, even though the gland is present, it is functionally turned off while the blocking antibodies
are present. This situation can go on for months, and treatment with levothyroxine is required, as
in the infant in this vignette, until the antibodies have disappeared.
Ascertaining the cause of a mother’s hypothyroidism is important. Mothers who have a history
of autoimmune hypothyroidism (Hashimoto thyroiditis), rarely if ever, have antibodies that cross
the placenta. Therefore, autoimmune hypothyroidism is unlikely to cause an issue with the infant
as long as the mother is taking her thyroid medication and is euthyroid. On the other hand, as
mentioned before, a mother with active or a history of Graves’ disease may pass antibodies
through the placenta to the infant (stimulating or blocking), which could cause either neonatal
Graves’ disease or CH. Thus infants of mothers with Graves’ disease, especially those who have
high antibody titers in the third trimester, deserve closer follow-up. This can include assessment
of the infant’s thyroid function and antibody titers.
Levothyroxine use during pregnancy is safe and will not affect the infant. In fact, maternal
thyroxine (or levothyroxine medication) crosses the placenta in small amounts during the first
half of pregnancy, when fetal thyroid hormone concentrations are low. This is necessary to keep
the fetus euthyroid.
Of the other response choices listed, antiphospholipid antibodies associated with lupus can cause
neonatal heart block, but would not affect thyroid function. DiGeorge syndrome, commonly
associated with hypoparathyroidism, can also be associated with hypothyroidism. However
DiGeorge syndrome is not associated with maternal transfer of thyroid antibodies.
Other possible reasons for hypothyroidism include transient causes, such as iodine overload or
maternal anti-thyroid medications.
It is very important for the general pediatrician to recognize that if there is any suspicion of CH,
treatment should be initiated with levothyroxine as early as possible. Treatment can be weaned
later if a transient cause of hypothyroidism is suspected. For infants with CH , the best
developmental outcomes occur with levothyroxine therapy started by 2 weeks of age at 10 μg/kg
or more per day, compared with lower doses or later start of therapy. There are only minor
differences in intelligence, school achievement, and neuropsychological test results in adults
with CH who were treated early with levothyroxine compared with control groups of classmates
and siblings. However, even with early treatment visuospatial processing, selective memory, and
sensorimotor deficits can occur. In contrast, the prognosis for normal mental and neurologic
performance is less certain for infants with CH who are not treated as early as possible, ideally
by age 2 weeks. If treatment is delayed even a few months, 77% of infants show some signs of
developmental delay and may have impairment of arithmetic ability, speech, or fine motor
coordination in later life.
PREP Pearls
• Imaging studies performed during the evaluation of congenital hypothyroidism can help
to determine whether there is a permanent versus transient cause of disease.
• A mother who is hypothyroid after surgery or radioactive iodine ablation for Graves’
disease and now taking levothyroxine, or who has active Graves’ disease during
pregnancy, may pass antibodies to the infant (stimulating or blocking) that could cause
either neonatal Graves’ disease or congenital hypothyroidism.
• Treatment for congenital hypothyroidism should never be delayed to obtain additional
imaging studies or further diagnostic workup. Treatment with levothyroxine should
always begin immediately to afford the best developmental outcome.
Suggested Readings
• American Academy of Pediatrics, Rose SR, Section on Endocrinology and Committee on
Genetics; American Thyroid Association, Brown RS; Public Health Committee, Lawson
Wilkins Pediatric Endocrine Society, Foley T, Kaplowitz PB, Kaye CI, Sundararajan S,
Varma SK. Update of newborn screening and therapy for congenital hypothyroidism.
Pediatrics. 2006;117:2290-2303. DOI: http://dx.doi.org/10.1542/peds.2006-0915.
Question 160
A 14-year-old adolescent is brought to the emergency department via emergency medical
services with complaints of shortness of breath, and moderately severe sharp chest and shoulder
pain after he was hit by a teammate during football practice.
The patient’s medical history is significant for mild persistent asthma and seasonal allergic
rhinitis, which are well controlled on leukotriene receptor antagonist monotherapy. The patient
did have symptoms of an upper respiratory infection and bronchitis during the preceding 3 days
and was completing treatment with azithromycin. There is no antecedent history of vomiting or
respiratory distress.
Of the following, the MOST likely explanation for this patient’s clinical presentation is
A. commotio cordis
B. pneumomediastinum
C. pneumothorax
D. splenic rupture
E. status asthmaticus
Signs and symptoms of pneumomediastinum include chest and neck pain, dyspnea, and sore
throat. Dysphonia may be seen. Subcutaneous emphysema and tactile subcutaneous crepitus is
frequently encountered. Subcutaneous air collections are often felt at the neck, but may also
involve the face and scalp. Hamman crunch describes precordial or substernal crepitance that is
typically synchronous with the patient’s heart beat. Hamman crunch is nearly pathognomonic for
subcutaneous emphysema.
Commotio cordis is a rare, but frequently fatal arrhythmia that occurs after the precordial chest is
struck or traumatized. This may occur in a motor vehicle crash or in a sport such as baseball or
football. Similar blunt trauma may be implicated in patients with pneumomediastinum without
commotio cordis.
While a pneumothorax is expected to present similarly with acute onset of chest pain and
dyspnea, symmetric lung aeration would not be expected nor is pneumothorax associated with
subcutaneous crepitus.
Spontaneous pneumothorax is more common in thin, tall male adolescents. A history of recurrent
pneumothorax warrants an evaluation for predisposing factors such as collagen vascular disease.
There is no cough, wheezing, prolongation of the expiratory phase of respiration, or silent chest
to suggest status asthmaticus in this patient. However, this patient’s mild asthma and mild
associated air trapping may contribute to the pathogenesis of pneumomediastinum.
Finally, splenic rupture may be the result of traumatic impact and may be associated with sharp
shoulder pain (Kehr sign). Other associated symptoms, however, include pain in the abdomen,
epigastrium, or left flank. In severe injuries to the spleen, the patient may present in hypovolemic
shock caused by massive blood loss.
PREP Pearls
• Subcutaneous emphysema in pneumomediastinum represents the tracking of free air from
ruptured alveoli along perivascular sheaths into the hilum and mediastinum.
• The most common causes of pneumomediastinum are infection and asthma.
Suggested Readings
• Bullaro F, Bartoletti S. Spontaneous pneumomediastinum in children: a literature review.
Pediatr Emerg Care. 2007;23(1):28-30 .
Question 161
A 6-month-old infant has a large ventricular septal defect complicated by congestive heart
failure. His corrective surgery has been delayed because of 2 hospitalizations for bronchiolitis
during which he lost weight. He is currently feeding 24 kcal/oz formula, but has not shown any
weight gain, and his weight is now below the third percentile for his age. The baby has a good
suck, but he takes no more than 60 to 75 mL every 4 hours.
Of the following, the BEST next step to increase this baby's energy intake is to
Standard infant formulas provide adequate nutrients to support growth of healthy term infants.
However, infants and children with complex medical needs may not receive appropriate nutrition
using standard formulas and may require feedings that differ by protein source, carbohydrate, fat
ratios, caloric density, and mineral and micronutrient content. For example, standard soy formula
does not provide enough calcium and phosphorus to prevent osteopenia in preterm infants, and
the increased aluminum content in soy formula may exacerbate this problem. Hydrolyzed and
amino acid-based formulas have been developed for children with cow’s milk allergy or at high
risk for atopic disease, and extensively hydrolyzed formulas have been used for short gut
syndrome, hepatobiliary and pancreatic disease, and autoimmune and immunodeficiency
diseases. Neither of these formula types would be required for the infant in the vignette whose
issues relate to inadequate intake rather than inadequate absorption or immunologic conditions.
The primary problems with these formulas include high cost and poor acceptance by infants.
Infant formulas have also been modified to manage gastroesophageal reflux by thickening, but
data to support this approach are limited.Children beyond infancy may require formula either as
the sole source of nutrition or as supplementary oral intake. Standard pediatric formulas provide
30 kcal/oz (1 kcal/mL) and provide sufficient vitamin and mineral content to prevent vitamin D,
calcium, phosphorus, and iron deficiency. Adult formulas have a higher calorie-to-nutrient ratio
than pediatric formulas. Adult formulas should not be used for low energy children (eg,
nonambulatory) because they may gain excessive weight despite being nutrient deficient.
PREP Pearls
• Children with chronic medical conditions such as congenital heart disease often require
increased caloric intake in order to meet energy requirements for normal growth and
development.
Suggested Readings
• Kleinman RE, Greer FR. Cardiac disease. Pediatric Nutrition. 7th ed. Elk Grove Village,
IL: American Academy of Pediatrics; 2014:1077-1094.
Question 162
A 16-year-old adolescent presents to your office because she is concerned that, unlike her
friends, she hasn’t started menstruating. Her mother reports that she and the patient’s sister had
menarche around 13 years of age. The patient’s past medical history is remarkable for surgery
for strabismus. She denies sexual activity. On physical examination, her height is less than the
fifth percentile. Her weight is at the 80th percentile. Her blood pressure and heart rate are
normal. She has a soft systolic murmur. The rest of her examination is unremarkable. She has a
sexual maturity rating of 2 for pubic hair and breast. Her urine pregnancy test is negative.
Of the following, the MOST appropriate next step in her evaluation is
B. a karyotype analysis
D. to provide reassurance
Turner syndrome (45 X gonadal dysgenesis) often presents during adolescence as primary
amenorrhea. Other features of Turner syndrome include short stature, aortic coarctation, widely
spaced nipples, webbed neck, cubitus valgus, strabismus, and congenital lymphedema. The
Turner Syndrome Consensus Study Group has published guidelines for the management of girls
and women with Turner syndrome. Management strategies address growth-promoting therapies
and the induction of puberty. The physical findings in the adolescent in this vignette suggest a
diagnosis of Turner syndrome and a karyotype analysis is the most appropriate next step in her
evaluation.
While psychological stress can be a reason for amenorrhea, in this scenario, there are other
findings that should alert the clinician to the possibility of other reasons for the patient’s
amenorrhea. Prolactinomas in young female adolescents can present as delayed menarche.
However, the other findings in this scenario make a prolactinoma less likely as the etiology of
this patient’s amenorrhea. Similarly, nonclassical adrenal hyperplasia (NCAH) can present with
menstrual irregularities like amenorrhea, but individuals with NCAH usually also have evidence
of hyperandrogenism with hirsutism and acne, as well as premature pubarche and tall stature
during adolescence.
PREP Pearls
• Primary amenorrhea is defined as the absence of menses by 15 years of age or within 5
years of breast development if that occurs before 10 years of age.
• Primary amenorrhea is most often caused by an anatomic or genetic abnormality.
• Gonadal dysgenesis resulting from chromosomal abnormalities accounts for
approximately 50% of cases of primary amenorrhea.
• Karyotype analysis should be done in the case of primary amenorrhea.
Suggested Readings
• Bondy CA. Care of girls and women with Turner syndrome: a guideline of the Turner
Syndrome Study Group. J Clin Endocrinol Metab. 2007;92(1):10-25.
American academy of pediatrics 529
American Academy of Pediatrics PREP 2016
Question 163
A 5-year-old boy with trisomy 21 and acute lymphoblastic leukemia, diagnosed 1 year ago, is
admitted to the hospital with increased work of breathing and dry cough that developed over the
last several days. The family reports travel to several cities in the United States over the last
month, but there were no known ill contacts or unusual exposures. On physical examination, the
boy’s temperature is 38°C, heart rate is 120 beats/min, respiratory rate is 60 breaths/min, blood
pressure is 95/65 mm Hg, and oxygen saturation is 85% in room air. He has subcostal and
intercostal retractions with clear breath sounds on auscultation of the lungs. With the exception
of alopecia and mild pallor, his physical examination is otherwise unremarkable.
Laboratory findings were as follows:
o Electrolytes, calcium, serum urea nitrogen, creatinine, serum bicarbonate, and
glucose levels were normal
o White blood cell count, 1,600/μL (1.6 × 109/L) with 30% neutrophils, 40%
lymphocytes, and 30% monocytes
o Hemoglobin, 9 g/dL (90 g/L)
o Platelet count, 130 × 103/μL (130 × 109/L)
o Lactate dehydrogenase, 1,100 U/L
A chest radiograph is obtained (Item Q163).
A. adenovirus
B. Cryptococcus neoformans
C. influenza virus
D. Pneumocystis jirovecii
E. Streptococcus pneumoniae
Pneumonia caused by adenovirus or influenza may also present with nonproductive cough,
tachypnea, and fever, but breath sounds are usually clear and there is a lesser degree of hypoxia
than is typical in Pneumocystis jirovecii pneumonia (PCP). Pneumonia caused by Cryptococcus
is characterized by cough, chest pain, and fever; chest radiography may show a solitary nodule or
focal or diffuse infiltrates. Patients with pneumococcal (Streptococcus pneumoniae) pneumonia
typically have productive cough, focal findings on chest auscultation (eg, bronchial breath
sounds, crackles, rales over affected area), and focal infiltrates on chest radiography.
Pneumocystis jirovecii pneumonia prophylaxis is recommended for all children with severe
immunosuppression (eg, HIV-infected children with low CD4+ T-lymphocyte counts, primary
immunodeficiency, other acquired cell-mediated immunodeficiency such as that following organ
transplantation, and patients receiving immunosuppressive chemotherapy). Oral TMP-SMX
(daily or three times per week) is the prophylactic drug of choice. If TMP-SMX is not tolerated
or cannot be used, alternative agents include oral dapsone (dosed daily or weekly), aerosolized
pentamidine (monthly), or oral atovaquone (daily).
PREP Pearls
• Pneumocystis jirovecii pneumonia (PCP) occurs almost exclusively in
immunocompromised hosts.
• Nonproductive cough, fever, and increased work of breathing accompanied by significant
hypoxia are the most common presenting features of PCP.
Suggested Readings
• American Academy of Pediatrics. Cryptococcus neoformans infections. In: Kimberlin
DW, Brady MT, Jackson, MA, Long SS, eds. Red Book: 2015 Report of the Committee
on Infectious Diseases. 30th ed. Elk Grove Village, IL: American Academy of Pediatrics;
2015:309-311 .
Question 164
A 6-year-old boy has suffered a severe traumatic brain injury and bilateral pulmonary contusions
after being struck by a car while walking. Computed tomography of the head shows extensive
subdural and subarachnoid hemorrhage, diffuse cerebral edema, ischemia, and herniation of the
cerebellar tonsils through the foramen magnum. The neurosurgeon does not consider him an
operative candidate because of the nonsurvivable nature of the injury. He has fixed and dilated
pupils; absent corneal, gag, and oculocephalic reflexes; no eye movement on cold caloric
stimulation; and no response to painful stimuli. He does not breathe over the ventilator. When he
is disconnected from the ventilator, his oxygen saturations decrease to the 70% range within
seconds. The apnea test could not be performed because of his respiratory instability. His parents
accept the inevitability of his death and express a strong desire to donate his organs.
D. recover all organs for donation in the operating room under anesthesia
Throughout most of human existence, death was defined by the cessation of breathing and
circulation. In 1981, the US President’s Commission released the Uniform Determination of
Death Act which defines death as either the irreversible cessation of circulatory or respiratory
functions or irreversible cessation of all functions of the entire brain, including the brainstem. In
short, the 2 paths to death include circulatory criteria and neurologic criteria, with the provision
that it be irreversible. Since the widespread adoption into clinical practice of brain death criteria
in the late 1960s, the majority of donated organs have been recovered from brain-dead donors.
An ethical corollary to the 1981 statement is that death should not be caused by the removal of
organs. Thus, objective and rigorous criteria must be applied and death should be declared before
organ removal for transplantation.
In 1987, and updated in 2011, the American Academy of Pediatrics published guidelines for
determining brain death in children and neonates. Prerequisites to testing include the absence of
neurologic function, with a known, irreversible cause of coma and the absence of potentially
reversible contributors such as abnormal vital signs, medications, or toxins. For example, testing
should not proceed if the patient exhibits even the slightest sign of neurologic function, such as
pupillary reactivity, spontaneous breathing, or response to stimulus. The clinical examination for
brain death consists of 2 full cranial, motor, and sensory neurologic examinations and apnea
testing, separated by age-dependent observation periods. The observation period should be 24
hours for full-term newborns (37 weeks’ gestational age to 30 days of age), and 12 hours for
infants and children (> 30 days to 18 years).
Apnea testing consists of disconnecting the ventilator, allowing the arterial carbon dioxide to rise
above 60 mm Hg, and observing for signs of breathing. If the child becomes unstable during that
period, as in the child in this vignette, testing should be stopped and the child cannot be declared
dead. In such an instance, an ancillary test such as a cerebral blood flow study or an
electroencephalography should be performed.
The local organ procurement organization (OPO) should be informed of any patient with a
severe neurologic injury, especially when brain death is suspected. Although discussions
regarding organ donation may occur at any time when initiated by the family, the family should
not be approached by anyone about organ donation before the declaration of death. Until such a
time, the child is still alive, and the family may perceive a conflict of interest between their child
and potential organ recipients even if the clinicians and OPO representatives have the best of
intentions. However, a clinician must pursue a diagnosis of brain death without delay if
suspected, much in the way he/she would auscultate the chest to diagnose death based on
circulatory criteria. Once brain death is diagnosed, the family should be approached by the OPO
American academy of pediatrics 534
American Academy of Pediatrics PREP 2016
representative regarding consent for organ donation. If consent is declined, support of breathing
and circulation should be stopped and the standard bereavement pathway should be followed.
There is no requirement for consent or agreement for withdrawal of support in this situation
because the diagnosis of death is valid from medical, ethical, and legal standpoints. If brain death
is suspected and a family chooses to withdraw life-sustaining therapies before the declaration of
death, the possibility of organ donation could be lost. However, prolonging futile care for the
patient approaches an ethical gray area. One approach is to inform the family that the usual
practice in such cases of devastating neurologic injury is to proceed along the brain death
pathway.
Informing the family that the boy is not a candidate for organ donation is not appropriate in this
case, because it is still possible he may be declared brain dead with the performance of an
ancillary study. Organ donation after cardiac death involves recovery of organs after death is
declared based on circulatory criteria in cases of failed resuscitation or after withdrawal of
support. In addition to some ethical considerations, organ donation after cardiac death is less than
ideal because organs become ischemic after circulation stops. Recovery of all organs for
donation in the operating room under anesthesia is not appropriate because organ donation
before declaring death is not ethical. Recovery of the corneas and skin only is not the most
appropriate next course of action, because brain death has not been diagnosed.
PREP Pearls
• The local organ procurement organization should be notified of any patient with a severe
neurologic injury, and should initiate discussions with families regarding organ
transplantation after the diagnosis of brain death has been made.
• The diagnosis of brain death should be pursued as soon as it is suspected.
• Death can be declared based on either circulatory (cardiac death) or neurologic criteria
(brain death).
Suggested Readings
• Nakagawa TA, Ashwal S, Mathur M, Mysore M; Society of Critical Care Medicine,
Section on Critical Care and Section on Neurology of the American Academy of
Pediatrics, and the Child Neurology Society. Guidelines for the determination of brain
death in infants and children: an update of the 1987 Task Force Recommendations.
Pediatrics. 2011;128:e720. DOI: http://dx.doi.org/10.1542/peds.2011-1511.
Question 165
An 18-year-old young man, hospitalized for perforated appendicitis, develops a rash. He
underwent laparoscopic appendectomy and has been receiving piperacillin-tazobactam for
peritonitis caused by Bacteroides fragilis. Vital signs show a temperature of 38.3°C, respiratory
rate of 20 breaths/min, blood pressure of 120/75 mm Hg, and a weight of 65 kg. On physical
examination, he has generalized abdominal tenderness and a diffuse morbilliform exanthem. You
suspect a drug reaction and choose to change the antibiotic regimen.
Of the following, the antibiotic expected to provide the BEST coverage for this patient’s
infection is
A. aztreonam
B. cefoxitin
C. clindamycin
D. imipenem
E. levofloxacin
Monobactams, such as aztreonam, have poor activity against anaerobic bacteria and are not
considered adequate as single therapy for polymicrobial intra-abdominal infections. Cefoxitin is
the most effective cephalosporin against B fragilis. However, 5% to 15% of isolates can be
resistant. Similarly, it is estimated that 5% to 10% of B fragilis will be resistant to clindamycin.
In national guidelines that address treatment of intra-abdominal infections in adults and children,
clindamycin is not recommended given increasing resistance in the B fragilis group.
Additionally, clindamycin does not treat gram-negative bacteria.
Older generation fluoroquinolones also have poor activity against anaerobes. While anaerobic
activity is improved in newer generation fluoroquinolones (eg, levofloxacin), national guidelines
recommend metronidazole be added to adequately cover anaerobic bacteria when
fluoroquinolones are used for the treatment of intra-abdominal infections. In the gastrointestinal
tract, anaerobes outnumber aerobic bacteria many times over. Therefore, any infection that
involves dissemination of gastrointestinal flora into a normally sterile site (eg, peritonitis
resulting from perforated appendicitis) must be considered polymicrobial with anaerobic bacteria
playing an important role. Selected antimicrobials must target all potentially involved pathogens.
Piperacillin-tazobactam can be an appropriate first line choice, as it targets gram-negative and
anaerobic bacteria in addition to treating susceptible gram-positive pathogens such as
Enterococcus species.
Anaerobic infections are common in children. Other clinical scenarios that may warrant
antimicrobial therapy directed against anaerobes include brain abscesses, oral and dental
infections, deep neck infections, pelvic infections, and necrotizing soft tissue infections. In
general, anaerobes should be considered potential pathogens in certain infections associated with
abscesses and in the setting of tissue destruction with associated gas formation.
PREP Pearls
• Susceptibility of anaerobic bacteria to carbapenems, metronidazole, and combinations of
a penicillin with a β-lactamase inhibitor has generally been preserved.
• Clindamycin is not recommended for anaerobic coverage of intra-abdominal infections
given increasing resistance in the Bacteroides fragilis group.
• Clinical scenarios that warrant antimicrobial therapy directed against anaerobes include
brain abscesses, oral and dental infections, deep neck infections, intra-abdominal
infections, pelvic infections, and necrotizing soft tissue infections.
American academy of pediatrics 537
American Academy of Pediatrics PREP 2016
Suggested Readings
• Brook I. Anaerobic infections in children. Adv Exp Med Biol. 2011;697:117-152. DOI:
http://dx.doi.org/10.1007/978-1-4419-7185-2_10.
Question 166
A 14-year-old adolescent presents to your office for a routine health supervision visit. He has no
complaints or concerns today and is not taking any medications. Vital signs show a temperature
of 37.8°C, heart rate of 60 beats/min, respiratory rate of 16 breaths/min, and blood pressure (BP)
of 162/90 mm Hg. His weight is 80.1 kg, height is 155 cm, and his body mass index is 33.3. His
physical examination is otherwise unremarkable. His repeat BP after 15 min of relaxation with
an appropriate size cuff is 162/94 mm Hg and 165/89 mm Hg.
Of the following, the MOST appropriate categorization of the patient’s elevated blood pressures
is
A. hypertensive emergency
B. malignant hypertension
C. masked hypertension
D. stage 1 hypertension
E. stage 2 hypertension
In the presence of a difference between systolic and diastolic BP readings categorization, the
higher value determines the BP category.
The recommendations of the 2004 National High Blood Pressure Education Program Working
Group guidelines are similar to the Joint National Committee (JNC 7) recommendations for
categorizing high BP in adults. These guidelines categorize BP readings in adults as:
1. Normal: systolic BP less than 120 mm Hg and diastolic BP less than 80 mm Hg
2. Prehypertension: systolic BP 120 to 139 mm Hg or diastolic BP 80 to 89 mm Hg
3. Stage 1 hypertension: systolic BP 140 to 159 mm Hg or diastolic BP 90 to 99 mm Hg
4. Stage 2 hypertension: systolic BP greater than or equal to 160 mm Hg or diastolic BP
greater than or equal to 100 mm Hg
For the 14-year-old male adolescent in this vignette with a height near the tenth percentile (155
cm), prehypertension is BP greater than 121/76 mm Hg and less than 125/80 mm Hg, stage 1
hypertension is BP greater than or equal to 125/80 mm Hg and less than 137/93 mm Hg, and
stage 2 hypertension is BP greater than or equal to 137/93 mm Hg (Item C166A).
Masked hypertension is defined as BP readings that do not meet the criteria for hypertension
based upon office readings, but are consistently elevated by out-of-office measurements. Masked
hypertension is the opposite of white coat hypertension wherein BP readings are consistently
elevated by office readings, but do not meet criteria for hypertension diagnosis based on out-of-
office readings. Increased use of 24-hour ambulatory BP monitoring in evaluation of elevated BP
readings in out-of-office settings has helped in better characterization of masked and white coat
hypertension. Increased long term risk of sustained hypertension and cardiovascular morbidity
has been reported with both white coat and masked hypertension.
Hypertensive emergency is defined as severely elevated BP readings, along with clinical features
of acute onset end-organ damage (encephalopathy; headache, seizures, mental status changes,
focal neurologic symptoms, visual disturbances, and heart failure; chest pain, palpitations,
shortness of breath). Patients with hypertensive emergency are at increased immediate risk and
require rapid reduction of blood pressure (less than 25% to 30% over the first several hours).
It is important to recheck and confirm systolic and diastolic BP greater than or equal to 95th
percentile on 3 or more separate occasions using the techniques detailed in the fourth task force
report. A detailed history and physical examination is indicated for all patients with elevated BP.
Physical examination should focus on clinical findings suggestive of secondary hypertension
Item C166B.
In all patients with hypertension (BP ≥ 95th percentile), further evaluation of blood urea
nitrogen, creatinine, electrolytes, urinalysis, complete blood cell count, fasting lipid profile,
fasting glucose, renal ultrasonography, and echocardiogram (also in patients with
prehypertension) is recommended. Fasting lipid profile and glucose is also recommended in
overweight and prehypertensive patients, as well as patients with diabetes or chronic kidney
disease and family history of hypertension or cardiovascular disease.
Detailed evaluation for secondary causes of hypertension such as plasma renin activity,
renovascular imaging (renal scan, duplex Doppler renal ultrasonography, arteriography), plasma
and urine steroids or catecholamines are indicated for young children with stage 1 hypertension
or adolescents with stage 2 hypertension.
PREP Pearls
• Children with high blood pressure (BP) readings should be categorized as stage 1 or stage
2 hypertension using the BP percentiles tables (age, gender, and height-based )
• Hypertensive emergency is defined as elevated BP readings, along with clinical features
of acute onset end-organ damage.
• In all pediatric patients with hypertension (BP ≥ 95th percentile), blood urea nitrogen,
creatinine, electrolytes, urinalysis, complete blood cell count, fasting lipid profile, fasting
glucose, renal ultrasonography, and echocardiogram is recommended.
• Detailed evaluation for secondary causes of hypertension is indicated for young children
with stage 1 hypertension or adolescents with stage 2 hypertension.
Suggested Readings
• Brady TM. Hypertension. Pediatr Rev. 2012;33(12):541-552. DOI:
http://dx.doi.org/10.1542/pir.33-12-541.
Question 167
A 4-month-old term male infant is brought to the office for evaluation of vomiting. The vomiting
began in the first weeks after birth and has been getting worse. The vomiting is nonbloody and
follows most feeds of cow’s milk-based formula. The infant’s mother reports intermittent bilious
emesis and feels they are increasingly projectile. His growth chart shows that his weight has
declined from the 50th to 25th percentile over the past 3 months. He is otherwise well, with
normal vital signs and no evidence of infection on examination. There are no associated
respiratory symptoms or color changes. The parents are mixing the formula correctly.
A. abdominal radiograph
B. gastric scintigraphy
C. pH impedence study
D. upper endoscopy
Gastroesophageal reflux occurs in all ages. Gastroesophageal reflux is the physiologic passage of
gastric contents into the esophagus. Gastroesophageal reflux disease (GERD) is the passage of
gastric contents into the esophagus associated with symptoms or problems. Gastroesophageal
reflux in infants is very common, occurring in two-thirds of all infants. The incidence is higher in
Western societies. There is a clear hereditary portion to GER and the associated complications
(ie, erosive esophagitis and Barrett esophagus, a condition where the lining of the esophagus is
damaged by gastric acid). The literature demonstrates an increased risk in certain populations,
including preterm or neurologically impaired infants, children with a history of structural or
dysmotility issues (esophageal atresia, hiatal hernia, and achalasia), and children with chronic
respiratory issues or with a history of lung transplant.
The evaluation of GER is complex because of the lack of a gold standard diagnostic test. Instead,
the current testing begins with a complete history and physical examination to document acid in
the esophagus, the correlation of acid to symptoms, and evaluation for complications of GER,
such as esophagitis. The tests most commonly used include an initial upper gastrointestinal series
to evaluate the anatomy when there are concerns for an esophageal web, hiatal hernia,
malrotation, or other structural anomaly. Pyloric ultrasonography should be used to evaluate
forceful vomiting to rule out pyloric stenosis in young infants. Esophageal pH monitoring can be
used to quantify the frequency and duration of GER and provides information on the temporal
association between GER and symptoms. Gastric scintigraphy evaluates for postprandial reflux
and aspiration, although this test is limited by a lack of consensus on standard technique or
normalized values. An abdominal radiograph will not be helpful in the diagnosis of GER.
Finally, endoscopy with biopsy provides direct visualization of the esophagus and tissue to
evaluate for esophagitis and other conditions, such as eosinophilic gastrointestinal disease, that
can clinically mimic GERD.
Gastroesophageal reflux disease is defined as GER with associated esophagitis, poor weight
gain, dysphagia, or abdominal and retrosternal pain. Extra-intestinal complications include
respiratory symptoms such as chronic cough, wheezing, laryngitis with or without hoarse voice,
dental erosions, and recurrent otitis media. Development of Barrett esophagitis or peptic
strictures is uncommon in children. The literature suggests that the contribution of GERD on
asthma is less than previously believed. Symptoms differ by age (Item C167A).
Gastroesophageal reflux disease is managed with both lifestyle changes and medications (Item
C167B) and can be handled by general pediatricians in most cases. Pediatric gastroenterology
should be consulted when primary treatment fails, if medication weaning fails, when children are
failing to thrive, or for other complications including significant family history, as noted in Item
C167B.
PREP Pearls
• Gastroesophageal reflux is the physiologic reflux of gastric contents into the esophagus.
• Gastroesophageal reflux disease (GERD) is the passage of gastric contents into the
esophagus associated with symptoms or problems.
• The differential diagnosis for GERD includes cow's milk and other protein intolerance,
structural etiologies (malrotation, pyloric stenosis, and gastric outlet obstruction),
gastritis, gastroparesis, and physiologic reflux.
• Initial evaluation includes a complete history and physical examination, followed by an
upper gastrointestinal series.
• Evaluation of GERD includes an extensive history, physical examination, and family
history, followed by appropriate testing and/or referral to pediatric gastroenterology.
Suggested Readings
• Craig WR, Hanlon-Dearman A, Sinclair C, Taback SP, Moffatt M. Metoclopramide,
thickened feedings, and positioning for gastro-oesophageal reflux in children under two
years. Cochrane Database Syst Rev. 2004;(4):CD003502.
doi:http://dx.doi.org/10.1002/14651858.CD003502.pub2.
Question 168
You are called to evaluate a baby in the newborn nursery with dysmorphic features. The nurse
tells you that the newborn has severe micrognathia and retrognathia with glossoptosis and is
having difficulty breathing. Physical examination of the newborn shows small, malformed ears
with extreme narrowing of the external auditory canals, a cleft palate, absent lashes and notching
of the lower eyelids, hypoplastic facial bones with a prominent nose, and downward-slanting
palpebral fissures. The mother has downward slanting palpebral fissures, mild hearing loss, and
hypoplasia of the zygomatic complex.
After immediately addressing the respiratory compromise, you explain to the family that the
baby is MOST likely to have
B. craniosynostosis
C. normal intelligence
D. renal anomalies
E. vertebral defects
Intelligence is usually normal, which is important for the family and care team to know from an
educational and therapeutic standpoint. Fertility is normal. Craniosynostosis is not a feature of
TCS, though patients usually have brachycephaly with bitemporal narrowing. Congenital heart
disease, renal anomalies, and vertebral defects are not commonly seen. Features of TCS are also
seen in Nager syndrome, Miller syndrome, Goldenhar syndrome, Pierre Robin sequence, and
nonsyndromic mandibular hypoplasia. With Nager syndrome and Miller syndrome, patients will
have the TCS facial dysmorphology, known as mandibular dysostosis, along with additional limb
anomalies. Nager syndrome patients have preaxial limb anomalies. Miller syndrome patients
have postaxial limb anomalies.
Major management concerns can be stratified by specific age groups (Item C168B).
PREP Pearls
• Treacher Collins syndrome is an autosomal dominant disorder characterized by a classic
facial dysmorphology that is characterized by hypoplasia of the zygomatic bones and
mandible, downslanting palpebral fissures, prominent nose, micrognathia and
retrognathia, external ear abnormalities, coloboma of the lower eyelid, absence of the
lower eyelashes, and anterior hair displacement onto the lateral cheekbones.
• Intelligence and fertility is normal with Treacher Collins syndrome.
• Careful attention to airway management and feeding difficulties is important in infancy
and early childhood.
Suggested Readings
• Katsanis SH, Jabs EW. Treacher Collins syndrome. GeneReviews.
http://www.ncbi.nlm.nih.gov/books/NBK1532.
American academy of pediatrics 552
American Academy of Pediatrics PREP 2016
Question 169
The 26-year-old mother of a patient seeks advice about the human papillomavirus (HPV)
vaccine. She wonders if she should pursue vaccination for herself. She will be turning 27 years
of age in 4 months and she tested positive for HPV on her most recent screening examination.
She has received conflicting information about the risks and benefits of this vaccine in cases
such as hers, and asks for information and a recommendation.
Of the following, the BEST recommendation for this woman is that she
C. should receive only the first 2 doses of HPV vaccine with a 4-week interval
The HPV vaccine is recommended for all males and females beginning at age 11 years, with the
option to initiate the series as early as age 9 years. The vaccine series should be offered to female
and male patients through age 26 and 21 years, respectively. In addition, unimmunized men ages
22 to 26 years who have sex with men or are immunocompromised should initiate the series.
Ideally, HPV vaccination should be administered before the initiation of sexual contact and
potential exposure to HPV. However, the vaccine should not be withheld from people who are
already sexually active. The HPV vaccine is recommended regardless of sexual orientation.
Three formulations of the HPV vaccine are licensed for use in the United States by the US Food
and Drug Administration (FDA) and currently recommended by the Centers for Disease Control
and Prevention (CDC). Bivalent (HPV2) and Quadrivalent (HPV4) were initially licensed for
use. Both vaccines protect against HPV types 16 and 18, which are responsible for 70% of cases
of cervical, 87% of anal, 60% of oropharyngeal, and 31% of penile cancers. In addition, HPV4
protects against HPV types 6 and 11, which are responsible for 90% of anogenital warts and
almost all cases of juvenile recurrent respiratory papillomatosis. In order to provide greater
coverage, a 9-valent vaccine, HPV9, was approved by the FDA on December 10, 2014, for use
in girls ages 9 through 26 years and boys ages 9 through 15 years. HPV9 protects against 5
additional HPV types (31, 33, 45, 52, and 58) which cause approximately 15-20% of vulvar,
vaginal, and cervical cancers. All formulations may be used in females. HPV2 should not be
used in males because of the limited efficacy in prevention of genital warts.
All HPV vaccines should be administered in a 3-dose schedule. The second dose is administered
1 to 2 months after the first dose and the third dose is given 6 months after the first dose. Four
weeks is the minimal interval between the first and second doses; 12 weeks between the second
and third doses; and 24 weeks between the first and the third doses. There is no accelerated
schedule for completing the series, so although the woman in the vignette will be older than 26
years at the time of series completion, the minimal intervals must be adhered to.
No clinical trials indicate a therapeutic effect on existing HPV infections or genital warts, but
patients infected with 1 or more HPV types still benefit from vaccination for protection against
the remaining HPV types in the vaccine. Hence, the mother in the vignette should be encouraged
to complete the HPV vaccine series despite being HPV positive. Testing for previous exposure to
HPV is not indicated before immunization. HPV vaccine may be administered to women with an
abnormal or equivocal Papanicolaou test result as well as to patients with a history of anogenital
warts.
The HPV vaccine is not recommended during pregnancy. The practitioner should inquire about
last menstrual period and potential for pregnancy in sexually active female patients; however, a
negative pregnancy test is not required before administering the vaccine.
PREP Pearls
• The human papillomavirus (HPV) vaccine is recommended beginning at age 11 years,
and may be initiated as early as age 9 years.
• The 3-dose HPV vaccine series should be offered to all female and male patients through
ages 26 and 21 years, respectively, and men ages 22 to 26 years who have sex with men
or are immunocompromised.
• The recommendations for minimal intervals between the 3 doses must be followed. Four
weeks is the minimal interval between the first and second doses; 12 weeks between the
second and third doses; and 24 weeks between the first and the third doses.
Suggested Readings
• AAP Committee on Infectious Diseases. HPV vaccine recommendations. Pediatrics.
2012;129(3):602-605. DOI:http://dx.doi.org/10.1542/peds.2011-3865.
Question 170
A 4-month-old infant presents to the emergency room for evaluation because her parents feel that
she has been “moving her legs less” over the past 2 weeks. On physical examination, she is a
happy infant in no distress who smiles at her mother and grabs her hair. The only abnormal
finding is flaccidity in her legs bilaterally. A complete blood cell count, complete metabolic
panel, creatine kinase, and coagulation profile are all unremarkable.
A. admit her to the hospital for overnight observation and a neurology consult
E. obtain an immediate magnetic resonance image of the thoracic and lumbar spine
While many types of childhood cancer can present with spinal cord compression in early
childhood, the most common include neuroblastoma and tumors of the central nervous system.
Neuroblastoma is an embryonal tumor of the peripheral nervous system and can arise in the
adrenal gland or in any of the sympathetic ganglia. It commonly arises in a paraspinal ganglion
and tends to track into the spinal canal through the neural foramina. This results in the
“dumbbell” sign (Item C170). While neuroblastoma rarely invades the spinal cord, it can cause
severe compression (Item C170), where the cord is not visible at all in the thoracic canal.
Once spinal cord compression has been identified, decompression must occur quickly.
Depending on the etiology of the compression, decompression can occur by surgical
laminectomy or emergent chemotherapy. If the spinal cord compression is caused by a tumor,
dexamethasone should be initiated immediately, as a reduction in inflammation can help reduce
the pressure on the spinal cord. If a tumor is noted, a pediatric oncologist should be emergently
consulted to determine the most appropriate method for cord decompression.
Admission for observation and a neurological evaluation are appropriate in this circumstance,
but only after imaging has been performed and spinal cord compression has been ruled out.
Discharge from the emergency room without imaging would not be the most appropriate
management in this scenario. While a physical therapy evaluation and program would be
appropriate to regain strength in the legs, it should come only after the diagnosis and
management plan have been initiated.
PREP Pearls
• New onset weakness is an emergency that requires immediate cross-sectional imaging of
the spinal cord from the suspected level of injury down.
• If cord compression is found to be caused by a tumor, dexamethasone should be initiated
and an emergent pediatric oncology consult should be obtained.
• Spinal cord decompression can be achieved surgically with a laminectomy, or medically
with chemotherapy or radiation, depending on the suspected cause of the compression.
Suggested Readings
• De Bernardi B, Balwierz W, Bejent J, et al. Epidural compression in neuroblastoma:
diagnostic and therapeutic aspects.Cancer Lett. 2005;228(1-2):283-299. DOI:
http://dx.doi.org/10.1016/j.canlet.2005.01.053
Question 171
You are seeing a 5-year-old boy and his adoptive parent because of educational concerns arising
in kindergarten. He was adopted 1.5 years ago, following a brief time in foster care. He has a
history of chronic and severe neglect by his biological parents. His adoptive parent says she
noted fairly quickly that he was developmentally behind, but had been told that with parental
attention and stimulation he was likely to catch up. However, that has not turned out to be the
case. Hearing and vision are normal for his age, but communication, as assessed by his speech
therapist, has not progressed beyond using occasional 2-word phrases. He is very active and is
"quite a handful" according to his mother. His attention and focus is poor, and although he does
make eye contact with his parent, he avoids eye contact with others. You think it is likely that
this patient has an intellectual disability and have recommended that he be seen by a
psychologist for IQ testing.
Of the following, the additional information needed to confirm your suspected diagnosis is
E. electroencephalogram
In order to establish the diagnosis of an ID, one cannot rely fully on IQ testing. A significant
deficit in adaptive functioning must be found before an ID diagnosis would be valid. According
to the current view of ID, someone who has an IQ more than 2 standard deviations below the
mean, which means a score of less than 70 but with normal range basic life skills for their age,
should not be diagnosed with ID. Adaptive life skills involve self care (such as feeding and
dressing yourself) and basic life planning (ie, getting yourself to where you need to go). The
ideal way to perform this adaptive life skill assessment is by using a standardized instrument
normed for age, which should go along with the IQ testing being performed.
While an autism spectrum disorder should be on this child's current differential diagnosis (the
reported poor eye contact raises this possibility), utilizing an autism symptom rating scale is not
the preferred option for a few reasons. The first is that the vignette specifically stated the
diagnosis being evaluated is an ID, for which both IQ and adaptive behavior assessments are the
preferred steps rather than having the parent fill out an autism symptom rating scale. The second
reason is that an autism symptom rating scale will not diagnose autism, but rather will yield a
score indicating the overall likelihood of autism and, if present, the extent of those symptoms.
When there is already sufficient reason to be suspicious of autism, as in this case, a clinical
assessment of the diagnostic characteristics of autism in the Diagnostic and Statiscal Manual of
Mental Disorders, Fifth Edition would be indicated. This child could have either ID, autism, or
both.
A brain magnetic resonance image would be indicated for any circumstance in which a child's
intellectual or other brain functioning is linked to specific neurological findings, such as spastic
hemiparesis, or has functioning that appears to be degenerative or progressive in nature.
Magnetic resonance imaging assessments for ID in general are not indicated. An
electroencephalogram (EEG) would be reasonable if there was any history of tonic-clonic
movements, sudden episodes of altered consciousness with loss of bowel or bladder control, or
possible post-ictal states. A child with recurrent seizures may appear more intellectually
impaired than they would be if their seizures were under control, so this test is occasionally
pertinent during the evaluation of an ID. For the child in this vignette, there is no reason for an
EEG at this time.
While it is true that a very young child with a history of being raised in a neglectful household
tends to increase their developmental gains after being placed in a more stimulating and
responsive environment, a reassurance-only approach would not be appropriate if the degree of
developmental delay is significant.
PREP Pearls
• Children with an IQ of less than 70 will not be diagnosed with an intellectual disability if
their adaptive behavior assessment is in the normal range.
• The diagnosis of intellectual disability and autism are separate from each other; a child
can have one or the other or both of these disorders.
Suggested Readings
• American Psychiatric Association. Diagnostic and Statistical Manual of Psychiatric
Disorders, Fifth Edition. Washington DC: American Psychiatric Publishing; 2014: 991
pp.
• Shea SE. Intellectual disability (mental retardation). Pediatr Rev. 2012;33(3):110-121.
DOI:http://dx.doi.org/10.1542/pir.33-3-110.
Question 172
A 2-day-old newborn is being evaluated for seizures by neurology and has had a computed
tomography of the brain that shows findings consistent with tuberous sclerosis. The neonatal
intensive care unit nurse tells you that there have been frequent wide complex ectopic beats, but
none have been captured on an electrocardiogram. On physical examination, you find a male
newborn with weight of 3.8 kg in no distress, heart rate of 130 beats/min, respiratory rate of 34
breaths/min, and blood pressure of 82/55 mm Hg. The heart rhythm is regular. The cardiac
examination is unremarkable: there is a normal S1and S2 without any murmurs, rubs, thrills, or
gallops. The point of maximal impulse is not displaced. There is no hepatosplenomegaly and the
femoral pulses are normal. An echocardiogram is ordered.
C. dilated cardiomyopathy
D. rhabdomyomas
E. tetralogy of Fallot
Rhabdomyomas are the most common cardiac tumor in young children and account for 80% of
those seen in infants younger than 1 month of age. More than 50% of children with
rhabdomyomas have TS complex. The other congenital heart lesions in the response choices,
including tetralogy of Fallot, atrial septal defect, and atrioventricular canal, may occur, but are
not linked to TS. Dilated cardiomyopathy is not likely. The rhabdomyomas may cause the
ventricular septum to enlarge because of the mass effect of the tumor.
Infants with TS and cardiac rhabdomyomas may present with a wide range of clinical symptoms.
They may be asymptomatic from a cardiac standpoint, or have symptoms of outflow obstruction
with abnormal atrioventricular (AV) valve function. If there is a mass on one of the AV valves,
there may be a regurgitant murmur in systole, or a murmur to suggest stenosis in diastole. If
there is outflow tract obstruction, there may also be systolic murmurs and, in the case of the right
ventricular outflow tract, there may be cyanosis. Patients have been reported with intractable
arrhythmias. The newborn described in the vignette has had a report to suggest premature
ventricular contractions. This may remain a minor issue or develop into a more serious
ventricular arrhythmia. The diagnosis of cardiac rhabdomyoma is usually made with
echocardiography and may be confirmed with magnetic resonance imaging. Patients commonly
have multiple lesions that range from millimeters to centimeters. They may cause little or no
hemodynamic consequence or require multiple medications such as amiodarone and propranolol
to control ventricular tachycardia. If there is evidence of obstruction, then surgical resection may
be recommended. Everolimus, an inhibitor of the mTOR pathway, can be used to treat
subependymal giant cell astrocytomas associated with TS and has been shown to be effective in
shrinking the cardiac rhabdomyomas in selected patients. The pathologic structure of the
rhabdomyomas may include Purkinje cells, and this may be a mechanism for pre-excitation seen
in some patients with this type of cardiac tumor. Patients with this finding may develop
supraventricular tachycardia.
Fortunately, however, the most likely course for cardiac rhabdomyomas diagnosed in infancy is
that of spontaneous regression. Surgical resection can be avoided unless there is obstruction or
arrhythmias that are unable to be controlled medically.
PREP Pearls
• All children with cardiac rhabdomyomas need evaluation for tuberous sclerosis.
• Cardiac rhabdomyomas found in infancy often regress spontaneously in the first year of
life.
Suggested Readings
• Benyounes N, Fohlen M, Devys JM, et al. Cardiac rhabdomyomas in tuberous sclerosis
patients: a case report and review of the literature. Arch Cardiovasc Dis. 2012;105(8-
9):442-445. DOI: http://dx.doi.org/10.1016/j.acvd.2012.01.009
Question 173
You are seeing a 12-year-old boy in your office with a complaint of right knee pain and swelling
of 4 days’ duration. He has pain with movement, making it difficult to walk. He has no fever,
rash, conjunctivitis, or diarrhea. There is no history of travel, injury, or tick exposure. The boy
was seen 2 weeks ago for fever and cough, at which time a throat culture was negative and he
was treated for suspected community-acquired pneumonia. He was then well until the knee
swelling occurred. On physical examination, you find an afebrile, well-appearing boy with
swelling, erythema, and decreased range of motion of the right knee. He winces when you
passively flex his knee. The remainder of the physical examination is unremarkable. The boy
was seen in the emergency department 2 days earlier, where an evaluation for septic arthritis was
negative.
Laboratory results from the emergency department show:
• White blood cells, 11,300/µL (11.3 x 109/L)
• Red blood cells, 4.5 x 106/µL (4.5 x 1012/L)
• Hemoglobin, 11.2 g/dL (112 g/L)
• Hematocrit, 37.1%
• Platelet count, 300 x 103/µL (300 x 109/L)
• Erythrocyte sedimentation rate, 25 mm/h
• C-reactive protein, 2.0 mg/L
• Synovial fluid analysis:
• Yellow and opaque
• White blood cell count, 20,000 cells
• 50% polymorphonuclear leukocytes
• Low viscosity
• Gram-stain negative
• Culture no growth for 48 hours
Of the following, the BEST next step in management of this boy’s symptoms is
A. azithromycin
B. doxycycline
C. naproxen
D. penicillin
E. prednisone
Reactive arthritis is associated with an infection outside the affected joint. Reactive arthritis is a
clinical diagnosis based on the presence of oligoarticular arthritis, usually of the lower
extremities, and exclusion of other types of arthritis, such as septic arthritis, Lyme arthritis, acute
rheumatic fever, trauma, neoplasm and osteomyelitis. Reactive arthritis is usually asymmetric
and affects large joints such as the knee, hip, and ankle. Sacroiliac joints and the joints of the
upper extremities can be affected. Reactive arthritis is commonly associated with sexually
transmitted diseases such asChlamydia and gonorrhea. All patients with a history of sexual
activity and arthritis should be screened for these diseases. Reactive arthritis is also associated
with other genitourinary, gastrointestinal, and upper respiratory infections. The arthritis can
appear within days or up to 6 weeks after the infection. After 6 weeks, the arthritis is considered
chronic and a rheumatology referral for possible autoimmune disease should be considered.
Treatment of reactive arthritis is supportive, with nonsteroidal anti-inflammatory drugs (Item
C173) and with a conservative approach, such as rest and cold therapy. Activity should be
limited secondary to pain and can resume as pain improves.
PREP Pearls
• Sexually active patients with arthritis should be screened for Chlamydia and gonorrhea.
• Reactive or postinfectious arthritis is associated with genitourinary, gastrointestinal, and
upper respiratory infections.
• Postinfectious arthritis should be managed initially with nonsteroidal anti-inflammatory
medications.
Suggested Readings
• Bernard R. Approach to the child with joint inflammation. Pediatr Clin North Am.
2012;59:245-262. DOI:http://dx.doi.org/10.10
Question 174
A 9-year-old girl presents to your office for evaluation of right wrist pain 2 days after falling
onto an outstretched arm during a soccer game. On physical examination, the girl is tender over
the right ulnar mid shaft with limited pronation and supination. Neurovascular examination of
the hand and wrist is unremarkable. Radiographs of the wrist demonstrate anterior bowing of the
shaft of the ulna.
A. allow return to activities without immobilization once she regains pronation and
supination
The medial border of the ulna should be perfectly straight on lateral radiography; curvature
suggests plastic deformity, bowing of the bone on radiographs without evidence of cortical
dysfunction. Similar to greenstick fractures, plastic deformity is only seen in pediatric patients
because of the increased flexibility of young bones. The presence of an ulnar deformity with
limited pronation and supination raises concern for a Monteggia lesion—fracture or deformation
of the ulna associated with a radiocapitellar dislocation. Although Monteggia lesions are rare,
failure to diagnose this condition can lead to subsequent disability. Therefore, any patient with a
midshaft or proximal ulnar injury, even patients with plastic deformity or mild greenstick
fracture, should undergo dedicated elbow radiography to examine the radiocapitellar joint. Item
C174 shows a greenstick fracture of the ulna with an associated radiocapitellar dislocation.
Monteggia lesions were classified by Bado into 4 types. The most common is a type 1 injury,
with fracture of the proximal or midshaft of the ulna and associated anterior dislocation of the
radial head. There are also Monteggia lesion variants, ulnar injuries with radial head subluxation
that were not included in Bado’s classification system. Monteggia injuries represent fewer than
1% of pediatric fractures and typically occur in children younger than 12 years. A Monteggia
lesion identified within 3 weeks of injury often can be treated with closed reduction. Monteggia
lesions with delayed diagnosis and fracture healing, or those unsuccessfully treated by closed
reduction, require open reduction with ulnar osteotomy to restore normal radiocapitellar joint
anatomy.
Elbow dislocations at the ulnar-trochlear joint are rare in children, with the incidence rising
during adolescence. Dislocations most often occur in the posterolateral direction. When an elbow
dislocation is suspected, radiographs should always be obtained before performing a reduction
maneuver, both to confirm the diagnosis and to look for associated fractures.
For the girl in the vignette, a return to sports without imaging to evaluate the elbow and
immobilization for the ulnar deformity would not be appropriate. Magnetic resonance imaging is
not indicated because a Monteggia lesion, if present, should be visible on plain radiography. Cast
immobilization of the wrist and forearm would be appropriate if no radiocapitellar disruption is
seen on elbow radiography. The patient should have close follow-up because of the risk for
radiocapitellar dislocation in the first few weeks after injury. If a Monteggia lesion is seen on
radiography, referral to orthopedic surgery would be indicated. Because the girl in the vignette
was injured only 2 days before presentation, the appropriate first step would be to attempt closed
reduction under sedation.
PREP Pearls
• For children with any type of acute ulnar deformity or fracture, dedicated elbow
radiographs should be obtained to assess for possible Monteggia lesion (radiocapitellar
joint dislocation).
• Elbow dislocation at the ulna/trochlear joint is uncommon in children.
Suggested Readings
• Weinstein SL, Flynn HM. Lovell & Winters Pediatric Orthopaedics. 7th ed. Philadelphia,
PA: Lippincott Williams & Wilkins; 2013.
Question 175
A 14-year-old patient requires treatment for a severe bacterial infection and meets inclusion
criteria to participate in a study examining the safety and appropriate dosing in children of a new
antibacterial drug. In addition to obtaining informed consent from the patient’s parents,
appropriate information to share with this patient includes:
• What the study is about
• Why the patient qualifies for the study
• That the study is voluntary
• What procedures are included in the study
• The potential benefits of participating in the study
• The risks of participating in the study
• That the patient will be treated the same whether enrolled in the study or not
• That the patient can withdraw from the study at any time
• An opportunity for the patient to ask questions
Of the following, the process outlined for the patient is BEST known as the
A. assent process
B. consent process
C. dissent process
D. permission process
E. regulatory process
Informed consent is the process by which full disclosure of research information (Item C175) is
discussed with the parent or guardian in nonscientific language, with the opportunity for
questions and clarification. This process should be conducted in a noncoercive setting and in a
manner such that the parent can comfortably make an informed decision about enrolling the child
in the research protocol. If a parent gives permission for a child to participate in a research
protocol without having received full disclosure of information regarding the research, that does
not constitute informed consent. The concepts of informed assent, dissent, and consent arise from
the ethical principles of patient autonomy and basic human rights. These principles are essential
for a good relationship between a physician and patient.
The research regulatory process covers vastly more than the consent or assent processes alone,
and includes issues such as IRB registration rules, financial disclosures of investigators, good
laboratory practice, investigational drug regulations, electronic record documentation, signature
rules, and more.
PREP Pearls
• The assent process requires that a reasonable effort be made to enable a child to
understand what his/her participation in research would entail and obtain the child’s
affirmative agreement to participate.
• Informed consent is the process by which full disclosure of research information is
discussed with the parent or guardian in nonscientific language, with the opportunity for
questions and clarification.
• The concepts of informed assent, dissent, and consent arise from the ethical principles of
patient autonomy and basic human rights. These principles are essential for a good
relationship between physician and patient.
Suggested Readings
• US Food and Drug Administration. Code of Federal Regulations Title 21. 21CFR50.55.
Available
athttp://www.accessdata.fda.gov/scripts/cdrh/cfdocs/cfcfr/cfrsearch.cfm?fr=50.55.
• US Food and Drug Administration. Guidance for institutional review boards and clinical
investigators. Available atwww.fda.gov/oc/ohrt/irbs/default.htm.
Question 176
You are following up on telephone messages at the end of a busy day in your pediatric practice
with the assistance of a medical student. One message is from a local attorney, asking if you
would be willing to serve as an expert witness in a medical malpractice case involving a pediatric
patient. The medical student asks if it is ethical for pediatricians to do this.
A. it is generally not ethical for physicians to receive compensation for serving as expert
witnesses in medical malpractice cases
D. physicians should not serve as expert witnesses within the state(s) in which they hold
medical licensure, to avoid conflicts of interest
E. physicians should only agree to testify in cases that are unrelated to their specific
medical specialties, to ensure objectivity
The American Academy of Pediatrics (AAP) Committee on Medical Liability and Risk
Management has issued a policy statement that provides guidance for pediatricians regarding
expert witness participation. The guidelines emphasize that the public interest and interests of the
medical and legal professions "are best served when scientifically sound and unbiased expert
witness testimony is readily available in civil and criminal proceedings." Pediatricians have an
ethical and professional duty to "assist in the administration of justice" as "members of the
medical community, patient advocates, and private citizens."
Regarding qualifications for physicians who serve as expert witnesses, the AAP recommends
that physicians should only contribute as medical experts to cases in which they possess true
expertise and related experience. Physicians serving as expert witnesses should hold current,
unrestricted medical licenses in their states of practice and be certified by the American Board of
Medical Specialties, American Osteopathic Association, or a board with equivalent standards. In
addition, expert witnesses must have been actively engaged in clinical practice in the area of
medicine about which they testify. The majority of their working time should not be spent doing
expert witness work unless they have retired from clinical practice.
Pediatricians serving as expert witnesses should take all steps needed to ensure that the medical
testimony they provide in legal proceedings is complete, accurate, unbiased, and based on an
excellent understanding of current medical evidence and standard practice related to the case for
which they testify.
It is ethical for physicians to testify as expert witnesses in medical malpractice cases, even when
they are not mandated by a court order to do so, as long as they adhere to ethical standards. The
AAP recognizes that pediatricians "have the professional, ethical, and legal duty to assist in the
legal process when medical issues are involved."
The assertion that physicians "should not serve as expert witnesses within the states in which
they hold medical licensure" is incorrect. In fact, some state courts have "locality rules,"
American academy of pediatrics 576
American Academy of Pediatrics PREP 2016
requiring that expert witnesses be familiar with the general practices of physicians in the same
community, or at least in similar communities. In some cases, state courts have not allowed
testimony from expert witnesses who have not practiced within that state.
Physicians should not agree to testify in cases unrelated to their specific medical specialties. The
AAP recommends that pediatricians should limit their participation as medical experts to cases in
which they possess genuine expertise.
PREP Pearls
• It is ethical for physicians to testify as expert witnesses, provided they take all steps
needed to ensure that the testimony they provide is complete, accurate, unbiased, and
based on a thorough understanding of current medical evidence and standard practice
related to the case.
• The AAP recommends that physicians should only contribute as medical experts to cases
in which they possess true expertise and related experience.
• Transcripts of courtroom testimony in medical malpractice cases may be submitted for
peer review.
Suggested Readings
• American Academy of Pediatrics Committee on Medical Liability and Risk Management.
Expert witness participation in civil and criminal proceedings. Pediatrics. 2009;124:428.
DOI: http://dx.doi.org/10.1542/peds.2009-1132.
Question 177
A newborn who was delivered at home is brought to your office 30 hours after birth for the first
newborn health supervision visit. She was born at 40 weeks’ gestation to a 42-year-old
primigravida mother by uncomplicated spontaneous vaginal delivery. The maternal prenatal
screening was normal, including negative group B streptococcal screening. The maternal history
was remarkable only for chronic hypertension. The newborn has been exclusively breastfeeding
since birth. Vital signs include an axillary temperature of 37°C, heart rate of 140 beats/min, and
respiratory rate of 50 breaths/min. Physical examination reveals a quiet, slightly ruddy newborn
weighing 2,400 g (<3%). There is jaundice of the face and chest, periodic breathing with rare 20
second pauses, slightly decreased tone, and listlessness. The mother describes her newborn as
being very quiet for the past 12 hours.
Of the following, the test MOST likely to establish the newborn’s diagnosis is a(n)
A. fractionated bilirubin
B. ionized calcium
C. serum glucose
E. venous hematocrit
Small-for-gestational age infants with a history of intrauterine growth restriction (IUGR) have
increased perinatal morbidity and mortality. Their intrauterine environment may cause chronic
fetal hypoxia and in utero demise. Diminished blood flow during uterine contractions
superimposed on a chronically stressed IUGR fetus may lead to intolerance of labor, meconium
aspiration, and birth asphyxia. After birth, the infant’s decreased subcutaneous fat and increased
surface area contribute to excessive heat loss and issues with thermoregulation. Chronic fetal
hypoxia may also stimulate erythropoiesis, leading to polycythemia, which may further
exacerbate hypoglycemia. The American Academy of Pediatrics Committee on the Fetus and
Newborn has recommended monitoring all SGA infants for hypoglycemia for the first 24 hours
after birth.
For the infant in the vignette, who has jaundice noted in only the face and chest, the fractionated
bilirubin is unlikely to be high enough to cause apnea and decreased tone. The infant is slightly
ruddy, suggesting a mildly elevated hematocrit, but it is unlikely to be the primary cause of the
infant’s clinical findings. Hypocalcemia may present with the findings noted in the vignette, but
is not associated with SGA infants unless asphyxia is present. Congenital cytomegalovirus
infection is associated with IUGR/SGA and may present with significant central nervous system
involvement. However, the underlying cause of the small size of the infant in the vignette is
more likely the maternal chronic hypertension.
PREP Pearls
• Small-for-gestational age (SGA) infants are at increased risk for hypoglycemia because
of insufficient hepatic and skeletal muscle glycogen stores. Some affected infants have
superimposed hyperinsulinemia.
• SGA infants with concurrent intrauterine growth restriction have increased perinatal
morbidity and mortality.
• SGA infants should be monitored for hypoglycemia for the first 24 hours after birth.
Suggested Readings
• American Academy of Pediatrics Committee on Fetus and Newborn. Postnatal glucose
homeostasis in late-preterm and term infants. Pediatrics. 2011;127:575-579. DOI:
http://dx.doi.org/10.1542/peds.2010-3851.
Question 178
A 10-year-old girl has a 2-year history of poor attention. At school, she stares out the window,
often unresponsive to her teacher’s directions, occasionally wanders around the playground, and
fails to come in on time at the end of recess. Her parents have noticed episodes of similar
behavior at home. One time, they noticed that the girl’s eyes were “beating” side-to-side for
about 10 min and she wouldn’t respond to them. The girl reports that she was awake during that
episode and could hear her parents, but didn’t feel like answering “stupid” questions. Her
physical examination is unremarkable.
C. obtain an electroencephalogram
D. referral to an ophthalmologist
E. referral to a psychologist
PREP Pearls
• Episodes of altered consciousness with lateral nystagmus are highly suggestive of a focal-
onset seizure.
• New-onset nystagmus requires prompt evaluation.
Suggested Readings
• Russ SA, Larson K, Halfon N. A national profile of childhood epilepsy and seizure
disorder. Pediatrics. 2012;129(2):256-264. DOI: http://dx.doi.org/10.1542/peds.2010-
1371.
Question 179
A 14-year-old adolescent presents with emesis this morning, following 3 days of worsening
abdominal pain and decreased energy. The patient has a history of pre–B cell acute
lymphoblastic leukemia diagnosed 7 years ago, which was treated with chemotherapy for 3
years. His disease recurred and he underwent bone marrow transplant earlier this year. He
subsequently developed graft versus host disease and was on a tapering course of prednisone
over the last 2 months. He was most recently taking 5 mg orally once daily until his prednisone
was discontinued 1 week ago. You order laboratory tests as a part of your evaluation.
A. hypochloremia
B. hyperglycemia
C. hypokalemia
D. hyponatremia
E. metabolic alkalosis
The causes of AI can be considered under 2 general headings: primary AI caused by destruction
of the adrenal glands themselves, and secondary AI caused by disordered HPA axis function. In
primary AI, hyponatremia and hyperkalemia occur because of mineralocorticoid deficiency. This
is seen in conditions such as congenital adrenal hyperplasia, Addison disease, or other conditions
leading to primary disease.
Secondary AI can occur after exogenous steroid treatment, after cure of Cushing syndrome, and
from hypothalamic or pituitary lesions. As occurred with the boy in the vignette, in some cases
the HPA axis remains suppressed despite prolonged tapering of exogenous glucocorticoids. The
resulting decreased adrenocorticotropic hormone secretion leads to isolated glucocorticoid
deficiency, which causes retention of free water and subsequent hyponatremia. Because the
renin-angiotensin system is not affected, potassium concentrations are generally normal.
However, if the adrenal glands have atrophied from prolonged suppression from exogenous
steroid use, then it is possible to have abnormalities in both sodium and potassium.
Other biochemical abnormalities that can occur in both forms of AI include metabolic acidosis
and hypoglycemia. Hypochloremia is not consistently associated with AI.
PREP Pearls
• Suppression of the hypothalamic-pituitary-adrenal axis after steroid withdrawal can lead
to secondary adrenal insufficiency (AI).
• In secondary AI, isolated hyponatremia without potassium abnormalities is a common
finding. This is in contrast to primary AI, where hyponatremia and hyperkalemia are
frequently observed.
Suggested Readings
• Antal Z, Zhou P. Addison disease. Pediatr Rev. 2009;30(12):491-493. DOI:
http://dx.doi.org/10.1542/pir.30-12-491.
• Loriaux DL. Adrenocortical insufficiency. In: Becker KL, ed. Principles and Practice of
Endocrinology and Metabolism. 3rd ed. Philadelphia, PA: Lippincott Williams &
Wilkins; 2001: 739-742.
Question 180
You have been caring for a 7-year-old girl with severe persistent asthma. She is being treated
twice daily with a combined high dose inhaled corticosteroid and long acting β-agonist, and once
daily with a leukotriene receptor antagonist. She demonstrates chronic rhinitis for which she is
treated daily with a nasal steroid and a nonsedating antihistamine. Her father and paternal
grandmother suffer from hay fever. She has never experienced an anaphylactic reaction to an
insect sting or food.
Despite her treatment and reported excellent adherence, the patient has an asthma control test
score of 17. She continues to require short acting β-agonist administration 3 to 4 times per week
for exercise intolerance, cough, or wheezing, and has required 3 courses of systemic steroids in
the last 4 months during weather changes or viral illnesses.
The family has a cat, 2 dogs, and a fish. Both parents smoke cigarettes outside of the home and
car. Prior radioallergosorbent tests for cat, dog, peanut, and egg allergies were negative.
On physical examination, she has a comfortable respiratory pattern. The lungs are well aerated,
but you note a prolongation of the expiratory phase of respiration with a scattered end expiratory
wheeze. Forced expiratory volume in 1 second (FEV1) is greater than 80% predicted, but the
FEV1/forced vital capacity ratio and maximal midexpiratory flow are mildly decreased.
The parents inquire about the utility of skin testing for allergies in their child.
Of the following, the BEST response to the parents’ question is
A. further skin testing is not cost effective and relocation of the family’s dog and cat is
recommended
B. further skin testing is reasonable for avoidance measures, but allergy therapy has
already been maximized
The patient in the vignette demonstrates chronic rhinitis and family members exhibit symptoms
that are consistent with atopy. Furthermore, there are multiple potential or proven aero-allergens
in the home, including pets and exposure to cigarette smoke.
The prevalence of asthma and allergic rhinitis have increased in parallel in recent decades.
Asthma and allergic rhinitis frequently coexist in the same patient. As many as 60% of asthmatic
individuals experience rhinitis and there is a corresponding 20% prevalence of asthma in patients
with allergic rhinitis.
The complexity found in the overlapping clinical spectra of asthma and allergic rhinitis is
exacerbated by the fact that asthma is not a single disease, but rather a variable and multifactorial
disease process that may be modified by genetic, epigenetic, and environmental factors. The
spectrum of atopic disease is similarly complex and different “endotypes” of atopic disease exist
that are unique with regard to their association with asthma. Furthermore, both serum specific
(immunoglobulin E [IgE]) and skin allergy tests are prone to variability in interpretation.
Therefore, it has been recommended that both skin testing and specific IgE testing be quantified
rather than simply reported as positive or negative.
The presence of sensitization to aero-allergens is a recognized risk factor for the development of
asthma. In children younger than 3 years of age with recurrent wheezing, evidence of
sensitization to 1 or more aero-allergens is considered one of the major criteria that predicts
wheezing at school age (asthma predictive index). Moreover, asthma is often preceded by the
presence of allergic rhinitis. Patients with allergic rhinitis without a history of asthma may
demonstrate asthma-like airway hyperreactivity when exposed to allergens to which they have
been sensitized. In addition, the inhalation of allergens in individuals with seasonal rhinitis has
been shown to induce bronchial inflammation. The duration of rhinitis may be an independent
risk factor for the development of asthma; in patients with moderate to severe persistent allergic
rhinitis without asthma, rhinitis duration of greater than 5 years has been found to be a
significant risk factor for severe airway hyperreactivity. Extensive research therefore suggests
that the upper (nose) and lower (lung) airways should be regarded as a pathophysiological
continuum rather than as distinct entities.
Compared to skin allergy testing, serum-specific IgE techniques are less time efficient and are
more expensive. Several studies have also found serum testing to be less sensitive than skin
testing for the detection of clinically relevant allergies. Serum testing allows evaluation of
patients with moderate to severe atopic dermatitis, and patients do not need to stop routine
antihistamine therapies prior to testing.
In this vignette, relocation of the family dog and cat is not recommended without evidence to
suggest aero-allergen sensitization. Cost effectiveness of allergen testing is likely to be favorable
in this patient with severe persistent asthma and the need for multiple physician visits. If
significant allergic disease is confirmed, additional therapies, including immunotherapy, may be
considered.
Finally, the vast majority of pediatric patients with moderate to severe persistent asthma have
normal forced expiratory volume in 1 second. The criteria for asthma-related severity and control
are clinically based and spirometric indices are only a portion of the appropriate assessments.
PREP Pearls
• The majority of pediatric patients with moderate-to-severe persistent asthma will
demonstrate a normal forced expiratory volume in 1 second with spirometric testing.
• Allergic disease and asthma frequently coexist in the same patient. Allergic rhinitis is a
recognized exacerbating factor in patients with asthma. Furthermore, allergic rhinitis may
precede the development of asthma and contribute independently to airway
hyperresponsiveness.
• Serum immunoglobulin E allergy testing is less sensitive than skin testing for the
detection of clinically relevant allergies.
• When allergies are suspected in patients with asthma, skin testing is warranted to help
direct therapy.
Suggested Readings
• Boulay ME, Boulet LP. The relationships between atopy, rhinitis and asthma:
pathophysiological considerations. Curr Opin Allergy Clin Immunol. 2003;3(1):51-55.
• Chinoy B, Yee E, Bahna SL. Skin testing versus radioallergosorbent testing for indoor
allergens. Clin Mol Allergy. 2005;3(1):4. DOI: http://dx.doi.org/10.1186/1476-7961-3-4.
Question 181
During his 9-month health supervision visit, you note that an infant has 2 lower central incisors.
The mother reports that she has not begun cleaning the infant’s teeth.
Of the following, the MOST appropriate guidance to provide this parent about her infant’s dental
health is:
B. fluoride toothpaste should not be used by children younger than 5 years of age
because of a high risk of toxicity
C. the initial dental examination should occur within 6 months of first dental eruption
and no later than 12 months of age
E. the parents’ dental hygiene has little impact on the infant’s oral health
Pediatricians also play a crucial role in supporting dental health, especially in the areas of
primary and secondary prevention. Primary prevention for dental caries is focused on parental
dental health because the child’s oral bacterial flora is determined by that of the mother, and if
the mother has heavy colonization with Streptococcus mutans and other cariogenic bacteria, the
child is at greater risk for early childhood caries. Secondary prevention focuses on managing risk
factors for early childhood caries once the bacterial milieu is established. The 3 components of
this approach are dietary counseling, promotion of oral hygiene, and use of fluoride. A diet high
in sugary carbohydrates provides the substrate that cariogenic bacteria ferment, leading to
lowered pH (which promotes further growth of cariogenic bacteria) and demineralization of
dental enamel. Therefore, dietary counseling to promote breastfeeding, to limit sugar, especially
in liquids, and to eliminate taking a bottle to bed are aspects of anticipatory guidance that
promote dental health.
Current recommendations for oral hygiene are that twice daily tooth brushing should start as
soon as the first tooth erupts and flossing should begin as soon as teeth contact each other. If the
child is at increased risk for early caries, a small amount of toothpaste can be used beginning in
infancy. The AAPD recommends a “smear” of fluoridated toothpaste for children younger than 2
years of age and a “pea-sized” amount for children aged 2 years through 5 years.
Fluoride use has led to a substantial decrease in dental caries. It is effective both topically and
orally, and home use, as well as professional application, has demonstrated benefit. Community
water fluoridation, when natural water fluoride levels are low, is the most cost effective way to
provide fluoride’s preventive benefits. Home-administered daily oral fluoride supplements are
beneficial in communities with low water fluoride content (Item C181). When that is not
available, and as a supplement, fluoridated toothpaste has proven efficacy. Professionally applied
topical fluorides in the form of gel, rinse, and varnish (most common) are safe and effective and
should be applied twice yearly.
PREP Pearls
• The first dental visit should occur within 6 months of first tooth eruption, but no later
than 12 months of age.
• Pediatricians should provide dental anticipatory guidance to discuss diet, dental hygiene,
and fluoride to promote good dental health.
• Topical and oral fluoride supplementation are effective in reducing dental caries.
Suggested Readings
• American Academy of Pediatric Dentistry. Guideline on fluoride therapy. American
Academy of Pediatric Dentistry website.
http://www.aapd.org/media/Policies_Guidelines/G_fluoridetherapy.pdf.
• American Academy of Pediatric Dentistry. Guideline on periodicity of examination,
preventive dental services, anticipatory guidance/counseling, and oral treatment for
infants, children, and adolescents. American Academy of Pediatric Dentistry website.
http://www.aapd.org/media/Policies_Guidelines/G_Periodicity.pdf.
Question 182
The local campaign to prevent teenage pregnancy is hosting its annual conference. The primary
focus of this year’s conference will be risks associated with teenage pregnancy and childbearing.
Of the following, the GREATEST risk associated with pregnancy in this age group is increased
Pregnant adolescents have a higher risk of complications, including placenta previa, pregnancy-
induced hypertension, and premature delivery. Additionally, adolescent mothers are less likely to
graduate from school and are more likely to live in poverty. Children born to adolescent parents
are more likely to have poorer health outcomes such as low birth weights and higher rates of
infant mortality, as well as lower educational achievement compared with children born to
nonadolescent parents.
Early and adequate prenatal care is important for the pregnant adolescent to try to ensure a
healthy pregnancy and birth outcomes. Pregnant adolescents should receive resources to assist
with smoking, alcohol, and drug cessation, if needed.
Adolescent childbearing has not been associated with an increased lifetime risk of breast cancer,
gestational diabetes, or chromosomal abnormalities. In fact, younger age at the time of
pregnancy has been associated with lower rates of these conditions.
PREP Pearls
• While adolescent birth rates have declined for the last few years, the United States
continues to have higher adolescent pregnancy and birth rates than other similarly
industrialized countries.
• Adolescent childbearing has been associated with negative consequences for both the
adolescents and their children.
• Early and adequate prenatal care is important for the pregnant adolescent to try to ensure
a healthy pregnancy and birth outcomes.
• Pregnant adolescents have a higher risk of complications including placenta previa,
pregnancy-induced hypertension, premature delivery, and higher rates of infant mortality.
Suggested Readings
• Elfenbein DS, Felice ME. Adolescent pregnancy. In: Kliegman RM, Stanton BF, St
Geme JW III, Schor, NE, Behrman RE, eds. Nelson Textbook of Pediatrics. 20th ed.
Philadelphia, PA: Saunders Elsevier; 2015:978-982.
Question 183
A 12-year-old boy with no significant medical history presents to your office with perineal pain,
difficulty urinating, and hematuria. His symptoms began 14 hours earlier after he slipped and
straddled his bicycle seat forcefully while attempting a jump off a retaining wall. At the time of
this injury, the boy immediately felt pain in his perineum, but did not seek medical attention. His
mother scheduled an appointment with you this morning after the boy reported he was having
difficulty urinating and had seen blood in his urine.
The boy is in no acute distress, but appears uncomfortable. His vital signs include a temperature
of 37.2°C, heart rate of 100 beats/min, respiratory rate of 14 breaths/min, and a blood pressure of
110/70 mm Hg. His physical examination reveals dried blood and a small amount of fresh blood
at the urethral meatus, along with perineal bruising. There are no lacerations visible on your
external genitourinary examination. Both of his testes are palpable and nontender, and his
abdomen is soft and nontender. There is no tenderness or instability with palpation over his
pelvic bones.
Based on your clinical findings, the MOST likely diagnosis for this boy is
B. bladder laceration
D. rhabdomyolysis
E. testicular contusion
Pediatric providers should recognize the clinical findings associated with urethral trauma. The
presence of blood at the urethral meatus, as noted in the boy in the vignette, is highly suggestive
of a urethral injury. This finding has been reported in up to 90% of patients with anterior urethral
injuries. Other clinical indicators of urethral injury may include voiding difficulty, scrotal
hematoma or perineal ecchymoses, and a high-riding prostate on rectal examination in males.
Urethral injuries may be associated with pelvic fractures. Posterior urethral injuries are typically
seen in patients with multisystem trauma and pelvic fractures, whereas anterior urethral injuries
typically result from a straddle impact and often occur as isolated injuries. Blunt trauma arising
from motor vehicle accidents, direct falls onto the perineum, and straddle injuries are the causes
of most pediatric urethral injuries. Penetrating injuries and injuries related to instrumentation of
the urethra occur much less commonly.
In patients with suspected urethral injuries, a urinary catheter should never be inserted until
retrograde urethrography is performed to assess whether the urethra is intact. Retrograde
urethography is the gold standard study for the diagnosis of urethral injury.
Injuries to the bladder may result from blunt or penetrating trauma, with motor vehicle accidents
being the most common mechanism in children. Most bladder injuries (>80%) are associated
with pelvic fractures and penetration of the bladder by a bony fragment. Clinical findings
associated with bladder laceration may include hematuria, dysuria, and pain in the lower
abdomen/pelvis. In cases of complete bladder rupture, patients may develop peritonitis and a
palpable fluid wave from leakage of urine into the peritoneal cavity. Although the boy in the
vignette presents with hematuria, an anterior urethral injury would be much more likely to result
from his mechanism of injury than a bladder laceration. In addition, he has no abdominal pain
and no findings suggestive of a pelvic fracture on physical examination.
Penile shaft fractures arise from traumatic rupture of the corpus cavernosum. These injuries
typically result from the traumatic impact of an erect penis against a hard surface. Patients with
penile shaft fractures generally report hearing a cracking sound at the time of injury and present
with pain, swelling, and deformity of the penile shaft. None of these findings is present in the
boy in the vignette.
Classic signs and symptoms for patients with rhabdomyolysis include weakness, myalgias, red or
brown discolored urine, and in some cases, difficulty with urination. Rhabdomyolysis may result
from traumatic causes such as crush injuries and compartment syndrome, as well as nontraumatic
causes such as excessive exertion, prolonged seizure activity, hypokalemia, metabolic
myopathies, illicit and prescription drugs, and infections. Anterior urethral injury is a much more
plausible explanation for the hematuria affecting the boy in the vignette given his history and
physical examination findings.
Scrotal trauma may occur as a result of straddle injuries, and injuries can range from minor
scrotal/testicular contusions to complete testicular rupture (a surgical emergency). Testicular
contusion may present with tenderness to palpation over the scrotum, scrotal edema, and
ecchymosis. The boy in the vignette has no tenderness on palpation of his testicles and has blood
at the urethral meatus that would not be explained by a testicular contusion.
PREP Pearls
• The presence of blood at the urethral meatus is highly suggestive of a urethral injury.
Other clinical indicators may include voiding difficulty, scrotal hematoma or perineal
ecchymoses, and displacement of the prostate on rectal examination in males.
• Retrograde urethrography is the gold standard study for the diagnosis of urethral injury.
• A urinary catheter should never be inserted in patients with suspected urethral injuries
until retrograde urethrography is performed to assess for an intact urethra.
Suggested Readings
• American College of Surgeons Committee on Trauma. Abdominal and pelvic trauma.
Advanced Trauma Life Support Student Course Manual. 9th ed. Chicago, IL: American
College of Surgeons; 2012.
Question 184
You are caring for a 16-year-old adolescent boy with advanced progressive muscular dystrophy.
He is currently intubated and sedated because of respiratory failure from pneumonia. This is his
fourth episode over the past year. Prior to this event, given the progressive nature of his illness,
his parents and primary care physician discussed end-of-life care with him. The patient indicated
his desire for withdrawal of life-sustaining treatment if he were to develop respiratory failure
again. Per his wishes, the endotracheal tube is removed. After 1 minute, he develops tachypnea
and gasping respirations.
Palliative care is the practice of integrating medical, spiritual, and psychosocial aspects of care
when making medical decisions based on quality of life for a patient in a family-centered
manner. Palliative care should begin for all children with progressive medical conditions early
enough in the course of disease so that rational decisions can be made with as little stress as
possible. As the practice of palliative care expands, primary care physicians will likely play a
more integral role. An important aspect of palliative care is defining the goals of end-of-life care
for a terminally ill child. Because of repeated respiratory infections from his progressive
neuromuscular condition, the child in the vignette prepared an advance directive indicating his
desire for withdrawal of life-sustaining therapies in the case of a subsequent episode of
respiratory failure. The principle of autonomy, or respect for persons, provides that an
appropriately mature 16-year-old child should ideally determine important decisions such as end-
of-life care. Although the decision-making power legally lies with his parents, every effort
should be made to incorporate the child’s wishes into such decisions.
Presumably, for the child in the vignette, open discussions occurred and decisions were
appropriately made in a relatively low-stress setting. A “do-not-resuscitate” order may be
defined in the outpatient setting, and family members can direct caregivers accordingly. It is not
uncommon for families to initially choose mechanical ventilation for a child like the boy in the
vignette, in case the cause of acute respiratory failure is quickly reversible. However, a
chronically ill or weak child often does not then wean from mechanical support easily. Terminal
extubation, which involves removal of an endotracheal tube from a ventilator-dependent patient,
is a reasonable option in such cases. For the boy in the vignette, because terminal extubation has
been chosen and the condition is not likely reversible, noninvasive positive pressure ventilation
would not be appropriate.
In terminal extubation, the patient and family should be informed that it is uncertain how long it
will take for the child to die. Opioids and/or benzodiazepines should be readily available, at
doses appropriate for the intent to treat symptoms of distress, as opposed to intentionally causing
or hastening death. Although these medications may in fact hasten death, the “doctrine of double
effect” provides that if a medication has 2 effects, 1 positive (treating symptoms) and the other
negative (hastening death), the intent of the positive effect can be honored. This distinction
should also be made clear to the family. Although it is practiced in some countries, euthanasia is
not widely accepted in the medical community.
For the boy in the vignette, administration of 5 mg/kg of intravenous morphine would be an
intentional overdose and would represent euthanasia. Administration of vecuronium without
adequate sedation is inhumane in any setting, and in this setting would also constitute euthanasia.
Comfort care should also be provided, but pharmacologic measures to relieve gasping are more
likely to be effective.
PREP Pearls
• It is ethically acceptable to treat symptoms of respiratory distress in a dying patient even
if it may hasten death because of the “doctrine of double effect.”
• Narcotics and benzodiazepines at standard doses are effective medications to treat the
discomfort of respiratory distress in dying patients.
• Palliative care should be incorporated early in the care of children with chronic or
progressive medical conditions, and ideally under minimally stressful conditions.
Suggested Readings
• Kang TI, Munson D, Hwang J, Feudtner C. Integration of palliative care into the care of
children with serious illness.Pediatr Rev. 2014;35:318. DOI:
http://dx.doi.org/10.1542/pir.35-8-318.
Question 185
A 16-year-old adolescent presents to the office for evaluation of a sore throat after the start of
school. She has been febrile and has discomfort with swallowing. Vital signs show a temperature
of 37.8°C, respiratory rate of 16 breaths/min, heart rate of 88 beats/min, blood pressure of 117/65
mm Hg, and a weight of 54 kg. On physical examination, she has erythema, edema, and exudates
of both tonsillar pillars, tender bilateral anterior cervical lymphadenopathy, and a scarlatiniform
rash. Laboratory data shows:
• White blood cells, 12,500/µL (12.5 x 109/L)
• Hemoglobin, 11.5 g/dL (115 g/L)
• Platelets, 320 x 103/µL (320 x 109/L)
• Differential, 1% segmented neutrophils, 18% bands, 78% lymphocytes, 2% monocytes,
1% eosinophils
• Rapid “strep” test, negative
A. age of patient
C. pharyngeal exudates
D. scarlatiniform rash
E. time of year
PREP Pearls
• It is not possible to distinguish pharyngitis caused by Streptococcus pyogenes from that
caused by Arcanobacterium haemolyticum on clinical grounds.
• While A haemolyticum can grow on the commonly used sheep blood agar, colonies are
small, have narrow hemolysis, and can be missed by laboratory personnel.
• Erythromycin is the drug of choice for treating A haemolyticum pharyngitis.
Suggested Readings
• American Academy of Pediatrics. Arcanobacterium haemolyticum infections. In:
Kimberlin DW, Brady MT, Jackson, MA, Long SS, eds. Red Book: 2015 Report of the
Committee on Infectious Diseases. 30th ed. Elk Grove Village, IL: American Academy
of Pediatrics; 2015:246-247.
Question 186
You are evaluating a 15-year-old adolescent in the emergency department (ED) for severe
headache. He has been complaining of headaches for the last year. You note 3 ED visits in the
last 6 months for headaches. His evaluation includes normal blood urea nitrogen, creatinine,
electrolytes, thyroid function tests, urinalysis, and head computed tomography. His vitals show a
temperature of 37˚C, heart rate of 120 beats/min, respiratory rate of 18 breaths/min, and a blood
pressure of 154/96 mm Hg. You make note that on his previous ED visits, his heart rate has
ranged 120 to 130 beats/min, and blood pressure has ranged from 149/84 to 166/92 mm Hg. His
physical examination is only significant for sweaty palms. You discuss the diagnostic evaluation
for this patient’s signs and symptoms with your residents.
Of the following, the MOST appropriate next test for evaluating this patient is
A. abdominal ultrasonography
The patient in the vignette has hypertension, tachycardia, and sweating, which is suggestive of
pheochromocytoma. His prior histories of recurrent emergency room visits are indicative of the
paroxysmal nature of the symptoms. Subsequent laboratory evaluation in this patient should
focus on identifying excess catecholamine secretion (plasma-fractionated metanephrines) as the
underlying cause of the patient’s symptoms.
Plasma fractionated metanephrines have a high sensitivity (> 95%) and low specificity (85%-
89%) for diagnosing pheochromocytoma. In patients with high pretest probability of
pheochromocytoma in the presence of clinical symptoms or strong family history, plasma
fractionated metanephrines is the initial diagnostic test. A 24-hour urine collection for
fractionated catecholamines and metanephrines is reported to have the highest sensitivity (98%)
and specificity (98%) for diagnosing pheochromocytoma. Urinary fractionated catecholamines
and metanephrines should be the first test in patients with low pretest probability for
pheochromocytoma (incidental adrenal mass without imaging characteristics consistent with
pheochromocytoma). In younger children in whom obtaining a complete 24-hour urine collection
is difficult, plasma fractionated metanephrines is the preferred initial test.
Total urinary metanephrines and vanillylmandelic acid on a spot sample have poor diagnostic
sensitivity and specificity, compared with fractionated plasma metanephrines, and are not
indicated as the next step for this patient.
American academy of pediatrics 602
American Academy of Pediatrics PREP 2016
PREP Pearls
• The classical symptoms associated with pheochromocytoma include paroxysmal
headache, sweating, and tachycardia, associated with elevated blood pressures.
• Measurement of fractionated metanephrines in urine or plasma is considered the most
sensitive biochemical test for evaluating patients suspected of having
pheochromocytoma.
• A 24-hour urine collection for fractionated catecholamines and metanephrines has the
highest sensitivity and specificity for diagnosing pheochromocytoma.
• Computed tomography and magnetic resonance imaging with contrast are preferred to
ultrasonography for identifying and localizing pheochromocytomas.
Suggested Readings
• Edmonds S, Fein DM, Gurtman A. Pheochromocytoma. Pediatr Rev. 2011;32(7):308-
310. DOI:http://dx.doi.org/10.1542/pir.32-7-308
Question 187
A 4-year-old boy is brought to the emergency department by his father after he found an empty
bottle of acetaminophen in his bedroom. His father is uncertain how many pills were in the
bottle. En route to the hospital, the child began having nonbloody and nonbilious emesis. His
father reports no chronic medical issues and no ill contacts. He is admitted for monitoring and
management. Physical examination 24 hours after admission showed a mildly jaundiced child.
The liver edge is palpable 2 cm below the right costal margin. Laboratory results obtained at
admission and at 12, 24 and 48 hours after admission are shown in Item Q187.
Which of the following 24-hour follow-up laboratory finding is MOST consistent with
impending liver failure in this child?
A. serum ammonia
B. blood glucose
C. aspartate aminotransferase
D. fibrinogen levels
The Pediatric Liver Failure Study Group defines ALF in children as:
1. Biochemical evidence of liver injury
2. No history of chronic liver disease
3. Coagulopathy not corrected by vitamin K
4. International normalized ratio greater than 1.5 in a patient with hepatic
encephalopathy or greater than 2.0 if no hepatic encephalopathy
Hepatic encephalopathy (HE) is difficult to assess in children, but is critical to the identification
of pending acute liver failure. There is a scale to assist with identifying the stages of HE (Item
C187).
The 4-year-old boy in the vignette has ALF caused by acetaminophen toxicity. Impending
hepatic failure is identified by increasing coagulopathy. Patients with liver failure will have
hypoglycemia caused by failure of synthesis and release. Ammonia levels are elevated associated
with increased glutamine and poor clearance. Fibrinogen levels are more often low, associated
with disseminated intravascular coagulation. Transaminases are elevated in this child because of
the acetaminophen toxicity and associated hepatitis. Elevated transaminases are associated with
inflammation and are not a good predictor of liver failure.
PREP Pearls
• Evidence of coagulopathy is a danger sign of impending acute liver failure.
• Acute liver failure is associated with hepatic encephalopathy, which is defined as having
4 stages.
• Acute liver failure is associated with hypoglycemia.
• The etiology of 50% of acute liver failure in children is idiopathic.
Suggested Readings
• Clemente MG, Schwarz K. Hepatitis: general principles. Pediatr Rev. 2011;32(8):333-
340. DOI:http://dx.doi.org/10.1542/pir.32-8-333.
Question 188
A 9-month-old infant diagnosed with achondroplasia presents to your office for a health
supervision visit. The parents also want to discuss recurrence risk of the achondroplasia for
future pregnancies. Both parents have achondroplasia.
Of the following, recurrence risks for achondroplasia for future live offspring of these parents is
BEST estimated at
A. 25%
B. 50%
C. 66%
D. 75%
E. 100%
Approximately 80% of individuals with achondroplasia have parents with average stature with
the mutation due to a de novo gene mutation unique to that individual. With a de novo gene
mutation, the parents would have a low risk of having another affected child. Twenty percent of
individuals with achondroplasia have at least 1 affected parent. An individual with
achondroplasia whose reproductive partner has normal stature would have a 50% risk of having a
child with achondroplasia (Item C188B).
When both parents have achondroplasia, the risk to the offspring is 50% for having a child with
achondroplasia, 25% of having a child with average stature and unaffected, and 25% for having a
homozygous achondroplasia conception, which is a lethal condition and would not result in a
live birth. Therefore, in the case vignette, given the 25% chance of an intrauterine lethal
condition, it would be a 66% (or two-thirds) chance of having a liveborn neonate with
achondroplasia (Item C188C).
PREP Pearls
• Achondroplasia, the most common skeletal dysplasia resulting in disproportionate small
stature, is an autosomal dominant condition caused by FGFR3 gene mutation.
• Individuals affected with achondroplasia have short arms and legs, macrocephaly, frontal
bossing, and midfacial retrusion. Normal intelligence and life span are expected, though
hypotonia in infancy can delay motor milestones.
• When both parents have achondroplasia, the risk to the offspring is 50% for having a
child with achondroplasia, 25% of having a child with average stature and unaffected,
and 25% for having a homozygous achondroplasia conception, which is a lethal condition
and would not result in a live birth.
Suggested Readings
• Pauli RM. Achondroplasia.
GeneReviews.http://www.ncbi.nlm.nih.gov/books/NBK1152/#achondroplasia.Genetic_C
ounseling.
• Trotter TL, Hall JG. Health supervision for children with achondroplasia. Pediatrics.
2005;116(3):771-783. DOI:http://dx.doi.org/10.1542/peds.2005-1440.
Question 189
This is the first health supervision visit for a healthy 12-month-old girl to your practice. When
you ask for a copy of the infant’s immunization record, the mother becomes anxious and states
that she doesn’t believe in immunizations. She adds that she has a niece with autism, so she has
no intention of allowing her own children to receive vaccines.
Despite one’s best efforts to educate patients and their families about the effectiveness of
vaccines, the incidence of adverse events related to vaccines, and the potential for true morbidity
and mortality associated with natural disease, some will decline vaccination. When this occurs, it
is recommended that the practitioner document the discussion, and request that the parents sign a
waiver affirming their decision not to vaccinate. The American Academy of Pediatrics (AAP)
has made available a “Refusal to Vaccinate” form which can be placed in the patient’s medical
record.http://www2.aap.org/immunization/pediatricians/refusaltovaccinate.html
Vaccines may not be administered without consent. Parents have the right to make informed
decisions for their children, so refusal to vaccinate is not considered medical neglect in healthy
children and should not be reported to child protective services. In general, it is recommended
that physicians continue to care for the patient and family. However, if the practitioner is too
uncomfortable to continue care, there is still a legal and ethical obligation to not abandon the
patient. The family must be given reasonable notice of the intent to terminate the relationship and
ample opportunity to arrange for alternative medical care.
PREP Pearls
• Practitioners should listen to concerns about immunizations, clarify and reaffirm accurate
beliefs, and correct any misconceptions.
• Open and ongoing discussions regarding vaccine concerns should be held in an effort to
partner with families and encourage vaccination.
American academy of pediatrics 612
American Academy of Pediatrics PREP 2016
• Parents have the right to refuse to vaccinate their children, in which case it is
recommended that the provider request the parents to sign a waiver.
Suggested Readings
• American Academy of Pediatrics. Clinical report: reaffirmation: responding to parents
who refuse immunization for their children. Pediatrics. 2013;131(5):e1696. DOI:
http://dx.doi.org/10.1542/peds.2013-043
Question 190
A 14-month-old boy whose parents recently emigrated from India presents to your office for a
health supervision visit. The boy is at the third percentile for weight and for height. He is pale
and his liver is palpable 3 cm below the costal margin. His parents bring with them laboratory
results from India that are shown in Q190.
C. iron overload
D. stroke
The complications associated with chronic iron overload include endocrinopathies such as
hypothyroidism, diabetes, hypogonadism, cardiomyopathy, and liver failure. Iron overload can
be managed or even prevented through the use of aggressive chelation therapy. The most
commonly used chelator is deferasirox, an oral, once a day medication. Some patients are unable
to be adequately chelated with deferasirox and require the subcutaneous or intravenous
administration of deferoxamine to maintain iron balance. The gold standard for assessing iron
overload is liver biopsy, although newer techniques using specially calibrated magnetic
resonance imaging are increasingly used. Although iron overload can be clinically monitored
through measurement of serum ferritin, the serum ferritin level may be affected by inflammation
and can vary widely. Magnetic resonance imaging of the liver and heart, calibrated with the
appropriate programing (T2*), can be used for noninvasive monitoring and quantification of iron
overload in centers where this technology is available.
Acute chest syndrome and stroke are complications of sickle cell disease and do not occur in β-
thalassemia major. Iron overload is the complication associated with frequent transfusions of
PRBC, not iron deficiency. Leg ulcers are a complication of diabetes mellitus or sickle cell
anemia, not thalassemia major.
PREP Pearls
• The absence of hemoglobin A (α2 ß2) on hemoglobin electrophoresis with only
hemoglobin A2 (α2 δ2) and F (α2 γ2) present is diagnostic of ß-thalassemia major.
• Thalassemia major is managed with chronic red blood cell transfusions, and the resultant
iron overload is the most common complication of its therapy.
• Iron overload causes endocrinopathies, hepatic dysfunction, and cardiomyopathies.
• Complications of iron overload and death and can be prevented through aggressive
chelation.
Suggested Readings
• Bell MD. Consultation with the specialist: red blood cell transfusions. Pediatr Rev.
2007;28(8):299-304. DOI:http://dx.doi.org/10.1542/pir.28-8-299.
Question 191
You are seeing an 11-year-old boy with a complaint of intermittent vision loss. He says that his
vision loss occurs suddenly in both eyes, lasts for a few minutes, and then resolves
spontaneously. He saw an optometrist 2 weeks ago and the family was told that there appeared to
be nothing wrong. He has no other physical complaints, including no headaches. His parents note
that there has been a significant amount of stress in the home recently. He often helps to care for
an ailing grandparent who is living with the family. Although his mother seems worried about
his complaints, the boy himself seems unconcerned. He has missed about 3 weeks of school over
the last 2 months because of this complaint. His parents both have a history of chronic medical
symptoms that limit their ability to work.
Despite the very low likelihood of finding any biological abnormalities on physical examination,
the best next step in care would be to perform a physical examination complete with extra
attention to the neurological system. Performing this examination serves to show that the boy has
been noticed and taken seriously, and shows the family that you are not brushing them off. Once
a provider has performed a thorough physical examination and can reassure the family that no
physical abnormalities have been found, psychosocial issues can be addressed further.
Delivering an "it's all in your head" message is usually counterproductive, as it may lead patients
or families to insist on more testing and more specialists to prove they were right.
Instead, the principles of treatment are to:
1. provide reassurance that the child appears healthy, as you cannot find evidence of
anything serious or permanent going wrong with his body
2. create a positive expectation that the symptom will improve in time on its own, that
things like "stress" can cause unusual symptoms in the body and that these symptoms
are time-limited
3. address any stress in his life either through household changes, psychological therapy,
or both
4. discourage any situations that lead to secondary gain from his symptomatic
complaints
You would like this family to get the message that despite what he may still intermittently say he
experiences that his life should go on as normal.
Ordering a brain magnetic resonance image, while not physically harmful, would be very
expensive and is medically unnecessary. Performing a lumbar puncture is an invasive and
stressful procedure, for which there is no clinical indication in this case. Referring the family for
family counseling might be an appropriate option if family dysfunction is discovered to be a core
problem. As family dysfunction has not been revealed to be the cause of his problems, a family
therapy recommendation is unlikely to be well received or pursued by the family at this time.
Referring him to psychiatry for having a conversion disorder without first performing a physical
examination is likely to be counterproductive. This is because the family may get stuck in a "the
doctor didn't believe me/take me seriously" situation. After the medical evaluation has been
performed, followed by the delivery of an assessment and recommendations, the family may
American academy of pediatrics 618
American Academy of Pediatrics PREP 2016
decide that they would like help in managing the patient's anxious or stressful adjustment
reactions and then can be referred to psychiatry.
PREP Pearls
• It is important not to label obvious conversion disorder symptoms as psychiatric without
first performing a thorough physical examination and making an interpersonal connection
about taking their complaints seriously.
• Medical and laboratory tests performed in the setting of a conversion disorder are
monetarily wasteful and do not provide any lasting reassurance to the patient or family.
• Extensive testing may promote, rather than alleviate, anxiety in the patient and family.
Suggested Readings
• Mink JW. Conversion disorder and mass psychogenic illness in child neurology. Ann NY
Acad Sci. 2013;1304:40-44. DOI:http://dx.doi.org/10.1111/nyas.12298.
Question 192
A 5-year-old boy is admitted to the hospital with a 2-week history of intermittent fever. He has
been having trouble walking today and is complaining of pain in his left knee. He is alert and
well perfused. His heart rate is 120 beats/min, respiratory rate is 18 breaths/min, and blood
pressure is 100/65 mm Hg. On physical examination, you note that he has conjunctival
petechiae, but no conjunctivitis or rhinorrhea. He has small, nontender lymph nodes in the
anterior cervical chain; he has no jugular venous distension. His neck is supple. His oropharynx
is clear, his chest is clear, his cardiac examination shows a regular rate and rhythm, and a normal
S1 and S2 with a 2/6 systolic murmur at the left mid-axillary line and the fourth intercostal
space. The abdominal examination shows a palpable spleen, but no hepatomegaly. The femoral
pulses are 2+. His left knee is swollen, erythematous, and warm to the touch. He is not able to
straighten his leg. He has not had any medications.
White blood cell count is 10,500/µL (10.5 x 109/L), 80% neutrophils, 18% lymphocytes, 2%
monocytes
Hemoglobin, 13 g/dL (130 g/L)
Hematocrit, 39%
Platelet count, 390 x 103/µL (390 x 109/L)
Erythrocyte sedimentation rate, 120 mm/h
Of the following, the MOST important diagnostic test that should be performed to evaluate the
etiology of this patient’s symptoms is
The Duke criteria for IE are shown in Item C192A and Item C192B. Clinical signs of IE include
fever, splenomegaly without hepatomegaly, conjunctival hemorrhages, a new murmur in the
mitral position, retinal and splinter hemorrhages, Osler nodes (tender lesions on the pads of the
fingers and toes), and Janeway lesions. The child in this vignette does not have signs of
congestive heart failure or a murmur to suggest severe valvular regurgitation. His laboratory
work is significant for leukocytosis and elevated erythrocyte sedimentation rate.
The patient described in this vignette may have 1 major criterion, if the mitral regurgitation
murmur is new, and 1 minor criterion, conjunctival hemorrhages, and would therefore be in the
category of possible IE. Multiple blood cultures are very important in being able to make a firm
diagnosis. The number of blood cultures needed and the volume of blood needed in each age
group varies. In a large meta-analysis in the adult population (23,313 patients), 2 sets of 3 blood
culture bottles (each 10 mL samples) done within 30 min was as effective as 3 sets of blood
cultures. In this setting, 2 aerobic and 1 anaerobic cultures were found to increase the yield, and
allowed for declaration of a positive blood culture when 2 were required to rule out a
contaminant. The most frequent pathogens were Staphylococcus aureus, Escherichia coli,
Klebsiella pneumoniae, Enterococcus, and coagulase-negative Staphylococcus. This volume of
blood is not realistic in younger pediatric patients.
Recommendations for infants and children have been made based on age and weight. Priority is
placed on aerobic cultures in the smaller infants:
For premature infants less than 1 kg, 1 blood culture with 2 mL of blood
For infants between 1.1 kg and 2 kg, 2 sets of blood cultures each 2 mL
For infants 2.1 to 12.7 kg, 1 set of 4 mL and 1 set of 2 mL
For children 12.8 to 36.3 kg, 2 sets of 10 mL each
For children over 36.3 kg, 2 sets of 20 to 30 mL, each which corresponds to the adult
recommendations
In summary, clinical findings are a crucial part of diagnosis in IE. Patients at high risk deserve
close scrutiny for physical signs of endocarditis, especially changes in their cardiac examination
such as a new regurgitant murmur in the setting of febrile illness. The mortality for infective
endocarditis is significant. Multiple blood cultures with adequate volume of blood are more
sensitive in making the diagnosis than an echocardiogram. The echocardiogram will be crucial in
the decision to obtain surgical consultation.
PREP Pearls
• Blood cultures are more sensitive in making the diagnosis of endocarditis than the
echocardiogram.
• A new regurgitant murmur, when present, is the most significant physical finding in
endocarditis.
• Recognize the clinical findings associated with infective endocarditis and provide
appropriate initial management
Suggested Readings
• Baron EJ, Miller JM, Weinstein MP, et al. A guide to utilization of the microbiology
laboratory for diagnosis of infectious diseases:2013 recommendations by the Infectious
Diseases Society of America (IDSA) and the American Society for Microbiology
(ASM)(a). Clin Infect Dis. 2013;57(4):e22-e121. DOI:
http://dx.doi.org/10.1093/cid/cit278.
• Gerwitz M, Tawburt K. Infective endocarditis and prevention. In: Allen HD, Driscoll DJ,
Shaddy RE, Feltes TF, eds. Moss and Adams’ Heart Disease in Infants, Children, and
Adolescents: Including the Fetus and Young Adult. 8th ed. Philadelphia, PA: Lippincott
Williams & Wilkins; 2012:1363-1376.
• Li JS, Sexton DJ, Mick N, et al. Proposed modification to Duke criteria of diagnosis of
infective endocarditis. Clin Infect Dis. 2000;30(4):633-638. DOI:
http://dx.doi.org/10.1086/313753.
• Patel R, Vetter EA, Harmisen WS, Schleck CD, Fadel HJ, Cockerill FR III. Optimized
pathogen detection with 30 compared to 20-milliliter blood culture draws. J Clin
Microbiol. 2011;49(12):4047-4051. DOI: http://dx.doi.org/10.1128/JCM.01314-11.
• Prendergast B, Tornes P. Valvular heart disease: changing concepts in disease
management: surgery for infective endocarditis: who and when? Circulation.
2010;121(9):1141-1152. DOI:
http://dx.doi.org/10.1161/CIRCULATIONAHA.108.773598.
Question 193
You are seeing an 8-year-old boy in your office for a health supervision visit. The child has been
developing normally. The mother mentions that he bruises easily, but has no other concerns. On
physical examination, the boy has smooth, velvety skin with several small bruises of various
ages. There are no petechiae. You are able to passively dorsiflex his fifth fingers past 90 degrees,
flex his thumbs to the forearms, and hyperextend his knees and elbows past 10 degrees. The boy
is able to place his palms flat on the floor with his knees fully extended.
Of the following, the clinical characteristic that would BEST support your suspected diagnosis is
C. occipital horns
E. skin laxity
Patients with EDS can have repetitive subluxations, sprains, and chronic joint pain. Muscle
hypotonia may cause a delay in motor development. Easy bruisability and bleeding are
associated with EDS. There are problems associated with fragile connective tissue such as poor
wound healing, hernias, cervical insufficiency, and rectal prolapse. Pregnant EDS patients are
considered high risk.
Preventive guidelines are available for patients with EDS including padding and protective
bandages for patients with skin disease, and a recommendation for closure of wounds in 2 layers
without tension. These patients may require longer periods for wound healing. If there is
significant bruising, contact sports should be avoided. Ascorbic acid supplementation may
decrease the bruising. If hypotonia, joint pain, or motor delay is present, physical therapy can be
beneficial. Joint pain associated with EDS and hypermobility may be managed with nonsteroidal
anti-inflammatory drugs (NSAIDs) and physical therapy, however, the use of NSAIDs may need
to be limited in patients with significant bruising. For patients with joint hypermobility, contact
sports and other sports that produce joint strain should be avoided, but other exercises such as
swimming may be beneficial for muscle strengthening. Echocardiography is recommended
before age 10 years and should be repeated periodically based on abnormalities found. Patients
with vascular subtypes should avoid contact sports, weight lifting, drugs that interfere with
platelet function, and invasive vascular surgery.
Long bone overgrowth is associated with Marfan syndrome. Occipital horns syndrome (OHS) is
characterized by wedge-shaped calcifications of the trapezius and sternocleidomastoid muscle at
the attachment to the occipital bone, lax skin, joint hypermobility, bladder diverticula, inguinal
hernias, and vascular tortuosity. Distal joint hypermobility associated with proximal muscle
contractures and muscle weakness is characteristic of Ullrich disease or scleroatonic muscular
dystrophy. In Ehlers-Danlos syndrome, the skin is hyperextensible and will quickly return to its
normal position when stretched. This is in contrast to cutis laxa and De Barsy syndromes, where
redundant skin hangs and slowly returns to its normal state. In these syndromes wound healing is
normal.
PREP Pearls
• Ehlers-Danlos syndrome has several subtypes, and typically presents with skin
hyperextensibilty, atrophic scarring, smooth velvety skin, and joint hypermobility.
• Ehlers-Danlos syndrome patients should be counseled regarding wound healing and
injury prevention.
• Echocardiography is recommended before age 10 years in patients with Ehlers-Danlos
syndrome and should be repeated periodically based on abnormalities found
Suggested Readings
• Callewaert B, Malfait F, Loeys B, Paepe AD. Ehlers-Danlos syndromes and Marfan
syndrome. Best Pract Res Clin Rheumatol. 2008;22(1):165-189. DOI:
http://dx.doi.org/10.1016/j.berh.2007.12.005.
Question 194
A 15-year-old female gymnast presents to your office for a health supervision visit. The
adolescent’s mother is concerned that she has not been sleeping well. The girl’s weight is at the
fifth percentile and height is at the 50th percentile. Her physical examination is unremarkable.
Her heart rate is 96 beats/min and her blood pressure is 128/76 mm Hg. You suspect that the
adolescent is using a performance enhancing drug.
A. androstenedione
B. coenzyme Q
C. erythropoietin
D. methylphenidate
E. phentermine
There is a common perception that athletes use performance-enhancing substances (PES) mainly
to enhance muscle bulk. Participants in sports such as football, body building, and wrestling,
where a muscular physique is perceived as advantageous, are likely to use anabolic steroids and
other compounds that promote weight gain and increased muscle mass. However, there are many
types of PES, and athletes may use medications and supplements for various reasons.
Performance-enhancing substances are medications or supplements that improve performance in
athletic activities or appearance. PES include medications taken for nontherapeutic purposes, or
in excess of therapeutic dose, and medications that cause weight loss or gain or alter body
composition. Supplements and vitamins, when used in excess of the recommended dose, and
substances taken to enhance oxygen-carrying capacity are other examples of PES. In addition,
some athletes use medications to prevent detection of another substance used to enhance
performance or appearance; these medications are also considered PES. Studies of children and
adolescents have demonstrated increased PES use over the past 2 decades. Increased youth
participation in sports and increased competitiveness of youth sports leagues are likely factors
influencing rates of PES use.
Androstenedione is a testosterone precursor with mild androgenic effects, though this substance
may have estrogenic effects in the presence of other, stronger androgens. Coenzyme Q is thought
to improve energy level and exercise tolerance, but is not commonly used by athletes and does
not have stimulant side effects. Abdominal discomfort is the most common side effect. Some
athletes use erythropoietin (EPO) to enhance oxygen-carrying capacity. Use of EPO can cause
hyperviscosity of the blood, which can lead to thrombosis, but would not cause the low weight or
sleep disturbance seen in the patient in this vignette. Phentermine is a stimulant and would have
effects that are similar to methylphenidate, but is much less commonly used by athletes.
PREP Pearls
• Participation in a sport that emphasizes a particular physique (eg, football, wrestling, or
gymnastics) is a risk factor for performance-enhancing substance use.
• Athletes participating in sports in which a lean physique is seen as desirable have an
increased rate of stimulant use to promote weight loss and to increase energy levels.
Suggested Readings
• AAP Committee on Sports Medicine and Fitness. Use of performance-enhancing
substances. Pediatrics. 2005;115(4):1103-1106. DOI: http://dx.doi.org/10.15
Question 195
A previously healthy 2-year-old boy presents to your office with several days of tactile fever and
nasal congestion interfering with sleep, clear rhinorrhea, and cough. On physical examination,
his temperature is 39°C, heart rate is 120 beats/min, and respiratory rate is 24 breaths/min. He is
fussy, rubbing his eyes and ears, but is consolable. His voice is hoarse and his posterior
oropharynx is erythematous without exudates. You note clear rhinorrhea and erythema of the
nasal mucosa. The tympanic membranes are mildly hyperemic and retracted. There is shotty,
bilateral cervical lymphadenopathy. Auscultation of the lungs reveals diffuse coarse upper
airway sounds. The remainder of the physical examination is unremarkable.
Of the following, the MOST appropriate therapy to recommend for this child is
A. acetaminophen
B. dextromethorphan
C. diphenhydramine
D. guaifenesin
E. pseudoephedrine
Decongestants, OTC antihistamines, antitussives, and expectorants are most commonly sold in
preparations containing multiple ingredients and often including antipyretics and analgesics.
Although symptomatic relief is the goal, evidence of efficacy in children is lacking. In young
children, the risk of adverse effects (eg, altered mental status, tachycardia, ataxia) is highest.
There have been a significant number of events of accidental overdose due to the combination of
ingredients in these OTC cough and cold products.
Viruses are the most common cause of upper respiratory tract infections in children (Item C195).
Clinical findings may include cough, congestion, sneezing, rhinorrhea, and fever during the
initial days of illness. The rhinorrhea may be clear or become yellowish-green within a few days.
In adults, illness typically lasts 5 to 7 days. Symptoms in children with upper respiratory tract
infection usually persist for at least 10 days, but lessen over time. The development of secondary
bacterial infection (eg, acute bacterial sinusitis, otitis media, pneumonia) is suggested by the
evolution or worsening of symptoms, especially fever, over several days.
PREP Pearls
• Over-the-counter cough and cold medications are of no benefit in young children with
upper respiratory tract infection.
• Over-the-counter cough and cold preparations can cause serious adverse effects in young
children.
• Viruses are the most common cause of upper respiratory tract infections in children.
• Rhinovirus causes most upper respiratory tract infections in children.
• Fever, cough, congestion, and rhinorrhea are the most common symptoms of upper
respiratory tract infection in children.
Suggested Readings
• American Academy of Pediatrics. American Academy of Pediatrics urges caution in the
use of over-the-counter cough and cold medicines.
http://www.generaterecords.net/PicGallery/AAP_CC.pdf.
Question 196
A 3-year-old, previously healthy girl presents to your office after she suddenly began crying and
complaining to her mother of burning in her mouth and throat. Just before these symptoms
began, the girl and her 4-year-old cousin were at home having a tea party with their dolls. The
girls had decorated their tea party table with some leaves taken from a houseplant in the room
(Item Q196).
Item Q196
Item 196. Houseplant. Reprinted with permission from the University of Illinois Extension.
On physical examination, the girl is nontoxic appearing and crying. She is afebrile and her vital
signs are within normal limits for her age. Her lips are slightly swollen and she is intermittently
drooling, although she was able to drink some water on the way to your office without difficulty.
Her lungs are clear and she is breathing comfortably without stridor or signs of respiratory
distress. Her skin is warm and dry without rash, and her extremities are well-perfused. The
remainder of your physical examination shows no other abnormalities. The girl takes no regular
medications and has no known allergies.
Of the following, the BEST next step in your management of this patient is
It is important for all pediatric providers to recognize the signs and symptoms that can arise from
ingestion of a toxic plant, and to understand the principles underlying appropriate management
of these ingestions. Children may be exposed to various potentially toxic plants, both within their
home environments and outdoors. Although these exposures rarely result in serious toxicity,
providers should be aware of the small number of plants that can have serious clinical effects
(Item C196).
Plant exposures account for about 5% of calls to US poison control centers annually, with the
vast majority affecting children younger than 6 years. Among small children, plant ingestions
generally occur inadvertently as children innocently explore their environments. Older children
and adolescents may ingest specific plants intentionally because of their psychoactive effects.
The specific plant causing symptoms for the girl in the vignette, philodendron, is a common
houseplant that causes mucous membrane irritation because of the presence of tiny needle-
shaped calcium oxalate bundles (raphides) throughout its structure. Other common houseplants
containing calcium oxalate raphides include dumb cane (Dieffenbachia), caladium, calla lily
(Zantedeschia), peace lily (Spathiphyllum), Jack-in the-pulpit (Arisaema atrorubens), and
elephant ear (Colocasia esculenta). When chewed, the calcium oxalate crystals in these plants
penetrate mucous membranes, causing immediate pain and inflammation, which is relatively
mild in most cases. Recommended treatment for symptomatic children is generally supportive,
and consists mainly of cleansing exposed skin and mucous membranes with water and/or mild
soap, administration of analgesics, and oral administration of cold fluids, popsicles, or ice.
For children presenting with compromise of the airway, breathing, or circulation after exposure
to any poisonous plant, advanced life support measures—including endotracheal intubation to
secure the airway when indicated—are paramount. In the United States, regional poison control
centers are invaluable resources that are available to clinicians at all times for guidance related to
toxic exposures, including exposure to potentially toxic plants. Pediatric providers may contact
their regional poison control center at 1-800-222-1222 for information related to signs and
symptoms that may arise from exposure to specific plants, as well as for guidance in managing
these exposures.
Oral diphenhydramine administration would not be the best initial step in management after
philodendron ingestion, because the pain and mucosal irritation caused by this plant arises from
microtrauma from calcium oxalate crystals and is not mediated by histamine. Diphenhydramine
may help to relieve pruritus in patients who have had toxic plant exposures, but pain is the main
symptom displayed by the girl in the vignette; oral analgesics would be the best next step in her
management.
Intramuscular epinephrine would not be recommended for treatment of the girl in the vignette,
because her symptoms are arising from mucosal irritation by calcium oxalate crystals in the
philodendron and not from a systemic allergic reaction.
For children with mild symptoms of ingestion of philodendron and related plant species who can
tolerate oral intake without difficulty, consultation with a gastroenterologist would not be
indicated.
PREP Pearls
• Philodendron is a common houseplant that causes mucous membrane irritation because
of the presence of tiny needle-shaped calcium oxalate bundles (raphides) throughout its
structure.
• Pediatric providers should be aware of those plants that can have serious toxic effects.
• Recommended treatment for children who are symptomatic after philodendron ingestion
is supportive, consisting of cleansing exposed skin and mucous membranes,
administration of analgesics, and oral administration of cold fluids, popsicles, or ice.
• Regional poison control centers are available at all times for guidance related to toxic
exposures, including exposures to potentially toxic plants.
Suggested Readings
• Carlson AG, Krenzelo EP. Poisonous plants. In: Erickson TB, Ahrens WR, Aks SE,
Baum CR, Ling LJ, eds. Pediatric Toxicology: Diagnosis and Management of the
Poisoned Child. New York, NY: McGraw-Hill; 2005:541-547.
Question 197
You are asked to assess a newborn, 30 minutes after delivery, for evaluation of tachypnea. The
newborn was delivered by vaginal delivery complicated by late decelerations and thick
meconium-stained amniotic fluid. He was vigorous at birth, therefore direct suctioning of the
trachea was not performed. He developed tachypnea and mild intermittent grunting 10 minutes
after delivery, which has persisted. Vital signs include a temperature of 37.2°C, heart rate of 160
beats/min, respiratory rate of 80 breaths/min, blood pressure of 60/38 mm Hg, and oxygen
saturation of 89% in room air. Physical examination is remarkable for subtle nasal flaring,
slightly diminished breath sounds on the right, and meconium staining of the nails and umbilical
cord. Arterial blood gas results show a pH of 7.21, PCO2 of 45 mm Hg, PO2 of 54 mm Hg,
bicarbonate of 15 mEq/L (15 mmol/L), and base deficit of –8. A chest radiograph is obtained
(Item Q197).
Item C197
Item C197. Moderate-sized right pneumothorax on plain radiograph. Reprinted with permission from Caldwell AL,
Goodpasture M, Alden M. Index of suspicion in the nursery. NeoReviews. 2010;11(4):e206 -e210.
Spontaneous pneumothorax in a healthy full-term neonate often goes undetected and resolves
without intervention. Physical examination findings suggestive of a pneumothorax include
tachypnea, grunting, and cyanosis with decreased breath sounds, chest wall asymmetry, and
shifting of the point of maximal cardiac impulse. The diagnosis is confirmed on chest
radiography, though transillumination may be performed in an emergent situation.
Symptomatic neonates with pneumothorax should be monitored closely for cardiorespiratory
compromise. If the neonate is hemodynamically stable, supportive therapy is appropriate.
Nitrogen washout therapy with 100% oxygen had been recommended until recently, because of
the belief that it would accelerate resolution. Newer studies have demonstrated that providing
just enough supplemental oxygen to maintain adequate oxygen saturation is equally effective and
does not carry the risks of hyperoxia.
American academy of pediatrics 642
American Academy of Pediatrics PREP 2016
The infant in the vignette has no evidence of hypovolemia or hypotension that would require a
normal saline bolus. Sodium bicarbonate is not indicated because the mild metabolic acidosis is
likely the result of perinatal events and will resolve over time without treatment.
PREP Pearls
• A spontaneous pneumothorax can be found in up to 2% of full-term neonates with no
lung disease.
• Physical examination findings suggestive of a pneumothorax include tachypnea,
grunting, and cyanosis with decreased breath sounds, chest wall asymmetry, and shifting
of the point of maximal cardiac impulse.
• Evidence of a tension pneumothorax including poor perfusion, hypotension, bradycardia,
increasing respiratory distress, or worsening metabolic acidosis requires immediate
drainage of the pleural air by needle thoracentesis or chest tube placement.
Suggested Readings
• Clark SD, Saker F, Schneeberger MT, Park E, Sutton DW, Littner Y. Administration of
100% oxygen does not hasten resolution of symptomatic spontaneous pneumothorax in
neonates. J Perinat. 2014;34:528-531. DOI:http://dx.doi.org/10.1038/jp.2014.55.
Question 198
A 3-year-old, previously healthy, fully immunized boy presents to the emergency department
after he had a seizure at home. He had a fever that morning and nonbloody diarrhea. The boy’s
mother reports that she gave him acetaminophen and some juice. When she laid him down for a
nap, his whole body stiffened and then started jerking rhythmically. This event lasted
approximately 2 minutes. Afterwards, he was limp and unresponsive. She called 911 and he was
brought by ambulance to the emergency department. On physical examination, his temperature is
38.9°C, blood pressure is 90/48 mm Hg, heart rate is 122 beats/min, and respiratory rate is 38
breaths/minute. He cries when you attempt to examine him, but is consoled by his mother. His
general physical and neurological examination results are unremarkable.
Of the following, the BEST next step in the management of this boy is to
A. obtain an electroencephalogram
Further diagnostic testing would be indicated if an underlying illness were clinically suspected.
For example, if there were clinical signs of meningitis such as persistent altered mental status or
meningismus, or signs of dehydration, then lumbar puncture or serum electrolytes should be
considered, respectively. If the seizure had a focal onset, such as starting on 1 side of the body,
then neuroimaging and electroencephalography would be appropriate. If there were postictal
hemiparesis or signs of increased intracranial pressure, such as gaze palsy or papillitis, then
computed tomography of the head would be the best test to perform. For a new focal seizure
without signs of increased intracranial pressure, magnetic resonance imaging of the brain is the
most sensitive test to detect a focal lesion.
PREP Pearls
• For a simple febrile seizure in a typically developing, previously healthy child, no further
diagnostic evaluation is needed.
• Magnetic resonance imaging of the brain is the most sensitive neuroimaging test for
causes of seizure, but computed tomography of the brain should be ordered if there is
urgency because of signs of increased intracranial pressure.
Suggested Readings
• AAP Subcommittee on Febrile seizures. Febrile seizures: guideline for the
neurodiagnostic evaluation of the child with a simple febrile seizure subcommittee on
febrile seizures. Pediatrics. 2011;127;389. DOI:http://dx.doi.org/10.1542/peds.2010-
3318.
Question 199
A full-term male 3.9-kg newborn develops jitteriness, tremors, and tachypnea 12 hours after
birth. The pregnancy was uncomplicated with no maternal history of infection or diabetes
mellitus. He is afebrile and his blood pressure is normal. His physical examination is
unremarkable. Serum glucose is 38 mg/dL (2.1 mmol/L). The remainder of his laboratory
evaluation, including electrolytes, is unremarkable.
Despite frequent feedings, hypoglycemia is recurrent and the newborn is started on dextrose
containing intravenous fluids. Over the next 48 hours, his glucose infusion rate approaches 15
mg/kg per minute to maintain glucose levels above 70 mg/dL (3.9 mmol/L). After 14 days,
intravenous fluids are still unable to be weaned without recurrence of hypoglycemia. At this
point, during a hypoglycemic event, the newborn is given glucagon 1 mg intravenously and, 20
minutes later, his glucose improves to 87 mg/dL (4.8 mmol/L). Serum ketones and free fatty
acids are not present.
The clues that PHHI is the cause of hypoglycemia in this case include the supraphysiologic
glucose requirements, as shown by the high glucose infusion rates and a positive response to
glucagon administration (a rise in glucose of over 30 mg/dL [1.7 mmol/L] after glucagon is
given). Additional diagnostic criteria include suppressed beta-hydroxybuterate (ketones) and free
fatty acids, and a measureable insulin level at the time of hypoglycemia. Infants with PHHI are
often large for gestational age. The diagnosis of PHHI is not usually considered until
approximately 2 weeks, because it can take that long for the aforementioned transient causes to
resolve.
The therapeutic goal for all hypoglycemic disorders is to maintain euglycemia, because a delay
in treatment confers a high risk of brain damage. Thus, in infants in whom PHHI is suspected, a
continuous glucose infusion should be rapidly established. Often a high glucose infusion rate is
needed, requiring a central line for venous access. The long-term treatment for PHHI is
diazoxide, an antihypertensive agent that opens the K-ATP channel of beta cells, thereby
decreasing insulin secretion. The recommended dose is 5 to 15 mg/kg per day, divided 2 or 3
times daily. Side effects include hypertrichosis and fluid retention.
A less commonly used treatment for PHHI is octreotide, which decreases insulin secretion by
hyperpolarization of the beta cells and inhibition of calcium channels. Octreotide is associated
with frequent treatment failure because of the development of tachyphylaxis. Recently,
octreotide was associated with the occurrence of necrotizing enterocolitis, therefore some centers
no longer recommend its use in neonates. For patients who fail medical treatment for PHHI,
pancreatectomy may be needed.
Other causes of hypoglycemia include panhypopituitarism, which can cause growth hormone
deficiency and/or adrenal insufficiency. Neither of these disorders should present with a
supraphysiologic glucose requirement; in such patients, the hypoglycemia does not resolve with
glucagon infusion as seen in the newborn in the vignette. Thus, administration of growth
hormone or hydrocortisone would not be the correct treatment for this patient.
Metabolic disorders including disorders of fatty acid synthesis, congenital disorders of
glycosylation, various congenital syndromes, glycogen storage diseases, and others may cause
hypoglycemia. Some of these disorders may respond to carnitine treatment. However, these are
unlikely diagnoses in this case, because they generally do not require supraphysiologic dosing of
glucose.
PREP Pearls
• Persistent hyperinsulinemic hypoglycemia of infancy (PHHI) is characterized by
hypoglycemia that persists beyond age 2 weeks, measurable insulin levels at the time of
hypoglycemia, suppression of free fatty acids and ketones, and a positive response to
glucagon administration (glucose rises > 30 mg/dL [1.7 mmol/L]).
• PHHI may require supraphysiologic glucose dosing.
• The recommended long-term treatment of PHHI is diazoxide, which opens the K-ATP
channel of beta cells, thereby decreasing insulin secretion.
• A delay in treatment of PHHI confers a high risk of brain damage. Ideally blood glucose
concentrations should be maintained in the normal range while undergoing diagnostic
evaluation.
Suggested Readings
• Lord K, De León DD. Monogenic hyperinsulinemic hypoglycemia: current insights into
the pathogenesis and management.Int J Pediatr Endocrinol. 2013;2013(1):3. DOI:
http://dx.doi.org/10.1186/1687-9856-2013-3.
• Mazor-Aronovitch K, Gillis D, Lobel D, et al. Long-term neurodevelopmental outcome
in conservatively treated congenital hyperinsulinism. Eur J Endocrinol. 2007;157(4):491-
497. DOI: http://dx.doi.org/10.1530/EJECopyright © 2016 American Academy of
Pediatrics. All rights reserved.
Question 200
A 9-year-old boy is brought for evaluation because of snoring nightly that wakes his parents in
the next room. He has been diagnosed with attention-deficit/hyperactivity disorder. He has
witnessed apneic periods, followed by gasping and partial awakening. He is tired and difficult to
awaken in the morning. He has nocturnal enuresis, which has not responded to bed alarms or
limitation of liquids prior to bed. On physical examination, he is morbidly obese with a body
mass index greater than the 98th percentile for age. His tonsils are 3+ and symmetric. He is sent
for a polysomnographic study, which reveals an apnea-hypopnea index of 30. The SpO2 nadir is
70%. Based on these results, the child is scheduled for tonsillectomy.
The parents ask about the clinical relevance of their son’s sleep apnea and the risks of
tonsillectomy.
Of the following, you are MOST likely to inform the parents that
C. the patient is at high risk for anesthesia-related complications and should be admitted
to the intensive care unit after tonsillectomy
D. their son will not require continuous positive airway pressure, as long as tonsils are
removed in a timely manner
The prevalence of OSA in the general pediatric population may approximate 6%. Boys are
affected at a higher rate than girls. Obesity is also a recognized risk factor. Snoring is frequent in
affected patients, but individuals with OSA should be differentiated from their counterparts with
habitual snoring but without obstructive events. Of note, patients with habitual snoring may be at
risk for development of OSA with age or with excessive weight gain. It is recommended that
clinicians inquire as to symptoms and sequelae of obstructive sleep apnea in all children who
habitually snore (snoring 3 or more nights per week) and that polysomnography (PSG) be
performed in children with suggestive symptoms or risk factors for OSA. In addition to snoring,
children with OSA may have witnessed apneic events; alternatively, they may gasp or snort with
frequent awakenings and disrupted sleep that leaves them poorly rested during the day.
Nocturnal enuresis is also associated with sleep disordered breathing and represents emptying of
the bladder during partial awakenings. Even with a thorough history, it is important to recognize
that clinical history and physical examination are often poor predictors of respiratory PSG
findings. The true prevalence of obstructive sleep disordered breathing is likely significantly
underestimated.
Associated physical findings in patients with OSA are numerous. The most common risk factor
is adenotonsillar hypertrophy. Other associated physical findings include obesity,
laryngomalacia, micrognathia, macroglossia, craniofacial abnormalities, dental malocclusion,
and hypotonia.
The functional symptoms and signs of sleep disordered breathing or OSA are variable, but
include deficits in cognitive and neuropsychological performance. Multiple studies have
demonstrated difficulties with attention, learning, executive function, and school performance in
affected children. Behavioral abnormalities have also been extensively described; the most
common manifestation is hyperactivity, but attention-deficit/hyperactivity disorder,
hypersomnolence, somatization, and depression have all been reported.
In children who are suspected as having OSA, an attended PSG study is recommended. An apnea
index of greater than 1 or an apnea-hypopnea index (AHI) of 1.5 is considered abnormal. For a
child who is 12 years of age or younger, surgical or medical treatment is generally favored in
patients with an AHI of 5 to 10 (mild-to-moderate OSA) or more than 10 (moderate-to-severe
OSA). It is recommended that children having either an SpO2 nadir of less than 80% on
preoperative PSG (or during recovery after surgical intervention) or an AHI greater than or equal
to 24 be admitted postoperatively because of an elevated risk for respiratory compromise.
Treatment options for OSA include adenotonsillectomy if there is confirmed OSA with evidence
of nasopharyngeal obstruction from adenoidal or tonsillar tissue. In children who are not surgical
American academy of pediatrics 650
American Academy of Pediatrics PREP 2016
candidates, positive pressure ventilatory support may be indicated in the form of continuous
positive airway pressure (CPAP) or bi-level positive airway pressure (bi-level PAP) during sleep.
As adherence and mid face growth defects may be problematic in children on chronic
noninvasive positive pressure ventilation (NIPPV), surgical intervention is warranted when an
anatomic abnormality is thought to be contributing to OSA.
PREP Pearls
• Health care providers should complete a thorough evaluation for sleep-related symptoms
and for sequelae of sleep disordered breathing (SDB) in children with habitual snoring.
• Obesity is a significant risk factor for SDB and obstructive sleep apnea (OSA).
• Adenotonsillar hypertrophy is a common anatomic risk factor for OSA and may
exacerbate OSA in obese individuals.
• Obesity is not a contraindication for adenotonsillectomy, but surgical management is
unlikely to result in full resolution of SDB and close follow-up is strongly recommended.
• Children with apnea-hypopnea index greater than or equal to 24 on polysomnography are
at elevated risk for anesthesia complications and should be monitored in an inpatient
setting after surgery.
Suggested Readings
• Costa DJ, Mitchell R. Adenotonsillectomy for obstructive sleep apnea in obese children:
a meta-analysis. Otolaryngol Head Neck Surg. 2009;140(4):455-460. DOI:
http://dx.doi.org/10.1016/j.otohns.2008.12.038.
Question 201
A 14-year-old adolescent fell while riding in a bicycle road race 3 days ago. He struck his head,
shoulder, and forearm, but he did not lose consciousness. He had a normal neurologic
examination in the emergency department on the day of the fall. You are seeing him for follow-
up. While discussing bicycle safety with you, he reports that his helmet hit the ground hard, but
there are only a few scratches on the plastic cover and the rigid foam component appears intact.
Of the following, the BEST advice to give this adolescent about bicycle helmets is
B. bicycle helmets should fit at the top of the forehead, and be tilted so that the occiput is
covered
Since 1999, the US Consumer Product Safety Commission (CPSC) has mandated safety
standards for bicycle helmets. Previously, the American National Standards Institute, Snell
Memorial Foundation, or the American Society for Testing and Materials endorsed voluntary
standards, and old helmets may carry this certification. However, all currently approved helmets
should have a sticker documenting CPSC approval, usually located on the inner liner. Bicycle
helmets can also be used for scooter-riding, roller and in-line skating, ice skating, and sledding,
but for other sports, different helmets are required in order to protect the participant from the
types of blows anticipated from those activities. The CPSC website provides a table listing the
appropriate helmet for each recreational activity (http://www.cpsc.gov/en/Safety-
Education/Safety-Guides/).
PREP Pearls
• Anticipatory guidance about bicycle helmet use should begin by the 2-year-old well
child visit.
• The US Consumer Product Safety Commission (CPSC) mandates safety standards for
bicycle helmets and a sticker documenting that approval should be found on any
helmet purchased.
Question 202
You are on a community health coalition along with staff from the local health department. A
recent health department report showed an increase in the chlamydia rate among teenagers.
Several coalition members insist that there should be a virginity pledge campaign in the local
schools to address this issue.
E. virginity pledges have been proven effective for the prevention of sexually
transmitted infections
The Community Preventive Services Task Force recommends comprehensive risk reduction
programs as an effective strategy to reduce adolescent pregnancy, HIV, and sexually transmitted
infections. Comprehensive educational programs include information about abstinence as the
best way to prevent pregnancy and sexually transmitted infections, but also include medically-
accurate information about contraception and condom use.
In the early 1990s, there were several virginity pledge programs that were designed to prevent
sexual risk taking behaviors among teenagers. Some studies have found that such pledges may
delay sexual initiation among some adolescents; however, other studies have found that
adolescents who broke their pledge used contraception less than their nonpledge peers.
Researchers have found that adolescents who received comprehensive sexuality education were
significantly less likely to report a pregnancy than those who received abstinence-only education.
The National Sexuality Education Standards provide guidance on the core content for
developmentally appropriate sexuality education for students in grades K through 12. These
standards, however, are not uniformly implemented.
Condom distribution programs have been shown not only to increase condom use, but also to
promote delayed sexual initiation among youth.
PREP Pearls
• The Community Preventive Services Task Force recommends comprehensive risk
reduction programs as an effective strategy to reduce adolescent pregnancy, HIV, and
sexually transmitted infections.
• Comprehensive educational programs include information about abstinence as the
best way to prevent pregnancy and sexually transmitted infections, but also include
medically-accurate information about contraception and condom use.
• The National Sexuality Education Standards provide guidance on the core content for
developmentally appropriate sexuality education for students in grades K through 12.
Suggested Readings
• Alford S, Advocates for Youth. Sex education programs: definitions & point-by-point
comparison. Advocates for Youth website.
http://www.advocatesforyouth.org/publications/publications-a-z/655-sex-education-
programs-definitions-and-point-by-point-comparison.
American academy of pediatrics 655
American Academy of Pediatrics PREP 2016
Question 203
A full-term newborn is jittery 3 days after delivery. The pregnancy was notable for limited
prenatal care, with the mother intermittently participating in a rehabilitation program because of
a history of substance abuse. The nurse describes the newborn as not feeding well with periods
of increased fussiness. On physical examination, the temperature is 37.3°C, heart rate is 140
beats/min, respiratory rate is 64 breaths/min, and blood pressure is 60/40 mm Hg. The newborn
has excoriations on the nose and chin, tachypnea without signs of distress, slightly increased
tone, and moderate tremors when disturbed. The blood glucose level is 60 mg/dL (3.3 mmol/L).
Maternal urine toxicology screening was positive for cocaine and tetrahydrocannabinol.
Of the following, the agent MOST likely to be related to the newborn’s clinical symptoms is
A. alcohol
B. cannabis
C. cocaine
D. methadone
E. methamphetamine
Many drugs used by mothers during pregnancy cross the placenta and affect the developing
fetus, including agents associated with substance abuse. The potential effects of these agents
include congenital malformations, prematurity, low birthweight, abnormal state regulation, and
neonatal withdrawal (Item C203B). Alcohol use is associated with congenital birth defects and
abnormal brain development, with the exposed fetus at risk for fetal alcohol spectrum disorder
(FASD). Congenital birth defects associated with FASD include cardiac defects (ventricular
septal defect, atrial septal defect), cleft palate, and renal anomalies (renal agenesis, renal
hypoplasia).
Newborns exposed to alcohol, barbiturates, and methamphetamines in utero may also display
symptoms of withdrawal. The severity of barbiturate withdrawal is similar to that seen with
opioids, and typically presents 4 to 7 days after birth. Withdrawal from alcohol or
methamphetamines begins within hours after birth. The symptoms of alcohol and
methamphetamine withdrawal are much less severe than those of opiate withdrawal. Although
no specific treatments exist for nonopioid withdrawal in infants, clinicians must consider
polysubstance exposure in newborns born to substance abusing mothers.
The physical examination findings of excoriations, tachypnea, increased tone, and moderate
tremors seen in the newborn in the vignette 3 days after delivery suggest opioid withdrawal.
Methamphetamine and alcohol withdrawal are less likely, as they typically occur shortly after
delivery. Although the urine toxicology screen of the mother in the vignette was positive for
cocaine and tetrahydrocannabinol, these agents are not typically associated with the classic
findings of neonatal abstinence syndrome.
PREP Pearls
• Newborns exposed to opioids, alcohol, barbiturates, or methamphetamines in utero are at
risk for symptoms of withdrawal.
• Neonatal abstinence syndrome (NAS) describes central nervous system, gastrointestinal,
and autonomic nervous system signs and symptoms seen in newborns with drug
withdrawal.
• Synthetic opioids, such as methadone, are not detected on many routine urine toxicology
screens.
Suggested Readings
American academy of pediatrics 658
American Academy of Pediatrics PREP 2016
Question 204
A 5-year-old, previously healthy boy presents to the emergency department with a 2-cm
laceration above his left eyebrow, sustained after hitting his head on a granite countertop. His
vital signs include a temperature of 37°C, heart rate of 140 beats/min, blood pressure of 90/50
mm Hg, respiratory rate of 30 breaths/min, and oxygen saturation of 100% on room air. He is
awake, alert, and anxious appearing. The repair requires sedation because of the boy’s age,
anxiety level, and proximity of the laceration to the eye. He was given a dose of intravenous (IV)
midazolam 0.1 mg/kg, but he remains awake, screaming, and thrashes his head when
approached. He is given a second dose of IV midazolam 0.1 mg/kg and falls asleep. His vital
signs after the second dose are a temperature of 37°C, heart rate of 90 beats/min, blood pressure
of 68/40 mm Hg, respiratory rate of 20 breaths/min, and oxygen saturation of 95% on room air.
Upon verbal stimulation, the boy opens his eyes, mumbles a few words, and falls back asleep.
His extremities are warm with a capillary refill time of 1 second.
Moderate sedation is often required for laceration repairs in children from toddler to early school
age, because of anxiety as well as pain. According to the American Academy of Pediatrics,
moderate sedation is a state of decreased level of consciousness with appropriate response to
physical stimulation or verbal commands. Monitoring should include continuous pulse oximetry,
visual assessment of ventilation, and noninvasive blood pressure measurement every 5 minutes.
For the boy in the vignette, the intended effect was not achieved after 1 dose of midazolam, as
evidenced by continued screaming and thrashing, therefore a second dose was given. Adverse
effects of benzodiazepines can include hypotension, bradycardia, and respiratory depression. The
following equation can be used to estimate minimum systolic blood pressure for a young child:
Systolic blood pressure = 70 mm Hg + (2 × child’s age in years)
The child in the vignette has a systolic blood pressure of 68 mm Hg, which qualifies him as
hypotensive. However, because he is not profoundly hypotensive, there is no indication of
significant overdose, and he has adequate clinical perfusion, the boy is not likely on the verge of
cardiovascular collapse.
PREP Pearls
• Benzodiazepines can cause hypotension by vasodilation, which decreases cardiac
preload and therefore cardiac output.
• Flumazenil is a reversal agent for benzodiazepines, but is not routinely recommended
because of rebound effects and a risk of seizures.
• Monitoring during moderate sedation should include continuous pulse oximetry,
visual inspection of breathing, and noninvasive blood pressure readings.
Suggested Readings
• American Academy of Pediatrics Committee on Drugs. Guidelines for monitoring
and management of pediatric patients during and after sedation for diagnostic and
therapeutic procedures: addendum. Pediatrics. 2002;110:836-838.
DOI:http://dx.doi.org/10.1542/peds.110.4.836.
Question 205
A 7-year-old, previously healthy Hispanic girl presents to your office for evaluation of daily
fever that has lasted 10 days. She has had associated night sweats, abdominal pain, and a 4 lb
weight loss. Her exposures include 2 dogs and 2 birds at home and consumption of unpasteurized
cheese from Mexico. Vital signs show a temperature of 39.5°C, respiratory rate of 22
breaths/min, blood pressure of 118/69 mm Hg, and a weight of 27 kg. Her physical examination
is unremarkable. Laboratory data shows:
• White blood cells, 4,400/µL (4.4 x 109/L)
• Hemoglobin, 11.5 g/dL (115 g/L)
• Hematocrit, 32.2%
• Platelets, 180 ×103/µL (180 x 109/L)
• Differential, 23% segmented neutrophils, 26% bands, 45% lymphocytes, 6%
monocytes
• Alanine aminotransferase, 55 U/L
• Aspartate aminotransferase, 59 U/L
• Urine analysis, negative
Of the following, the test that is MOST likely to establish the diagnosis is
A. abdominal ultrasonography
B. blood culture
C. chest radiograph
E. urine culture
Although there are various definitions of fever of unknown origin (FUO), some experts consider
FUO as fever of at least 8 days’ duration in which the diagnosis in not readily apparent after a
thorough history and physical examination. Of all etiologies, infections account for the largest
proportion of cases of FUO. In the United States, 100 to 200 cases of brucellosis are reported
annually. Children are estimated to account for up to one-third of all cases of brucellosis in
endemic countries, although underreporting is possible.
While abdominal ultrasonography may show enlargement of the liver and spleen, this would not
be a specific finding. Abdominal ultrasonography could be helpful in the diagnosis of
hepatosplenic cat scratch disease caused by Bartonella henselae, where microabscesses may be
visualized. However, this patient does not have feline exposure, making this a much less likely
possibility.
Pneumonia can be occult and should be considered in cases of FUO. Chest radiograph can
additionally help identify hilar lymphadenopathy and chest masses. However, this patient lacks
respiratory complaints or pulmonary findings on physical examination, making pneumonia less
likely. Additionally, given her exposure history, Epstein-Barr virus infection is less likely.
However, Epstein-Barr virus should be considered in cases of FUO, especially those with
transaminitis. Lastly, the negative urinalysis makes the likelihood of urinary tract infection,
diagnosed with urine culture, low.
PREP Pearls
• Indolent infections, such as Brucella, should be considered in the differential
diagnosis of fever of unknown origin, especially in the context of consumption of
unpasteurized dairy products. A blood culture can reveal the diagnosis in the acute
phase of infection.
American academy of pediatrics 664
American Academy of Pediatrics PREP 2016
Suggested Readings
• American Academy of Pediatrics. Brucellosis. In: Kimberlin DW, Brady MT,
Jackson MA, Long SS, eds. Red Book: 2015 Report of the Committee on Infectious
Diseases. 30th ed. Elk Grove Village, IL: American Academy of Pediatrics;
2015:268-270.
Question 206
A 7-year-old girl presents to the emergency department with complaints of “pain in her private
area” and “blood in her underwear.” According to the mother, the child slipped today while
playing on monkey bars. On physical examination, her vital signs show a temperature of 37.8°C,
heart rate of 70 beats/min, respiratory rate of 16 breaths/min, and blood pressure of 90/60 mm
Hg. Her physical examination is significant for a medium- to large-sized vulvar hematoma with
superficial abrasions (Item Q206).
Item Q206
Item Q206. Findings for the girl described in the vignette. Reprinted with permission from Johnson CF. Pediatr Rev.
2006; 27:17
You discuss the differential diagnosis of straddle injury versus sexual abuse with the medical
students.
Of the following, the MOST accurate statement regarding this patient’s presentation and clinical
findings is
C. sexual abuse is the likely explanation for the patient’s presentation and findings
A detailed history and physical examination is warranted in all cases of children with genital
trauma. The extent of perineal injury on examination should correlate with the history to confirm
nonsexual trauma as the cause of the patient’s injury. Caregivers of patients with accidental
genital trauma generally seek immediate medical attention. The verbal child can give a
supportive history of the accident and there may be witnesses (other family members) to the
event. Physical examination in the presence of a chaperone should be performed in a patient with
a history of perineal injury. In premenarcheal girls, the genital examination is best performed in
the supine (frog leg) and the prone (knee chest) position. The prone (knee chest) examination is
indicated in cases with suspected vaginal trauma. The examination may be difficult in a patient
with a recent history of trauma and pain from the trauma. In these cases, local anesthesia
(application of 2% lidocaine) or procedural sedation (such as in patients requiring sutures) is
indicated.
Vulvar hematomas and superficial lacerations (vulvar and vaginal) are the usual straddle injuries
in girls. Vulvar hematomas may vary in size following a straddle injury. The vulvar area in
young girls is highly vascular, with loose subcutaneous tissues increasing the risk for large
hematoma formation. Therefore, complaints of bleeding or blood stains on the underwear are
common in patients with straddle injuries and vulvar hematomas. Urinary retention may be
associated with vulvar hematomas and the physician needs to ensure proper voiding before
discharging the patient home. Patients with large hematomas and urinary retention may need
temporary bladder drainage. Most hematomas are usually managed conservatively with adequate
pain control, rest, ice packs, and tub baths. Patients are advised to rest on their side or use a foam
or air-filled rubber doughnut (while sitting) to avoid pressure injury of the swollen external
genitalia. Surgical intervention may be needed in patients with injury to the pelvic floor, urethra,
or increasing hematoma size despite adequate conservative management.
Straddle injuries may lead to unilateral and superficial lacerations of the vagina and vulva. The
patient in the vignette has lacerations in the right hymenal wall and fourchette. Identification of
vaginal lacerations from accidental trauma is important, as penetrating injuries (from sexual
abuse) are usually associated with vaginal lacerations in children. Bleeding from hymenal
injuries is often minimal and usually requires no treatment. Lacerations of the vaginal mucosa
may need suturing under anesthesia.
genital trauma, most children do not present until weeks or months after the abuse. In patients,
U- or V-shaped clefts (notches) of the posterior rim (from 3 o’clock to 9 o’clock), indicative of
healing after a laceration and attenuation or decreased width (less than 1 mm) of the posterior
hymen, are suggestive for underlying sexual abuse. It is important to note that only a small
percentage of sexually abused children have an abnormal genital or anal finding. Clinicians
should also be aware of the age-related hymenal changes and normal anatomic variations of the
hymen, which may be confused with features of sexual abuse. Midline sparing (linea
vestibularis), developmental variants (fenestrated hymen, failure of midline fusion), labial
adhesions, and dermatologic conditions such as lichen sclerosus and pemphigoid may be
confused with features of sexual abuse.
For the patient in the vignette, the presenting history, symptoms (of blood in the underwear), and
physical examination (vulvar hematoma and acute superficial lacerations) are consistent with
straddle injury.
PREP Pearls
• Vulvar hematomas and superficial lacerations are consistent with straddle injuries in
girls.
• Identification of vaginal lacerations from accidental trauma is important because
sexual abuse is commonly associated with vaginal lacerations.
• Only a small percentage of sexually abused children have an abnormal genital or anal
finding.
• A knowledge of the age-related hymenal changes and normal anatomic variations of
the hymen is important, as these may be confused with features of sexual abuse.
Suggested Readings
• Dowd MD, Fitzmaurice L, Knapp JF, Mooney D. The interpretation of urogenital
findings in children with straddle injuries. J Pediatr Surg. 1994;29(1):7-10.
Question 207
You are seeing a 12-year-old girl in follow-up. She has a 3-month history of worsening right
upper quadrant abdominal pain and bloody diarrhea. She reports 2 to 4 liquid, bloody bowel
movements daily. She also reports nausea and a 10 lb weight loss. She denies recent travel,
reptile exposure, or ill contacts.
A. abdominal ultrasonography
B. colonoscopy
D. enteroclysis
All gastrointestinal (GI) bleed evaluations should begin with assessment of vital signs and
hemodynamic stability with resuscitation as clinically indicated. A complete history and physical
examination follows stabilization and should include visualization of a stool sample. Testing for
fecal occult blood to assess for the presence of blood is essential whenever there is a concern for
American academy of pediatrics 670
American Academy of Pediatrics PREP 2016
GI bleeding. Stool cultures and Clostridium difficile toxin should be completed to evaluate for
infectious etiologies. A complete blood cell count (CBC) and coagulation studies will assess for
anemia and coagulopathy. If blood loss is active or ongoing, serial CBC studies may be needed
to determine need for more urgent interventions. Imaging studies may include an abdominal
radiograph to evaluate for foreign bodies and free air, and an upper gastrointestinal (UGI) series
to evaluate for anatomic abnormalities, including duplication cysts and abdominal
ultrasonography with Doppler to assess for portal hypertension. Tagged red blood cell scans and
angiography can be used, but are not effective at localization if the rate of blood loss is low, as is
typical in children. A Technetium-99 scan can identify a Meckel diverticulum. Endoscopy of the
GI tract can aid in both diagnosis and therapy, particularly when bleeding occurs from the upper
GI tract.
The vignette describes a child with a history and laboratory studies that are most consistent with
inflammatory bowel disease (IBD) without evidence of an acute abdomen. A colonoscopy will
be the best test to aid in the diagnosis. Abdominal ultrasonography, computed tomography of the
abdomen and pelvis, magnetic resonance imaging enterography, and enteroclysis would all
evaluate the GI anatomy and may identify inflammation, but will not confirm the diagnosis of
IBD.
PREP Pearls
• Gastrointestinal bleeding occurs at all ages.
• History and physical examination can give some clues to etiology.
• Testing for fecal occult blood should be used to confirm the presence of blood.
• Colonoscopy is used to confirm the diagnosis of inflammatory bowel disease.
Suggested Readings
• Dubinsky M. Special issues in pediatric inflammatory bowel disease. World J
Gastroenterol. 2008;14(3):413-420.http://www.wjgnet.com/1007-9327/14/413.pdf.
Question 208
A 4-year-old boy presents to your practice with developmental delays, unusual movements, and
cerebral palsy. He was a full term spontaneous vaginal delivery without complications during
pregnancy or delivery. Initial development was normal for the first few months, which then
slowed compared to his peers. The mother states he had low tone and delays noted by 6 months
of age. He currently is at the developmental level of a 1-year-old child. He can sit, but does not
walk. He has twisting movements and abnormal posturing of his extremities. Recently, he began
biting his fingers and lips, as well as head banging. The neurologic examination reveals dystonia,
spasticity, extensor plantar reflexes, and increased deep tendon reflexes. You suspect a genetic
disorder. You order a chromosomal microarray and a karyotype that are both unremarkable.
Due to the recent changes in behaviors, his neurologic examination, and history, the BEST next
test to aid in diagnosis in this child is
A. total homocysteine
C. urine mucopolysaccharides
D. urine porphyrins
Treatment is directed at controlling the uric acid production with allopurinol for the urate
nephropathy, gouty arthritis, tophi, and nephrolithiasis; however, this treatment has no impact on
the behavioral and neurologic symptoms of the disorder. Baclofen and benzodiazepines can be
used for the spasticity. Patients typically require therapies, including habilitative, behavioral, and
psychiatric therapy. Protective equipment may be necessary to reduce trauma secondary to self-
injurious behaviors.
Total homocysteine would be a good screening test for homocystinurias that are characterized by
a Marfanoid phenotype, developmental delay, ectopia lentis, severe myopia, tall stature, and
thromboembolism that are not consistent with this patient’s phenotype.
Urine mucopolysaccharides would be indicated in a child with developmental regression and
progressive coarsening of the facial features. Urine porphyrins should be obtained in a patient
with life-threatening acute neurovisceral attacks of severe abdominal pain, tachycardia,
hypertension, mental status changes, convulsions, peripheral neuropathy, and hyponatremia.
Very long chain fatty acids are an excellent screening test for peroxisomal disorders that present
with sensorineural hearing impairment, ocular abnormalities (retinopathy, cataracts, optic nerve
atrophy), developmental delay, and a classic dysmorphic appearance.
PREP Pearls
• Lesch-Nyhan syndrome is an uric acid metabolism disorder that presents with motor
dysfunction resembling cerebral palsy, behavioral disturbances, cognitive
impairment, self-injurious behaviors, and hyperuricemia.
• A pathognomonic clinical manifestation of Lesch-Nyhan disorder is self-injurious
behaviors.
• The best screening test is an urinary urate-to-creatinine ratio that should be greater
than 2.0 in a child who is younger than 10 years of age who has Lesch-Nyhan
syndrome. Hyperuricosuria and hyperuricemia (serum uric acid concentration > 8
mg/dL [476 µmol/L]), while often present, are not sensitive or specific enough to
confirm the diagnosis.
Suggested Readings
• Fu R, Ceballos-Picot I, Torres RJ, et al. Genotype-phenotype correlations in
neurogenetics: Lesch-Nyhan disease as a model disorder. Brain. 2014;137(Pt
5):1282-1303. DOI: http://dx.doi.org/10.1093/brain/awt202.
• Jinnah HA, Ceballos-Picot I, Torres RJ, et al. Attenuated variants of Lesch-Nyhan
disease. Brain. 2010;133(Pt 3):671-689. DOI:
http://dx.doi.org/10.1093/brain/awq013.
Question 209
A 1-month-old male infant is brought to your office by his mother for his first health supervision
visit. Pregnancy, labor, and delivery were uncomplicated. He was born appropriate for
gestational age at 39 weeks of gestation and was discharged at 3 days of age. He stayed 1 extra
day in the hospital for mild jaundice and delayed passage of his first stool. He is primarily
breastfed and has been gaining weight along the 25th percentile for his age. He passes stools
every 3 days and his mother thinks he may have constipation, but states that another pediatrician
assured her that this was normal for breastfed babies. The mother denies any significant family
history, though you note that she has iris heterochromia. On physical examination, the infant has
mild abdominal fullness, but no organomegaly or palpable masses. On digital rectal examination,
the anal canal is tight and a small amount of stool squirts out as you withdraw your finger from
the anus. The remainder of his physical examination is unremarkable.
Of the following, the BEST test to confirm the suspected diagnosis for this infant is
A. celiac panel
E. thyroid panel
Delayed passage of meconium, defined as beyond 48 hours after birth, can be indicative of a
serious problem such as bowel atresia or obstruction, imperforate anus, meconium plug, or
Hirschsprung disease. It is important for pediatric health care providers to recognize the potential
significance of this delay. Neonates who pass their first meconium stool after 48 hours but before
72 hours, and are otherwise well appearing, should undergo a thorough physical examination.
The conditions that may be commonly associated with delayed passage of meconium should be
carefully considered and excluded. These infants should be followed closely and evaluated
promptly if they develop symptoms of abdominal distention, bilious vomiting, or constipation.
Any neonate who fails to pass meconium within the first 72 hours after birth should be evaluated
for Hirschsprung disease.
Meconium ileus in the newborn is almost always caused by cystic fibrosis. These infants fail to
pass meconium stools, and often have marked abdominal distention with bilious emesis soon
after birth. Occasionally, infants with cystic fibrosis may have lesser degrees of meconium
impaction and a presentation that is more benign. The diagnosis of cystic fibrosis is made
American academy of pediatrics 676
American Academy of Pediatrics PREP 2016
through immunoreactive trypsinogen levels on the newborn screening test or sweat chloride
levels in the older infant or child.
Congenital hypothyroidism should be considered in infants who present with prolonged jaundice
and constipation. However, these infants will have additional signs and symptoms, which the
infant in this vignette did not have, such as lethargy, hypothermia, feeding problems, poor weight
gain, macroglossia, umbilical hernia, large fontanels, hypotonia, and dry skin.
PREP Pearls
• Passage of meconium occurring beyond the first 48 hours after birth is considered
delayed.
• Delayed or absent passage of meconium can indicate a serious problem, such as
bowel atresia or obstruction, imperforate anus, meconium plug, or Hirschsprung
disease.
• Neonates who pass their first meconium stool between 48 and 72 hours after birth,
but are otherwise well-appearing, should undergo a thorough physical examination,
with close follow-up and further evaluation performed as indicated.
• Any neonate who fails to pass meconium within the first 72 hours after birth should
be evaluated for Hirschsprung disease.
Suggested Readings
• Branski D, Dennis NR, Neale JM, Brooks LJ. Hirschsprung's disease and
Waardenburg's syndrome. Pediatrics. 1979;63(5):803-805.
http://pediatrics.aappublications.org/content/63/5/803 .
Question 210
An 18-month-old boy presents to the emergency department with a 3-day history of high, spiking
fevers and malaise. On physical examination, his temperature is 39.2°C, heart rate is 142
beats/min, and blood pressure is 90/48 mm Hg. He is alert and cranky, but consolable by his
mother. He is flushed, but has no rash. He appears well-hydrated. His liver and spleen are not
enlarged. He has no retractions or nasal flaring. His lungs are clear to auscultation. His complete
blood cell count results are shown:
White blood cell count 5,600/µL (5.6 x 109/L)
Hemoglobin 11.2 g/dL (112 g/L)
Platelet count 467 x 103/µL (467 x 109/L)
Absolute neutrophil count 320/µL
Of the following, over the next 48 hours, this child is MOST likely to develop
E. renal failure
A diffuse petechial rash and epistaxis would be most associated with severe thrombocytopenia.
Isolated, severe neutropenia should not impact hemostasis or increase the risk of bleeding.
While leukemia can present with low blood cell counts and fever, it would be unusual for it to
present as isolated neutropenia. Leukemia is a space-occupying lesion that reduces the bone
marrow medullary space available for normal hematopoiesis. This can result in pancytopenia.
The normal hemoglobin, platelet count, and absence of a palpable spleen all would suggest that
leukemia is not the etiology of this child’s acute illness.
A purpuric rash rising up the legs to the buttocks would suggest Henoch-Schönlein purpura
(HSP). This is an inflammatory disease of the small blood vessels. While a rash is the most
typical sign, it can also cause renal failure. Henoch-Schönlein purpura is not associated with
high, spiking fevers or neutropenia. The clinical picture for the boy in this vignette is not
consistent with HSP. Although the neutropenia in this vignette is most likely caused by viral
suppression of neutrophil production and is most likely transient, other possible causes of the
neutropenia include autoimmune neutropenia and congenital neutropenia. Autoimmune
neutropenia is most common in the first 2 years of life. Children with autoimmune neutropenia
tend not to develop invasive bacterial infections and it typically resolves spontaneously over
months to years. To assess the risk of neutropenia being a presentation of congenital neutropenia
(for example, severe congenital neutropenia), it would be important to ascertain whether the
child has ever had a normal neutrophil count. No mention of prior bacterial infections was made
in the vignette, however, it is certainly possible that an 18-month-old boy could present with an
undiagnosed congenital neutropenia. Severe congenital neutropenia (SCN) is most often caused
by a mutation in the ELANE gene (19p13.3). If there is suspicion for SCN, the gene can be
sequenced to establish the diagnosis.
Although the child in the vignette is not reported to be taking any medications, there are many
drugs that have neutropenia as a potential adverse effect. These include many antimicrobials,
anti-epileptics, chemotherapeutics, immunosuppressives, and many others. It is important to
review every medication and supplement the patient is taking to ascertain the risk of drug-
induced neutropenia. Often, the treatment is to stop or dose reduce the offending agent.
American academy of pediatrics 679
American Academy of Pediatrics PREP 2016
PREP Pearls
• Neutropenia with a normal hemoglobin and normal platelet count during a febrile
illness is most likely transient viral suppression.
• Severe congenital neutropenia and autoimmune neutropenia should be included in the
differential diagnosis of an 18-month-old patient presenting with severe neutropenia.
• Severe congenital neutropenia is often caused by a mutation in the ELANE gene
(19p13.3).
Suggested Readings
• Boxer LA. Neutrophil abnormalities. Pediatr Rev. 2003;24(2):52-62. DOI:
http://dx.doi.org/10.1542/pir.24-2-52.
• Segel GB, Halterman JS. Neutropenia in pediatric practice. Pediatr Rev.
2008;29(1):12-24. DOI:http://dx.doi.org/10.1542/pir.29-1-12.
• Walkovich K, Boxer LA. How to approach neutropenia in childhood. Pediatr Rev.
2013;34(4):173-184. DOI:http://dx.doi.org/10.1542/pir.34-4-173.
Question 211
You have been taking care of a 10-year-old boy for most of his life, along with his 3 siblings.
Unfortunately, this young boy developed acute myelogenous leukemia around 8 years of age.
Throughout his treatment, you have continued to be in close contact with his family. Today, you
received a notice from the local oncology team that he died from complications of his disease. In
addition to dealing with your own internal reactions to this loss, you begin to wonder how best to
interact with his family going forward.
Of the following, the MOST appropriate action for you to take with this family is
B. avoid bringing up any reminders of this boy at future appointments with his siblings
C. explain how the Kübler-Ross grief stages will be occurring for each of his siblings
D. offer the advice that children younger than 5 years of age should not attend a funeral
E. refrain from expressing any tearful emotions when interacting with his parents
What helps a grieving family is for a pediatrician to listen and support them during their process
of loss and bereavement. Scheduling an appointment with the family about 1 month after the
death of the child provides a good time to address coping concerns, and to reinforce with parents
that their decisions were thoughtful and based on what was best at the time. Providers should
also address the siblings directly in order to be someone for them who is not embarrassed to talk
about the deceased child and the changes in their lives.
The Kübler-Ross stages of grief are very commonly referenced in popular culture, but should not
be viewed as a dogmatic step by step process for everyone going through grief. While the
“stages” of denial, anger, bargaining, depression, and acceptance are clearly recognizable grief
reactions, not everyone experiences them. In fact, a more current view is that most children and
adults are quite resilient to loss, particularly if they live in a supportive personal environment,
and therefore might not exhibit any external symptoms of grief. There is neither any specific
need for people to progress through the Kübler-Ross stages of grief, nor a reason to believe there
is something wrong with a child that does not exhibit significant grief reactions.
The decision of whether or not a child should attend a funeral is often a challenging one for
families. There is no right or wrong answer, nor should it be based on the child’s age alone.
Conversant children can be given the option of attending, but should first be told what they are
likely to see and hear. If a child does attend a funeral, a supportive person should be available to
them to be responsive to their needs and who would be willing to leave with them should the
child wish to do so. Funerals are occasions for the living to process their loss and not every child
will benefit from that experience.
Parents usually appreciate a treatment provider expressing emotion about a child’s death. A
pediatrician’s affect and attitude can leave a lasting impression, particularly at the time of death.
Whether or not their care provider expresses emotion, there is significant value in just being
present with the family to provide support without having to worry about finding just the right
thing to say.
Children understand death through their developmental level, personal experiences, and the
family’s religious and cultural beliefs. Children younger than 2 years of age may not understand
death, but they do understand separation and the emotions of others. Preschool age children will
develop an understanding about death as different from being alive, but might still see death as
American academy of pediatrics 682
American Academy of Pediatrics PREP 2016
temporary or wonder what dead people “do.” Elementary school age children understand that
because others can die that they too can die, generally understand the permanence of death, and
some may have a stage of being preoccupied with the concept of death.
PREP Pearls
• Scheduling an appointment with the family about 1 month after the death of a child
provides a good time to address coping concerns.
• Children can be given the option of attending of attending a funeral, but should first
be told what they are likely to see and hear while they are there.
Suggested Readings
• Linebarger JS, Sahler OJZ, Egan KA. Coping with death. Pediatr Rev.
2009;30(9):350-356. DOI:http://dx.doi.org/10.1542/pir.30-9-350.
Question 212
A 10-month-old male infant is brought to the office with 6 days of fever and a new rash. His
mother states that he has been difficult to feed for 2 days because he is so irritable. He usually
breastfeeds for 20 min every 3 hours, but has only been feeding for 5 min every 2 hours. He has
had 2 wet diapers in the last 24 hours. He has not been given any medication.
No one else is ill at home. The infant’s family is Asian American and he is in day care 3 days a
week.
Vital signs show a temperature of 39.5°C rectally, respiratory rate of 30 breaths/min, heart rate of
160 beats/min, and a blood pressure of 90/65 mm Hg. Physical examination shows an alert, but
very fussy infant. His conjunctiva are injected, but there is no purulent drainage. The lips are red
and cracked. There are no mouth ulcers. His tongue is red with white papilla. The neck, chest,
and cardiac examinations are unremarkable. The liver edge is at the right coastal margin. No
spleen is palpated. His hands and feet have mild edema. There is an erythematous maculopapular
rash on his trunk and arms.
Laboratory results are as follows:
• White blood cells, 10,500/µL (10.5 x 109/L) with 65% neutrophils, 25% lymphocytes,
10% atypical lymphocytes
• Hemoglobin, 9.5 g/dL (95 g/L)
• Hematocrit, 30.1%
• Erythrocyte sedimentation rate, 60 mm/h
• C-reactive protein, 4.5 mg/L
• Urinalysis shows white blood cells
• Alanine aminotransferase, 50 U/L
• aspartate aminotransferase, 45 U/L
Of the following, the BEST next combination of diagnostic and therapeutic maneuvers is
As the etiology is still not known, there is no definitive diagnostic test available for KD and the
diagnosis must be made on clinical grounds with supporting laboratory tests. Kawasaki disease is
more common in boys. The major symptom of KD is fever for at least 5 days. Four additional
clinical signs and symptoms are needed to make the diagnosis of typical KD, including cervical
lymphadenopathy, red and cracked lips or strawberry tongue, nonpurulent conjunctivitis, a
polymorphous rash, and redness and edema of hands and feet, with peeling of the fingers and
toes in the later part of the illness. If more than 4 of the principal symptoms are present along
with fever, the diagnosis may be made before the fifth day of the illness.
The highest risk for development of CAA is in the first 2 weeks of the illness, and administration
of IVIG is needed within the first 10 days of febrile illness to help prevent that sequelae.
Coronary artery (CA) findings are not required to make the diagnosis of KD, although this may
be helpful in making a diagnosis in patients with atypical KD. Extreme irritability, sterile pyuria,
and thrombocytosis are frequently seen in KD, but are not 1 of the 5 principle findings used for
the diagnosis of KD.
Prior to the use of IVIG, thrombocytosis with platelet counts up to 1 million/μL were seen and
thought to contribute to the damaging effects of coronary artery vasculitis. Patients with
persistent fever despite treatment with IVIG are at a higher risk of CAA development. A second
dose of IVIG, infliximab, or steroids, as well as continuation of high dose ASA, are among the
recommendations in this situation. In less complicated cases where the patient becomes afebrile
after initial IVIG administration, the high dose ASA is decreased to low dose ASA.
Recommendations for further treatment and follow-up depend on the presence or absence of
CAA or the less severe finding of CA ectasia. Follow-up at 2 weeks and 6 weeks after discharge
is recommended, as a minimum, even if no coronary involvement was seen initially.
American academy of pediatrics 685
American Academy of Pediatrics PREP 2016
If CAA have been identified, more intense follow-up will be needed. Cardiac stress testing and
angiography may be required in these cases. If the CAA are large, anticoagulation will be
needed. Over time, CAA can cause CA stenosis and myocardial ischemia.
A rapid diagnosis of KD must be made because of the risk of CAA. Diagnosis and treatment of
KD with IVIG should not be delayed while waiting for an echocardiogram. Treatment with IVIG
will help prevent development of CAA. A very high risk group for development of CAA are
younger male children of Asian heritage, such as described in the vignette. With or without
CAA, patients require cardiology follow-up.
The infant in this vignette does not have respiratory symptoms, therefore azithromycin for
pneumonia would not be the ideal treatment for him. Irritability would create concern for
meningitis, but the other clinical findings would not make that the most likely diagnosis. A
urinary tract infection would not cause the combination of findings described. Kawasaki disease
does share several similar clinical findings with scarlet fever. The rash in that case is more likely
to be “sand paper” in appearance. This and the additional findings of pyuria without bacteria in
this infant would make a throat culture and penicillin not the next step in this patient.
PREP Pearls
• Kawasaki disease (KD) is a clinical diagnosis that needs to be made as soon as
possible to allow initiation of intravenous immunoglobulin treatment and prevent
development of coronary artery aneurysms.
• An echocardiogram is needed to evaluate for coronary artery involvement, but not to
make the diagnosis, except in cases that do not meet criteria for typical KD.
Suggested Readings
• American Academy of Pediatrics. Kawasaki disease. In: Kimberlin DW, Brady MT,
Jackson MA, Long SS, eds. Red Book:2015 Report of the Committee on Infectious
Diseases. 30th ed. Elk Grove Village, IL: American Academy of Pediatrics;
2015:494-500.
• Burns JC, Kushner HI, Bastian JF, et al. Kawasaki disease: a brief history. Pediatrics.
2000;106(2):e27.http://pediatrics.aappublications.org/content/106/2/e27.full.pdf+html
Question 213
You are seeing a 12-year-old girl in your office with complaint of generalized pain and back pain
that is worsened by exercise. The girl has muscle aches and joint pain that she is not able to
localize well. She has noticed recurrent ankle swelling that lasts a few hours with each episode.
She rates the pain as a 3 out of 10 at its mildest, and 7 out of 10 at its worst. The girl has pain
daily, which is not associated with time of day, and is worsened by exercise. Her review of
symptoms is positive for fatigue, headache, difficulty sleeping, abdominal pain with cramping,
alternating loose stools and constipation, and occasional mouth sores. She has missed several
days of school because of pain, but has kept up with her schoolwork. When asked about changes
at home, the girl states that her parents are going through a divorce.
Of the following, the BEST next step in the evaluation and management of this girl would be
a(n)
D. referral to a gastroenterologist
The pain associated with APS can be localized or widespread. There may be skin color changes
or swelling that comes and goes. The onset of APS may be acute or triggered by a preceding
injury, and symptoms may increase overtime. Patients with APS often are affected by
widespread musculoskeletal pain; chronic fatigue; sleep disturbance; and cognitive and mood
disorders. Patients with chronic pain often display other somatic symptoms such as headache,
dysautonomia, subjective soft tissue swelling, and irritable bowel syndrome. There are several
subtypes of amplified pain syndromes.
Complex regional pain syndrome presents as continuing pain that is disproportionate to the
inciting event. Some cases are preceded by nerve injury, but this may be absent. Categories
include sensory dysfunction, such as hyperalgesia and/or allodynia, and vasomotor dysfunction,
presenting with temperature asymmetry or sweating changes, and patient-reported edema. There
must be no other diagnosis that better explains these signs or symptoms.
Localized idiopathic pain is characterized by pain localized to 1 limb that persists for 1 week
with treatment or 1 month without treatment, no history of trauma or disease that can explain the
symptoms.
American academy of pediatrics 688
American Academy of Pediatrics PREP 2016
Several conditions may occur concurrently with APS, including irritable bowel syndrome,
chronic fatigue syndrome, interstitial cystitis, chronic headache, functional abdominal pain, or
conversion disorders. Psychosocial factors such as stress, injury, resilience, academic factors,
peer relationships, family relationships, parenting style, and the medical system’s response to the
pain can have a significant effect on the level of pain in APS. Several psychological factors can
also have an effect on pain in APS patients, such as pain avoidance or acceptance,
catastrophizing, coping, cognition, intelligence, motivation and perception.
The approach to treatment for APS should be multidisciplinary. Helpful approaches include
exercise, desensitization, stress management, counseling, and self-regulation with purposeful
relaxation.
There is little evidence available to guide the treatment of pediatric pain with medication.
Several of the medications commonly used effectively in adults are “off label” in the pediatric
population. Medications commonly used to treat pediatric pain include acetaminophen,
nonsteroidal anti-inflammatory drugs, and Cox-2 inhibitors. Some anticonvulsants, such as
gabapentin and pregabalin, have been used to effectively treat chronic pain. Antidepressants have
been prescribed, but there is no significant evidence of effectiveness. The most commonly used
antidepressants are amitriptyline and nortriptyline. Duloxetine has been used, but is not labeled
for pediatric use. Opioids are used to treat pain in patients with cancer, but have limited use for
other types of chronic pain, and safety data regarding their long-term use are limited. Other
medications, with limited evidence regarding effectiveness, include clonidine, dexmedetomidine,
lidocaine, and mexiletine.
PREP Pearls
• Amplified pain syndromes are diagnoses of exclusion. Other conditions must be ruled
out before diagnosis.
• Multifactorial treatment including exercise, desensitization, stress management, and
counseling should be used in treating patients with amplified pain syndromes.
• Amplified pain syndromes often occur concurrently with irritable bowel syndrome,
chronic fatigue syndrome, interstitial cystitis, chronic headache, functional abdominal
pain, or conversion disorders.
Suggested Readings
• Goodard JM Chronic pain in children and young people. Curr Opin Support Palliat
Care. 2011;5(2):158-163. DOI:http://dx.doi.org/10.1097/SPC.0b013e328345832d.
• Hoffart CM, Wallace DP Amplified pain syndromes in children: treatment and new
insights into disease pathogenesis. Curr Opin Rheumatol. 2014;26(5):592-603. DOI:
http://dx.doi.org/10.1097/BOR.0000000000000097.
Question 214
You are speaking with a group of pediatric residents about exercise in children. You emphasize
the importance of routinely discussing the benefits of exercise with young patients and their
families.
Of the following, the MOST accurate statement to include in your discussion is that
B. increased physical activity in children correlates with lower rates of coronary artery
disease in adults, even if they are not physically active as adults
C. physical activity during childhood is correlated with improved bone mineral density
in adulthood
E. the majority of high school students in the United States participate in 60 or more
minutes of moderate physical activity each day
Recess, walking, or biking to school, after-school activity programs, and other community
recreation programs can all boost physical activity levels. Children are more likely to participate
in organized sports if their parents support their athletic endeavors and when their peers
participate in sports. Lower socioeconomic status has been correlated with lower rates of sports
participation, whereas the availability of inexpensive, easily accessible sports opportunities in the
community has been positively correlated with sports participation. Pediatricians can promote
physical activity by checking that gross motor skills are being learned in early school-age
children and by providing developmentally appropriate activity guidelines and resources to
families.
Children with siblings are no more likely than only children to engage in regular physical
activity. While physically active adults have lower rates of heart disease, there is no direct link
between physical activity during childhood and coronary disease in adulthood. Studies have
shown that some school-based physical activity programs can improve fitness, increase the
amount of time children and teens participate in physical activity, and lead to decreased screen
time, but these effects are not sustained when the school programs end.
PREP Pearls
• Physical activity during early childhood is correlated with improved bone mineral
density in adolescence and adulthood.
• Most children and adolescents do not meet recommended levels of physical activity.
Suggested Readings
• Dobbins M, Husson H, DeCorby K, LaRocca RL. School-based physical activity
programs for promoting physical activity and fitness in children and adolescents aged
6 to 18. Cochrane Database Syst Rev. 2013;2:CD007651.
DOI:http://dx.doi.org/10.1002/14651858.CD007651.pub2.
Question 215
A 6-year-old boy presents to your office with a several month history of worsening nasal
congestion, thick nasal discharge, and cough disrupting sleep, despite multiple courses of
antibiotics. The boy has asthma and uses an inhaled steroid appropriately for maintenance
therapy, but recently has been having frequent exacerbations. Although recent testing did not
reveal allergies, he has been using nasal steroid sprays and oral antihistamines. He has a history
of multiple upper respiratory tract infections and foul smelling stools.
On physical examination, the boy’s temperature is 37°C, heart rate is 110 beats/min, respiratory
rate is 20 breaths/min, blood pressure is 95/65 mm Hg, and weight is 22 kg (tenth percentile).
Examination of the head and neck reveals deviation of the nasal septum to the right, hypertrophy
of the nasal turbinates with mucopurulent discharge, and nasal polyps in the left naris. The
tympanic membranes are normal. The posterior oropharyngeal mucosa has a cobblestone
appearance and the tonsils are enlarged without exudates. Auscultation of the lungs is significant
for a prolonged expiratory phase with mild, intermittent wheezing throughout.
Of the following, the MOST likely cause of this patient’s prolonged symptoms is
B. defective phagocytosis
D. HIV infection
Chronic sinusitis is an inflammatory disorder of the sinuses and nasal passages that lasts 12
weeks or longer. In children, the signs and symptoms of chronic sinusitis include mucopurulent
nasal drainage, cough, nasal congestion or obstruction, and facial fullness or pain. Many factors
may contribute to or exacerbate chronic sinusitis including allergic rhinitis, immunodeficiency or
other systemic diseases (eg, granulomatosis with polyangiitis), defects in mucociliary clearance
(eg, cystic fibrosis), environmental irritants, recurrent viral infections, and anatomic
abnormalities.
Although allergic disorders are commonly implicated in chronic sinusitis, the boy in the vignette
does not have a proven allergy and has not responded to nasal corticosteroid sprays and oral
antihistamines. Allergic fungal sinusitis is an uncommon form of chronic sinusitis, often
mistaken for a paranasal sinus tumor, caused by an allergic reaction to aerosolized environmental
fungi in an immunocompetent individual. Surgery and immunomodulation are required for
treatment.
Immune disorders that cause defects in phagocytosis can present with sinusitis (eg, leukocyte
adhesion defect, Shwachman-Diamond syndrome), but they are rare and typically also associated
with cutaneous and gastrointestinal infections in the first years after birth. Humoral immune
disorders, such as immunoglobulin A deficiency and common variable immunodeficiency, occur
more commonly and can present with disease isolated to the respiratory tract. Infection with
human immunodeficiency virus (HIV) can cause recurrent bacterial infections, but it would be
unlikely that chronic, refractory bacterial infection would occur in the respiratory tract without
other manifestations of HIV infection (eg, diarrhea, wasting, fungal infections, viral infections).
There is a strong association between asthma and chronic sinusitis; approximately 20% of
patients with chronic sinusitis have asthma and nearly two-thirds of patients with asthma have
sinus disease. Evidence demonstrates that treatment of sinusitis may improve asthma symptoms,
but the converse is unproven.
PREP Pearls
• Cystic fibrosis is caused by a mutation in the cystic fibrosis transmembrane
conductance regulator (CFTR) protein, which is found in all exocrine cells and
regulates the transport of chloride and other ions.
• Chronic sinusitis is an inflammatory disorder of the sinuses and nasal passages that
lasts 12 weeks or longer.
• The signs and symptoms of chronic sinusitis include mucopurulent nasal drainage,
cough, nasal congestion or obstruction, and facial fullness or pain.
• Factors that may contribute to or exacerbate chronic sinusitis include allergic rhinitis,
immunodeficiency, mucociliary clearance defects, environmental irritants, recurrent
viral infections, and anatomic abnormalities.
Suggested Readings
• Cherry JD, Mundi J, Shapiro NL. Rhinosinusitis. In: Cherry J, Demmler-Harrison GJ,
Kaplan SL, Steinbach WJ, Hotez P, eds. Feigin & Cherry’s Textbook of Pediatric
Infectious Diseases. 7th ed. Philadelphia, PA: Elsevier Saunders; 2014:193-203.
• Desrosiers M, Evans GA, Keith PK, et al. Canadian clinical practice guidelines for
acute and chronic rhinosinusitis. Allergy Asthma Clin Immunol. 2011;7:2. DOI:
http://dx.doi.org/10.1186/1710-1492-7-2.
Question 216
A community hospital calls the pediatric emergency department where you are working seeking
to transfer an 11-year-old boy with a grade IV laceration to his left kidney. This injury was
sustained when the boy fell 10 feet from a tree, landing on his left side. The transferring
physician asks for your recommendation regarding insertion of an indwelling bladder catheter to
monitor the boy’s urine output prior to transporting him to your hospital.
Of the following, the MOST accurate statement regarding this patient is that
All pediatric providers should recognize the contraindications to bladder catheterization after
acute trauma. Insertion of urinary catheters in patients with traumatic injuries can help to relieve
urinary retention, decompress the bladder, and allow for monitoring of urinary output, which
serves as an index of tissue perfusion. Urinary catheters should not be inserted, however, if any
of the following contraindications are present: gross hematuria, inability to void, unstable pelvic
fracture, blood at the urethral meatus, perineal ecchymoses or scrotal hematoma, or a high-riding
or nonpalpable prostate on rectal examination. For patients presenting with any of these
contraindications, retrograde urethrography must be performed to confirm an intact urethra
before urinary catheter insertion.
A rectal examination should always be performed in patients presenting after abdominal and
pelvic trauma beforeinsertion of a bladder catheter. Goals of the rectal examination include
assessment of sphincter tone and integrity of the rectal mucosa, determination of prostate
position in male patients (high-riding prostate suggests urethral disruption), assessment for gross
blood (which may indicate bowel perforation), and identification of pelvic fractures. In female
patients, a vaginal examination should also be performed to exclude lacerations from bony
fragments from pelvic fractures or from penetrating wounds, prior to urinary catheterization.
Prostate enlargement is not a contraindication to bladder catheterization. High-riding position of
the prostate or the inability to palpate the prostate indicates disruption of the urethra and are
contraindications.
The presence of red blood cells on urinalysis in patients presenting for evaluation of traumatic
injuries who do not have gross hematuria or blood at the urethral meatus is not a contraindication
for bladder catheterization.
Urologic consultation is not needed for patients without any of the previously discussed
contraindications to urinary catheter placement.
PREP Pearls
• Insertion of urinary catheters in patients presenting with traumatic injuries can help to
relieve urinary retention, decompress the bladder, and allow for monitoring of urinary
output.
• Urinary catheters should not be inserted in patients who have experienced an acute
trauma and have any of the following contraindications: gross hematuria, inability to
void, unstable pelvic fracture, blood at the urethral meatus, perineal ecchymoses,
scrotal hematoma, or a high-riding or nonpalpable prostate on rectal examination.
• A rectal examination should always be performed in patients presenting after
abdominal and pelvic trauma before insertion of a bladder catheter.
Suggested Readings
• American College of Surgeons Committee on Trauma. Abdominal and pelvic trauma.
Advanced Trauma Life Support Student Course Manual. 9th ed. Chicago, IL:
American College of Surgeons; 2012:122-140.
• Garcia CT, Thompson VT. Genitourinary trauma. In: Fleisher GR, Ludwig S, eds.
Textbook of Pediatric Emergency Medicine. 6th ed. Philadelphia, PA: Lippincott
Williams & Wilkins; 2010:1316-1327.
Question 217
You are called to see a full-term newborn who is found to have abdominal distension with
delayed passage of stool at 28 hours after birth. The pregnancy was remarkable for diet-
controlled gestational diabetes and echogenic bowel on the second trimester ultrasonography.
The newborn has been breastfeeding well with good urine output and no passage of stool.
Physical examination reveals an alert, active newborn with a markedly distended, firm abdomen,
and an externally patent rectum. In the course of the subsequent evaluation, a barium enema is
performed (Item Q217).
Item Q217
Item Q217. Radiograph for the newborn described in the vignette. Courtesy of Donald Frush, MD.
Of the following, the test that is MOST likely to reveal the underlying cause of this newborn’s
clinical and radiographic findings is a
A. chromosome analysis
C. rectal biopsy
Item C217A
More than 99% of healthy full-term newborns will pass a stool within 48 hours after delivery.
Intestinal obstruction should be suspected in a newborn with delayed passage of stools,
abdominal distension, and vomiting. The initial assessment should include a physical
examination and plain abdominal radiography. This is often unable to distinguish an emergent
condition such as midgut volvulus from a condition responsive to conservative care, therefore, a
newborn suspected of having an intestinal obstruction should be evaluated by a pediatric
surgeon. The decision, made in consultation with both a pediatric surgeon and a radiologist,
about whether to perform an upper gastrointestinal series or a contrast enema will be guided by
the suspicion of either an upper or lower bowel obstruction.
Lower bowel obstruction is associated with delayed passage of stool (Item C217B). With the
exception of anorectal malformations, a contrast enema is generally the best diagnostic study to
perform. A recent review supports an association of cystic fibrosis with meconium ileus, but not
meconium plug syndrome. If present, the location of meconium plugs will distinguish meconium
plug syndrome/small left colon syndrome from meconium ileus. Because 13% of patients with
meconium plug syndrome are subsequently found to have Hirschsprung disease, newborns with
meconium plugs should have further evaluation if issues with passage of stool persist.
The newborn in the vignette has clinical and radiographic features suggestive of meconium ileus,
which is likely the result of cystic fibrosis, and a sweat test should be performed for diagnosis.
Chromosome disorders can be associated with bowel obstruction, but typically not with
meconium ileus. A maternal hemoglobin A1c level would not be useful, because the barium
enema does not demonstrate localized small caliber of the descending colon as would be seen in
small left colon syndrome. There is no evidence of a transition zone on the barium enema that
would suggest Hirschsprung disease; therefore a rectal biopsy would be unlikely to yield a
diagnosis. Congenital cytomegalovirus has been associated with a pseudo-Hirschsprung disease
presentation, but this is not supported by the findings on the barium enema.
PREP Pearls
• Intestinal obstruction should be suspected in a newborn with delayed passage of
stools, abdominal distension, and vomiting.
• A newborn suspected of having an intestinal obstruction should be evaluated by a
pediatric surgeon, because the physical examination and plain abdominal radiography
may not distinguish an emergent condition such as midgut volvulus from a condition
responsive to conservative care.
Suggested Readings
• Keckler SJ, St Peter SD, Spilde TL, et al. Current significance of meconium plug
syndrome. J Pediatr Surg. 2008;43:896-898. DOI:
http://dx.doi.org/10.1016/j.jpedsurg.2007.12.035.
• Levine D. Overview of echogenic masses and calcifications in the fetal abdomen.
UpToDate. Available online only for subscription.
Question 218
A 2-day-old male neonate is being cared for in the well-baby nursery. His mother tells the nurse
that she is concerned about his vision. She notices that he stares to the side for 20 to 30 seconds
every couple of hours. There are no facial twitches, cyanosis, or other symptoms. He was born
by normal spontaneous vaginal delivery at 39 weeks’ gestational age, with no complications
during pregnancy, labor, or delivery. His Apgar scores were 8 and 9 at 1 and 5 minutes,
respectively. His birth weight was 3.2 kg and head circumference was 34.5 cm. His physical and
neurologic examinations are unremarkable. The neonate is transferred to the neonatal intensive
care unit where treatment with antibiotics and acyclovir are started. His initial laboratory
evaluation is normal, including complete blood cell count with differential, comprehensive
metabolic panel, serum herpes simplex virus polymerase chain reaction (PCR), and urinalysis.
Lumbar puncture results are normal, including cerebrospinal fluid protein, glucose, cell count
with differential, and herpes simplex virus PCR. Blood, urine, and cerebrospinal fluid cultures
are pending.
Of the following, the MOST likely to reveal the cause of the neonate’s symptoms is a(n)
B. ophthalmology consultation
The most common cause of neonatal seizure is hypoxic ischemic encephalopathy. Other
common causes of neonatal seizure include intracranial hemorrhage, infection, and brain
malformation. Less common causes of seizure in the neonatal period include neonatal stroke,
hypoglycemia, electrolyte abnormalities (eg, hypocalcemia, hyponatremia, or hypernatremia),
inborn errors of metabolism, and epilepsy syndromes.
There are many treatable causes of neonatal seizure, and time to diagnosis and treatment can be
critical, therefore the initial diagnostic evaluation is broad. This evaluation includes laboratory
testing for infection and electrolyte and metabolic abnormalities. Brain infection can be
devastating; therefore empiric antibiotics and antiviral treatment should be started urgently.
Anticonvulsant treatment should also be started when clinical suspicion for seizure is high. Brain
imaging is almost always performed in these cases; head ultrasonography or computed
tomography is often performed first because of their availability, but MRI is preferred.
For the neonate in the vignette, ophthalmology consultation is unlikely to be helpful in
determining the cause for seizures. Urine cytomegalovirus (CMV) culture would be appropriate
if there were signs suggestive of congenital CMV infection, such as microcephaly,
hepatomegaly, characteristic skin rash, thrombocytopenia, or abnormal liver function tests. Urine
homovanillic acid and vanillylmandelic acid are elevated in neuroblastoma, which presents with
opsoclonus, myoclonus, and ataxia. Opsoclonus is an involuntary, conjugate, random, jerky eye
movement, which this neonate does not have. Urine organic acids can be abnormal in some
inborn errors of metabolism, which can present with neonatal seizures. However, the lack of
acidosis makes an organic acidemia unlikely in this case.
PREP Pearls
• Tonic horizontal eye deviation is a common manifestation of seizure in full-term
neonates.
• The diagnosis of seizure is often made clinically in preterm and full-term neonates,
because an electroencephalogram can be normal even during a seizure at this age.
Suggested Readings
• Gillam-Krakauer M, Carter BS. Neonatal hypoxia and seizures. Pediatr Rev.
2012;33;387. DOI:http://dx.doi.org/10.1542/pir.33-9-387.
• Mosley M. Neonatal seizures. Pediatr Rev. 2010;31;127. DOI:
http://dx.doi.org/10.1542/pir.31-3-127.
Question 219
An 8-year-old boy with type 1 diabetes mellitus diagnosed 2 years ago presents for a follow-up
visit. Review of systems is significant for increased fatigue and constipation, but he is otherwise
doing well.
The boy’s physical examination is significant for height that has dropped from the tenth
percentile last year to less than the third percentile this year. His examination is otherwise
unremarkable. There is no goiter, skin changes, or abnormal findings at his insulin injection sites.
His hemoglobin A1c level is 6.9%.
Of the following, the MOST likely explanation for his poor growth is
A. autoimmune hypothyroidism
B. celiac disease
C. gastroparesis
D. pernicious anemia
E. thyroid cancer
The International Society for Pediatric and Adolescent Diabetes (ISPAD) 2009 Consensus
Guidelines for Children and Adolescents with Diabetes also recommend screening of thyroid
function by measuring TSH and thyroid antibodies at diagnosis and every second year thereafter
in asymptomatic individuals without goiter or in the absence of thyroid autoantibodies. More
frequent assessment is indicated if antibodies are present or symptoms develop (eg, poor growth
or goiter). It is important for pediatricians to recognize that goiter may not be present in children
with thyroid disease.
Many other autoimmune conditions are associated with type 1 diabetes. Celiac disease occurs
with increased frequency in patients with type 1 diabetes (1%-16% of individuals compared with
0.3%-1% in the general population). Symptoms of celiac disease include diarrhea, weight loss or
poor weight gain, growth failure, abdominal pain, chronic fatigue, malnutrition due to
malabsorption, and unexplained hypoglycemia or erratic blood glucose concentrations. The ADA
recommends that practitioners consider screening children with type 1 diabetes for celiac disease
soon after the diagnosis, as well as in cases with a positive family history of celiac disease,
symptoms consistent with the diagnosis, or frequent unexplained hypoglycemia or deterioration
in glycemic control. ISPAD guidelines are more stringent, and recommend screening at the time
of diagnosis, annually for the first 5 years and every second year thereafter. Although the child in
this vignette could have celiac disease, hypothyroidism occurs far more commonly and thus is
the most likely explanation for his poor growth.
American academy of pediatrics 707
American Academy of Pediatrics PREP 2016
The degree of glucose control and duration of diabetes mellitus are linked to long-term
complication rates. In long-term (20-year) follow-up of patients with type 1 diabetes mellitus,
extensive clinical trials have shown that glomerular filtration rate (and overall kidney function) is
improved in patients treated with intensive insulin therapy earlier in their disease and correlates
with the degree of glucose control. Other outcomes improve as well. Monitoring for associated
disorders and long-term glucose control is important to minimize these complications.
PREP Pearls
• Autoimmune thyroid disease is the most common autoimmune disorder associated
with diabetes.
• Routine assessment of thyroid autoimmunity and thyroid function should be
performed every 1 to 2 years in children with type 1 diabetes, and sooner if the child
develops symptoms of thyroid disease, such as poor growth.
• Goiter may not always be present in children with thyroid disease.
Suggested Readings
• American Diabetes Association. Standards of medical care in diabetes--2014.
Diabetes Care. 2014;37(suppl 1):S14-80. DOI: http://dx.doi.org/10.2337/dc14-S014.
• DCCT/EDIC Research Group, de Boer IH, Sun W, Cleary PA, et al. Intensive
diabetes therapy and glomerular filtration rate in type 1 diabetes. N Engl J Med.
2011;365:2366-2376. DOI: http://dx.doi.org/10.1056/NEJMoa1111732.
• Kordonouri O, Maguire AM, Knip M, et al. ISPAD Clinical Practice Consensus
Guidelines 2009 Compendium: Other Complications and Associated Conditions.
Pediatr Diabetes. 2009;10(suppl 12):204-210. Available
at:http://onlinelibrary.wiley.com/doi/10.1111/j.1399-5448.2009.00573.x/full.
• Perros P, Singh RK, Ludlam CA, Frier BM. Prevalence of pernicious anaemia in
patients with Type 1 diabetes mellitus and autoimmune thyroid disease. Diabet Med.
2000 Oct;17(10):749-751. DOI: http://dx.doi.org/10.1046/j.1464-5491.2000.00373.x.
• Vergouwe Y, Soedamah-Muthu SS, Zgibor J, et al. Progression to microalbuminuria
in type 1 diabetes: development and validation of a prediction rule. Diabetologia.
2010;53(2):254-262. DOI: http://dx.doi.org/10.1007/s00125-009-1585-3.
American academy of pediatrics 708
American Academy of Pediatrics PREP 2016
Question 220
A 7-month-old infant is brought to your office for a follow-up evaluation. She was recently
hospitalized for the third time with bronchiolitis. On each occasion, she has been treated with a
short acting ß-agonist and systemic steroids. There is no history of pneumonia or recurrent otitis
media. The infant was full term at birth. The parents report that they have heard “whistling and
rattling” in their infant’s chest since she was 1 week of age. The infant suffers from
gastroesophageal reflux and has been treated with ranitidine.
On physical examination, you note a thriving infant in no respiratory distress. The respiratory
pattern is comfortable, but the respiratory rate is mildly elevated at 48 breaths/min. You note a
monophonic coarse wheeze, which is more notable at the left chest. The remainder of the
examination is unremarkable. You review the most recent chest radiograph and recognize a
double aortic arch.
Of the following, the study MOST likely to confirm your suspected diagnosis is
The normal aortic arch is left-sided. Anomalies of the anatomy of the aortic arch are best
understood through a review of embryogenesis. During early fetal development, 6 pairs of
primitive aortic arches are formed in sequence. The final anatomic structure is a result of both
regressive and derivational components. The arches that persist normally in humans are the
fourth and sixth. Fourth arch derivatives contribute to a portion of the normal left aortic arch.
The proximal portions of the sixth arches become the mediastinal portion of the pulmonary
arteries and the distal portions form the ductus arteriosus, which generally involutes after birth
but which may persist.
Vascular anomalies that may be associated with airway and esophageal compression include:
• Double aortic arch
• Right aortic arch
• Left aortic arch with aberrant right subclavian artery or right descending aorta and
right ligamentum arteriosum
• Anomalous innominate artery
• Cervical aortic arch
• Pulmonary artery sling
A vascular ring occurs when the trachea and esophagus are entirely encircled by vascular
structures. Vascular rings comprise approximately 1% of congenital cardiovascular anomalies. A
double aortic arch, where the right- and left-sided aortic arches surround the trachea and
esophagus, is the most common etiology of vascular ring and accounts for 50% to 60% of
occurrences. A vascular ring may also be created by a right aortic arch with aberrant left
subclavian artery and left ligamentum arteriosum. Infants with vascular rings generally present
early in life with wheezing, stridor, and dysphagia caused by this anatomic compression.
An anomalous innominate artery arises either more distally or more leftward as compared to
normal and compresses the trachea anteriorly. In a pulmonary artery sling, the left pulmonary
artery originates from the right pulmonary artery and then courses to the left between the trachea
American academy of pediatrics 710
American Academy of Pediatrics PREP 2016
and the esophagus; compression most commonly occurs at the distal trachea or right mainstem
bronchus. The esophagus may be compressed, but esophageal function is generally maintained.
Prenatal diagnosis of congenital vascular rings and slings has advanced with the utilization of 3
vessel tracheal views on fetal ultrasonography and fetal echocardiography. The postnatal
diagnostic evaluation of suspected airway compression by vascular malformations includes plain
chest radiography, barium esophagoscopy, echocardiography, bronchoscopy, and computed
tomography (CT). Plain chest radiographs showing abnormal laterality of the aortic arch should
increase the clinical suspicion in an infant or child with suggestive symptomatology, but further
imaging will be required to confirm the diagnosis. For the infant in this vignette, CT will provide
excellent visualization of airway compression and is the most likely of the choices to confirm the
diagnosis. More recently, magnetic resonance (MR) imaging has become the preferred study in
defining vascular anatomy, but is not one of the choices.
PREP Pearls
• Vascular malformations and related compressive airway lesions should be considered
in all infants with recurrent or chronic airway symptomatology, and particularly in
those where symptom onset has been atypically early in life or where response to
standard medical therapies has been poor.
• The double aortic arch is the most common etiology of a vascular ring.
• Although a plain chest radiograph, barium esophagram, or echocardiogram may
provide support for a diagnosis of a compressive vascular malformation, further
imaging with computed tomography or magnetic resonance is generally required for
definitive diagnosis and surgical planning.
Suggested Readings
• Haramati LB, Glickstein JS, Issenberg HJ, Haramati N, Crooke GA. MR imaging and
CT of vascular anomalies and connections in patients with congenital heart disease:
significance in surgical planning. Radiographics. 2002;22(2):337-349..
• Kellenberger CJ. Aortic arch malformations. Pediatr Radiol. 2010;40(6):876-884.
DOI:http://dx.doi.org/10.1007/s00247-010-1607-9.
• Kussman BD, Geva T, McGowan FX. Cardiovascular causes of airway compression.
Paediatr Anaesth. 2004;14(1):60-74. DOI: http://dx.doi.org/10.1046/j.1460-
9592.2003.01192.x.
• Li S, Luo G, Norwitz ER, et al. Prenatal diagnosis of congenital vascular rings and
slings: sonographic features and perinatal outcome in 81 consecutive cases. Prenat
Diagn. 2011;31(4):334-346. DOI:
Question 221
A family seeks your advice about a planned wilderness camping trip in Belize with their 4-year-
old and 10-year-old children. In addition to endemic dengue, there has been an outbreak of
malaria in the area they will visit. You provide prescriptions for malaria prophylaxis and discuss
options for insect bite prevention with them.
A. botanical insect repellents are the only products recommended for skin application for
children younger than 5 years of age
C. low concentration (7%) DEET has the same efficacy and duration of action as higher
concentration (20%-50%) formulations
DEET is active against several species of mosquito, and it has been tested for protection against
other insects, including ticks and bed bugs, but its effectiveness against these other arthropods is
less clear. Efficacy and duration of action of DEET increases with concentration, and it is most
effective when used in concentrations between 10% and 35%. Activity plateaus and even
decreases at concentrations above 30% to 50%. It is considered safe for use by infants and by
pregnant and lactating women. The American Academy of Pediatrics recommends use of DEET
in concentrations less than 30% on the exposed skin of infants and children 2 months of age or
older.
Several other products have been developed for use against both mosquitoes and other
arthropods. Botanicals are particularly sought after because many consider them safer and they
are biodegradable, which synthetic compounds such as DEET are not. Most botanicals are made
from plant essential oils derived from leaf or fruits. Oil of lemon eucalyptus has efficacy equal to
lower concentration DEET products (7%-15%) and a duration of action of 4 to 7 hours for
preventing mosquito bites. It is also effective against flies and gnats, but not against ticks. It is
not recommended for infants younger than 6 months of age. Permethrin, a synthetic pyrethroid,
is effective against ticks, mosquitoes, and other arthropods, as well as for treatment of head lice
and scabies. While not recommended for skin application for arthropod bite protection,
permethrin is very useful for treating clothing, camping gear, and bed nets. It must be reapplied
after every 5 washings to remain effective. Permethrin is more effective than DEET in protecting
against ticks.
PREP Pearls
• DEET is a highly effective mosquito repellant that is used in concentrations between
10% and 35%; it is considered safe for topical use by infants 2 months of age and
older, as well as pregnant and lactating women.
American academy of pediatrics 714
American Academy of Pediatrics PREP 2016
• Icaridin is equally effective against mosquitos compared to DEET; the World Health
Organization recommends it as the first line mosquito repellant product, but is not
approved for children younger than 2 years of age.
• Despite concerns and case reports, DEET has a good safety profile.
• Some botanicals provide activity against mosquitoes equivalent to low concentration
DEET.
• Permethrin applied to clothing, bed nets, and camping gear is effective against ticks,
mosquitoes, and other arthropods. It is not recommended for skin use for arthropod
protection.
Suggested Readings
• American Academy of Pediatrics. Insect repellents. Healthy Children
website.http://www.healthychildren.org/English/safety-prevention/at-
play/Pages/Insect-Repellents.aspx.
• Katz TM, Miller JH, Hebert AA. Insect repellents: historical perspectives and new
developments. J Am Acad Dermatol. 2008;58(5):865-871. DOI:
http://dx.doi.org/10.1016/j.jaad.2007.10.005.
Question 222
The mother of a 19-year-old young man with moderate intellectual disability calls you because
she wants resources for her son. She recently heard about Intermediate Care Facilities for
Individuals with Intellectual Disabilities (ICF/IID) offered by the Centers for Medicare and
Medicaid Services and asks for more information.
C. individuals can apply for and qualify for ICF/IID services even if they are not eligible
for Medicaid
E. to qualify for ICF/IID, individuals must receive or need intensive training, treatment,
and health services
PREP Pearls
• Intermediate Care Facilities for Individuals with Intellectual Disabilities must provide
a protected residential setting with 24-hour supervision and ongoing evaluation,
planning, coordination, and integration of services.
• Implementation and availability of these services varies by state and region.
Suggested Readings
• Centers for Medicare & Medicaid Services, US Department of Health and Human
Services. Intermediate Care Facilities for Individuals with Intellectual Disabilities
(ICF/IID). Centers for Medicare & Medicaid Services
website.http://www.cms.gov/Regulations-and-
Guidance/Legislation/CFCsAndCoPs/Intermediate-Care-Facilities-for-Individuals-
with-Intellectual-Disabilities-ICF-IID.html.
Question 223
A 6-year-old girl is struggling in first grade. Her teacher suggested that she be evaluated for
attention-deficit/hyperactivity disorder. In your office, the girl is very shy and sits quietly in a
chair, playing a game on a tablet computer for the entire visit. Her parents tell you she sometimes
does not hear them or forgets what she is supposed to be doing. They have also noticed staring
spells, lasting 5 to 10 seconds. These sometimes occur when she is in the middle of a sentence,
and when the spell is over, she picks right up where she left off and completes the sentence. Her
physical and neurological examinations are unremarkable.
Of the following, the MOST likely to reveal the cause of the girl’s symptoms is a(n)
A. electroencephalogram
Other causes of inattention do not typically cause the clinical presentation seen in the girl in the
vignette. The altered mental status seen in hypoglycemia lasts longer and does not include
automatisms, and seizures are typically generalized tonic-clonic convulsions, not absence
seizures. So it is unlikely that fasting blood glucose levels will lead to the correct diagnosis in
this girl. Similarly, anxiety does not cause brief periods of behavioral arrest with automatisms, so
evaluation with an anxiety scale would not be helpful in this case. Testing this girl for attentional
disorder would also not be helpful in revealing this girl’s diagnosis.
In absence seizures the brain is almost always structurally normal, therefore brain imaging does
not need to be performed for the girl in the vignette.
PREP Pearls
• Absence seizures typically present at about 5 years of age.
• Absence seizures are characterized by behavioral arrest for 5 to 10 seconds,
sometimes with an automatism such as eye fluttering, then a quick return to baseline.
Suggested Readings
• Floet AMW, Scheiner C, Grossman L. Attention-deficit/hyperactivity disorder.
Pediatr Rev. 2010;31(2):56-69. DOI:http://dx.doi.org/10.1542/pir.31-2-56.
• Sidhu R, Velayudam K, Barnes G. Pediatric seizures. Pediatr Rev. 2013;34(8):333-
342. DOI:http://dx.doi.org/10.1542/pir.34-8-333
Question 224
An 18-month-old girl presents to the emergency department because of increasing lethargy. Two
hours earlier, she was found playing with an open pill bottle containing her grandmother's
clonidine tablets. The family is uncertain whether she ingested any pills and the girl did not have
any pill fragments in her mouth. Her vital signs show a temperature of 37°C, heart rate of 80
beats/min, blood pressure of 65/25 mm Hg, respiratory rate of 20 breaths/min, and pulse
oximetry of 100% on room air. Physical examination reveals a sleepy child who opens her eyes
and cries with stimulation. Her pupils are 2 mm, equal, and reactive, there is no nystagmus, and
her gag reflex is intact. Her lungs are clear. The girl’s muscle tone is normal and she exhibits
purposeful movements in response to stimulation. Her extremities are cool with a capillary refill
of 4 seconds.
Clonidine is primarily an α-2 adrenergic agonist. In the central nervous system, it inhibits
presynaptic norepinephrine release and neurotransmission, causing sedation. This mechanism
also decreases sympathetic outflow from the vasomotor center, causing decreased arterial blood
pressure. Clonidine’s action as a partial α-2 antagonist in the medulla also stimulates the
excitatory cardiac vagal reflex and the inhibitory baroreceptor reflex, which also contribute to
decreased cardiac output. Through its peripheral α-1 agonist activity, clonidine can transiently
produce hypertension.
Clonidine is widely used in adults as an antihypertensive medication and for drug withdrawal
prophylaxis, and is used in children for its behavioral effects. It is a common agent identified in
both intentional and accidental poisonings in children. Hypotension occurs within 30 to 60
minutes of clonidine ingestion. Peak plasma concentrations occur within 3 to 5 hours, and the
plasma half-life is 12 to 16 hours. Symptoms of clonidine toxicity can mimic narcotic ingestion,
with miosis, lethargy, bradycardia, hypotension, and respiratory depression; some improvement
in mental status in response to naloxone can be seen. However, the degree of hemodynamic
compromise with bradycardia and hypotension is usually more marked in clonidine ingestion
than in opioid toxicity, whereas respiratory depression is typically not quite as severe. Other
common ingestions that can cause shock and hypotension include other antihypertensives,
benzodiazepines, barbiturates, iron, and tricyclic antidepressants. Treatment generally includes
fluid administration, decontamination, and, in more severe cases, administration of inotropic and
vasopressor agents. Specific antidotes exist depending on the agent, such as glucagon for
calcium channel blockers, and sodium bicarbonate for tricyclic antidepressants.
Treatment of shock and hypotension from a clonidine overdose should include aggressive
administration of crystalloid fluids to restore preload in the setting of vasodilation. However,
reversing the cardiovascular effects of vasodilation and decreased inotropy from reduced
sympathetic outflow may require an adrenergic agonist infusion such as dopamine, epinephrine,
or norepinephrine. Activated charcoal and whole bowel irrigation can be considered, but
restoration of circulation takes precedence. Sodium bicarbonate is helpful in tricyclic
antidepressant toxicity, but not for clonidine. Syrup of ipecac is no longer recommended for
childhood poisonings because of a lack of evidence that it improves outcomes and because it
may delay administration or reduce the effectiveness of other therapies, including activated
charcoal.
PREP Pearls
• Common pediatric medication ingestions causing hypotension include
antihypertensives, benzodiazepines, barbiturates, iron, and tricyclic antidepressants
• Clonidine is primarily a central α-2 agonist, causing sedation, respiratory depression,
and reduced sympathetic outflow, with the cardiovascular effects of hypotension and
bradycardia.
• Clonidine toxicity can mimic opioid toxicity, but usually presents with more severe
cardiovascular effects and less severe respiratory compromise.
Suggested Readings
• Fiser DH, Moss MM, Walker W. Critical care for clonidine poisoning in toddlers.
Crit Care Med. 1990;18(10):1124-1128. Abstract available at
http://www.ncbi.nlm.nih.gov/pubmed/2209040.
• Spiller HA, Klein-Schwartz, Colvin JM et al. Toxic clonidine ingestion in children. J
Pediatr. 2005;146(2):263-266. DOI:http://dx.doi.org/10.1016/j.jpeds.2004.09.027.
Question 225
A 12-year-old boy with Crohn disease is hospitalized with fever, abdominal pain, and bloody
diarrhea. He was recently hospitalized with blunt abdominal trauma complicated by intestinal
perforation. During the past hospitalization, he underwent resection of the terminal ileum and
received a course of piperacillin-tazobactam for peritonitis. Current vital signs show a
temperature of 39.6°C, respiratory rate of 24 breaths/min, heart rate of 143 beats/min, blood
pressure of 120/69 mm Hg, and a weight of 32.6 kg. On physical examination, he is pale and has
abdominal distention and diffuse tenderness. Laboratory data shows:
• White blood cells, 17,000/µL (17.0 x 109/L)
• Hemoglobin, 7.8 g/dL (78 g/L)
• Hematocrit, 25.9%
• Platelets, 260 x 103/µL (260 x 109/L)
• Differential, 52% segmented neutrophils, 20% bands, 8% lymphocytes, 17%
monocytes, 3% eosinophils
• Clostridium difficile polymerase chain reaction, positive
A. antibiotic susceptibility
B. episode number
C. prior surgery
D. severity of illness
E. underlying condition
It is important to know that testing for C difficile infection in infants is not recommended.
Additionally, testing for children between 1 and 3 years of age is recommended only in the
setting of diarrhea after evaluating for other (eg, viral) etiologies. Approximately one-third of
babies 0 to 1 months of age are carriers of C difficile. The carriage rate in children approximates
non hospitalized adult rates of 0% to 3% by the age of 3 years. Clinical illness is rarely reported
before 2 years of age.
Metronidazole resistance in C difficile is rare and does not influence management decisions.
While episode number does affect the choice of therapy, there is nothing to suggest a recurrent
episode of infection for the patient in this vignette. While previous surgery and underlying
conditions are considered risk factors for acquiring C difficile infections, they do not influence
management. High risk surgeries include any manipulation of the gastrointestinal tract, including
gastrostomy and jejunostomy tubes. Medical risk factors include underlying bowel disease and
impaired humoral immunity.
PREP Pearls
• Management of Clostridium difficile infection is influenced both by the severity of
the presentation and whether the current episode represents a disease recurrence.
• Testing for C difficile infection in infants is not recommended and testing for children
between 1 and 3 years of age is recommended only in the setting of diarrhea after
testing for other (eg, viral) etiologies.
• C difficile is a common health care-associated pathogen and environmental control is
critical in preventing its spread.
American academy of pediatrics 724
American Academy of Pediatrics PREP 2016
Suggested Readings
• American Academy of Pediatrics. Clostridium difficile infection in infants and
children. Pediatrics. 2013;13(1):196-200. DOI: http://dx.doi.org/10.1542/peds.2012-
2992 .
• American Academy of Pediatrics. Clostridium difficile. In: Kimberlin DW, Brady
MT, Jackson MA, Long SS, eds. Red Book:2015 Report of the Committee on
Infectious Diseases. 30th ed. American Academy of Pediatrics; 2015:298-301 .
Question 226
A 12-month-old girl is brought to your office for a regular health supervision visit. The parents
inform you that the patient was evaluated in the emergency department (ED) 2 days ago for
tactile temperatures at home. You review the records from the ED and make note that the patient
had a temperature of 38.9°C and an unremarkable physical examination. The patient had a
catheterized urine analysis showing a specific gravity of 1.030, a pH of 6.0, 3+ leukocyte
esterase, 1+ proteinuria, 1+ blood, and positive nitrites. Her urine microscopy showed 50 to 100
white blood cells/high power field (HPF), less than 5 red blood cells/HPF, bacteria, and no
crystals.
You discuss with the parents the sensitivity of urinalysis in diagnosing urinary tract infection
(UTI) in children.
Of the following, the positive results in this child with the HIGHEST sensitivity for
presumptively diagnosing UTI is
A catheterized or suprapubic aspirated (SPA) urine specimen for both urinalysis and culture
before an antibiotic is started should be sent in a febrile infant with no apparent source of
infection. Urine culture, on a sample collected by a bag applied to the perineum, has an
unacceptably high false-positive rate (88%) and has clinical implications only when cultures
yield negative results.
Both urinalysis results that suggest infection (pyuria and/or bacteriuria) and the presence of at
least 50,000 colony-forming units/mL of uropathogen cultured from an appropriately collected
urine specimen (obtained through catheterization or SPA) are required for confirming the
diagnosis of UTI. Urine test strip analysis cannot substitute for urine culture to document the
presence of UTI in a patient. The urine test strip analysis should be used in conjunction with
culture for diagnosing UTI. As urine culture results are not immediately available, a urine test
strip analysis suggestive of UTI diagnosis helps in presumptively treating a patient at initial
presentation. Urinalysis can be performed on a bag urine specimen. However, a properly
collected urine specimen via catheterization or SPA can be sent for both culture and urinalysis. It
is important that the urine specimen be tested within 1 hour after voiding, with maintenance at
room temperature or within 4 hours after voiding, with the specimen being kept refrigerated to
ensure sensitivity and specificity of the urinalysis.
In urinalysis, identification of leukocyte esterase (marker of pyuria or white blood cells in urine)
and nitrite on urine test strip analysis, and urine microscopic examination for white blood cells
(WBCs) and bacteria have been commonly used for diagnosing UTI. As seen in Item C226,
positive results for leukocyte esterase, nitrites, and urine microscopy has the highest sensitivity
(99.8%), followed by leukocyte esterase and nitrites (93%), and leukocyte esterase only (83%)
for diagnosing UTI.
In patients with asymptomatic bacteriuria, urine culture is positive and the leukocyte esterase test
is negative. Asymptomatic bacteriuria is often observed in school aged and older girls, and less
frequently in young infants. Asymptomatic bacteriuria should not be treated with antibiotics, as
antimicrobial treatment may do more harm than good. Asymptomatic bacteriuria can be
distinguished from true UTI by the presence of pyuria in UTI. False-positive (low sensitivity)
results for leukocyte esterase may be seen in patients with fever and from other causes or after
vigorous exercise. In the presence of pyuria, a UTI should be confirmed with a positive urine
culture result, as pyuria alone does not confirm the diagnosis of UTI.
A positive nitrite test is indicative of the conversion of dietary nitrates to nitrites by urinary
pathogens. This reaction requires approximately 4 hours, and is therefore not a sensitive marker
for identifying UTI in young children who void frequently. The test is also negative for urinary
pathogens (enterococcus) that do not reduce nitrate to nitrite. A positive urine nitrite test has high
specificity (98%) and therefore low false-positives.
Urine microscopy requires additional equipment and technical expertise. Standard urine
microscopy includes microscopic examination of unstained urine for white blood cells (pyuria >
5 WBCs/high power field or 25 WBC/µL) and bacteria (bacteriuria, presence of any bacteria per
high power field). Enhanced urinalysis includes Gram staining of uncentrifuged urine and using
a counting chamber for assessment of pyuria (> 10 WBC/µL) and bacteriuria (1 gram-negative
rod in 10 oil immersion fields). Enhanced urine analysis has been reported to have higher
sensitivity, specificity, and positive predictive value than the standard urinalysis, and is preferred
with the availability of equipment and trained personnel.
PREP Pearls
• A catheterized or suprapubic aspirated urine specimen for both urinalysis and culture
before an antibiotic is started should be sent in a febrile infant with no apparent source of
infection.
• Both urinalysis results that suggest infection (pyuria and/or bacteriuria) and the presence
of at least 50,000 colony-forming units per mL of an uropathogen cultured from an
appropriately collected urine specimen are required for confirming the diagnosis of
urinary tract infection (UTI).
• Positive leukocyte esterase test has high sensitivity, while positive urine nitrites have
high specificity for diagnosing UTI.
• In patients with asymptomatic bacteriuria, the urine culture is positive and leukocyte
esterase test is negative.
Suggested Readings
• American Academic of Pediatrics. Urinary tract infection: clinical practice guideline for
the diagnosis and management of the initial UTI in febrile infants and children 2 to 24
months. Pediatrics. 2011;128(3);595-610. DOI: http://dx.doi.org/10.1542/peds.2011-
1330.
• Finnell SM, Carroll AE, Downs SM. Technical report: diagnosis and management of an
initial UTI in febrile infants and young children. Pediatrics. 2011;128(3):e749-e770.
DOI: http://dx.doi.org/10.1542/peds.2011-1332.
Question 227
A 2-year-old boy is brought for evaluation after passing 3 large, maroon-colored bowel
movements in the last 12 hours. His parents deny recent fever, nausea, vomiting, diarrhea, or
abdominal pain. On physical examination, the patient’s heart rate is 140 beats/min and his blood
pressure is 80/50 mm Hg. He is somewhat pale, but well appearing with an unremarkable
abdominal examination.
A. barium enema
B. computerized tomography of the abdomen
C. plain radiograph of the abdomen
D. technetium-99m pertechnetate scintiscan
E. upper gastrointestinal series with small bowel follow through
Meckel diverticulum may present in several ways, including gastrointestinal bleeding, bowel
obstruction, and diverticulitis with or without perforation. In addition, in rare cases, a Meckel
diverticulum may be found in a hernia into the vitelline duct, resulting in umbilical drainage.
Obstructive symptoms can occur secondary to intussusception, hernia, or volvulus involving the
diverticulum. Obstruction is more common in adults than children. Patients present with
abdominal pain, distention, nausea, and vomiting. On physical examination, patients have
abdominal distention, tenderness with palpation, hypoactive bowel sounds, and sometimes a
mass or signs of peritonitis.
Infrequently, symptoms are similar to those of appendicitis with fever, right lower quadrant
abdominal pain, nausea, and vomiting. The physical examination is the same as an acute
appendicitis.
Evaluation for Meckel diverticulum should include a complete blood cell count and basic
metabolic panel to evaluate for dehydration and anemia. Imaging studies may include an
abdominal radiograph, contrast study of the bowel, ultrasound, computed tomography, or
magnetic resonance imaging of the abdomen and pelvis. These studies are all nonspecific and are
best used in cases when obstruction is suspected. For patients with a bleeding presentation, such
as the child in this vignette, a technetium-99 pertechnectate scan (Meckel scan) is the best study
for evaluation and diagnosis.
The sensitivity and specificity of a Meckel scan are 80% to 90% and 95%, respectively, but this
decreases with age. Once identified, excision is the treatment of choice for Meckel diverticulum.
The child in the scenario has a Meckel diverticulum and is presenting with a gastrointestinal
bleed, best identified by technetium-99 pertechnectate scan. Barium enema is best used for
evaluation of constipation and adds little to an evaluation for acute gastrointestinal bleeding.
Computed tomography of the abdomen may identify obstruction and inflammation, but is
unlikely to provide diagnosis in this young child. A radiograph of the abdomen will evaluate for
a foreign body, pneumatosis, and evidence of obstruction, but will not make the diagnosis of
Meckel diverticulum. Maroon-colored stools in this child are indicative of bleeding from the
lower gastrointestinal tract, so an upper gastrointestinal series with small bowel follow through is
unlikely to provide a diagnosis.
PREP Pearls
• Meckel diverticulum are most likely to present in the first 2 years of life.
• Meckel diverticulum can present with gastrointestinal bleeding, obstruction, or
inflammation with perforation.
• Technetium-99 pertechnectate scan is the best test to diagnose a Meckel diverticulum
with ectopic gastric tissue.
Suggested Readings
• Murphy SM. Management of bloody diarrhoea in children in primary care. BMJ.
2008;336(7651):1010-1015. DOI: http://dx.doi.org/10.1136/bmj.39542.440417.BE.
• Pepper VK, Stanfill AB, Pearl RH. Diagnosis and management of pediatric appendicitis,
intussusception, and Meckel diverticulum. Surg Clin North Am. 2012;92(3):505-526.
DOI: http://dx.doi.org/10.1016/j.suc.2012.03.011.
Question 228
A 3-year-old boy with a history of learning disabilities presents to your office as a new patient
with macrocephaly, axillary and inguinal freckling, short stature, and 8 café-au-lait macules over
5 mm in diameter (Item Q228). His blood pressure is 130/88 mm Hg.
Given his genetic disorder, the MOST likely cause of his hypertension is
Hypertension is common in NF1 and can occur at any age. In many cases, it is classified as
essential hypertension; however, a subset of individuals will have a characteristic NF1
vasculopathy that leads to renal artery stenosis, aneurysms, aortic stenosis, or coarctation of the
aorta. Renal artery stenosis could be intrinsic in nature, arising from arterial dysplasia, or
extrinsic, caused by a plexiform neurofibroma or other abdominal mass. Therefore, a
renovascular or cardiovascular cause of hypertension should be investigated in any patient with
an NF1 clinical phenotype. Pheochromocytoma manifesting as hypertension is more common in
adults with NF1. Ambulatory blood pressure monitoring should be performed at least once
annually in patients with NF1.
Hyperthyroidism and neuroblastoma can cause hypertension, but are not frequent in patients with
NF1. Most individuals with neuroblastoma have sporadic neuroblastoma. Congenital heart
defects can occur in NF1, more commonly associated with whole gene deletions. Most valvular
problems become evident at later ages and are caused by NF1 vasculopathy. The most common
valvular problem is pulmonic stenosis.
Wilms tumor is an embryonal malignancy of the kidney that is the most common renal tumor in
childhood. It can be found in WAGR (Wilms tumor-aniridia-genital anomalies-retardation)
syndrome, Denys-Drash syndrome, Beckwith-Wiedemann syndrome, and hemihyperplasia, but
not commonly with NF1.
PREP Pearls
• Neurofibromatosis 1 (NF1) is characterized by multiple café-au-lait macules, axillary and
inguinal freckling, cutaneous and plexiform neurofibromas, and iris Lisch nodules.
• Hypertension is common in NF1 patients occurring at any age and requires careful blood
pressure monitoring.
• Neurofibromatosis 1 vasculopathy can lead to renal artery stenosis, aneurysms, valvular
stenosis, or coarctation of the aorta.
Suggested Readings
• Friedman JM. Neurofibromatosis 1. GeneReviews.
http://www.ncbi.nlm.nih.gov/books/NBK1109/.
• Han M, Criado E. Renal artery stenosis and aneurysms associated with
neurofibromatosis. J Vasc Surg. 2005;41(3):539-543.
• Hersh JH. Health supervision for children with neurofibromatosis. Pediatrics.
2008;121(3):633-642. DOI: http://dx.doi.org/10.1542/peds.2007-3364.
Question 229
3-day-old, full-term female newborn is brought to the office for a scheduled visit. The mother’s
pregnancy, labor, and delivery were uncomplicated, and the infant was discharged from the
hospital 24 hours after birth. Her parents report that she has been doing well, with normal
feeding and stooling patterns. However, today they noticed a red “blotchy” rash on her body. The
rash seems to come and go. On physical examination, you note an erythematous macular
eruption with a few 1- to 2-mm papules and pustules, mostly on the face and trunk (Item Q229).
The examination is otherwise unremarkable.
Of the following, the MOST likely cytologic finding associated with this rash is
Differentiation from other pustular eruptions of the newborn is important to rule out more serious
infectious or dermatologic conditions. Usually these rashes can be distinguished by the history
and clinical findings; however, cytologic findings can be helpful in making the diagnosis.
Scrapings of scabies lesions may reveal mites, eggs, or excrement on mineral oil preparation. A
gram-positive cocci found on staining of pustular material is typical of staphylococcal
folliculitis. A Tzanck smear of herpetic lesions may demonstrate multinucleated giant cells. If
pseudohyphae are seen on potassium hydroxide preparation, congenital candidiasis is the most
likely cause of the pustular eruption.
PREP Pearls
• Erythema toxicum neonatorum is a benign, asymptomatic, self-limited condition usually
diagnosed clinically, that occurs most often in full-term newborns.
• Erythema toxicum neonatorum usually appears in the first 4 days and tends to remit and
recur during the first 2 weeks after birth.
• A predominance of eosinophils will be revealed on a Wright or Giemsa stained smear of
pustular material from an infant with erythema toxicum neonatorum.
Suggested Readings
• Udassi S, Marshall S, Acharya R, Tuli SY. Erythema toxicum neonatorum. Cons
Pediatricians. 2014;13(11):494 .
• Johr RH, Schachner LA. Neonatal dermatologic challenges. Pediatr Rev. 1997;18(3):86-
94 http://www.newnanpediatrics.com/resources/Student-Articles/Neonatal-Dermatologic-
Challenges.pdf .
• Paller AS, Mancini AJ. 2011. Cutaneous disorders of the newborn. In: Hurwitz Clinical
Pediatric Dermatology: A Textbook of Skin Disorders of Childhood and Adolescence.
4th ed. Philadelphia, PA: Saunders Elsevier; 2011:16-18.
Question 230
You see two 8-week-old infants consecutively in your general pediatrics practice for health
supervision visits. The first infant was born at 30 weeks gestational age, while the second infant
was born full term. The medical student working with you asks what will happen to the
hematocrit of the preterm infant relative to the full term infant.
Of the following, you inform the medical student that, compared with the full term infant, this
preterm infant will
A. reach the nadir of erythrocyte production at the same time, but will recover faster
B. reach the nadir of erythrocyte production at the same time, but will require longer to
recover
C. reach the nadir of erythrocyte production sooner and will require longer to recover
D. reach the nadir of erythrocyte production sooner, but will recover faster
E. take longer to reach the nadir of erythrocyte production and will require longer to recover
Other factors that can lead to the anemia of prematurity include repeated phlebotomy in sick or
premature neonates, a reduced lifespan for the red blood cells, and iron depletion. Although iron
depletion is not the cause of the anemia of prematurity, it may impair the ability to recover.
Given the reduced iron stores present in the premature infant, it is important to initiate iron
supplementation by 8 weeks of life.
PREP Pearls
• The greatest increase in fetal red blood cell mass occurs in the final trimester of
pregnancy, so premature infants are born with a lower hematocrit than full term infants.
• Premature infants have impaired production of erythropoietin.
• When the impaired production of erythropoietin is combined with the lower hematocrit at
birth, this results in an earlier, deeper, and longer nadir in the hematocrit. This is known
as the anemia of prematurity.
Suggested Readings
• Bishara N, Ohls RK. Current controversies in the management of the anemia of
prematurity. Semin Perinatol. 2009;33(1):29-34. DOI:
http://dx.doi.org/10.1053/j.semperi.2008.10.006.
• Kett JC. Anemia in infancy. Pediatr Rev. 2012;33(4):186-187. DOI:
http://dx.doi.org/10.1542/pir.33-4-186.
• Roth P. Anemia in preterm infants. Pediatr Rev. 1996;17(30):370.
http://pedsinreview.aappublications.org/content/17/10/370 .
Question 231
An 8-year-old girl is brought to see you because of school problems. Her school performance has
been poor this year, and her teacher reports that she has had poor attention in class. The mother
says that for more than 2 months she has resisted and sometimes refused to go to school, and
when at school, she sometimes goes to the office crying that she needs to go home. She denies
having any peer problems at school. At home, she has developed some sleep problems as well,
which the mother has addressed by staying in her room with her until she falls asleep. She has a
significant stressor in that her close friend’s family moved away this past summer, and she talks
about missing her friend. When you talk to the girl, she tells you that she has a specific worry
that someone will hurt her mother when she is not there. Her mother emphatically denies that
domestic violence is occurring at home.
A. adjustment disorder
B. attention-deficit/hyperactivity disorder
C. oppositional defiant disorder
D. separation anxiety disorder
E. specific phobia
Separation anxiety disorder may include physical symptoms, such as stomach aches or
headaches when anticipating or experiencing a separation, which a parent might misinterpret as
evidence of a medical disorder. First line treatment includes cognitive behavioral therapy and
parental coaching for how to help their child master their fears. When a child has a separation
anxiety disorder, there often are other family members with anxiety difficulties.
PREP Pearls
• Some degree of separation anxiety is often seen in the preschool years and during the first
month of school entry, which parents and teachers resolve through reassurance.
• Separation anxiety disorder should be suspected in cases of school refusal or when
separation anxiety lasts more than 4 weeks.
Suggested Readings
• Bagnell AL. Anxiety and separation disorders. Pediatr Rev. 2011;32(10):440-446. DOI:
http://dx.doi.org/10.1542/pir.32-10-440.
• Ginsburg GS, Kendall PC, Sakolsky D, et al. Remission after acute treatment in children
and adolescents with anxiety disorders: findings from the CAMS. J Consult Clin Psychol.
2011;79(6):806-813. DOI: http://dx.doi.org/10.1037/a0025933.
American academy of pediatrics 742
American Academy of Pediatrics PREP 2016
• Mohatt J, Bennett SM, Walkup JT. Treatment of separation, generalized, and social
anxiety disorders in youths. Am J Psychiatry. 2014;171:741-748. DOI:
http://dx.doi.org/10.1176/appi.ajp.2014.13101337.
Question 232
You are called to evaluate a term female newborn who is noted to have lymphedema. Her
prenatal ultrasonography suggested a horseshoe kidney. She is in respiratory distress, requires
intubation, and is stabilized from a respiratory standpoint. During intubation, you notice that her
neck has thickened folds. On physical examination, her heart rate is 160 beats/min, blood
pressure is 90/60 mm Hg in her right arm, and 70/50 mm Hg in her right leg. Her oxygen
saturation is 95% in the right arm. She is noted to have a 1/6 systolic murmur at the right upper
sternal border. Her femoral pulses are 1+. You are worried about congenital heart disease. You
consult cardiology and genetics to help evaluate the newborn.
A. DiGeorge syndrome
B. Down syndrome
C. Kartagener syndrome
D. Marfan syndrome
E. Turner syndrome
Among the other responses, none would be as likely to be seen in females with the constellation
of findings described. Patients with Down syndrome can have lymphedema, but more commonly
have an atrioventricular (AV) septal defect (or AV canal) and no murmur at birth. They are much
less likely to have a coarctation of the aorta. DiGeorge syndrome would be expected in patients
with conoseptal defects such as tetralogy of Fallot or an interrupted aortic arch. Kartagener
syndrome (lack of normal ciliary function and chronic infections) is associated with heterotaxy
lesions (abnormal orientation of internal organs), single ventricle physiology (often with an AV
canal type defect), and abnormal pulmonary vein anatomy. Typically, this would not feature
aortic position murmurs or lymphedema as seen in the newborn in this vignette. Marfan
syndrome is associated with aortic root dilation that is not likely to present in the newborn
period. It does not cause coarctation of the aorta.
PREP Pearls
• Turner syndrome is associated with coarctation of the aorta and bicuspid aortic valve and
is seen in females only.
• Lymphedema is common in newborns with Turner syndrome.
Suggested Readings
• American Academy of Pediatrics. Care of girls and women with Turner syndrome: a
guideline of the Turner syndrome study group. Pediatrics. 2009;123(5):1423. DOI:
http://dx.doi.org/10.1542/peds.2009-0394.
• Bondy C, Bakalov VK, Cheng C, Olivieri L, Rosing DR, Arai AE. Bicuspid aortic valve
and aortic coarctation are linked to deletion of the X chromosome short arm in Turner
syndrome. J Med Genet. 2013;50(10):662-665. DOI:
http://dx.doi.org/10.1136/jmedgenet-2013-101720.
• Goldmuntz E, Crenshaw M, Lin AE. Genetic aspects of congenital heart defects. In:
Allen HD, Driscoll DJ, Shaddy RE, Feltes TF, eds. Moss and Adams’ Heart Disease in
Infants, Children, and Adolescents: Including the Fetus and Young Adult. 8th ed.
Philadelphia, PA: Lippincott Williams & Wilkins; 2013:617-643.
• Park MK. Obstructive lesions. Park’s Pediatric Cardiology for Practitioners. Philadelphia,
PA: Saunders Elsevier; 2014:184-205.
• Wong SC, Burgess T, Cheung M, Zacharin M. The prevalence of Turner syndrome in
girls presenting with coarctation of the aorta. J Pediatr. 2014;164(2):259-263. DOI:
http://dx.doi.org/10.1016/j.jpeds.2013.09.031.
Question 233
You are seeing a 7-year-old boy in your office, which is a medical home with comprehensive
care systems available. The boy, recently diagnosed with Becker muscular dystrophy, was
referred to neurology 2 weeks ago for a positive Gower sign and elevated muscle enzymes. He
has normal intelligence and is able to walk and perform activities of daily living normally. The
family is struggling to deal with this new diagnosis. They have concerns regarding his prognosis,
educational needs, and care planning.
Of the following, the MOST accurate statement regarding this child is that
A. the family should contact the Department of Health for community resources
B. he is not disabled enough to qualify for educational modifications
C. he will need monthly health maintenance visits
D. his neurologist should be his primary medical provider
E. your office will coordinate specialist referrals for disease management
Significant needs and psychosocial factors affect the families of CSHCN. Most will have a
significant financial burden because of medical expenses that are not covered by insurance.
Household income may be reduced when 1 or both parents are needed as full or part-time
caregivers for the child, and are therefore unable to work or must have reduced work hours.
Many families have difficulty navigating the medical system. CSHCN are frequently seen by
multiple specialists in various settings such as the hospital, emergency room, and outpatient
office. Navigating these entry points can be difficult. Other unmet needs of CSHCN include
mental health services to help the patient and family cope with the medical condition and its
impact on their lives. Many CSHCN also need specialized equipment, including communication
and mobility aids. The medical home model offers needed family support along with care
coordination. Information for providers regarding setting up a medical home can be found online
at http://www.medicalhomeinfo.org and http://www.medicalhomeportal.org/diagnoses-and-
conditions.
Families of CSHCN often need to work with the school district to provide appropriate
educational opportunities and improve educational outcomes. Children with disabilities will
qualify for special education under the Individuals with Disabilities Education Improvement Act
of 2004. Families of children eligible for this program work with their schools to develop
individualized education plans (IEPs). The parents of children with disabilities who do not
qualify for special education services may request specific accommodations such as assistive
technology, extra time for school work because of hospitalization, and physical assistance such
as wheelchair ramps or special seating arrangements. These resources are available through the
Rehabilitation Act of 1973, Section 504. The primary care provider should be able to discuss the
programs available with families of CSHCN.
PREP Pearls
• As a medical home, the pediatric practitioner’s office should coordinate specialist
referrals for disease management.
• Any child with a chronic medical condition can qualify for a 504 educational plan
designed to prevent the condition from having a significant impact on the child’s
academic career.
• Children with disabilities qualify for special education under the Individuals with
Disabilities Education Improvement Act of 2004
Suggested Readings
• American Academy of Pediatrics Council on Children With Disabilities. Provision of
educationally related services for children and adolescents with chronic diseases and
disabling conditions. Pediatrics. 2007;119:1218-1223. DOI:
http://dx.doi.org/10.1542/peds.2007-0885.
• Sadof MD, Nazarian B. Caring for children who have special health-care needs: a
practical guide for the primary care practitioner. Pediatr Rev. 2007;28:e36-e42. DOI:
http://dx.doi.org/10.1542/pir.28-7-e36.
Question 234
A 17-year-old adolescent presents to your office for a preparticipation physical examination.
You inquire about nutritional supplement and drug use. He tells you that he takes creatine to
enhance his performance in lacrosse at the recommendation of his coach. He plans to play
lacrosse at the college level next year. You discuss the use of creatine and other supplements
with him.
Of the following, the MOST accurate statement to include in your discussion is that
Physicians should discuss supplement use with young patients because children and adolescents
may harbor misconceptions about their risks and benefits.
Creatine has rarely been associated with renal failure in case reports; in most cases, the
individuals affected had underlying kidney disease. Creatine does not increase the risk of
rhabdomyolysis or arrhythmias. Creatine has not been banned by any major sports organization,
such as the National College Athletic Association. However, athletes should be counseled that
legal supplements can be contaminated with other substances that may be banned.
PREP Pearls
• Creatine can enhance performance in sports that involve brief, high-intensity activities,
such as shot put, but it does not improve endurance.
• The use of supplements that typically have mild or no adverse effects, like creatine and
caffeine, is an ethical gray area.
Suggested Readings
• Castillo EM, Comstock RD. Prevalence of use of performance-enhancing substances
among United States adolescents. Pediatr Clin North Am. 2007;54(4):663-675, ix-x. .
• Dandoy C, Gereige RS. Performance-enhancing drugs. Pediatr Rev. 2012;33(6):265-271;
quiz 271-262. DOI: http://dx.doi.org/10.1542/pir.33-6-265.
Question 235
An 8-year-old, previously healthy, fully immunized boy presents to your office with bilateral
facial swelling that developed over the last 24 hours. Throughout the past week, the patient had a
subjective fever, myalgias, and a mild cough. On physical examination, his temperature is
37.5°C, heart rate is 100 beats/min, and respiratory rate is 18 breaths/min. The boy’s voice is
hoarse. There is bilateral, tender, firm, nonerythematous swelling of the pre-auricular area that
extends to the angle of the mandible. His teeth and gingiva are normal, with mild erythema of the
buccal mucosa. His posterior oropharynx is erythematous with enlarged tonsils without exudates.
He has clear rhinorrhea, erythema of the nasal mucosa, and mildly hyperemic and retracted
tympanic membranes. There is shotty bilateral cervical lymphadenopathy. The remainder of the
physical examination is unremarkable.
A. HIV
B. Klebsiella species
C. mumps
D. parainfluenza virus
E. Staphylococcus aureus
There are many causes of parotitis in children (Item C235); bilateral involvement in a non–toxic-
appearing child is most likely viral in origin. Parotitis can be the first manifestation of human
immunodeficiency virus (HIV) infection acquired at birth in an otherwise healthy child, but HIV
is a less common cause of bilateral parotitis than other viruses in healthy children. Mumps
parotitis continues to occur in individuals who are unimmunized or have waning immunity, but
most often is preceded by a prodrome of fever, headache, anorexia, and malaise.
Acute bacterial parotitis is most commonly caused by Staphylococcus aureus in all age groups.
Patients typically present with fever and the sudden onset of swelling, warmth, and tenderness
over the parotid gland; disease typically is unilateral, and purulent discharge may be visible from
the Stensen duct on the affected side. Acute bacterial parotitis more commonly occurs in
neonates, the elderly, and hospitalized patients, with gram-negative (eg, Klebsiella species) and
fungal infections occurring in addition to those caused by Staphylococcus aureus. Parotitis
caused by anaerobic bacteria (eg, Peptostreptococcus, Prevotella) usually is associated with
dental infection.
Patients with common viral causes for parotitis (including mumps) are treated with analgesics,
antipyretics, and hydration. Patients with HIV infection should be evaluated for antiretroviral
therapy. Treatment of acute bacterial parotitis should be directed toward the most likely
causative organisms.
PREP Pearls
• The differential diagnosis of preauricular swelling includes the following conditions:
parotitis, preauricular cyst, sebaceous cyst, lymphadenitis, sialolithiasis, tuberculosis,
actinomycosis, and neoplasm.
• Mumps parotitis continues to occur in unimmunized individuals or those with waning
immunity.
• The treatment of viral parotitis is supportive.
• Staphylococcus aureus is the most common cause of acute bacterial parotitis in all age
groups.
• Acute bacterial parotitis is most commonly seen in neonates, the elderly, and hospitalized
patients.
Suggested Readings
• American Academy of Pediatrics. Mumps. In: Kimberlin DW, Brady MT, Jackson, MA,
Long SS, eds. Red Book: 2015 Report of the Committee on Infectious Diseases. 30th ed.
Elk Grove Village, IL: American Academy of Pediatrics; 2015: 564-568.
• Campbell JR. Parotitis. In: Cherry J, Demmler-Harrison GJ, Kaplan SL, Steinbach WJ,
Hotez P, eds. Feigin & Cherry’s Textbook of Pediatric Infectious Diseases. 7th ed.
Philadelphia, PA: Elsevier Saunders; 2014:189-193.
Question 236
You are seeing a 19-month-old boy in your office for follow-up 2 days after a 5-day
hospitalization for treatment of orofacial burns. The boy chewed on a frayed extension cord last
week and he sustained a 1 x 1.5 cm full-thickness burn to his right oral commissure and lower
lip, as well as a small partial-thickness burn to the right side of his chin.
His parents tell you that the boy has been doing well since discharge from the hospital. He has
had no fevers and he has been able to tolerate fluids and soft foods. His pain has been well
controlled. In your office, the boy’s vital signs are normal for his age and he appears well.
Physical examination reveals only the healing burns at the right oral commissure and lower lip
with eschar formation, and a small healing, partial thickness burn to the right side of his chin.
There are no signs of infection. The boy’s mucous membranes are moist, and his extremities are
warm and well-perfused. The boy’s parents ask you about potential for delayed complications
from his recent injury.
All pediatric providers should understand the sequelae associated with electrical burns. Electrical
burns are responsible for more than 4,000 emergency department visits and more than 1,500
deaths annually. Many of these injuries involve children. Up to one-third of electrical burns are
household burns, which typically occur when young children come into contact with low-voltage
(<200 V) alternating current from household appliances, extension cords, or place objects into
electrical outlets. Medium (200-1,000 V) and high-current (>1,000 V) injuries are typically seen
in adolescents as a consequence of risk-taking behaviors, utility workers, and people exposed to
electrical storms.
Electrical burn injuries arise from thermal energy that is released as an electrical current passes
through the body. The degree of thermal energy produced depends on the voltage of electric
current contacted. The extent of electrical injury depends on the resistance of body structures
through which the current passes, the type of current (alternating or direct), the duration of
contact with the electrical source, the intensity of the current, and the pathway travelled by the
current. Current preferentially flows through body tissues with low electrical resistance, such as
blood vessels, nerves, and muscles. Alternating current is more dangerous than direct current,
causing muscle tetany because of the continual contraction and relaxation of the muscle with
each cycle.
Acute hemolytic anemia is not a typical complication seen after electrical burn injuries.
Acute renal failure may occur in patients with severe burn injuries, because of direct injury to the
kidney, renal hypoperfusion, or excessive deposition of myoglobin from injured muscle tissue. It
would be very unlikely to develop in this boy, whose injuries were limited to his mouth and right
side of the chin, and who is well-appearing with normal urine output 1 week after the injury.
Although victims of severe electrical injuries, including lighting strikes, can present with hearing
loss, the development of progressive sensorineural hearing loss in this boy over the next 6 to 12
weeks is extremely unlikely. Damage to the central nervous system including brain, spinal cord,
peripheral nerves, and sympathetic fibers may result from electrical injuries. Immediate clinical
manifestations may include pain, loss of consciousness, respiratory center paralysis, confusion,
visual changes, deafness, sensory deficits, seizures, motor deficits, memory loss, and intracranial
American academy of pediatrics 757
American Academy of Pediatrics PREP 2016
Acute myocardial injury and life-threatening ventricular arrhythmias can occur as an immediate
consequence of electrical injuries. If an electrical current flows across the heart (which occurs
more often when the flow takes an arm-to-arm pathway), myocardial injury and cardiac
arrhythmias (including ventricular fibrillation or asystole) may occur. However, in the acute
setting, a patient with a normal electrocardiogram does not appear to be at significant risk for
later development of arrhythmias.
Musculoskeletal sequelae may arise from electrical injuries, with the potential for tissue to
become edematous and necrotic, or the development of compartment syndrome. Muscle tetany
can result when there is contact with alternating electrical current. Respiratory failure may result
if tetany involves the chest wall muscles.
Renal failure can arise from the direct effect of electrical current on kidney tissue, decreased
perfusion to the kidney, or renal tubular injury secondary to deposition of myoglobin from
extensive muscle damage.
Pulmonary, abdominal, and ocular complications (including the development of cataracts) may
also occur, but these are not common sequelae of electrical burn injuries.
In terms of prevention of electrical injuries, it is essential that caregivers keep all electrical
outlets covered with safety covers and keep children away from all electrical cords. Adolescents
should be counseled on the dangers of climbing electrical structures and taught to avoid other
risk-taking behaviors. To avoid potentially life-threatening injury from lighting strikes, children
should be taught that during storms they must stay indoors, exit the water immediately, and
avoid contact with metal objects.
PREP Pearls
• Injury to the oral cavity and lips from low-voltage household electrical current typically
occurs when an infant places a household electrical cord in his or her mouth.
• Oral cavity and lip burns require very close follow-up to prevent severe scarring and
contractures. Patients are at risk for severe bleeding from the labial artery when the
eschar separates 1 to 2 weeks after the injury.
• Up to one-third of electrical burns are household burns, which typically affect young
children who place objects into electrical outlets or come into contact with low-voltage
(<200 V) alternating current from household appliances or extension cords.
• Acute myocardial injury and life-threatening ventricular arrhythmias can occur as an
immediate consequence of electrical injuries. Patients with normal electrocardiograms in
the acute setting do not appear to be at significant risk for developing arrhythmias in the
future.
Suggested Readings
• Joffe MD. Burns. In: Fleisher GR, Ludwig S, eds. Textbook of Pediatric Emergency
Medicine. 6th ed. Philadelphia, PA: Lippincott Williams & Wilkins; 2010:1281-1288.
• Pham TN, Gibran NS. Thermal and electrical injury. Surg Clin North Am.
2007;87(1)185-206. DOI: http://dx.doi.org/10.1016/j.suc.2006.09.013.
• Reed JL, Pomerantz WJ. Emergency management of pediatric burns. Pediatr Emerg
Care. 2005;21(2):118-129.
Question 237
You are seeing a 1-week-old newborn in your office during a newborn follow-up visit. The
mother is worried because her newborn has not begun to “unfold” after being born. The mother
tells you that the obstetrician found decreased amniotic fluid on the fetal ultrasonography 6
weeks before her infant was born. Since birth, her newborn always sleeps with her legs flexed up
towards her abdomen, with her head tilted to the side. While being held in her mother’s arms,
you see an active newborn with her head tilted to the right, her chin tilted to the left, and
adduction of both forefeet (Item Q237).
Item Q237. Forefoot of the infant described in the vignette. Reprinted with permission from
Smith BG. Pediatr Rev. 2009;30(8):287-294.
Of the following, the finding MOST likely to be appreciated during this newborn’s examination
is
A. camptodactyly
B. clavicular crepitus
C. genu recurvatum
D. hip dislocation
E. jaw subluxation
The pelvic obliquity seen in MBS leads to an abduction contracture of the hip on the concave
side of the newborn, whereas the contralateral side develops an adduction contracture (Item
C237). This positioning worsens between 2 and 5 months after birth as the legs become more
extended, with an increased risk of hip dislocation on the adducted side. A recent single-center
study found that 12% of infants with CMT had DDH. Although the American Academy of
Pediatrics does not include CMT as a risk factor in the screening guidelines for DDH, some
authors suggest obtaining an ultrasound or anteroposterior pelvis radiograph in affected infants at
the same age as standard DDH screening guidelines. Malposition of the knees and feet may
occur with MBS. Congenital talipes equinovarus (clubfoot), metatarsus adductus, and tibial
torsion have also been associated with intrauterine compression.
Item C237. A newborn with moulded baby syndrome. Reprinted with permission from Buxton
RA. The moulded baby syndrome. Curr Orthop. 2001;15(6):439-443.
Torticollis is the most commonly associated finding in infants with plagiocephaly. Infants with
torticollis often have some deformation of the occiput in utero, and some authors postulate that
affected infants preferentially sleep on these flat occipital surfaces, exacerbating the development
of plagiocephaly. Early referral to a physical therapist is recommended to teach caregivers to
perform and monitor stretching exercises. Health care providers should also encourage “tummy
time” while the infants are awake and monitored, to decrease time spent supine with the head in
the infant’s “preferred” position.
Genu recurvatum, or congenital dislocation of the knee, is a rare condition that can be either a
malformation associated with a genetic condition, such as Larsen syndrome, or an isolated
deformation related to oligohydramnios. Jaw subluxation is extremely rare in infants and a few
case reports document an association with trauma. Camptodactyly is a flexion deformity of the
proximal interphalangeal joints with the fifth finger always affected, usually with a genetic
cause, and is autosomal dominant. Clavicular crepitus suggests an underlying fracture that might
cause preferential head positioning. None of these findings are commonly associated with MBS.
PREP Pearls
• Moulded baby syndrome (MBS) may include head molding, postural torticollis,
congenital scoliosis, pelvic obliquity with altered hip movement, and malposition of the
knees and feet.
• MBS has been attributed to fetal positioning against the mother’s spine, along with
prolonged intrauterine constraint later in pregnancy
• Torticollis is the most commonly associated finding in infants with plagiocephaly.
• Although the American Academy of Pediatrics does not include congenital muscular
torticollis in the screening guidelines for developmental dysplasia of the hip, some
authors suggest ultrasonography or anteroposterior pelvis radiography in affected infants.
Suggested Readings
• Buxton RA. The moulded baby syndrome. Curr Orthop. 2001;15:439-443. DOI:
http://dx.doi.org/10.1054/cuor.2002.0232.
• Joiner ER, Andras LM, Skaggs DL. Screening for hip dysplasia in congenital muscular
torticollis: is physical examination enough? J Child Orthop. 2014;8:115-119. DOI:
http://dx.doi.org/10.1007/s11832-014-0572-5.
• Kuo AA, Tritasavit S, Graham JM. Congenital muscular torticollis and positional
plagiocephaly. Pediatr Rev. 2014;35:79-87. DOI: http://dx.doi.org/10.1542/pir.35-2-79.
• Rubio AS, Griffet JR, Caci H, Berard E, El Hayek T, Boutte P. The moulded baby
syndrome: incidence and risk factors regarding 1,001 neonates. Eur J Pediatr.
2009;168:605-611. DOI: http://dx.doi.org/10.1007/s00431-008-0806-y.
• Smith BG. Lower extremity disorders in children and adolescents. Pediatr Rev.
2009;30:287-294. DOI: http://dx.doi.org/10.1542/pir.30-8-287.
Question 238
A 14-year-old adolescent girl presents to your office with right hand and arm pain for the past
month, which is worse at night. The pain is most severe in the palm of her hand and involves her
arm up to her shoulder. She has numbness and tingling in her index and middle fingers. The
patient states that running warm water over her hand helps. She had been taking ibuprofen, but
states that it is not helping anymore. She denies hand or arm weakness. There is no history of
trauma. She plays violin in the school orchestra and is having difficulty now because of the pain.
Her physical examination is unremarkable, with no skin rashes or discolorations. Her
neurological examination is also unremarkable. She has normal sensation and strength in the
entire right upper extremity.
Carpal tunnel syndrome starts with intermittent numbness and tingling in the wrist and first 3
digits of the hand. Pain occurs in the same area and commonly extends up to the shoulder. The
absence of neck pain helps differentiate carpal tunnel syndrome from a cervical radiculopathy.
As carpal tunnel syndrome progresses, the pain, numbness, and tingling become constant and are
often worse at night. Findings on neurologic examination and electromyography/nerve
conduction study are often normal until the later stages. The diagnosis is usually made clinically,
and if treated in the early stages, progression can be prevented.
The carpal tunnel syndrome seen in the girl in the vignette is likely the result of repetitive hand
movements from playing violin. She may need to change her technique or shorten playing times.
If the pain is severe and persistent (=6 months), hand muscle weakness or evidence of
denervation on electromyography/nerve conduction study may be seen; then surgical
decompression should be considered.
Pain clinic referral may provide symptomatic relief; however, this is not the best choice to treat
mild symptoms. The other response choices are not effective treatments for carpal tunnel
syndrome.
PREP Pearls
• Violin, viola, guitar, and clarinet players are at risk for carpal tunnel syndrome.
• In carpal tunnel syndrome, pain can involve the entire arm but spares the neck.
Suggested Readings
• Dawson DM. Entrapment neuropathies of the upper extremities. N Engl J Med.
1993;329(27):2013-2018. DOI: http://dx.doi.org/10.1056/NEJM199312303292707.
• O'Connor D, Marshall S, Massy-Westropp N. Non-surgical treatment (other than steroid
injection) for carpal tunnel syndrome. Cochrane Database Syst Rev. 2003;(1):CD003219.
DOI: http://dx.doi.org/10.1002/14651858.CD003219.
Question 239
A 16-year-old African American adolescent presents to the emergency department in his
hometown in Wisconsin, where it is nearing the end of winter. He complains of cramps and
contractions in his hands that have lasted for hours. He experienced tingling in his arms and legs
during basketball practice intermittently over the last 2 weeks, but assumed this was because of
dehydration. He is concerned because the cramping in his hands today did not resolve with fluid
intake.
While taking vital signs, the technician in the emergency department accidently left the blood
pressure cuff inflated on the patient’s arm. After 2 minutes, the adolescent’s hand cramping
significantly worsened. His wrist and metacarpophalangeal joints flexed and his fingers
adducted. You quickly remove the blood pressure cuff and examine the patient. His physical
examination is unremarkable, except for facial twitching at the nose and lip, elicited when
tapping the angle of the jaw.
Of the following, the measurement MOST likely to reveal the cause of the patient’s symptoms is
A. 1,25-dihydroxyvitamin D
B. 25-hydroxyvitamin D
C. alkaline phosphatase
D. creatine kinase
E. parathyroid hormone
The most common reason for hypocalcemia to occur is vitamin D deficiency. Natural sunlight is
the major source of vitamin D for children and adolescents, therefore inadequate exposure during
the winter months may lead to low vitamin D levels. People with naturally dark skin tone require
at least 3 to 5 times longer sun exposure to produce the same levels of vitamin D as those with
light skin tone. Thus, for the adolescent in the vignette, vitamin D deficiency would be the most
likely cause of hypocalcemia.
The laboratory findings in vitamin D deficiency are summarized in Item C239. Early in the
development of vitamin D deficiency, serum parathyroid hormone (PTH) levels increase, which
is followed later by increased alkaline phosphatase levels. Calcium and phosphorus may initially
be normal. 25-hydroxyvitamin D, with a circulating half-life of 2 to 3 weeks, is the major
circulating form and is the best indicator of vitamin D status.
Although the patient in this vignette has muscle aches, measurement of creatine kinase is not
relevant when considered along with the patient’s history and physical examination. Levels of
PTH and alkaline phosphatase will both be elevated in the presence of hypocalcemia, however,
they can be elevated due to any cause of increased bone turnover, therefore measurement will not
identify the specific diagnosis.
The 2006 American Academy of Pediatrics guidelines on calcium and vitamin D intake include
recommended sources of nutritional intake to optimize bone health. Guidelines from the
Pediatric Endocrine Society also recognize the need to optimize nutritional intake, given the
increased use of sunscreen during the summer and limited sun exposure during winter months.
PREP Pearls
• Measurement of 25-hydroxyvitamin D (the storage form of vitamin D) is the definitive
test for diagnosing vitamin D deficiency.
• Vitamin D deficiency is the most common cause of low calcium levels in adolescence,
especially during the winter months when sun exposure is limited.
Suggested Readings
• Greer FR, Krebs NF; American Academy of Pediatrics Committee on Nutrition.
Optimizing bone health and calcium intakes of infants, children, and adolescents.
Pediatrics. 2006;117(2):578-585. DOI: http://dx.doi.org/10.1542/peds.2005-2822.
• Holick MF, Binkley NC, Bischoff-Ferrari HA, et al; Endocrine Society. Evaluation,
treatment, and prevention of vitamin D deficiency: an Endocrine Society clinical practice
guideline. J Clin Endocrinol Metab. 2011;96(7):1911-1930. DOI:
http://dx.doi.org/10.1210/jc.2011-0385.
Question 241
A 6-year-old boy comes for evaluation after he was diagnosed with amblyopia at a recent
optometry evaluation. He was born at term with no complications, has only had a few minor
childhood illnesses, and has had no eye trauma. On past routine physical examination, he had a
normal cover-uncover test and pupillary light reflex.
Of the following, the MOST accurate statement about this child’s amblyopia is that it
To date, there have been limited data about the efficacy of vision screening of young, preliterate
children for preventing amblyopia. The United States Preventive Services Task Force
recommends vision screening at least once between 3 and 5 years of age, but does not currently
recommend earlier screening because of the absence of data. However, with the development of
handheld autorefractors and photoscreeners, pediatricians can effectively screen children
between the ages of 6 months and 3 years, detecting the presence and magnitude of optical
(refractive) and physical abnormalities at younger ages. The greatest barriers to implementing
screening for these younger ages are cost for equipment, personnel, time, and the lack of assured
reimbursement for this service by third party payers. The American Academy of Pediatrics states
that it is unlikely that health care providers will adopt this early screening rapidly.
Treatment for amblyopia starts with addressing underlying factors such as strabismus or media
opacity. The next step is corrective lenses, and for some children this will provide sufficient
treatment to correct the amblyopia. However, many more children will be treated either with
occlusion or penalization of the better eye. For 3- to 7-year-old children, patching the good eye
for 2 to 6 hours per day produces excellent improvement for both those with moderate amblyopia
(visual acuity 20/40-20/100) and severe amblyopia (visual acuity 20/100-20/400). However,
compliance is difficult with this regimen and the incidence of bullying against these children is
substantial. Instead, many families prefer atropine penalization (ie, blurring vision in the better
eye with atropine drops), and for children with moderate amblyopia, the results of this treatment
are similar to that of patching. Treatment continues to evolve with ongoing research into the
duration needed for patching and the frequency of atropine drop use (weekend only versus daily
treatment). While treatment is more effective, the earlier it is started, the evidence suggests that
even older children may benefit, particularly if they had not previously received amblyopia
treatment. Nearly half of adolescents aged 13 to 17 years at initiation of patching showed 10
letters of improvement or more in visual acuity at the completion of treatment.
PREP Pearls
• Amblyopia is a neurologically based decrease in visual acuity.
• Amblyopia can be secondary to strabismus, refractive errors, or visual deprivation.
• Vision screening is necessary to detect amblyopia, and routine vision screening
should begin between 3 and 5 years of age. Newer technology may allow for earlier
vision screening.
• Treatment for amblyopia involves treating any underlying condition and providing
corrective lenses. Patching or atropine penalization of the better eye is often
necessary and has good results.
Suggested Readings
• American Academy of Pediatrics. Instrument-based pediatric vision screening policy
statement. Pediatrics. 2012;130(5):983-986. DOI:
http://dx.doi.org/10.1542/peds.2012-2548.
Question 242
A local high school parent-teacher association invites you to speak at its next meeting about drug
use among adolescents. During the presentation, a mother asks about trends in substance abuse
among adolescents.
According to the 2013 YRBS, 66% of high school students have had at least 1 drink in their
lifetime. This represents a decrease from 81% in 1999. Defined as at least 1 drink in the 30 days
preceding the survey, 35% of high school students currently drink alcohol. Approximately 19%
of students initiated alcohol use before 13 years of age.
Marijuana is a commonly used substance among adolescents. In 2013, 41% of high school
students had used marijuana at least once. This represents a decrease from 47% in 1999. Defined
as using marijuana at least once in the 30 days preceding the survey, 23% of high school students
currently use marijuana. Approximately 9% of students initiated marijuana use before 13 years
of age.
In 2011, boys reported using alcohol on school property slightly more than females. The average
age of first marijuana use reported in 2011 was 17.5 years, which was similar or slightly older
than the average age in recent years.
PREP Pearls
• Substance use can put youth at greater risk for other health problems such as injuries,
violence, and sexually transmitted infections.
• According to the 2013 Youth Risk Behavior Survey, 66% of high school students have
had at least 1 drink in their lifetime, and 41% of high school students had used marijuana
at least once.
Suggested Readings
• Kann L, Kinchen S, Shanklin SL, et al. Youth Risk Behavior Surveillance — United
States, 2013. MMWR Morbid Mortal Wkly Rep. 2014;63(SS04):1-168.
http://www.cdc.gov/mmwr/preview/mmwrhtml/ss6304a1.htm.
• Substance Abuse and Mental Health Services Administration, US Department of Health
and Human Services. Results from the 2011 National Survey on Drug Use and Health:
summary of national findings. Substance Abuse and Mental Health Services
Administration website.
http://www.samhsa.gov/data/sites/default/files/Revised2k11NSDUHSummNatFindings/R
evised2k11NSDUHSummNatFindings/NSDUHresults2011.htm .
Question 243
A 6-year-old boy develops a fever to 39.4°C, cough, and rhinorrhea. After 2 days of symptoms,
his parents notice a mass in the middle of his neck and bring him to your office for evaluation
(Item Q243). Upon further reflection, his parents recall that the mass presented once before when
the boy had a similar infection, but then disappeared after his infection resolved. On physical
examination, the neck mass moves upwards with protrusion of the tongue.
Item Q243. Neck mass of the patient in the vignette. Reprinted with permission from Geddes G,
Butterly MM, Patel SM, Marra S. Pediatric neck masses. Pediatr Rev. 2013;34(3):115-125.
Branchial cleft cysts are the most common congenital neck lesions, accounting for approximately
20% to 30% of all pediatric neck masses. They most often appear on the lateral side of the neck.
Complications of branchial cleft cysts include recurrent infection and fistula formation.
Cystic hygromas are spongy, mobile, nontender lesions located in the posterior triangle of the
neck, most frequently on the left side. Large lesions can result in airway compromise. If a large
cystic hygroma is detected prenatally, delivery should be performed at a center capable of
managing the airway and lesion at the time of birth. Fine needle aspiration should be avoided
when diagnosing cystic hygromas because hemorrhage into the lesion may cause rapid
expansion.
An ectopic thyroid gland may be located anywhere along the path of descent of the thyroid
during its embryologic development, most commonly at the base of the tongue. Although these
are often midline, they can occur in many locations and present far less commonly as a midline
mass compared with a thyroglossal duct cyst. In most cases ultrasonography is necessary to
identify the location of an ectopic thyroid gland in a child with hypothyroidism, because there
are usually no external signs of the thyroid’s location.
Thyroid nodules are relatively uncommon in children. However, they have a high potential for
malignancy, estimated to range anywhere from 9% to 50%. Because many children and
adolescents with thyroid cancer have metastatic lesions in the cervical lymph nodes at
presentation, they most often present with a lateral neck mass.
PREP Pearls
• Thyroglossal duct cysts are anterior neck midline lesions that generally present after an
upper respiratory tract infection.
• Branchial cleft cysts are the most common congenital neck lesions, and most often appear
laterally.
Suggested Readings
• Geddes G, Butterly MM, Patel SM, Marra S. Pediatric neck masses. Pediatr Rev.
2013;34;115-125. DOI: http://dx.doi.org/10.1542/pir.34-3-115.
• Hopwood NJ, Kelch RP. Thyroid masses: approach to diagnosis and management in
childhood and adolescence. Pediatr Rev. 1993;14(12).
http://pedsinreview.aappublications.org/content/14/12/481.abstract.
Question 244
You are caring for an 8-year-old hospitalized boy who is receiving chemotherapy for acute
lymphoblastic leukemia. He was admitted for fever, neutropenia, and septic shock 5 days ago
and has improved after receiving fluid resuscitation, stress dose steroids, intravenous
vancomycin and cefepime, and a dopamine infusion. He was weaned off the dopamine after 24
hours. His indwelling central line culture grew Pseudomonas aeruginosa, and after 48 hours,
based on sensitivity results, the vancomycin was discontinued. Today, the boy had 8 loose,
watery, foul-smelling stools. Stool studies have been sent.
A. discontinuation of cefepime
B. fluconazole, 12 mg/kg intravenously every 24 hours
C. ganciclovir, 5 mg/kg intravenously every 12 hours
D. linezolid, 10 mg/kg orally every 8 hours
E. metronidazole, 10 mg/kg orally every 6 hours
For the patient in the vignette, risk factors for pseudomembranous colitis include the antecedent
history of broad-spectrum antibiotics and immunocompromise. Either oral metronidazole or oral
vancomycin is the best therapy for pseudomembranous colitis. Discontinuation of cefepime is
not advisable because treatment of his central line infection is necessary. Fluconazole and
ganciclovir will not be helpful, because the cause of the diarrhea is not likely to be fungal or
viral. Oral linezolid is effective against gram-positive bacteria, but it is not a first-line therapy for
C difficile infection because its spectrum is too broad, and it is a bacteriostatic agent.
PREP Pearls
• Pseudomembranous colitis is caused by C difficile infection in the setting of altered
immunity, motility, or balance of colonic flora.
• Treatment of pseudomembranous colitis is oral metronidazole or oral vancomycin.
• Reducing the use of broad-spectrum antibiotics is important in the prevention of
pseudomembranous colitis.
Suggested Readings
• Feigin RD. Antimicrobial agent-induced pseudomembranous colitis. Pediatr Rev.
1981;3:147-152. DOI: http://dx.doi.org/10.1542/pir.3-5-147.
• Mezoff EA, Cohen MB. Clostridium difficile infection. In: Kliegman RM, Stanton BF,
St. Geme JW III, Schor NF, Behrman RE, eds. Nelson Textbook of Pediatrics. 20th ed.
Philadelphia, PA: Saunders Elsevier; 2015:1434-1436 .
Question 245
A 17-month-old girl presents to the emergency department with a limp for 1 week. Her parents
have noted progressive swelling of the left knee. Fever has not been documented, but she has
been on scheduled ibuprofen since the onset of illness. She has had concurrent nasal symptoms
and mild cough. Vital signs show a temperature of 37.2°C, respiratory rate of 24 breaths/min,
heart rate of 130 beats/min, blood pressure of 91/53 mm Hg, and a weight of 10.2 kg. On
physical examination, she has warmth, a notable effusion, and limited range of motion of the left
knee. Laboratory data shows:
A. cefotaxime
B. clindamycin
C. linezolid
D. trimethoprim
E. vancomycin
Approximately 40% of Kingella are resistant to clindamycin and all are resistant to glycopeptide
antibiotics, including vancomycin. Kingella is a gram-negative pathogen. Clindamycin, linezolid,
and vancomycin have good gram-positive activity and would be considered in osteoarticular
infections caused by gram-positive pathogens such as Staphylococcus aureus; however, these
antibiotics have no activity against Kingella. Kingella are also resistant to trimethoprim.
However, the combination of trimethoprim with sulfamethoxazole would be effective against
Kingella.
In young children, in comparison with older children and adults, disease occurs in healthy
individuals without underlying conditions. Daycare attendance may be a risk factor for Kingella
colonization.
PREP Pearls
• Kingella can be an asymptomatic colonizer of the posterior pharynx of young children.
• Kingella is increasingly recognized as a cause of osteoarticular infections, including
septic arthritis, osteomyelitis, spondylodiscitis, and tenosynovitis in young, healthy
children.
• Kingella kingae infections are usually susceptible to β-lactam antibiotics, including
penicillins and cephalosporins.
Suggested Readings
• American Academy of Pediatrics. Kingella kingae infections. In: Kimberlin DW, Brady
MT, Jackson MA, Long SS, eds. Red Book: 2015 Report of the Committee on Infectious
Diseases. 30th ed. Elk Grove Village, IL: American Academy of Pediatrics; 2015:500-
501.
• Yagupsky P, Porsch E, St Geme J III. Kingella kingae: an emerging pathogen in young
children. Pediatrics. 2011;127(3):557-565. DOI: http://dx.doi.org/10.1542/peds.2010-
1867 .
Question 246
A 9-year-old boy presents to your office with the chief complaint of having cola-colored urine
for 2 days. His review of systems is significant for an upper respiratory tract infection-like illness
3 weeks ago. On physical examination, the boy has normal growth parameters. He has a
respiratory rate of 18 breaths/min, heart rate of 94 beats/min, and blood pressure of 130/90 mm
Hg. The remainder of the physical examination is unremarkable. A urine test strip analysis
demonstrates a specific gravity of 1.015, pH of 5.5, 3+ blood, 2+ leukocyte esterase, and no
protein or nitrites.
You suspect acute poststreptococcal glomerulonephritis (PSGN) as the cause of the patient’s
symptoms and clinical findings. A resident working with you asks how to distinguish PSGN
from other forms of glomerulonephritis.
Of the following, you are MOST likely to tell the resident that
E. serology for streptococcal infections is rarely positive when nephritis is diagnosed in both
poststreptococcal glomerulonephritis and membranoproliferative glomerulonephritis
The timing of infectious illness and acute nephritis can provide clues to the presenting nephritis.
In patients with postinfectious GN, history of illness and acute nephritis is usually separated by 7
to 21 days. However, the onset of acute GN within days of viral infection is seen in
immunoglobulin A (IgA) glomerulonephritis, Alport syndrome, and membranoproliferative
glomerulonephritis (MPGN).
Initial urine microscopy shows hematuria, pyuria (glomerular inflammation), and red blood cell
casts. Subsequent urine microscopy in patients with postinfectious GN may show persistent
microscopic hematuria (which may persist for up to 1 to 3 years in some cases). This has not
been associated with a worse prognosis in the patients.
The treatment for acute GN is usually supportive and includes fluid restriction, low sodium diet,
and diuretics or vasodilators if patients have edema and hypertension. Renal biopsy is rarely
indicated in patients with suspected postinfectious GN. Presence of severe azotemia, rapidly
rising serum creatinine (suggestive of rapidly progressive glomerulonephritis), or persistently
low complement levels (beyond 6-8 weeks) are indications for renal biopsy in patients with
suspected postinfectious GN. Membranoproliferative GN can be differentiated from
postinfectious GN by the characteristic renal biopsy findings in each condition.
Patients with IgA nephropathy often present after an upper respiratory infection, similar to the
presentation of patients with postinfectious GN. Potential distinguishing features from
postinfectious GN include a shorter time between the antecedent illness and nephritis and normal
complement levels. History of episodes of gross hematuria prior to the first episode or after the
initial episode of nephritis is also suggestive of IgA nephritis, as recurrences are rare in
postinfectious GN.
American academy of pediatrics 785
American Academy of Pediatrics PREP 2016
PREP Pearls
• Poststreptococcal glomerulonephritis (PSGN) related to group A, β-hemolytic
streptococcal infection is the most common acute nephritis in children worldwide.
• It is difficult to differentiate membranoproliferative glomerulonephritis (MPGN) from
postinfectious glomerulonephritis, especially in patients with no history of preceding
infection or in patients with a short duration between infection and onset of nephritis.
• Hypocomplementemia resolves by 6 to 8 weeks in most cases of postinfectious
glomerulonephritis (GN).
• Persistently decreased C3 levels are suggestive of MPGN.
• A low C3 level in acute GN indicates activation of the alternative pathway of
complement (as in postinfectious GN or MPGN).
Suggested Readings
• Eison TM, Ault BH, Jones DP, Chesney RW, Wyatt RJ. Post-streptococcal acute
glomerulonephritis in children: clinical features and pathogenesis. Pediatr Nephrol.
2011;26(2):165-180. DOI: http://dx.doi.org/10.1007/s00467-010-1554-6.
• Massengill SF. Hematuria. Pediatr Rev. 2008;29(10):342-348. DOI:
http://dx.doi.org/10.1542/pir.29-10-342.
• Varade WS. Nephritis. In: McInerny TK, Adam HM, Campbell DE, Kamat DM, Kelleher
KJ, eds. American Academy of Pediatrics Textbook of Pediatric Care. 19th ed. Elk Grove
Village, IL: American Academy of Pediatrics; 2009:2291-2292.
Question 247
An 18-month-old boy is brought to the office for a health supervision visit. He was delivered by
spontaneous vaginal delivery at 27 weeks of gestation due to maternal preeclampsia. His birth
weight was 980 g. He has been growing and developing normally with no significant medical
issues to date.
On physical examination, he is alert and energetic without scleral icterus or jaundice. His height
and weight are at the 25th percentage for age. Abdominal examination is notable for
hepatomegaly, with his liver edge palpable 4 cm below the right costal margin. The liver is firm
and nontender. No splenomegaly is palpated.
Laboratory studies show:
• Aspartate aminotransferase, 35 U/L
• Alanine aminotransferase, 41 U/L
• Bilirubin, 1.5 mg/dL (25.7 µmol/L)
• Direct bilirubin, 0.1 mg/dL (1.7 µmol/L)
• Platelets, 500 x 103/µL (500 x 109/L)
• α-fetoprotein, 100,000 ng/mL (100,000 µg/L)
A. abdominal radiograph
Hepatomegaly is a nonspecific finding and not a reliable predictor of liver disease. There is
significant variability in the size and shape of the liver. If an enlarged liver is suspected, physical
examination should assess the liver span at the mid-clavicular line using palpation and
percussion. The liver edge should be less than 2 cm below the costal margin at the mid-clavicular
line, with the exception of children younger than 2 years of age, when it may be palpable to 3.5
cm below the costal margin. The normal values for liver span are based on gender, age, and body
weight, with a variability of +/- 2 to 3 cm. An abdominal ultrasonography can be used to confirm
the size of the liver and provides some assessment of density and evaluation for a liver mass, if
present. Laboratory studies may include a complete blood cell count, comprehensive metabolic
panel, total and direct bilirubin, γ-glutamyl transferase, and an assessment of synthetic function
with a prothrombin time and partial thromboplastin time. Evaluations for infectious etiologies
and storage disease should be considered based on the history. Liver biopsy may be required to
confirm a diagnosis.
The differential diagnosis for hepatomegaly is quite broad and is summarized in Item C247.
Splenomegaly occurs with portal hypertension, storage disease, inflammation, infection, and
malignancy.
Abdominal radiographs are not effective at assessing liver size or density. Magnetic resonance
cholangiopancreatography is used to assess bile ducts and anatomy, and may be useful in the
evaluation of hepatomegaly, but it is not the first imaging modality in the evaluation. Similarly,
nuclear medicine liver and spleen scans and positron emission tomography may be used in the
evaluation of hepatomegaly to assess blood flow or for tumor evaluations, but are not used to
confirm hepatomegaly or as the initial screen for a mass.
PREP Pearls
• Hepatomegaly should be confirmed by physical examination and abdominal
ultrasonography.
• Liver size varies with age, sex, and body weight.
• Hepatomegaly may be associated with an acute viral illness.
• Children with hepatomegaly without evidence of recent acute viral illness should be
referred to pediatric gastroenterology for further evaluation.
Suggested Readings
• Abu-Wasel B, Walsh C, Keough V, Molinari M. Pathophysiology, epidemiology,
classification and treatment options for polycystic liver diseases. World J
Gastroenterol. 2013;19(35):5775-5786.
DOI:http://dx.doi.org/10.3748/wjg.v19.i35.5775.
Question 248
A 2-month-old infant presents to your office for a health care supervision visit. On examination,
you note disproportionate small stature with proximal shortening of the arms and legs, trident
hands, hypotonia, a large head, frontal bossing, and midfacial hypoplasia. You suspect a skeletal
dysplasia.
Of the following, the MOST likely complication to increase the risk of death in infancy in this
patient would be
B. hydrocephalus
The management of achondroplasia depends on the complications that may arise. Middle ear
dysfunction is common, requiring tympanostomy tube placement and close monitoring of
hearing. If significant dysfunction is present, a delay in language development can occur. A
newborn screening hearing test should be checked initially and repeated by 1 year of age. An
orthopedist should investigate any progressive bowing of the legs. A sleep study should be
performed during infancy because of the risk for obstructive sleep apnea (OSA). However, many
patients develop OSA as an older child or adult, so careful attention to this clinical manifestation
is required during each well child visit. Obstructive sleep apnea may be caused by a combination
of midfacial retrusion resulting in a smaller airway size, hypertrophy of the lymphatic ring,
abnormal innervation of the musculature in the airway, or, in the worst case scenario,
craniocervical constriction. Treatment for OSA in patients with achondroplasia may include
adenotonsillectomy, weight reduction, continuous positive airway pressure, or tracheostomy in
extreme cases.
Significant short stature is universally present with achondroplasia and growth charts specific for
patients with achondroplasia should be utilized for routine growth checks. Obesity is common
and can worsen the morbidity associated with lumbar stenosis and contribute to joint problems.
The development of kyphosis typically occurs in the first 3 years of life, and therefore should be
clinically assessed by physical examination every 6 months during that time period. Lumbar
spinal stenosis is more common in adulthood. A clinical history and neurologic examination is
warranted every 3- to 5-years once an individual with achondroplasia reaches adulthood to look
for spinal stenosis.
Screening of developmental milestones throughout infancy and childhood is important.
Development attainment should be compared to specific milestone assessments standardized to
patients with achondroplasia. A speech evaluation is recommended by 2 years of age if any
delays are noted.
Of the answer choices in the vignette, craniocervical junction compression would lead to the
highest risk of death in infancy due to central hypopnea. Hydrocephalus, lumbar spinal stenosis,
and OSA are common complications during childhood, but are not typically associated with a
high risk of mortality. Lumbar spinal stenosis is more commonly seen as an adult complication.
A hypoplastic thorax is not commonly seen with achondroplasia, but can be seen in other skeletal
dysplasias, such as thanatophoric dysplasia, campomelic dysplasia, or achondrogenesis.
PREP Pearls
• Craniocervical junction compression can increase the mortality risk in infancy in
patients with achondroplasia.
• Every infant with achondroplasia should have a computed tomography or magnetic
resonance imaging of the craniocervical junction, overnight polysomnography, and a
thorough neurologic examination for signs of cervical myelopathy. The best
predictors for craniocervical junction constriction include lower limb hyperreflexia or
clonus, central hypopnea on sleep study, or reduced foramen magnum size on
computed tomography of the craniocervical junction.
• Achondroplasia is the most common inherited skeletal dysplasia that results in
disproportionate small stature, with affected individuals having short arms and legs,
macrocephaly, frontal bossing, and midfacial retrusion. Normal intelligence and life
span is expected, although hypotonia in infancy can delay motor milestones.
• Other common complications of achondroplasia include hydrocephalus, delayed
motor milestones, sleep apnea, middle ear dysfunction, hearing loss, kyphosis,
bowing of the legs, obesity, and lumbar spinal stenosis.
Suggested Readings
• Ireland PJ, Johnson S, Donaghey S, et al. Developmental milestones in infants and
young Australasian children with achondroplasia. J Dev Behav Pediatr.
2010;31(1):41–47. DOI: http://dx.doi.org/10.1097/DBP.0b013e3181c72052 .
• Pauli RM. Achondroplasia. GeneReviews.
http://www.ncbi.nlm.nih.gov/books/NBK1152/ .
• Pauli RM. Achondroplasia. In: Cassidy SB, Allanson JE, eds. Management of
Genetic Syndromes. 3rd ed. New York, NY: John Wiley & Sons; 2010:17-37 .
• Trotter TI, Hall JG. Health supervision for children with achondroplasia. Pediatrics.
2005;116(3):771-783. DOI:http://dx.doi.org/10.1542/peds.2005-1440 .
• Wynn J, King TM, Gambello MJ, Waller DK, Hecht JK. Mortality in achondroplasia
study: a 42-year follow-up. Am J Med Genet A. 2007;143A(21):2502-2511. DOI:
http://dx.doi.org/10.1002/ajmg.a.31919 .
Question 249
You are examining an African-American male newborn at 36 hours of age. He was a full-term
newborn without complications. There were no maternal problems during pregnancy. The
newborn has been rooming in with his mother and doing well. He is alert, active, and well-
appearing, with normal vital signs. The patient’s mother states that she noticed tiny pustules on
his face, chin, and neckline at birth, but these have begun to disappear. On physical examination,
you find a few 2-mm vesicles and white pustules on the trunk and neckline without surrounding
erythema or induration. There are several faint brownish macules with a collarette of scale in the
perioral area (Item Q249). The remainder of the newborn’s physical examination is
unremarkable.
Item Q249
A. congenital candidiasis
C. impetigo neonatorum
D. miliaria
Differentiating this benign condition from other pustular eruptions seen in newborns is important
because of the potential for serious illness if the infant has a congenital infection. Laboratory
findings can be helpful in corroborating or excluding suspected clinical diagnoses. Congenital
candidiasis presents at birth or in the first few days after birth as a diffuse erythematous papular,
pustular, exfoliating rash that may involve the palms and soles. Potassium hydroxide preparation
of the pustular material reveals pseudohyphae or spores and Candida spp will be found on
culture. Erythema toxicum neonatorum is characterized by evanescent blotchy erythematous
macules in combination with 1- to 3-mm papules and pustules that occur predominantly on the
face, trunk, and extremities, sparing the palms and soles. Cytologic examination of a smear from
a pustule will reveal a predominance of eosinophils on Wright or Giemsa stain and culture will
be negative. Impetigo neonatorum usually presents as superficial vesicular, pustular, or bullous
lesions on an erythematous base. These lesions are easily ruptured, leaving a denuded red moist
surface with later crust formation. Gram-positive cocci and neutrophils will be seen on Gram
stain and a culture will yield streptococci or staphylococci. Pinpoint 1- to 2-mm clear vesicles
without surrounding erythema are characteristic of miliaria crystallina. Myriads of tiny pink
papules are characteristic of miliaria rubra. Miliaria is most often distributed in the intertriginous
regions, on the face, or areas that have been occluded, such as the upper back, and usually is
associated with warming of the infant. Lymphocytes may be seen on Wright stain or Gram stain
of a smear of the contents of a miliaria vesicle and culture is negative.
PREP Pearls
• Transient neonatal pustular melanosis (TNPM) is characterized by sterile superficial
vesicles or pustules that rupture easily, leaving hyperpigmented macules surrounded
by a fine white “collarette” of scale.
• The pustular lesions of TNPM resolve spontaneously within 24 to 48 hours, leaving
hyperpigmented macules that fade gradually over several weeks to months.
• Neutrophils and cellular debris are seen on Wright stain smear of TNPM lesions.
Suggested Readings
• Johr RH, Schachner LA. Neonatal dermatologic challenges. Pediatr Rev.
1997;18(3):86-94. DOI:http://dx.doi.org/10.1542/pir.18-3-86.
• Paller AS, Mancini AJ. Cutaneous disorders of the newborn. In: Hurwitz Clinical
Pediatric Dermatology: A Textbook of Skin Disorders of Childhood and
Adolescence. 4th ed. Philadelphia, PA: Elsevier Saunders; 2011:18-19.
Question 250
A 16-year-old female adolescent presents to your office with a swollen and tender left lower leg.
She had been previously well, with no fevers, weight loss, or rash. She denied any trauma or
known insect bites. She has been on the track team and has denied ever smoking. She has not
been sexually active, has not been taking birth control pills, has not been on any long trips, and
has not been immobilized for any period of time. On physical examination, she is well appearing.
Her temperature is 37°C, her heart rate is 82 beats/min, blood pressure is 108/78 mm Hg, and her
respiratory rate is 20 breaths/min. Her height and weight are both at the 50th percentile for age.
Her left lower leg is visibly larger than her right and is tender to the touch posteriorly. She has
normal pedal pulses bilaterally and a normal gait.
You refer her for ultrasonography of the left lower extremity, which reveals a nonocclusive
venous thrombus in the popliteal vein.
Of the following, further evaluation and management of this patient should include
C. factor V Leiden, a spiral chest computed tomography, and, if both are normal, no
anticoagulation
The anti-β2-glycoprotein and anticardiolipin antibodies are known risk factors for thrombosis.
Circulating phospholipids are required to form a normal clot. The presence of auto-antibodies to
phospholipids increases their thrombogenicity. In extreme forms, the presence of
antiphospholipid antibodies can lead to sudden, life-threatening end organ damage through
synchronous thrombus formation in vital organs, frequently with a consumptive coagulopathy
and bleeding. When this occurs, it is known as the catastrophic anti-phospholipid antibody
syndrome (APLAS). The treatment of catastrophic APLAS includes anticoagulants, plasma
exchange, and immune suppression with corticosteroids or other immune suppressants.
For the patient in the vignette, the best combination of next steps would include testing for
antiphospholipid antibodies, pulse oximetry, and anticoagulation with enoxaparin. A chest
radiograph would not be warranted, as a chest computed tomography would be far more
sensitive than a chest radiograph to assess for a pulmonary embolus. Anticoagulation with
aspirin alone would be inadequate therapy for a DVT. There would be no reason to image the
patient’s brain in this scenario. The only risk for a central nervous system embolus would be if
there was a large patent foramen ovale or other right to left intracardiac shunt. Given the rarity of
that occurrence and the absence of any central nervous system symptoms, a brain magnetic
resonance image would not be warranted.
PREP Pearls
• Patients who present with a new thrombosis should have a workup for heritable
thrombophilia including factor V Leiden, prothrombin gene mutation, protein C and S
levels, and antithrombin III levels.
• Patients who present with a new thrombosis should have a workup for acquired
thrombophilia including an evaluation for antiphospholipid antibodies (lupus
anticoagulant, anticardiolipin antibodies, and anti-β2-glycoprotein antibodies).
• Anticoagulation with enoxaparin is easier and requires less monitoring than warfarin.
Suggested Readings
• Hunt BJ. Pediatric antiphospholipid antibodies and antiphospholipid syndrome.
Semin Thromb Hemost. 2008;34(3):274-281. DOI: http://dx.doi.org/10.1055/s-0028-
1082271.
• Roth M, Manwani D. Thrombotic disorders. Pediatr Rev. 2011;32(1):41-43. DOI:
http://dx.doi.org/10.1542/pir.32-1-41.
• Yang JY, Chan AK. Pediatric thrombophilia. Pediatr Clin North Am.
2013;60(6):1443-1462. DOI:http://dx.doi.org/10.1016/j.pcl.2013.09.004.
Question 251
You are seeing a 10-year-old girl who has been performing repetitive cleaning behaviors over the
past 3 to 4 months. She repeatedly cleans and organizes her bedroom and frequently washes her
hands. She is experiencing dysfunction because of these behaviors, which interfere with her
ability to fall asleep at night and get to school. The frequent washing is causing red cracked skin
on her hands. The mother expresses significant frustration about dealing with these behaviors,
and says that their home life is now strained because of it.
B. prescribe clomipramine
C. prescribe clonazepam
An anti-streptolysin O (ASO) titer might be included within an OCD workup that is suspecting a
pediatric autoimmune neuropsychiatric disorder associated with Streptococcus (PANDAS) or
streptococcal immune reaction condition having caused the symptoms, but it would not be the
best next step in care for several reasons. The first is that the ASO titer only indicates that the
child may have experienced a streptococcal infection in the preceding 6 months, which is an
extremely common circumstance among healthy children and not indicative of any current
disorder. Secondly, associating an autoimmune etiology for a child's OCD does not currently
open up any other valid treatment options; clinical care is essentially the same regardless of
whether or not the OCD may have been triggered in an autoimmune fashion. A third reason is
that this child has no clinical history of acute onset of OCD symptoms in the month immediately
following a streptococcal infection, which would be the expected time course of an autoimmune-
triggered OCD syndrome.
OCD in children is to help parents understand what is happening with their child, and coach
them on how best to help their child as they struggle to manage their obsessive impulses.
PREP Pearls
• Obsessive compulsive disorder (OCD) is a very biological rather than social
environment generated disorder.
• Obsessive compulsive disorder treatment with cognitive behavior therapy (CBT) and
selective serotonin reuptake inhibitor (SSRI) treatment is highly effective.
• While there are rare cases in which OCD may have been triggered by an autoimmune
reaction to infection (such asStreptococcus), the best clinical treatment for the OCD is
still the use of SSRIs and CBT.
Suggested Readings
• Franklin ME, Sapyta J, Freeman JB, et al. Cognitive behavior therapy augmentation
of pharmacotherapy in pediatric obsessive-compulsive disorder: the Pediatric OCD
Treatment Study II (POTS II) randomized controlled trial. JAMA.
2011;306(11):1224-1232. DOI: http://dx.doi.org/10.1001/jama.2011.1344.
• Stewart SE, Hezel D, Stachon AC. Assessment and medication management of
paediatric obsessive-compulsive disorder.Drugs. 2012;72(7):881-893. DOI:
http://dx.doi.org/10.2165/11632860-000000000-00000.
Question 252
A 4-year-old boy was noted to have a heart murmur when seen for a febrile illness by your
partner 2 weeks ago. The murmur was described as a 3/6 systolic murmur. He is here today for
follow-up and is well. He has no history of syncope, palpitations, or exercise intolerance. The
family history is negative for any individuals with sudden death, cardiomyopathy, or early
coronary artery disease.
Physical examination shows a heart rate of 76 beats/min, respiratory rate of 18 breaths/min, and
blood pressure of 90/65 mm Hg. The lungs were clear to auscultation, the cardiac examination is
significant for a regular rate and rhythm, and S1 and S2are normal. There is a 2/6 systolic
murmur at the left lower sternal border. There is no radiation to the neck or axilla. There is no
rub or gallop. On abdominal examination, there is no hepatosplenomegaly. His femoral pulses
are 2+ and normal.
Of the following, the additional finding that would be MOST consistent with an innocent
murmur in this child is
Innocent murmurs are common in childhood. There are 4 distinct types of innocent murmurs in
children:
• Still’s murmur (vibratory murmur heard at the left mid sternal border and apex)
• Pulmonary flow murmur in infancy (heard in the left upper sternal border and into the
axilla)
• Pulmonary flow murmur (heard in the left upper sternal border in later childhood)
• Venous hum (a holosystolic murmur heard in the supraclavicular area on either the
right or left side, which may change or become inaudible when the child’s head is
turned to the left or right)
Other than the venous hum, innocent murmurs are systolic, less than 3/6 in intensity, and do not
obscure S1. If a murmur does obscure S1, it is occurring when no blood flow should be audible,
as this is the phase of isovolumic contraction when the atrioventricular valves have closed and
the aortic and pulmonic valves have not yet opened.
Differentiation of an innocent murmur from a pathologic murmur is usually done by history and
physical examination. If there are symptoms of decreased cardiac output, cyanosis, palpitations,
or chest pain, there is a higher likelihood of cardiac pathology.
The other murmurs offered as response choices have qualities that are not innocent. Diastolic
murmurs suggest a regurgitant lesion, such as aortic and pulmonary insufficiency. A systolic
murmur in the mitral position would suggest mitral regurgitation. A continuous murmur heard
over the scapula suggests a patent ductus arteriosus. A systolic murmur in the right upper sternal
border (aortic position) would suggest aortic stenosis.
The low pitched diastolic murmur at the right mid sternal border is consistent with tricuspid
inflow and, although not usually audible, might be heard with a large atrial septal defect. A mid
systolic murmur in the third left intercostal space suggests mitral regurgitation. A murmur in the
right upper sternal border is never considered normal, as this is the aortic position. A patient with
hyperdynamic circulation, as seen with anemia or infection, can have a murmur in the left upper
sternal borders that is louder than the usual innocent pulmonary flow murmur. A vibratory
murmur, which is an innocent finding that is also musical and is heard at the left lower sternal
border and toward the apex, is consistent with a Still’s murmur.
PREP Pearls
• Diastolic murmurs are not innocent.
• Murmurs that obscure S1 or are greater than 3/6 in intensity are not innocent.
• Innocent murmurs are common in children. There are 4 distinct types of innocent
murmurs in children:
o Still’s murmur (vibratory murmur heard at the left mid sternal border and apex)
o Pulmonary flow murmur in infancy (heard in the left upper sternal border and into
the axilla) oPulmonary flow murmur (heard in the left upper sternal border in later
childhood)
o Venous hum (a holosystolic murmur heard in the supraclavicular area on either
the right or left side, which may change or become inaudible when the child’s
head is turned to the left or right)
Suggested Readings
• Etoom Y, Ratnapalua S. Evaluation of children with heart murmurs. Clin Pediatr
(Phila). 2014;53(2):111-117. DOI:http://dx.doi.org/10.1177/0009922813488653.
• Frank JE, Jacobe KM. Evaluation and management of heart murmurs in children. Am
Fam Physician. 2011;84(7):793-800.
• Menashe, V. Heart Murmurs. Pediatr Rev 2007; 28 (4):e19-e22. DOI:
http://dx.doi.org/10.1542/pir.28-4-e19.
• Park MK. Physical examination: auscultation. Park’s Pediatric Cardiology for
Practitioners. 8th ed. Philadelphia, PA: Saunders Elsevier; 2014:9-40.
• Punnoose AR, Burke AE, Golub RM. Innocent (harmless) heart murmurs in children.
JAMA. 2012;308(3):305. DOI:http://dx.doi.org/10.1001/jama.2012.6223.
• Teitel DF, Cassidy SC, Fineman JR. History and physical examination. In: Allen HD,
Driscoll DJ, Shaddy RE, Feltes TF, eds.Moss and Adams’ Heart Disease in Infants,
Children, and Adolescents: Including the Fetus and Young Adult. 8th ed.
Philadelphia, PA: Lippincott Williams & Wilkins; 2013:69-81.
Question 253
A 13-year-old adolescent boy presents to your office with complaint of joint pain that started 3
months ago. He complains of lower back, left hip, right knee, and left ankle pain that is worse in
the morning and improves with exercise. Rest makes his back pain worse. On physical
examination, the patient has tenderness over the right tibial tuberosity and the left calcaneus.
There is swelling at the insertion of the Achilles tendon. He has pain with passive movement and
decreased range of motion of his left hip.
Of the following, the additional feature MOST supportive of your suspected diagnosis is a
E. paraspinal tenderness
Spondyloarthritis is a classification of several different arthridites that can affect the spine and
sometimes the sacroiliac joints. Spinal involvement is often not present at disease onset, but
develops over time. Ten percent to 20% of patients with spondyloarthritis develop symptoms in
childhood, which is known as juvenile spondyloarthritis (JS). Clinical features of
spondyloarthritis include enthesitis, peripheral arthritis, axial arthritis, uveitis, gastrointestinal
inflammation, and cardiac manifestations. In spondyloarthritis, the peripheral arthritis is usually
an asymmetric arthritis of the large joints of the lower extremity. Hip involvement is more
frequent in the pediatric population. Tarsitis is more common in spondyloarthritis than in other
types of arthritis. It is difficult to predict which patients will progress from spondyloarthritis to
AS.
Enthesitis is inflammation of the entheses, the bone insertion sites for ligaments, tendons, fascia,
and joint capsules. Enthesitis causes pain, swelling, and tenderness and presents more commonly
in pediatric disease. Enthesitis can also occur with various types of arthritis, as well as overuse or
traction injuries such as Osgood-Schlatter syndrome. Enthesitis-related arthritis (ERA) is defined
as arthritis or enthesitis with sacroiliac joint tenderness or inflammatory spinal pain and 2 of the
following:
• the presence of HLA-B27
• acute anterior uveitis
• onset in a boy older than 6 years
• family history of first-degree relative with AS, ERA, sacroiliitis with inflammatory
bowel disease, reactive arthritis, or acute anterior uveitis
Axial arthritis is associated with inflammatory back pain that has an insidious onset, occurs in
the second half of the night, improves with exercise but not with rest, and is associated with
morning stiffness. There can be buttock pain that alternates, as in spondyloarthritis.
Inflammatory back pain is not as common in the pediatric population as it is in the adult
population. Sacroiliitis is often the first sign of axial involvement in AS. Silent sacroiliitis is
found in 21% of children with spondyloarthritis. Sacroiliitis can usually be seen on plain
radiography. However, in cases in which plain radiography is normal and spondyloarthritis is
suspected, a magnetic resonance imaging scan of the pelvis with short T1 inversion recovery
(STIR) images is best to make the diagnosis.
American academy of pediatrics 808
American Academy of Pediatrics PREP 2016
Elevated C-reactive protein level can be associated with AS. Other laboratory studies are of
limited value in diagnosing AS, but may be useful to rule out other diseases.
A positive HLA-B27 is associated with AS, but is not diagnostic. Six percent to 10% of the
white population is HLA-B27 positive, limiting its diagnostic usefulness. Only 20% of people
who are HLA-B27 positive and have a family history of AS will go on to develop AS. Although
a family history of AS in an HLA-B27–positive patient is a diagnostic factor, HLA-B27
positivity in the absence of a family history of AS is not. A patient with inflammatory back pain
with peripheral enthesitis and a first-degree relative with psoriasis would meet criteria for
psoriatic arthritis, not AS. A history of illness 2 weeks before the onset of symptoms would more
likely be associated with a postinfectious or reactive arthritis. Paraspinal tenderness is more
likely to be caused by musculoskeletal pain than muscle strain.
PREP Pearls
• Anklylosing spondylitis is a clinical diagnosis that includes enthesitis, peripheral
arthritis, axial arthritis, uveitis, gastrointestinal inflammation, and cardiac
manifestations.
• Sacroiliitis is often the first sign of axial involvement in ankylosing spondylitis.
• HLA-B27 is associated with increased risk of uveitis in ankylosing spondylitis.
Suggested Readings
• Ramanathan A, Srinivasalu H, Colbert RA. Update on juvenile spondyloarthritis.
Rheum Dis Clin North Am. 2013;39:767-788. DOI:
http://dx.doi.org/10.1016/j.rdc.2013.06.002.
• Tse SML, Laxer RM. New advances in juvenile spondyloarthritis. Nat Rev
Rheumatol. 2012;8:269-279. DOI:http://dx.doi.org/10.1038/nrrheum.2012.37.
American academy of pediatrics 809
American Academy of Pediatrics PREP 2016
Question 254
A 15-year-old adolescent presents to your office for evaluation of a neck injury that occurred
earlier that day. During football practice, the left side of his neck was struck by another player’s
leg during a tackling drill. He immediately developed left-sided neck pain, accompanied by pain
and weakness in his left arm. You review the recommended evaluation and treatment for this
injury with the boy and his family.
Of the following, radiographic studies are MOST strongly warranted in the presence of
A. bilateral symptoms
Stinger injuries are common in contact sports, especially football. Typically the diagnosis can be
made based on clinical symptoms and physical examination findings. Recurrence is common.
There are 3 mechanisms for these mild brachial plexus injuries: direct force applied to the
brachial plexus, as happened to the boy in the vignette; forced lateral bending of the neck
causing compression of the cervical nerve roots, leading to ipsilateral symptoms; or traction
applied to the nerve roots, causing contralateral symptoms. Athletes typically report pain
radiating down the arm. Weakness and paresthesias in the arm and shoulder are also common.
Symptoms generally last less than 24 hours.
On physical examination, affected individuals often have weakness of the deltoid, biceps, and
rotator cuff muscles. Sensory changes may also be seen. Radiographic studies may be indicated
to rule out other injuries, such as a cervical spine fracture or spinal cord injury, but are not
needed to diagnose a stinger.
Traction is a common mechanism for mild brachial plexus injuries, so the injury mechanism for
the boy in the vignette should not prompt concern. Tenderness at the site of impact is expected
for most soft tissue injuries. Biceps weakness is a common clinical sign of a stinger injury. The
duration of the boy’s symptoms does not suggest a severe injury.
PREP Pearls
• Mild brachial plexus injuries, commonly referred to as burners or stingers, are
common in contact sport athletes.
• Bilateral or lower extremity symptoms suggest cervical spine injury and should
prompt cervical spine immobilization and radiographic studies.
Suggested Readings
• Sarwark JF, LaBella C. Pediatric Orthopaedics and Sports Injuries: A Quick
Reference Guide. 2nd ed.Elk Grove Village, IL: American Academy of Pediatrics;
2014: 795 pp.
• Standaert CJ, Herring SA. Expert opinion and controversies in musculoskeletal and
sports medicine: stingers. Arch Phys Med Rehabil. 2009;90(3):402-406.
Question 255
An 8-year-old boy presents to the emergency department with a 3-day history of right facial
swelling and poor oral intake. On physical examination, his temperature is 39°C and vital signs
are normal for age. His right cheek is markedly swollen and firm, with mild erythema and
moderate tenderness to palpation. Examination of the oropharynx reveals dental caries,
malodorous breath, and right mandibular gingival swelling with erythema. There is purulent fluid
observed near the base of the first molar and palpation of the tooth elicits pain. The remainder of
the physical examination is unremarkable.
A. ampicillin-sulbactam
B. cefotaxime
C. metronidazole
D. tetracycline
E. vancomycin
Most orofacial infections arise from periapical, periodontal, or pericoronal dental infection that
spreads by the path of least resistance into surrounding (potential) spaces and tissues. Spread of
infection into the fascial spaces can result in significant facial swelling and fever, such as that
described in the vignette. Odontogenic infections in children usually are localized; significant
spread can lead to respiratory compromise and life-threatening complications.
Serious odontogenic infection should be treated with intravenous antibiotics and, if indicated,
surgical drainage. Given the polymicrobial nature of infection, with anaerobic organisms playing
an important role in severe disease, broad-spectrum antimicrobial therapy with activity against β-
lactamase–producing anaerobic and aerobic organisms is optimal.
Cephalosporins, such as cefotaxime, are second-line agents for the treatment of odontogenic
infection. They effectively treat β-lactamase–producing oral streptococci and gram-negative
pathogens such as Eikenella, but must be combined with metronidazole for anaerobic coverage.
Metronidazole has excellent activity against anaerobic pathogens but no activity against aerobic
gram-positive and gram-negative organisms. Tetracycline is not recommended for children
younger than 9 years because it permanently stains teeth by becoming incorporated into newly
formed dentin. In addition, there is increasing resistance to tetracycline among gram-positive and
gram-negative organisms. Although vancomycin has broad-spectrum activity against aerobic
gram-positive organisms (eg, methicillin-resistant Staphylococcus aureus [MRSA], streptococci),
it is not necessary in the treatment of odontogenic infection as more narrow-spectrum agents will
suffice. Mild odontogenic infection may be treated with oral penicillin, amoxicillin, or
clindamycin, but as resistance to these agents is increasing, patients must be monitored closely
for clinical improvement.
PREP Pearls
• Odontogenic infections typically are polymicrobial, with oral facultative streptococci
and anaerobic bacteria commonly involved.
• Broad-spectrum antimicrobial therapy with activity against beta-lactamase–producing
anaerobic and aerobic organisms is optimal for the treatment of odontogenic
infections.
• Spread of odontogenic infection into the fascial spaces can result in significant facial
swelling and fever; significant spread can lead to respiratory compromise and life-
threatening complications.
Suggested Readings
• Garcia C, McCracken GH Jr. Antibacterial therapeutic agents. In: Cherry J,
Demmler-Harrison GJ, Kaplan SL, Steinbach WJ, Hotez P, eds. Feigin & Cherry’s
Textbook of Pediatric Infectious Diseases. 7th ed. Philadelphia, PA: Elsevier
Saunders; 2014:3182-3241.
• Simos C, Gonzalez BE. Infections of the oral cavity. In: Cherry J, Demmler-Harrison
GJ, Kaplan SL, Steinbach WJ, Hotez P, eds. Feigin & Cherry’s Textbook of Pediatric
Infectious Diseases. 7th ed. Philadelphia, PA: Elsevier Saunders; 2014:140-155
Question 256
A 3-week-old full-term newborn, with fever and decreased oral intake over the past 24 hours, is
admitted to a teaching hospital where you are the supervising pediatrician. Her plan of care
includes obtaining blood, urine, and cerebrospinal fluid cultures, and initiating empiric
intravenous antibiotic therapy. As you discuss this plan with the admitting resident, you ask how
she will manage the newborn’s procedure-related pain. The resident replies that she generally
does not use any pain management modality when performing procedures in newborns, as they
are much less affected by these procedures than older children.
Of the following, the MOST accurate statement about pain management for patients of this age is
that
B. their level of procedure-related pain is overestimated by parents and health care providers
D. they typically display a less pronounced physiologic response to pain than older children
It is imperative for all pediatric providers to understand the effects of patients’ developmental
stage on their response to pain and to apply this knowledge in implementing appropriate pain
management plans for children of various ages. The field of knowledge related to pain response
in children and pediatric pain management techniques has advanced significantly over the past 2
to 3 decades. Regarding newborns specifically, an emerging body of literature highlights the
critical importance of appropriate pain management. Historically, analgesics were rarely
administered to newborns, based on the theory that they experience less pain because of the
immaturity of their nervous systems. Until the late 1980s, in fact, newborns often underwent
certain surgical procedures without anesthesia.
Recent studies suggest that newborns may actually have an increased sensitivity to pain, which
may be attributable to the fact that although their ascending nerve pathways can transmit painful
stimuli to the brain, descending inhibitory pathways are not yet established. Data demonstrate
that neonates display a more pronounced physiologic response to pain and require higher serum
concentrations of analgesics to modulate pain compared with older children. Studies also
indicate that repeated exposure to painful stimuli in the neonatal period can increase sensitivity
to subsequent painful stimuli as well as routine handling. In addition, infants who experience
painful procedures may develop altered responses to future painful episodes, even those that
occur well beyond infancy. Pain may even be a contributing factor in the occurrence of
intraventricular hemorrhage in preterm infants.
A single modality approach to the management of procedure-related pain is not the best strategy
for infants or older children. A multimodal approach incorporating environmental, behavioral,
and pharmacologic interventions is much more effective.
Pain levels related to procedures are generally underestimated rather than overestimated, largely
because of the inability of caretakers to reliably assess pain in infants.
Current literature contradicts the theory that infants lack the cognitive ability to remember
painful experiences. In fact, recent studies indicate that experiencing pain during infancy may
alter a child's responses to future painful episodes. Studies also indicate that compared with older
children, newborns may display a greater physiologic response to pain.
PREP Pearls
• Inadequate pain management in infants can lead to significant short- and long-term
sequelae.
• Pain should be managed prophylactically whenever possible.
• A multimodal approach to pain management incorporating environmental, behavioral,
and pharmacologic interventions is the recommended approach for children of all
ages.
• Parental input and involvement should be incorporated into the pain management
plans of all children.
Suggested Readings
• Anand KJ, International Evidence-Based Group for Neonatal Pain. Consensus
statement for the prevention and management of pain in the newborn. Arch Pediatr
Adolesc Med. 2001;155:173-180. DOI:http://dx.doi.org/10.1001/archpedi.155.2.173.
• Gray L, Garza E, Zageris D, Heilman KJ, Porges SW. Sucrose and warmth for
analgesia in healthy newborns: an RCT .Pediatrics. 2015;135:e607-e614. DOI:
http://dx.doi.org/10.1542/peds.2014-1073.
• Zempskey WT, Schecter NL. What’s new in the management of pain in young
children. Pediatr Rev. 2003;24:337-348. DOI: http://dx.doi.org/10.1542/pir.24-10-
337.
Question 257
A 33-week gestation newborn who was delivered at home is brought to the emergency
department via ambulance. The pregnancy was complicated by limited prenatal care and
maternal substance abuse. The mother was unable to describe the events surrounding the labor
and delivery. Your quick assessment reveals a heart rate of 90 beats/min and a respiratory rate of
50 breaths/min. Physical examination demonstrates a jittery newborn with decreased perfusion,
moderate grunting, and cool extremities.
Of the following, the MOST likely cause of the clinical findings seen in this newborn is
A. drug withdrawal
B. hypothermia
C. intraventricular hemorrhage
D. sepsis
E. surfactant deficiency
All newborns are at risk for convective, evaporative, and conductive heat loss at the time of
delivery. This heat loss is exaggerated in newborns born prematurely because of decreased
epidermal thickness, decreased brown fat, and increased surface area. Optimizing the
temperature in the delivery room, gently drying the neonate, and replacing any wet towels and
blankets with dry warm blankets can minimize heat loss in the full-term and late-preterm
newborn. Unless clinically unstable, full-term and late-preterm newborns should be placed on the
mother’s chest shortly after birth for both bonding and enhanced thermoregulation.
A radiant warmer should be available in the delivery room for management of newborns who are
unstable or premature. This will provide the resuscitation team ready access to the newborn
while minimizing heat loss. The amount of heat delivered by the radiant warmer must be
carefully monitored. Many radiant warmers allow a temperature probe to be placed on the
newborn’s skin that allows auto regulation of the overbed heat output. If auto regulation is
unavailable, and the radiant warmer temperature is manually set to deliver maximum heat, the
newborn’s temperature should be monitored frequently to avoid hyperthermia.
The clinical signs and symptoms of cold stress can mimic those of neonatal sepsis or drug
withdrawal, though sinus bradycardia is generally not present in those conditions. Severe
intraventricular hemorrhage (IVH) can also have similar findings as cold stress, but most IVH
cases occur in premature infants born at less than 32 weeks of gestation. Cold stress can lead to
surfactant deficiency in affected premature infants, but the mild tachypnea without other signs of
respiratory distress in the newborn in the vignette makes surfactant deficiency unlikely.
PREP Pearls
• Newborns are at risk for convective, evaporative, and conductive heat loss at the time
of delivery.
• A newborn with cold stress presents with tachypnea and peripheral vasoconstriction.
The peripheral vasoconstriction leads to decreased perfusion, poor color, and cool
extremities.
• Biochemical findings associated with cold stress include metabolic acidosis and
hypoglycemia.
• Unless clinically unstable, full-term and late-preterm newborns should be placed on
the mother’s chest and gently dried shortly after birth for both bonding and enhanced
thermoregulation.
Suggested Readings
• American Academy of Pediatrics. Textbook of Neonatal Resuscitation. 6th ed. Elk
Grove Village, IL: American Academy of Pediatrics; 2011:329 pp.
• Laptook A, Jackson GL. Cold stress and hypoglycemia in the late preterm (“near-
term”) infant: impact on nursery of admission. Semin Perinatol. 2006;30:24-27. DOI:
http://dx.doi.org/10.1053/j.semperi.2006.01.014.
• Sedin G. To avoid heat loss in very preterm infants. J Pediatr. 2004; 145: 720-722.
DOI:http://dx.doi.org/10.1016/j.jpeds.2004.08.065.
Question 258
You are seeing a 6-year-old boy in follow-up from a visit 1 month ago for newly noted
intermittent exotropia. He was referred to an ophthalmologist, who found an unremarkable
examination and recommended monitoring. The boy’s mother notices his eye wandering several
times a day now. In addition, she feels he looks excessively tired in the afternoons and she is
putting him down for naps, but he says he is not tired at all. He goes to sleep easily at night,
sleeps through the night, and wakes up without difficulty. He does not snore. He has not had
fevers, headaches, or difficulty with walking or running. His physical examination, including
neurological examination, is unremarkable.
D. polysomnography
The diagnosis of myasthenia gravis is confirmed with serum myasthenia gravis antibody titers. In
hospitalized patients, edrophonium can be given intravenously to help confirm the diagnosis, but
is not used as commonly as it was in the past. Edrophonium is a short-acting acetylcholinesterase
inhibitor which, when given intravenously, will cause a patient with myasthenia gravis to have a
sudden improvement in symptoms. The patient may also experience sudden salivation, and
possibly defecation or bradycardia because of its parasympathomimetic effects. Another method
used to diagnose myasthenia gravis is a nerve conduction study with repetitive stimulation.
Repetitive stimulation is a specialized test that confirms the presence of a defect in signal
transmission at the neuromuscular junction. This is almost always caused by myasthenia gravis,
but the diagnosis must be confirmed with antibody testing. Initial treatment of myasthenia gravis
is usually with oral pyridostigmine, a longer-acting acetylcholinesterase inhibitor than
edrophonium. Chronic immunosuppression is sometimes necessary.
The boy in the vignette appeared sleepy due to ptosis that worsened in the afternoon, not fatigue,
so evaluation for causes of chronic fatigue, such as Epstein-Barr virus antibody titers or
polysomnography, are not needed. A trial of stimulants is also not indicated for ptosis. Magnetic
resonance imaging of the brain is not helpful in diagnosing disorders of the neuromuscular
junction such as myasthenia gravis.
PREP Pearls
• Drooping eyelids can be the result of fatigue or ptosis in patients with myasthenia
gravis.
• New onset of intermittent ocular misalignment and intermittent ptosis is suggestive of
myasthenia gravis.
Suggested Readings
• Index of suspicion. Pediatr Rev. 2000;21(9):315-320. doi:
http://dx.doi.org/10.1542/pir.21-9-315.
American academy of pediatrics 822
American Academy of Pediatrics PREP 2016
• Patterson MC, Gomez MR. Muscle disease in children: a practical approach. Pediatr
Rev. 1990;12(3):73-82. DOI:http://dx.doi.org/10.1542/pir.12-3-73.
Question 259
A 12-year-old boy with spastic quadriplegic cerebral palsy, seizure disorder, osteopenia, and
intellectual disability sustains a complex fracture of his left femur after an accidental fall. He is
subsequently wheelchair bound for 2 months. He is brought to your office for evaluation of new
onset anorexia, nausea, abdominal pain, and polyuria.
Current medications include:
• Valproic acid, 500 mg orally twice daily
• Methylphenidate hydrochloride, 36 mg orally once daily
• Vitamin D3, 1,000 IU orally once daily
Of the following, the laboratory test MOST likely to have abnormal results is
A. platelet count
D. serum potassium
E. urine culture
Liver disease or hypokalemia might cause similar symptoms to those of the child in the vignette,
but in the setting of immobilization, hypercalcemia is more likely. A urinary tract infection also
might mimic these findings, but the boy has no fever or dysuria. Low platelet count could present
with easy bruising or even internal bleeding, but would not be related to immobilization.
Of note, this patient is also taking vitamin D, which could potentially raise calcium levels
further. However, he is only taking 1,000 IU of vitamin D per day. Although the recent Institute
of Medicine (IOM) Report on Vitamin D recommends only 600 IU of vitamin D per day in a
child of this age, the IOM, the Endocrine Society, and the European Food Safety Authority all
note the safe upper limit of vitamin D intake to be up to 4,000 IU per day for children 11 years of
age or older. Thus, for the child in the vignette, immobility is likely the major contributor to his
hypercalcemia, not his vitamin supplementation.
PREP Pearls
• Hypercalcemia can occur after prolonged periods of immobilization, especially in
growing children and adolescents.
• The Institute of Medicine recommends a daily allowance of 600 IU per day of
vitamin D for children 1 year of age or older.
Suggested Readings
• Ross AC, Taylor CL, Yaktine AL, et al. Dietary Reference Intakes for Vitamin D and
Calcium; Food and Nutrition Board,.Dietary Reference Intakes for Calcium and
Vitamin D. Washington, DC: National Academies Press.
2011.http://www.iom.edu/Reports/2010/Dietary-Reference-Intakes-for-calcium-and-
vitamin-D.aspx.
• Vogiatzi MG, Jacobson-Dickman E, Deboer MD; Drugs, and Therapeutics
Committee of The Pediatric Endocrine Society. Vitamin D supplementation and risk
of toxicity in pediatrics: a review of current literature. J Clin Endocrinol Metab.
2014;99(4):1132-1141. DOI: http://dx.doi.org/10.1210/jc.2013-3655.
American academy of pediatrics 825
American Academy of Pediatrics PREP 2016
Question 260
You are evaluating a 15-year-old female adolescent with the chief concern of hoarseness of
voice. Two weeks prior to presentation, the patient’s father reports a history of acute and near
complete absence of voice for 1 to 2 days. Although she can now speak, her voice is still
abnormal with lower volume, lower pitch, and a persistent hoarse quality. There has been no
stridor, cough, wheezing, or fever. Symptoms of gastroesophageal reflux are denied. Nasal
symptoms have been notably absent. The patient is an avid cheerleader.
On physical examination, the patient is well appearing in no acute distress. Her voice is “raspy”
and low pitched. There is no stridor or respiratory distress. A head, eyes, ears, nose, and throat
examination is unremarkable. Nasal mucosa is normal in appearance. The oropharynx is clear.
Her neck is supple without palpable lymphadenopathy. Lungs are clear bilaterally without
wheezing, crackles, or differential aeration. Cardiac, abdominal, extremity, and neurologic
examinations are unremarkable.
Of the following, the MOST likely etiology of this patient’s hoarseness of voice is
A. allergic rhinitis
C. laryngeal papillomatosis
D. vocal abuse
The larynx descends in anatomic location from birth to adulthood. The normal position of the
vocal cords is C4 at birth, with descent to C5-C6 by adolescence and further to C7 in adulthood.
Laryngeal growth also occurs in concert with linear growth. Laryngeal growth normally
accelerates during infancy and preschool years and then again during adolescence. These
changes in the structure and function of the pediatric larynx can lead to dysphonia.
Infections (most commonly viral pathogens) are common etiologies for acute laryngitis. In these
patients, hoarseness of voice is often experienced in addition to more classic symptoms of an
upper respiratory tract infection, such as fever, cough, and rhinitis. Vocal symptoms in this
setting typically resolve within 7 to 10 days. If symptoms persist, an evaluation of the airway for
alternate etiologies of chronic laryngitis is warranted.
Laryngeal stenosis and laryngeal webs or subglottic cysts may narrow the flow of air sufficient
to create a “breathy” voice or one with abnormal frequency or resonance. Gastroesophageal
reflux (GER) and laryngopharyngeal reflux have been a recent topic of significant interest in the
evaluation of children with nonspecific airway symptoms including chronic hoarseness. In
addition, the population of children with defined eosinophilic esophagitis and related
gastrointestinal and airway symptoms has grown in recent decades. The extent to which
diagnostic evaluations for these disorders are pursued is generally dependent on clinical history.
An empiric trial of therapies for GER may be considered if visual findings at bronchoscopy or
endoscopy are classic. Although young children may have esophageal evidence of GER disease
without relatable symptoms such as heartburn, dyspepsia, vomiting, or dysphagia, older patients,
such as the adolescent in this vignette, are more likely to exhibit classic symptomatology.
The most feared etiology of chronic hoarseness of voice is also one of the rarest. Laryngeal
papillomatosis is infrequently encountered, affecting 4.5 out of 100,000 children; it is caused by
human papilloma virus 6 and 11. Papillomas grow over time, with a predilection for the vocal
cords, larynx, and bronchi; progressive growth and a recurrent pattern may cause respiratory
insufficiency and death through airway obstruction. This disease is manageable with surgery and
antiviral therapy.
Vocal cord paralysis may cause hoarseness and may occur after cardiac surgery or with injury to
the recurrent laryngeal nerve during thyroid or other surgical interventions at the cervical region.
Alternate etiologies for acute and chronic hoarseness include intubation trauma, allergic rhinitis,
and environmental allergens or irritants. Furthermore, in patients treated for asthma, inhalational
therapies such as short acting β-agonists and inhaled corticosteroids may cause hoarseness,
presumably from local irritation.
In patients with chronic hoarseness, an evaluation of the upper airway by otolaryngology head
and neck surgery (OHNS or ENT) is warranted to evaluate for alternate etiologies. If vocal abuse
is confirmed, voice rest and therapy may establish more functional speech patterns in order to
limit inflammatory response and restore normalcy of voice.
PREP Pearls
• The most common etiology for acute hoarseness is an uncomplicated viral illness
with associated laryngitis.
• The most common etiology for chronic hoarseness of voice is vocal abuse.
• Iatrogenic hoarseness may be induced by inhalational therapies for asthma.
• A laryngoscopic evaluation of the upper airway is warranted in patients with chronic
hoarseness of voice.
Suggested Readings
• Connelly A, Clement WA, Kubba H. Management of dysphonia in children. J
Laryngol Otol. 2009;123(6):642-647.
DOI:http://dx.doi.org/10.1017/S0022215109004599.
• Kelchner LN, Brehm SB, de Alarcon A, Weinrich B. Update on pediatric voice and
airway disorders: assessment and care.Curr Opin Otolaryngol Head Neck Surg.
2012;20(3):160-164. DOI:http://dx.doi.org/10.1097/MOO.0b013e3283530ecb.
Question 261
An 8-year-old boy presents to your office for evaluation of sleep problems. His bedtime is 8:30
PM, but he usually falls asleep while watching television around 11:00 PM. Once he falls asleep,
he sleeps soundly during the night. He awakens at 6:30 AM, but is often drowsy and difficult to
arouse. He does not snore or mouth breathe, wet the bed, or have a history of behavioral health
problems. He is performing adequately in school. On physical examination, he has a body mass
index that is at the 79th percentile for his age, 2+ tonsils, and patent nares. The remainder of his
examination is unremarkable.
Of the following, the MOST appropriate initial management for this child is to
The child in the vignette has a history typical of delayed sleep phase, therefore further
interventions beyond sleep hygiene counseling are not recommended at this time. He does not
demonstrate snoring, apnea, mouth breathing, or nasal congestion that might suggest obstructive
sleep apnea and has no other symptoms of parasomnias. Therefore, a sleep study should be
considered if he does not respond to sleep hygiene intervention or if further symptoms are noted.
Since he has no signs of nasal congestion, topical steroids are unlikely to affect his sleep.
Allowing catch-up sleep on weekends can actually worsen delayed sleep phase syndrome and
thus is discouraged. Appropriate physical activity should be encouraged for all children and
exertion during the afternoon is unlikely to affect his sleep.
PREP Pearls
• The presence of a bedroom television has been associated with later bedtimes,
difficulty falling asleep, and decreased total sleep hours.
• Electronic media in the child’s bedroom has been associated with increased risk for
obesity, substance use, and exposure to sexual content.
• The pediatrician should ask 2 key questions regarding media use at each well child
visit: “How much recreational screen time does your child or adolescent consume
daily?” and “Is there a TV set or an internet-connected electronic device in the child’s
or adolescent’s bedroom?”
• Anticipatory guidance about media exposure should include discussion of screen time
limits, appropriate use of social media, and developing a family media plan that
includes restrictions on inappropriate or excessive media use while promoting family
time.
Suggested Readings
• American Academy of Pediatrics. Children, adolescents, and the media. Pediatrics.
2013;132(5):958-961. DOI:http://dx.doi.org/10.1542/peds.2013-2656.
• American Academy of Pediatrics. Policy statement–children, adolescents, obesity and
the media. Pediatrics. 2011;128:201-205. DOI: http://dx.doi.org/10.1542/peds.2011-
1066.
• Bhargava S. Diagnosis and management of common sleep problems in children.
Pediatr Rev. 2011;32(3):91-98. DOI:http://dx.doi.org/10.1542/pir.32-3-91.
• Garrison MM, Liekweg K, Christakis DA. Media use and child sleep: the impact of
content, timing and environment.Pediatrics. 2011;128:29-35. DOI:
http://dx.doi.org/10.1542/peds.2010-3304.
• Strasburger VC. Pediatricians, schools, and media. Pediatrics. 2012;129(6):1161-
1163. DOI:http://dx.doi.org/10.1542/peds.2011-3213.
Question 262
A 16-year-old adolescent presents to the office for a health supervision visit. Upon review of his
social history, you discover that his grades have dropped significantly over the last year. He was
also recently suspended for coming to school intoxicated. He reports daily marijuana use. He
states that he drinks alcohol and smokes marijuana when alone and to feel better about life. His
friends have told him to stop using marijuana because he forgets things when he is using it.
B. identifying referral options and facilitating the patient’s follow-through with them
Adolescents with a CRAFFT score of 1 should receive brief counseling regarding the effects of
substance use. A score of 2 or more is considered positive.
The adolescent in this scenario reports daily marijuana use. According to the Screening to Brief
Intervention (S2BI)- based approach to Clinical Screening Brief Intervention and Referral to
Treatment (SBIRT, Item C262), which triages adolescents based on risk categories for substance
use disorders, this patient would be considered at risk for severe substance use disorder. In
addition to assessing for problems and advising the patient to quit, identifying referral options
and facilitating the patient’s follow through would be the most appropriate next step. Referring
providers should track the adolescent’s progress through follow-up appointments or phone calls.
While providing information about the adverse effects of alcohol and drugs may be helpful to
this individual, more intensive intervention would be required in this situation. Similarly,
working with the parents to restrict the adolescent’s access to drugs and alcohol would not be
sufficient. If an adolescent with a CRAFFT score of 2 or more has no major problems and has
the perceived self-efficacy for behavioral change, a brief in-office intervention may be sufficient,
but the typical follow-up would be less than 2 months. Breaking confidentiality for the patient’s
safety should be considered in this situation; however, notifying law enforcement is not
necessarily indicated.
PREP Pearls
• Guidelines for adolescent preventive services recommend that providers screen
adolescents at least annually for substance use.
• The CRAFFT screening tool is a 6-item assessment of adolescents’ substance use.
• SBIRT is the framework for in-office intervention and referrals when using and
interpreting the CRAFFT.
Suggested Readings
• American Academy of Pediatrics. Substance use screening, brief intervention, and
referral to treatment for pediatricians.Pediatrics. 2011;128(5):e1330-e1340. DOI:
http://dx.doi.org/10.1542/peds.2011-1754.
• Massachusetts Department of Public Health Bureau of Substance Abuse Services.
Provider guide: adolescent screening, brief intervention, and referral to treatment for
alcohol and other drug use using the CRAFFT screeing tool. Masseachusetts
Department of Substance Abuse Services
website.http://www.mass.gov/eohhs/gov/departments/dph/programs/substance-
abuse/prevention/screening-brief-intervention-and-referral-to.html.
Question 263
You are caring for a 16-year-old adolescent with cystic fibrosis. His forced expiratory volume in
1 second is moderately decreased at 1.5 L (45% predicted). He is chronically infected with
Pseudomonas aeruginosa and methicillin-resistant Staphylococcus aureus. He was most recently
hospitalized with acute and non-massive hemoptysis. He does not have pulmonary hypertension
and has not required chronic supplemental oxygen. Eight months ago, he was admitted to the
intensive care unit when he required noninvasive respiratory support with bi-level positive
pressure ventilation during a pulmonary exacerbation. At his parents’ request, you begin to
discuss lung transplant evaluation and this patient’s advance care directives.
B. discussions regarding advance directives are premature and should not occur until the
patient has experienced additional cystic fibrosis-related complications
C. earlier rather than later discussion of advance care directives may ease anxiety and
improve quality of life in patients with chronic disease
D. the involvement of palliative care sends a message that the medical team is “giving
up” on the patient
E. the patient should not participate in advance care directive discussions because he has
not yet reached the age of legal consent
The Patient Self Determination Act of 1991 mandates that patient preferences as to advance
directives (AD) be actively solicited and documented for all adult patients. In contrast, no similar
legislation has been directed at children and adolescents. It is generally appreciated that end-of-
life discussions allow patients and families to examine their disease and their treatment
preferences. When inquiries are made as to end-of-life preferences, literature has shown that the
majority of families choose home or hospice as a preferred location of death for their child. In
instances where care directives are not obtained, children are more likely to expire in the
hospital.
The World Health Organization (WHO), in 2008, defined palliative care for children as “the total
active care of the child’s body, mind, and spirit,” along with “support to the family.”
Furthermore, they outlined that palliative care “begins when the illness is diagnosed and
continues regardless of whether or not a child receives treatment directed at the disease.” An
effective palliative care approach is multidisciplinary in nature and includes the family, as well
as community resources. In keeping with this WHO recommendation, discussions regarding ADs
are not dependent on the stage of disease and can proceed during active treatment. The intent of
these discussions should not be interpreted as the health care team “giving up.”
Even prior to the age of legal consent, it has become apparent that children and adolescents will
be more educated and aware of care directives if they are involved and have a voice in these
discussions. In clinical research studies, the “assent” of younger children is routinely sought.
Most pediatric palliative care has focused on children with HIV and cancer. In a 2014
investigation of US parents’ and caregivers’ awareness of ADs, 61.6% of respondents reported
that they had never heard of ADs and 81.8% had never discussed one previously. Only 3% had
created an AD for their child.
The community of pediatric care providers will continue to benefit from a standardized approach
to planning for end-of-life care. Best practice guidelines were developed in England in 2007. In
the United States, the Children’s Hospice and Palliative Care Services Project (ChiPPS) aims to
enhance pediatric hospice and palliative care education, access, and quality.
American academy of pediatrics 837
American Academy of Pediatrics PREP 2016
PREP Pearls
• The approach to end-of-life planning with an advanced directive discussion should
take place early in the chronic disease course, with updates to the plan as needed.
• The approach to the end-of-life care planning for a patient with chronic and life-
limiting illness is multidisciplinary, and should include various members of the
medical and health care team, as well as family and community resources.
Suggested Readings
• Brook L, Hain R. Predicting death in children. Arch Dis Child. 2008;93(12):1067–
1070. DOI:http://dx.doi.org/10.1136/adc.2007.127332.
• Liberman DB, Pham PK, Nager AL. Pediatric advance directives: parents’
knowledge, experience and preferences.Pediatrics. 2014;134(2):e436-e443. DOI:
http://dx.doi.org/10.1542/peds.2013-3124.
• Lyon ME, Garvie PA, Briggs L, et al. Is it safe? Talking to teens with HIV/AIDS
about death and dying: a 3-month evaluation of Family Centered Advance Care
(FACE) planning - anxiety, depression, quality of life. HIV AIDS (Auckl).
2010;2:27–37. http://www.ncbi.nlm.nih.gov/pmc/articles/PMC3218704/.
• Lyon ME, Garvie PA, McCarter R, Briggs L, He J, D’Angelo LJ. Who will speak for
me? Improving end-of-life decision-making for adolescents with HIV and their
families. Pediatrics. 2009;123(2):e199-e206.
DOI:http://dx.doi.org/10.1542/peds.2008-2379.
• Lyon ME, Jacobs S, Briggs L, Cheng YI, Wang J. A longitudinal, randomized,
controlled trial of advance care planning for teens with cancer: anxiety, depression,
quality of life, advance directives, spirituality. J Adolesc Health. 2014;54(6):710-717.
DOI: http://dx.doi.org/10.1016/j.jadohealth.2013.10.206.
• World Health Organization. Definition of palliative care for children. World Health
Organization website.http://www.who.int/cancer/palliative/definition/en/.
Question 264
A 1-day-old female newborn, with an unremarkable prenatal history, is admitted to the neonatal
intensive care unit for increasing lethargy and feeding difficulties. The baby was born at term, by
normal spontaneous vaginal delivery, after spontaneous rupture of membranes with clear
amniotic fluid. Apgar scores were 8 and 9 at 1 and 5 minutes, respectively. Prenatal screening for
infection was negative. The newborn took 20 mL of formula within 1 hour of birth, but has taken
less with each subsequent feed. She has to be awakened and stimulated in order to feed. Upon
admission, the newborn is minimally arousable with painful stimuli. Her vital signs show a
temperature of 37°C, pulse of 150 beats/min, respiratory rate of 20 breaths/min, blood pressure
of 60/40 mm Hg, and oxygen saturation of 100% on room air. Her pupils are 4 mm, equal, and
sluggish. Her lungs are clear to auscultation bilaterally and her heart has a regular rate and
rhythm. The newborn’s muscle tone is decreased.
Of the following, the test MOST likely to reveal this newborn’s diagnosis is
A. blood culture
D. echocardiography
The many causes of coma in neonates can be divided by organ system, and the differential
diagnosis for each case can be narrowed based on the clinical presentation. Infectious causes are
common, and can include bacterial, viral, and parasitic etiologies. There is often an antecedent
history of prolonged rupture of membranes, maternal fever, or a positive screen for group B
Streptococcus. Pathogens of neonatal sepsis that are not routinely screened for include
Escherichia coli, Listeria monocytogenes, and herpes simplex virus. Congenital heart disease is
another important cause of poor feeding and lethargy in neonates. Although cyanotic congenital
heart disease is usually apparent at birth, infants with patent ductus arteriosus–dependent lesions
such as hypoplastic left heart syndrome, interrupted aortic arch, and critical aortic stenosis may
appear acyanotic and otherwise normal at birth, but deteriorate within the first few days when the
ductus arteriosus closes. Fetal tachycardia and cardiomyopathy can also cause progressive poor
feeding and lethargy. None of these cardiac problems would cause hyperammonemia. Maternal
intoxication, perinatal asphyxia, hypoglycemia, and birth trauma are causes of neonatal coma
that can be identified through maternal or perinatal history; however, none of these would
present with hyperammonemia.
The other main category of causes of neonatal coma is inborn errors of metabolism. These
infants appear normal at birth because in utero the metabolite causing encephalopathy, usually a
small molecule that can readily cross the placenta, is cleared by the mother’s metabolism. For
these infants the time of onset of signs and symptoms of a metabolic crisis will vary, but in
severe cases can occur within hours or days. Poor feeding, lethargy, respiratory depression, and
hypotonia are common initial signs. Acute encephalopathy presenting in an infant with an
unremarkable history and unremarkable results of routine blood tests, cultures, and chest
radiography should alert the clinician to consider an inborn error of metabolism. Acidosis,
hyperammonemia, and hypoglycemia can cause coma, intracranial hypertension, and
hemodynamic compromise. Suspected metabolic disease should prompt urgent evaluation of pH,
lactate level, electrolytes, liver function tests, ammonia, and glucose level. More common
metabolic disorders (Item C264) include organic acidemias, urea cycle defects, glycogen storage
diseases, galactosemia, and fatty acid oxidation defects. The diagnosis can often be made with
evaluation of urine organic acids.
While blood culture and evaluation for herpes simplex virus could reveal an infectious diagnosis
for the infant in this vignette, a metabolic cause of this presentation is more likely. Brain
magnetic resonance imaging can detect brain malformations that can cause encephalopathy, but
those infants generally have an abnormal neurologic status immediately after birth.
Echocardiography would be important if congenital heart disease were suspected, but such
infants generally do not have hyperammonemia.
PREP Pearls
• Common causes of lethargy and coma in a neonate include infection, cardiac
abnormalities, and metabolic disorders.
• Newborns with inborn errors of metabolism, cardiomyopathy, or patent ductus
arteriosus (ductal) dependent cardiac lesions may appear normal at birth then develop
poor feeding and lethargy in the first few days after birth.
• Neonatal hyperammonemia can be caused by urea cycle defects, fatty acid oxidation
defects, and organic acidemias.
Suggested Readings
• Donn SM, Banagale RC. Neonatal hyperammonemia. Pediatr Rev. 1984;5:203-
208.http://pedsinreview.aappublications.org/content/5/7/203 .
• Levy PA. Inborn errors of metabolism: part 1: overview. Pediatr Rev. 2009;30:131-
138. DOI:http://dx.doi.org/10.1542/pir.30-4-131.
• Levy PA. Inborn errors of metabolism: part 2: overview. Pediatr Rev. 2009;30:e22-
e28. DOI:http://dx.doi.org/10.1542/pir.30-4-e22.
Question 265
An 11-month-old female infant presents to the emergency department with fever and lethargy.
She was seen by her primary care physician 8 days ago with upper respiratory tract symptoms
and a cough. Vital signs show a temperature of 39.8°C, respiratory rate of 60 breaths/min, heart
rate of 178 beats/min, blood pressure of 118/57 mm Hg, and a weight of 10.09 kg. On physical
examination, she is lethargic and has a full anterior fontanelle. Laboratory data shows:
• White blood cells, 12,400/µL (12.4 x 109/L)
• Hemoglobin, 11 g/dL (110 g/L)
• Platelets, 450 x 103/µL (450 x 109/L)
• Differential, 27% segmented neutrophils, 30% bands, 38% lymphocytes, 5% monocytes
• Cerebrospinal fluid (CSF) white blood cells, 2,200/µL (98% segmented neutrophils)
• CSF red blood cells, 24/μL
• CSF glucose, 20 mg/dL (1.1 mmol/L)
• CSF protein, 375 mg/dL
• CSF Gram stain, gram-negative diplococci
Of the individuals who had contact with this patient, the BEST candidate for chemoprophylaxis
is the
While a childcare center contact is considered a close contact, exposure 10 days prior to
symptom onset precludes concern for infection in this patient. The incubation period for N
meningitides is 1 to 10 days and symptomatic infection usually occurs in less than 4 days.
Healthcare personnel with direct exposure to the patient’s secretions are candidates for
chemoprophylaxis. The resident who took the history in the emergency department does not
require chemoprophylaxis. Obtaining a nasal wash, as in response choice E, would be considered
direct exposure to respiratory secretions. However, this sample was obtained more than 7 days
before the symptom onset, so the nurse does not require chemoprophylaxis.
PREP Pearls
• Antimicrobial prophylaxis is recommended for close contacts of patients with
invasive meningococcal disease, including household members, daycare, or childcare
center contacts with whom contact occurred at any time during the seven days before
symptom onset, and anyone with direct exposure to the index case’s oral secretions in
the 7 days before symptom onset.
• Chemoprophylaxis for Neisseria meningitidis should be administered within 24 hours
with rifampin, ceftriaxone, or ciprofloxacin.
American academy of pediatrics 844
American Academy of Pediatrics PREP 2016
• Healthcare personnel without direct exposure to the patient’s secretions are not
candidates for chemoprophylaxis.
Suggested Readings
• American Academy of Pediatrics. Meningococcal infections. In: Kimberlin DW,
Brady MT, Jackson MA, Long SS, eds. Red Book: 2015 Report of the Committee on
Infectious Diseases. 30th ed. Elk Grove Village, IL: American Academy of
Pediatrics; 2015:547-558.
• Crohn AC, MacNeil JR, Clark TA, et al. Prevention and control of meningococcal
disease: recommendations of the advisory committee on immunization practices.
MMWR Morbid Mortal Wkly Rep. 2013;62(RR02):1-
28.http://www.cdc.gov/mmwr/preview/mmwrhtml/rr6202a1.htm.
Question 266
A 10-year-old boy presents to your office with the chief complaints of increased thirst,
micturition, and nighttime bed wetting for the last 6 months. According to his mother, the patient
attained bladder and bowel control by 4 years of age. The patient has also been complaining of
headaches on waking up in the morning, which have been increasing in severity for the last
week. He has also had vomiting associated with headaches on waking up for the last 2 days.
There is no history of encopresis. They moved to a new home and the patient is attending a new
school for the last year. He has been an above average student. Physical examination reveals a
temperature of 37.8°C, heart rate of 76 beats/min, respiratory rate of 16 breaths/min, blood
pressure of 98/50 mm Hg, and normal growth parameters. He is in no pain currently.
His urinalysis demonstrates a specific gravity of 1.005, pH of 6.0, and no blood, leukocyte
esterase, protein, or nitrites.
A. psychogenic polydipsia
B. small bladder
C. stress
Polyuria is characterized by an increased total urine volume resulting from an underlying defect
in water balance. This presents with the excretion of large volumes of dilute urine, as seen in
diabetes mellitus (osmotic diuresis), diabetes insipidus (anti-diuretic hormone disorders), and
psychogenic polydipsia. It is important to note that children with polyuria may have nocturia or
nocturnal enuresis; however, the more frequently reported symptoms of frequency, nocturia, or
enuresis may not be associated with increased urinary volume (or polyuria).
Recent onset of increased thirst, micturition, and nocturnal enuresis (and nocturia) is indicative
of increased urine volume in the patient. His urinalysis shows a specific gravity of 1.005, which
also points towards an underlying urine concentration defect associated with increased urine
volume. Absence of glucose in the urine rules out diabetes mellitus as the cause of his polyuria.
Diabetes insipidus (DI) occurs secondary to either decreased secretion of anti-diuretic hormone
(ADH) known as central DI, or renal resistance to ADH effects known as nephrogenic DI.
Patients with DI present with polyuria, polydipsia, and increased thirst. Patients with DI have
decreased urine osmolality and associated serum osmolality that may be normal (older patients
able to increase their water intake) or increased (in patients unable to increase their water intake,
as in neonates or those with developmental delay). A detailed neurologic examination, including
examination of the spine, is vital in any patient presenting with recent onset abnormal voiding
patterns. The history of headaches more prominent on waking up in the morning and association
with vomiting is indicative of intracranial pathology, as noted for the patient in this vignette.
Central DI is characterized by decreased ADH secretion. It could be idiopathic (most common)
or secondary to central nervous system tumors, infiltrative lesions (histiocytosis), and trauma
(surgical or nonsurgical).
longer indicated for the treatment of primary nocturnal enuresis. If there is no response to
vasopressin and the urine osmolality remains at less than 100 mOsm/kg (100 mmol/kg) over
baseline, the diagnosis of nephrogenic DI is confirmed.
Psychogenic polydipsia presents with hyponatremia associated with a low urine osmolality,
consistent with water overload. Maximal urine concentration is usually impaired (500 to 600
mOsm/kg) compared to that in normal patients (800 mOsm/kg or more). Water deprivation in
these patients will increase the urine osmolality (> 500 mOsm/kg) and there will be no response
to desmopressin because endogenous production and release of ADH is normal.
Recent onset stress such as moving to a new home and school can sometimes lead to secondary
nocturnal enuresis in children. However, the patient’s symptoms of morning headaches
increasing in severity and vomiting are suggestive of an intracranial pathology.
Symptoms of bladder dysfunction (associated with underlying small bladder capacity, overactive
bladder, dysfunctional voiding) include urinary frequency, urgency, and urge incontinence.
Incontinence in such patients may present with daytime or nocturnal enuresis. Constipation is a
commonly associated symptom in patients with bladder dysfunction. Examination of the spine is
vital in any patient presenting with abnormal voiding patterns. Skin abnormalities of the spine
such as tuft of hair, vascular lesions (hemangioma), or discoloration of the skin overlying the
spine are suggestive of an underlying vertebral or spinal lesion. Spinal cord lesions (even very
low sacral lesions associated with normal lower extremity function) are associated with bladder
dysfunction because bladder control is below the level for lower extremity function in the spinal
cord. Bladder function is evaluated with renal bladder ultrasonography, voiding
cystourethrogram, and urodynamic studies (intravesical pressures and volume of fluid during
filling, storage, and voiding). In the vignette, urinary tract infection is unlikely in the absence of
fever or urinary symptoms such as dysuria, flank pain, or burning micturition. The absence of
leukocyte esterase, nitrites, and bacteria on urinalysis also rules out urinary tract infection as the
underlying cause of the patient’s symptoms.
PREP Pearls
• Enuresis is diagnosed in children aged 5 years or older who void in bed or on clothes
twice or more per week for 3 consecutive months.
• Polyuria (excretion of large volumes of dilute urine) may be seen in diabetes mellitus
(osmotic diuresis), diabetes insipidus (antidiuretic hormone disorders), and
psychogenic polydipsia.
• Recent onset of increased thirst, micturition, and nocturnal enuresis (and nocturia) is
indicative of increased urine volume in a patient.
• Diabetes insipidus (DI) occurs secondary to either decreased secretion of antidiuretic
hormone (ADH) known as central DI, or renal resistance to ADH effects known as
nephrogenic DI.
Suggested Readings
• Linshaw MA. Back to basics: congenital nephrogenic diabetes insipidus. Pediatr Rev.
2007;28(10):372-380. DOI:http://dx.doi.org/10.1542/pir.28-10-372.
• Miller RS, Libber Sm, Plotnick LP. Polyuria. In: McInerny TK, Adam HM, Campbell
DE, Kamat DM, Kelleher KJ, eds.American Academy of Pediatrics Textbook of
Pediatric Care. 19th ed. Elk Grove Village, IL: American Academy of Pediatrics;
2009:1670-1675.
• Nevéus T, von Gontard A, Hoebeke P, et al. The standardization of terminology of
lower urinary tract function in children and adolescents: report from the
Standardization Committee of the International Children's Continence Society. J Urol.
2006;176(1):314-324. DOI: http://dx.doi.org/10.1016/S0022-5347(06)00305-3 .
• Saborio P, Tipton GA, Chan JCM. Diabetes insipidus. Pediatr Rev. 2000;21(4):122-
129. DOI:http://dx.doi.org/10.1542/pir.21-4-122.
Question 267
A 3-year-old boy presents to your office with a history of recurrent rectal prolapse without
visible blood. He began having prolapse with bowel movements 6 weeks ago. It occurs with
every bowel movement and resolves without intervention. His bowel movements are described
as clusters of grapes. His mother denies fever, recent illness, diarrhea, poor growth, or other
chronic medical issues. Physical examination shows height and weight at the 50th percentile for
age. His abdomen is mildly distended with positive bowel sounds. His rectal examination
identifies a normally placed anus without visible hemorrhoids or fissures.
A. celiac disease
B. cystic fibrosis
C. functional constipation
D. rectal polyps
E. ulcerative colitis
Rectal prolapse is defined as either mucosal or full thickness protrusion of the rectum through
the anus. On physical examination, concentric rings of rectal mucosa can be seen (Item C267).
Rectal prolapse has a male predominance. The differential diagnosis includes constipation, acute
diarrhea, cystic fibrosis, parasitosis, polyps, malnutrition, increased intra-abdominal pressure,
and conditions that result in pelvic floor weakness.
Item C267
Treatment involves management of the underlying cause, when identified. Conservative therapy
with manual reduction of the prolapse and aggressive management of the underlying etiology is
often successful. Emergent evaluation is indicated if the prolapse cannot be reduced. If prolapse
recurrence persists, surgical intervention is recommended.
PREP Pearls
• Constipation is the most common cause of rectal prolapse.
• Conservative medical therapy is often successful.
• Cystic fibrosis and parasites should be ruled out prior to surgical intervention.
• Rectal prolapse has a male predominance.
Suggested Readings
• Gourgiotis S, Baratsis S. Rectal prolapse. Int J Colorectal Dis. 2007;22(3):231-243 .
• Melton GB, Kwaan MR. Rectal prolapse. Surg Clin North Am. 2013;93(1):187-198.
DOI:http://dx.doi.org/10.1016/j.suc.2012.09.010.
Question 268
You are called to assess a female newborn for unusual limb findings. Pregnancy was remarkable
for decreased fetal movement. There was no history of maternal uterine fibroids or
oligohydramnios. Delivery was via cesarean delivery secondary to transverse fetal position.
Weight and length are appropriate for gestational age. On physical examination, you note that the
shoulders are internally rotated, the elbows are extended, the wrists are flexed with ulnar
deviation, the fingers are stiff, and the thumbs are positioned in the palms (Q268). On the lower
extremities, the hips are bilaterally dislocated with knees extended, and the feet have
equinovarus contractures. Breathing is normal and unlabored. Facies are non-dysmorphic.
B. Crouzon syndrome
D. peroxisomal disorders
E. Prader-Willi syndrome
The diagnosis of amyoplasia, as in the newborn in this vignette, should be considered when the
following clinical manifestations are present:
• Symmetric congenital rigid contractures
• Internal rotation of the shoulder, fixed extension of the elbows, pronation of the
forearm, flexion of the wrist
• Significant equinovarus deformity of the foot
• Marked decrease in muscle mass of limbs
• Lack of flexion creases on fingers and hands
• Dimples over affected joints
• Unaffected, mobile trunk
• Nevus flammeus in the mid-face or glabellar region
• Digital reductions (12%)
• An alert infant with non-dysmorphic facies
• Negative family history for arthrogryposis
Making an accurate diagnosis of AMC is essential so the provider can monitor for other
problems that may be associated with the diagnosis and provide accurate prognostic guidelines.
Thirty percent to 40% of patients have early feeding problems, often necessitating gastrostomy
tube or nasogastric tube feedings. Feeding problems are typically short lived, lasting until about
4 months of age. Increased rates of bowel atresia and abdominal wall musculature deficiencies
have been noted due to vascular pathology. Some individuals have respiratory problems initially.
Patients frequently require extensive physical therapy, casting, and orthopedic procedures. Most
have gross and fine motor delays, but have normal intelligence and communication skills.
Individuals are quite capable of excelling scholastically, but will require extensive parental
support because of their physical limitations. Patients are at risk for scoliosis in childhood and
arthritis in affected joints starting in the third decade of life. Dilated ureters and transient
hydronephrosis are present in some individuals. Mothers often report a lack of fetal movement
known as akinesia during pregnancy.
PREP Pearls
• Arthrogryposis multiplex congenita presents with congenital nonprogressive contractures of 2
or more joints of the body. Patients have symmetric congenital rigid contractures involving
internal rotation of the shoulders, fixed extension of the elbows, pronation of the forearm,
flexion of the wrist, and significant equinovarus deformity of the foot.
• Patients with arthrogryposis have a marked decrease in muscle mass of limbs along with non-
dysmorphic facies.
• Arthrogryposis multiplex congenita is a heterogeneous condition that could be secondary to
disorders of the central or peripheral nervous system, maternal myasthenia gravis, connective
tissue disorders leading to decreased fetal movements, vascular causes, uterine crowding
caused by uterine fibroids or other uterine anomalies, environmental factors, maternal
infections (cytomegalovirus or toxoplasmosis), or teratogenicity.
Suggested Readings
• Gordon N. Arthrogryposis multiplex congenita. Brain Dev. 1998;20(7):507-511. DOI:
http://dx.doi.org/10.1016/S0387-7604(98)00037-0.
• Haliloglu G, Topaloglu H. Arthrogryposis and fetal hypomobility syndrome. Handb Clin
Neurol. 2013;113:1311-1319. DOI: http://dx.doi.org/10.1016/B978-0-444-59565-2.00003-4.
• Hall JG, Aldinger KA, Tanaka KI. Amyoplasia revisited. Am J Med Genet A.
2014;164(3):700-730. DOI:http://dx.doi.org/10.1002/ajmg.a.36395.
• Rink BD. Arthrogryposis: a review and approach to prenatal diagnosis. Obstet Gynecol Surv.
2011;66(6):369-377. DOI:http://dx.doi.org/10.1097/OGX.0b013e31822bf5bb.
Question 269
The parents of an 8-month-old male infant bring him to your office for follow-up after a recent
episode of otitis media. He completed a 10-day course of amoxicillin and is clinically improved.
His fever resolved after 3 days and his activity is back to normal. The infant’s mother is
concerned because he has loose stools and diaper rash. He is breastfeeding well, but is not eating
as much pureed food as he did before the illness. On physical examination, you find
erythematous papules and pustules on the penis, scrotum, and inner thighs (Item Q269). There
are bilateral middle ear effusions and a few white plaques on the buccal mucosa. The remainder
of the physical examination is unremarkable.
Item Q269
The presumptive cause of diaper dermatitis should be identified to select the appropriate
treatment. Antibiotics should be used to treat bacterial infections; Streptococcus typically causes
a perianal erythematous rash, whereas Staphylococcus may cause cellulitis or bullous impetigo
on the buttocks. Topical antibiotic cream or ointment may be used for well-localized superficial
bacterial skin infections. Oral antibiotics should be prescribed for widespread lesions, cellulitis,
systemic symptoms, or if there is a concurrent bacterial infection, such as otitis media. More
severe infection may require parenteral antibiotic treatment.
In contrast to candidiasis, irritant diaper dermatitis typically spares the intertriginous creases and
involves the convex surfaces of the buttocks. Both may involve the lower abdomen, proximal
thighs, and perineal area. The primary method of treatment for irritant dermatitis is frequent
diaper changes to keep the area clean and dry. A protective barrier cream, such as zinc oxide or
petrolatum-based preparations, can aid in healing. When applied at every diaper change such
creams are effective in forming a barrier to protect the skin from ongoing contact with stool and
urine. Topical corticosteroids should rarely be used, and only to treat severely inflamed irritant
diaper dermatitis. Only low-potency nonhalogenated topical corticosteroid creams should be
used, sparingly, twice daily for no longer than 3 to 5 days.
PREP Pearls
• The treatment of diaper dermatitis should be directed toward the most likely etiology.
• When oral thrush coexists with perineal candidiasis, an oral antifungal suspension
should be used to treat both concurrently.
• The mainstay of treatment of irritant diaper dermatitis is to minimize direct skin
contact with urine and feces.
Suggested Readings
• Paller AS, Mancini AJ. Cutaneous disorders of the newborn. Hurwitz Clinical
Pediatric Dermatology: A Textbook of Skin Disorders of Childhood and
Adolescence. 4th ed. Philadelphia, PA: Saunders Elsevier; 2011:20-23.
American academy of pediatrics 858
American Academy of Pediatrics PREP 2016
Question 270
A 6-year-old boy presents to the emergency department with pallor, leg pain, and bruising. The
results of a complete blood cell count are shown:
• White blood cell count 11,600/µL (11.6 x 109/L)
• Hemoglobin 7.2 g/dL (72 g/L)
• Platelet count 67 x 103/µL (67 x 109/L)
• % neutrophils 3%
• % leukemic blasts 26%
A blood sample is sent for flow cytometry to determine whether the leukemic blasts are
lymphoid or myeloid. The resident working with you asks why this is important to do at this
time.
Flow cytometry maps the surface antigens on cells. The cells are incubated with antibodies to
surface markers that are conjugated to fluorochromes. After incubation, the cells are drawn in a
single file through the flow cytometer in which various lasers hit the cells. If the wavelength of
light emitted by the laser excites the fluorochrome conjugated to the antibody, a different
wavelength of light is emitted by the fluorochrome that can be detected by the flow cytometer. If
that second wavelength is detected, then the targeted surface marker is present on the cell. Acute
lymphoblastic leukemia and AML have markedly different patterns of surface markers, making
flow cytometry an effective way to rapidly and accurately distinguish between the two. Flow
cytometry can also be used to distinguish between different subtypes of ALL, including B and T
cell.
PREP Pearls
• Acute lymphoblastic leukemia (ALL) and acute myeloblastic leukemia (AML) have
markedly different prognoses and treatments.
• Flow cytometry can rapidly and accurately distinguish AML from ALL.
• Leukopheresis is indicated only in the context of hyperleukocytosis (generally when
the white blood cell count is > 100,000/μL).
Suggested Readings
• Hutter JJ. Childhood leukemia. Pediatr Rev. 2010;31(6):234-241. DOI:
http://dx.doi.org/10.1542/pir.31-6-234.
• Pearce JM, Sills RH. Consultation with the specialist: childhood leukemia. Pediatr
Rev. 2005;26(3):96-104. DOI:http://dx.doi.org/10.1542/pir.26-3-96.
• Pui CH, Carroll WL, Meshinchi S, Arceci RJ. Biology, risk stratification, and therapy
of pediatric acute leukemias: an update. J Clin Oncol. 2011;29(5):551-565. DOI:
http://dx.doi.org/10.1200/JCO.2010.30.7405
Question 271
You have just diagnosed a 9-year-old boy with attention-deficit/hyperactivity disorder, following
your usual data-gathering steps from both school and home that helped characterize the problem.
After discussing the treatment options, the parents have elected to initiate methylphenidate and
plan a follow-up appointment with you in 4 weeks.
Of the following adverse effects, the MOST likely to occur in this patient is
A. dysphoria
B. hallucinations
C. headaches
D. hypertension
E. tics
Of the options listed in the vignette, headaches are the most likely to be experienced by this
child. Usual care monitoring of stimulant use includes following the child's height and weight,
checking blood pressure and pulse after treatment initiation to ensure the child did not have an
outlier response, and reviewing treatment response repeatedly at least every 6 months after
reaching an effective dosage to assess for loss of benefit as the child grows and develops.
PREP Pearls
• The most common adverse effects of stimulant medication include appetite suppression,
insomnia, headaches, stomach aches, dry mouth, and nausea.
• Children receiving stimulant therapy should be monitored at least every 6 months to
measure growth, screen for new adverse effects, and identify any loss of treatment
effectiveness.
Suggested Readings
• Hilt RJ. Monitoring psychiatric medications in children. Pediatr Ann. 2012;41(4):157-
163. DOI: http://dx.doi.org/10.3928/00904481-20120307-13.
• Pierce K. Treatment of ADHD. Pediatr Ann. 2011;40(11):556-562. DOI:
http://dx.doi.org/10.3928/00904481-20111007-06.